Edited By Deeangelee Pooran-Kublall, MD/MPH · 9.5 Preparing for the MCAT: Biochemistry in the...

395

Transcript of Edited By Deeangelee Pooran-Kublall, MD/MPH · 9.5 Preparing for the MCAT: Biochemistry in the...

Page 1: Edited By Deeangelee Pooran-Kublall, MD/MPH · 9.5 Preparing for the MCAT: Biochemistry in the Chemical and Physical Foundations of Biological Systems Section 9.6 Preparing for the
Page 2: Edited By Deeangelee Pooran-Kublall, MD/MPH · 9.5 Preparing for the MCAT: Biochemistry in the Chemical and Physical Foundations of Biological Systems Section 9.6 Preparing for the

MCAT®528AdvancedPrepforAdvancedStudents

EditedByDeeangeleePooran-Kublall,MD/MPH

Page 3: Edited By Deeangelee Pooran-Kublall, MD/MPH · 9.5 Preparing for the MCAT: Biochemistry in the Chemical and Physical Foundations of Biological Systems Section 9.6 Preparing for the

TheKaplanMCAT528Team

DeeangeleePooran-Kublall,MD/MPHEditor-in-Chief

ChristopherDurlandKaplanMCATFaculty,Editor

MatthewDominickEggertKaplanMCATFaculty,Author

SamerT.IsmailKaplanMCATFaculty,Author/Editor

ThomasC.C.Sargent,IIKaplanMCATFaculty,Author

LauraAmblerKaplanMCATFaculty,Author

MCATfacultywriters/contributers:MarilynEngel,JasonPflieger,UneebQureshi,NehaRao,CharlesRichards,NoahSilva

Countless thanks to Kim Bowers; Eric Chiu; Samantha Fallon; Owen Farcy; Dan Frey; RobinGarmise;RitaGarhaffner; JoannaGraham;AdamGrey;AllisonHarm;AlexMacnow,MD;AaronLemon-Strauss; Keith Lubeley; Petros Minasi; John Polstein; Rochelle Rothstein, MD; LarryRudman;SylviaTidwellScheuring;CarlySchnur;LeeWeiss;andmanyotherswhomadethisprojectpossible.

Page 4: Edited By Deeangelee Pooran-Kublall, MD/MPH · 9.5 Preparing for the MCAT: Biochemistry in the Chemical and Physical Foundations of Biological Systems Section 9.6 Preparing for the

AbouttheMCATThestructureofthefoursectionsoftheMCATisshownbelow.BiologicalandBiochemicalFoundationsofLivingSystemsTime

95minutes

Format

•59questions•Scorerange:118and132•Mostquestionsarepassage-based,butsomearediscrete(standalone)questions

WhatItTests

•Biochemistry:25%•Biology:65%•GeneralChemistry:5%•OrganicChemistry:5%

ChemicalandPhysicalFoundationsofBiologicalSystemsTime

95minutes

Format

•59questions•Scorerange:118and132•Mostquestionsarepassage-based,butsomearediscrete(standalone)questions

WhatItTests

•Biochemistry:25%•Biology:5%•GeneralChemistry:30%•OrganicChemistry:15%•Physics:25%

Psychological,Social,andBiologicalFoundationsofBehaviorTime

95minutes

Format

•59questions•Scorerange:118and132•Mostquestionsarepassage-based,butsomearediscrete(standalone)questions

WhatIt

•Biology:5%

Page 5: Edited By Deeangelee Pooran-Kublall, MD/MPH · 9.5 Preparing for the MCAT: Biochemistry in the Chemical and Physical Foundations of Biological Systems Section 9.6 Preparing for the

Tests •Psychology:65%•Sociology:30%

CriticalAnalysisandReasoningSkills(CARS)Time

90minutes

Format

•53questions•Scorerange:118and132•Allquestionsarepassage-based.Therearenodiscrete(standalone)questions.

WhatItTests

Disciplines:

•Humanities:50%•SocialSciences:50%

Skills:•FoundationsofComprehension:30%•ReasoningWithintheText:30%•ReasoningBeyondtheText:40%

TotalTestingTime

375minutes(6hours,15minutes)

Questions

230

ScoreRange

472to528

TheMCATalsotestsfourScientificInquiryandReasoningSkills(SIRS):

1.KnowledgeofScientificConceptsandPrinciples2.ScientificReasoningandProblem-Solving3.ReasoningabouttheDesignandExecutionofResearch4.Data-BasedandStatisticalReasoning

TheMCATisacomputer-basedtest(CBT)andisofferedatPrometriccentersduringalmosteverymonth of the year.There are optional breaks between each section, and a lunchbreakbetween thesecondandthirdsectionsoftheexam.

RegisteronlinefortheMCATatwww.aamc.org/mcat.

Forfurtherquestions,contacttheMCATteamattheAssociationofAmericanMedicalColleges:

MCATResourceCenterAssociationofAmericanMedicalColleges

Page 7: Edited By Deeangelee Pooran-Kublall, MD/MPH · 9.5 Preparing for the MCAT: Biochemistry in the Chemical and Physical Foundations of Biological Systems Section 9.6 Preparing for the

HowThisBookWasCreatedTheKaplanMCAT528 bookwas created to give advancedMCAT students an edge on theMCATexam.ThisbookhighlightsthecontentareasontheMCATwhosemasterywillhelpstudentsachievetheirhighestpossibleMCATscoreonTestDay.Tothatend,wehadlearningscienceexpertspollallofourexpertMCATfacultyandtrainerstodiscernthemosthigh-yieldandhigh-difficultytopicsforstudents.Basedon theresultsof thatstudy,wewereable to identify thecontent topics thateven themost advanced studentsmight need an extra. Then,we had ourmost qualified itemwriters createchallengingtest-likepassagestoprovidethatextrapractice.Additionally,theKaplanMCAT528bookcontainsstrategiccalloutsderivedfromKaplan’sexperienceintest-taking.Infact,skillsareprovidedtohelpstudentsworkthroughthetoughpassagesandgettheacademicboonthatwillallowthemtogetthehighestscorepossible(528)onthe2015MCAT.

A teamof highly dedicatedwritersworked very long hours to create this resource.However, thisbook was submitted for publication in April 2014. For any updates after this date, please visitwww.kaplanmcat.com.

If you have any questions about the content presented here, [email protected]. For other questions not related to content, [email protected].

Thisbookhasseenatleastfiveroundsofreview.Tothatend,theinformationpresenteditistrueandaccurate to the best of our knowledge. Still, your feedback helps us improve our prep materials.Please notify us of any inaccuracies or errors in the books by sending an email [email protected].

Page 8: Edited By Deeangelee Pooran-Kublall, MD/MPH · 9.5 Preparing for the MCAT: Biochemistry in the Chemical and Physical Foundations of Biological Systems Section 9.6 Preparing for the

UsingThisBookKaplan’sMCAT528bringsthebestofKaplan’sclassroomexperiencetoyou—rightinyourhome,atyourconvenience.ThisbookoffersthesameKaplanstrategiesandpracticethatmakeKaplanthe#1choice forMCATprep.After all, twice asmanydoctorspreparedwithKaplan for theMCAT thanwithanyothercourse.

ThisbookisdesignedtohelpyouapproachthemostchallengingtopicscoveredontheMCATinastrategicway.ItrepresentsjustoneofthepracticeresourcesavailabletoyoufromKaplan.Additionalresources are available in your Online Center, including more practice questions, video sciencereview,andfull-lengthpracticeexams.RegisterforyourOnlineCenteratkaptest.com/booksonline.

NomatterhowconfidentyouareinthecontentfortheMCAT,pleaseunderstandthatcontentreview—nomatter how thorough—isnot sufficient preparation for theMCAT!TheMCAT tests not onlyyour scienceknowledge, but alsoyour critical reading, reasoning, andproblem-solving skills.DonotassumethatsimplymemorizingthecontentwillearnyouhighscoresonTestDay;tomaximizeyourscore,youmustalsoimproveyourreadingandtest-takingskillsthroughMCAT-stylequestionsandpracticetests.

That is precisely what this book strives to offer: challenging, MCAT-style worked examples andpracticetargetingthemosthigh-yieldanddifficulttopicsontheexam.

STRATEGICOVERVIEWSThisbooksimplifiestheskillstestedbytheAAMCandKaplan’sMethodsforapplyingthoseskillsinthedifferentcontentareasbyansweringthreemainquestions:

1)Whataretheskills/strategies/contentyouneedtoknow?2)Howaretheskills/strategies/contentpresentedontheexam?3)Howcanyougettheedgeinthatparticularskill/strategy/contentarea?

MCATPracticeInthisbook,MCATPracticeisprovidedintwoforms:

1)Thefirsttypeofpracticeisintheformofworkedexamples.ThesepassagesandquestionsetsweredesignedforthestudentstobeabletotryontheirownAND/ORseehowanexpertwouldworkthroughthemusingKaplan’smethods,therebygivingyouthetoolsyouneedtosucceedonTestDay.2)Thesecondtypeofpracticeisintheformofpracticepassages/sections(inthebookandonline)thathelpyouapplythestrategiesandtipsthatweredemonstratedintheworkedexamplessothatyougainmasteryandcanusethemseamlesslyonTestDay.

SIDEBARSThefollowingisalegendofthefourtypesofsidebarsyou’llfindinKaplan’sMCAT528:

Page 9: Edited By Deeangelee Pooran-Kublall, MD/MPH · 9.5 Preparing for the MCAT: Biochemistry in the Chemical and Physical Foundations of Biological Systems Section 9.6 Preparing for the

•KeyConcept:Thesesidebarsdrawattentiontothemostimportanttakeawaysinagiventopic,andsometimesoffersynopsesoroverviewsofcomplexinformation.Ifyouunderstandnothingelse,makesureyougettheKeyConceptsforanygivensubject.•MCATExpertise:ThesesidebarspointouthowinformationmaybetestedontheMCAT,orofferkeystrategypointsandtest-takingtipsthatyoushouldapplyonTestDay.•ThingstoWatchOutFor:Thesesidebarswillwarnyouofcommontrapsstudentsfallintowhenansweringaspecificquestiontype.•Takeaways:Thesesidebarswillhelpyouunderstandthepointofaquestionsoyouwon’tjustwalkawayfromitboggeddownwithdetails.

Tothisend,thisisyourbook,sowriteinthemargins,drawdiagrams,highlightthekeypoints—dowhateverisnecessarytohelpyougetthathigherscore.Welookforwardtoworkingwithyouasyouachieveyourdreamsandbecomethedoctoryoudeservetobe!

Page 10: Edited By Deeangelee Pooran-Kublall, MD/MPH · 9.5 Preparing for the MCAT: Biochemistry in the Chemical and Physical Foundations of Biological Systems Section 9.6 Preparing for the

ContentsTheKaplanMCAT528Team

AbouttheMCAT

HowThisBookWasCreated

UsingThisBook

RelatedMCATTitles

UNITI:MCATBASICSANDTESTSTRATEGY

CHAPTER1:AbouttheMCAT

1.1TheFourSectionsoftheMCAT

1.2FiveCommonMisconceptionsabouttheMCAT

1.3WhattheMCATWillLookLike

CHAPTER2:ReadingtheKaplanWay

2.1HowtoReadStrategicallyUsingKeywords

2.2HowtoCriticallyAnalyzePassages

2.3HowtoAttackDifferentPassageTypes

CHAPTER3:Kaplan’sQuestionandAnswerStrategy

3.1KaplanMethodforQuestions

3.2WrongAnswerPathologies

3.3SignsofaHealthyAnswer

3.4GettingtheEdgeUsingtheQuestionStrategy

3.5ConceptandStrategySummary

UNITII:STRATEGICINQUIRYANDREASONINGSKILLS

CHAPTER4:Skills1and2

4.1WhatAreSkills1and2?

4.2HowWillSkills1and2BeTested?

4.3GettingtheEdgewithSkills1and2

CHAPTER5:Skill3

Page 11: Edited By Deeangelee Pooran-Kublall, MD/MPH · 9.5 Preparing for the MCAT: Biochemistry in the Chemical and Physical Foundations of Biological Systems Section 9.6 Preparing for the

5.1WhatIsSkill3?

5.2FundamentalConceptsofSkill3

5.3HowWillSkill3BeTested?

5.4GettingtheEdgeinSkill3Questions

CHAPTER6:Skill4

6.1WhatIsSkill4?

6.2FundamentalConceptsofSkill4

6.3HowWillSkill4BeTested?

6.4GettingtheEdgeinSkill4

UNITIII:SCIENCESUBJECTREVIEW

CHAPTER7:ScienceUnitOverview

7.1TopicOverview

7.2PassagesasWorkedExamples

7.3PassagesasPracticeProblems

CHAPTER8:BehavioralSciences

8.1ReadingthePassage

8.2AnsweringtheQuestions

8.3GettingtheEdgeinBehavioralSciences

8.4Step-by-StepGuidetoBehavioralSciencesPassages

8.5PreparingfortheMCAT:Psychology

8.6PreparingfortheMCAT:Sociology

8.7BehavioralSciencesWorkedExamples

BehavioralSciencesPassageIExplanation

BehavioralSciencesPassageIIExplanation

8.8BehavioralSciencesPractice

CHAPTER9:Biochemistry

9.1ReadingthePassage

9.2AnsweringtheQuestions

9.3GettingtheEdgeinBiochemistry

9.4Step-By-StepGuidetoBiochemistryPassages

Page 12: Edited By Deeangelee Pooran-Kublall, MD/MPH · 9.5 Preparing for the MCAT: Biochemistry in the Chemical and Physical Foundations of Biological Systems Section 9.6 Preparing for the

9.5PreparingfortheMCAT:BiochemistryintheChemicalandPhysicalFoundationsofBiologicalSystemsSection

9.6PreparingfortheMCAT:BiochemistryintheBiologicalandBiochemicalFoundationsofLivingSystemsSection

9.7BiochemistryWorkedExamples

BiochemistryPassageIExplanation

BiochemistryPassageIIExplanation

9.8BiochemistryPractice

CHAPTER10:Biology

10.1ReadingthePassage

10.2AnsweringtheQuestions

10.3GettingtheEdgeinBiology

10.4Step-By-StepGuidetoBiologyPassages

10.5PreparingfortheMCAT:Biology

10.6BiologyWorkedExamples

BiologyPassageIExplanation

BiologyPassageIIExplanation

10.7BiologyPractice

CHAPTER11:GeneralChemistry

11.1ReadingthePassage

11.2AnsweringtheQuestions

11.3GettingtheEdgeinGeneralChemistry

11.4Step-By-StepGuidetoGeneralChemistryPassages

11.5PreparingfortheMCAT:GeneralChemistry

11.6GeneralChemistryWorkedExamples

GeneralChemistryPassageIExplanation

GeneralChemistryPassageIIExplanation

11.7GeneralChemistryPractice

CHAPTER12:OrganicChemistry

12.1ReadingthePassage

12.2AnsweringtheQuestions

12.3GettingtheEdgeinOrganicChemistry

Page 13: Edited By Deeangelee Pooran-Kublall, MD/MPH · 9.5 Preparing for the MCAT: Biochemistry in the Chemical and Physical Foundations of Biological Systems Section 9.6 Preparing for the

12.4Step-By-StepGuidetoOrganicChemistryPassages

12.5PreparingfortheMCAT:OrganicChemistry

12.6OrganicChemistryWorkedExamples

OrganicChemistryPassageIExplanation

OrganicChemistryPassageIIExplanation

12.7OrganicChemistryPractice

CHAPTER13:Physics

13.1ReadingthePassage

13.2AnsweringtheQuestions

13.3GettingtheEdgeinPhysics

13.4Step-By-StepGuidetothePhysicsPassageandQuestions

13.5PreparingfortheMCAT:Physics

13.6PhysicsWorkedExamples

PhysicsPassageIExplanation

PhysicsPassageIIExplanation

13.7PhysicsPractice

UNITIV:CRITICALANALYSISANDREASONINGSKILLS

CHAPTER14:ArgumentsandFormalLogic

14.1WhatIsanArgument?

14.2WhatAretheElementsofArguments?

14.3HowWillArgumentsBeTested?

14.4WhatIsFormalLogic?

14.5WhatAretheElementsofFormalLogic?

14.6HowWillFormalLogicBeTested?

14.7GettingtheEdgeUsingArgumentsandFormalLogic

CHAPTER15:CARSQuestionTypes

15.1WhatKindsofQuestionsWillYouBeAsked?

15.2FoundationofComprehensionQuestions

15.3ReasoningWithintheTextQuestions

15.4ReasoningBeyondtheTextQuestions

Page 14: Edited By Deeangelee Pooran-Kublall, MD/MPH · 9.5 Preparing for the MCAT: Biochemistry in the Chemical and Physical Foundations of Biological Systems Section 9.6 Preparing for the

15.5HowWillCARSQuestionTypesAppearontheExam?

15.6GettingtheEdgeUsingCARSQuestionTypes

CHAPTER16:ReasoningWithinPassages

16.1CARSWorkedExampleI:APhilosophyPassage

16.2CARSWorkedExampleII:ASocialSciencePassage

16.3CARSPracticePassageI

16.4CARSPracticePassageII

Page 15: Edited By Deeangelee Pooran-Kublall, MD/MPH · 9.5 Preparing for the MCAT: Biochemistry in the Chemical and Physical Foundations of Biological Systems Section 9.6 Preparing for the

RelatedMCATTitlesAvailableinPrintandDigitalEditions

KaplanMCATBehavioralSciencesReview*KaplanMCATBiologyReview

KaplanMCATBiochemistryReview*KaplanMCATGeneralChemistryReviewKaplanMCATOrganicChemistryReviewKaplanMCATPhysicsandMathReview

KaplanMCATCriticalAnalysisandReasoningSkillsReview*

*includesonlinepractice

AvailableinPrintwithBonusAppIncludedKaplanMCATFlashcards+App

AvailableinPrintGettingIntoMedicalSchool

Page 16: Edited By Deeangelee Pooran-Kublall, MD/MPH · 9.5 Preparing for the MCAT: Biochemistry in the Chemical and Physical Foundations of Biological Systems Section 9.6 Preparing for the

I

MCATBasicsandTestStrategy

Page 17: Edited By Deeangelee Pooran-Kublall, MD/MPH · 9.5 Preparing for the MCAT: Biochemistry in the Chemical and Physical Foundations of Biological Systems Section 9.6 Preparing for the

CHAPTERONE

AbouttheMCATAtthebeginningofthisbook,yousawanoutlineoftheMCATandapercentageofthegeneraltopicstested. In this chapter, we will take a closer look at the overall meaning behind those statistics.Additionally, units two through four of this book will breakdown the content areas and skillsspecificallytestedontheexam.

1.1TheFourSectionsoftheMCATThreeofthefoursectionsontheMCATtestyourbasicsciencecontentknowledgebyrequiringyoutocriticallyusetheinformationratherthanjustprovideindividualscientificfacts.Therefore,studentsshouldknowhowtointegrateandanalyzeinformationindifferentcontextsusingvariousskillsandcontentdatabases.

Thelastsection,CriticalAnalysisandReasoningSkills,isauniquepartoftheexaminthatitisapuretestofcriticalthinking.Passagesontopicswithinthesocialsciencesandhumanitiesarepresentedandthen a series of questions asks you to reason about the material presented–just as you would beexpectedtodoinmedicalschoolandinyourmedicalcareers.

BIOLOGICALANDBIOCHEMICALFOUNDATIONSOFLIVINGSYSTEMSInthissectionoftheMCAT,youwillhavetodemonstrateanunderstandingofthebasicprocessthatfoster life, such as growing, reproducing, acquiring energy, etc.Equally important in the study ofmedicine is your knowledge of how cells and organ systems within an organism act bothindependentlyandinconcerttoaccomplishtheseprocesses.

CHEMICALANDPHYSICALFOUNDATIONSOFBIOLOGICALSYSTEMSInthissection,youwillberequiredtocombineyourknowledgeofthebasicphysicalscienceswiththat of the biological sciences. Therefore, an understanding of the basic chemical and physicalprinciples that underlie themechanismsoperating in the humanbody, andyour ability to apply anunderstandingofthesegeneralprinciplestolivingsystems,willbeessential.

PSYCHOLOGICAL,SOCIAL,ANDBIOLOGICALFOUNDATIONSOFBEHAVIORThissectionisanessentialadditiontotheMCATsinceitassessesyourabilitytoimplementresearchandstatisticalprincipleswithintherealmofbehavioralandsocioculturaldeterminantsofhealthandhealthoutcomes.Basically,youarerequiredtointegratepsychological,sociological,andbiologicalbasesofbehaviorsandrelationships.

Page 18: Edited By Deeangelee Pooran-Kublall, MD/MPH · 9.5 Preparing for the MCAT: Biochemistry in the Chemical and Physical Foundations of Biological Systems Section 9.6 Preparing for the

CRITICALANALYSISANDREASONINGSKILLSThis unique section asks you to analyze scenarios rooted in the social sciences and humanitiesdisciplines.Itisimportanttonotethatunliketheothersections,specificknowledgeisnotrequiredforthis section, as all of the information is presented in thepassages.Someof the subject areas fromwhichcontentisdrawnincludeethicsandphilosophy,culturalstudies,andpopulationhealth.

Page 19: Edited By Deeangelee Pooran-Kublall, MD/MPH · 9.5 Preparing for the MCAT: Biochemistry in the Chemical and Physical Foundations of Biological Systems Section 9.6 Preparing for the

1.2FiveCommonMisconceptionsabouttheMCAT#1TheMCATisaContentTest,SummingUptheCoursesITookinUndergradYes, theMCAT does contain a lot of content—two semesters each of physics, general chemistry,organic chemistry, and biologyplus, a semester each of biochemistry, psychology, and sociology.Butwhile you do need to know about theDoppler effect, theHenderson–Hasselbalch equation forbuffers, acyl substitution reactions, and the hormones that govern themenstrual cycle (sometimescalledtheHPO,orhypothalamic-pituitary-ovarianaxis),contentaloneisnotsufficientforexcellentMCATperformance.

Rather,criticalthinking—theabilitytoreason,tointegrate,tolookataprobleminacreativewayandfindefficientmethodstosolveit—istheprimarydriverofahighscore.

Why is this? Well, schools can get a sense of your content knowledge by looking at yourundergraduateorpost-baccalaureategrades.Butthethinkingprocessandabilitytousethesesciencesisnot testedevenlyacrossschools; thus, theMCATactsasagreatequalizer, testingyourability tothink—andnotjustmemorize.Andperhapsmostimportantly,criticalthinkingunderliesyourabilitytosucceedasaphysician.

Considerthepatientcomingintotheemergencydepartmentwithacuteabdominalpainoffourhours’duration.Sure,youcouldmemorizeallofthepossiblediagnoses,work-ups,andtreatmentsforeverycondition that causes abdominal pain … or could you? The differential diagnosis (list of likelycauses)isextensive;butconsideringtheageofthepatient,thepatient’sgender,comorbidities(otherillnessesheorshehas),andthedescriptionofthepain,youcanreasonwhatquestionswouldbebesttoasktodecideonthediagnosis.

#2TheMCATlikestoTesttheExceptions,theUnusualExamples,theEsotericContentThis is a commonmisconception about theMCAT,which leadsmany premedical students to takeadditional coursework that is not necessary for success on Test Day. While advanced organicsynthesis, anatomy and physiology, and modern physics can show up in an MCAT passage, theoutside knowledge required by the AAMC still adheres to the eight-semester sequence previouslymentioned.

It’s certainly not a bad idea to takemore advanced science courses if your schedule permits—anunderstandingofanatomyandphysiologybeforeyougettomedicalschoolwillundoubtedlymakecadaverdissectionabiteasier—butrecognizethatthesecoursesshouldnotbetakenspecificallyfortheMCAT.Alltheinformationnecessarytoanswerthequestionswillbeinthepassages,orinoutsideknowledgeaslistedbytheAAMC’scontentoutlines.

#3PassagesareIncludedontheMCATtoSlowMeDownStudents sometimes assume that passages are included as background information for thoseunfamiliar with the content covered in a given set of questions. Therefore, they misinterpret thepassagesasmerelyintroducingatimecrunch,ratherthanbeingacriticalpartofthetest.

Page 20: Edited By Deeangelee Pooran-Kublall, MD/MPH · 9.5 Preparing for the MCAT: Biochemistry in the Chemical and Physical Foundations of Biological Systems Section 9.6 Preparing for the

Thechangetopassage-basedquestionsin1992camefromafarmoresophisticateddrivethantiming:theyrequireyou to integratenewinformationwith thecorpusofknowledgeyoualreadyhave,andseehowtheyjive together.MCATpassageswill frequentlychallengecommonassumptionsaboutagivenscientificprocess,orintroduceanexperimenttestingthevalidityofascientificidea.Onlybyreading the passage and actually seeingwhat happens can you be prepared for the accompanyingquestions.

Medicine is a field requiring continuous learning. Our advancements in technology belie ouradvancements in understanding the human body. Much like how you will have to integrate newinformationwithwhatyoualreadyknowwhilereadingMCATpassages,youwillhavetostayabreastoftheneweststudiesinmedicinethroughacademicjournals,conferences,andtrainings.Admissionscommittees (and your future patients!) are very interested in your ability to adjust to new data, tomanipulateit,andtoabsorbitintoyourschemataofhowtheworldworks.

#4I’llNeverUsethisInformationAgain—EspeciallyasaDoctor!The concepts and critical thinking that underlie theMCAT are both important to decisions you’llmakeasadoctor.We’vediscussedthecriticalthinking,butwhyaretheseconceptsimportant?There’sprobablynobetterwaytoproveitthanwithafewexamples.

Whenapatientbreaksabone,thetranslationalforcesandtorquesstillactingonthebonecanbeusedtopredictwhatstructuresmightbedamagedifthefractureisangulatedordisplaced(movedfromitsstarting position). We also must understand these forces and torques if we are to reset the bonecorrectly.

Acid and base chemistry dictates the blood disturbances we see in everything from chronicobstructivepulmonarydisease (COPD), toaltitudesickness, toacutekidney failure.Wefurtherusetheprinciplesofacid–basechemistryandthesemipermeablemembranetoincreasetheexcretionoftoxins;apatientwithanoverdoseofaspirin(acetylsalicylicacid)canexcretemoreofthetoxinwhenit isdeprotonated, since it takes on a negative charge and thus cannot cross the cellmembrane toreenter thebodyfromtherenal tubules.Urinaryalkalization(whentitratedcorrectly)canthereforehelpavoidatoxicoverdose.

ThecontinuityequationandBernoulliprincipleexplainthepathophysiologyofanumberofvalvularandvascular disorders in thebody. In fact, oneof thediagnostic findings in valvular stenosis (thenarrowing of a heart valve) is an increased velocity of blood flow. Physicians know from thecontinuity equation that as cross-sectional area decreases, velocity increases (assuming a constantflowrate/cardiacoutput).

Isomerismisacriticalconsiderationindrugdesign.Considertheproton-pumpinhibitoromeprazole(used for gastroesophageal reflux disease, peptic ulcers, and other acid-excess states). When thismedicationwasgoingtocomeoffpatent,anewdrugwasdeveloped:esomeprazole.Takealookatthe names there.Omeprazole is a racemicmixture; esomeprazole is only theS-enantiomer of thesamedrug.Yetthereceptorhereisachiral!Thus,forahugedifferenceincost,thepatientseesverylittledifferencewhentakingonedrugversustheother.Yetapatientwillbethankfulwhenthetherapyyouprescribedoesn’tbreakthebank!

Page 21: Edited By Deeangelee Pooran-Kublall, MD/MPH · 9.5 Preparing for the MCAT: Biochemistry in the Chemical and Physical Foundations of Biological Systems Section 9.6 Preparing for the

Therearehundredsofadditionalexamples.But,tobeclear,drawingouttheseconnectionsbetweenscienceandmedicine,andmakingthemmoreexplicit,isacriticalcomponentoftheMCAT.

#5TheMCATisnotParticularlyPredictiveofMySuccessinMedicalSchoolWhile itmayhavebeenabitharder todrawacorrelationbetweenyourSATscoreandsuccess inundergrad, the MCAT has been demonstrated multiple times to be highly predictive of first- andsecond-year grades in medical school and success on the United States Medical LicensingExamination,Step1(USMLE,orthe“Boards”).AlandmarkstudybyEllenJulianin2005foundthattheMCATwas59percentcorrelatedwithfirst-andsecond-yeargrades,46percentcorrelatedwithclerkship (third-year) grades, and 70 percent correlatedwith Step 1 scores. Thiswas significantlyhigher than undergraduateGPA alone, at 54 percent, 36 percent, and 49 percent, respectively. Thebrieftakeaway:dominatingtheMCATbodeswellforyoursuccessinmedicalschool.

Page 22: Edited By Deeangelee Pooran-Kublall, MD/MPH · 9.5 Preparing for the MCAT: Biochemistry in the Chemical and Physical Foundations of Biological Systems Section 9.6 Preparing for the

1.3WhattheMCATWillLookLikePASSAGESPassageson theMCATarewritten to testscienceconcepts in thecontextof livingsystems. Inotherwords,youwillnot seeapassagedescribinga rollercoastercardescendinga trackatanangleθ,withagivenheighthandcoefficientofkineticfrictionμkthatisaccompaniedbyquestionsaskingforplug-and-chug application of these principles. Rather, solution chemistry could be tested as anunderlyingthemeinourunderstandingofurolithiasis(theformationofkidneyandbladderstones);organic oxidation and reduction mechanisms as a component of the metabolism of toxins likeethanol;andatomicabsorptionandemissionspectrometryasitrelatestobioluminescence.

The recommendations, as made by the AAMC, include two semesters each of physics, generalchemistry, organic chemistry, and biology; one semester each of psychology, sociology, andbiochemistry;andanunderstandingofstatisticsandresearchdesign.Notethat,whileitisnotgivenitsown section, biochemistry will make up a full 25 percent of the Biological and BiochemicalFoundationsofLivingSystemsandoftheChemicalandPhysicalFoundationsofBiologicalSystemssectionsoftheexam.

QUESTIONSANDSKILLSAfull-lengthMCAT2015willcontainquestionsdividedintofourScientificInquiryandReasoningSkills(SIRS).WhiletheseskillsarefurtherexplainedinUnitII,itisworthwhiletonoteherethattherewill be agreatly increasednumberofquestions focusingon researchdesignandbias (Skill 3), aswell as data interpretation and statistical analysis (Skill 4). These previously made a minimalappearanceontheMCAT,butwillnowconstituteasignificantproportionofthequestions—perhapsabout20percent,combinedbetweenthetwoskills.

Page 23: Edited By Deeangelee Pooran-Kublall, MD/MPH · 9.5 Preparing for the MCAT: Biochemistry in the Chemical and Physical Foundations of Biological Systems Section 9.6 Preparing for the

ContentUpdatesBIOCHEMISTRYInanAAMCsurveyofmedicalschoolfaculty,biochemistrywasratedthemostimportantscienceforstudentstomasterforthemedicalschoolcurriculaofthefuture(averagescore3.34onafive-pointLikert scale,with5being thehighest).Sixof the top-ten rated topics in thesurveybelonged to therelatedfieldofcellandmolecularbiology.Soit’sclearthatbiochemistryisconsideredimportantforthemedicalstudentofthefuture.

PSYCHOLOGYANDSOCIOLOGYGiventheexpandingdiversityinAmericansociety,ourinterconnectednessduringthedigitalage,andtheagingofthepatientpopulation,therehasbeenanincreasingfocusinmedicalschoolsonculturalsensitivity.Further,manyofthetopcausesofmorbidityandmortalityintheUnitedStatesarecausedbybehavioralandenvironmentaldeterminantsofhealth:smokinganddruguse,dietandexercise,andinequitiesincareduetosocioeconomicstatus.

ThreemainthemeswereidentifiedforwhythismaterialshouldbeincludedintheMCAT.First,thediversetheoreticalframeworksusedinthebehavioralandsocialsciencesunderscoretheimportanceof thinking through “complex (and often chaotic) systems”—like the biopsychosocial model—tounderstandthepatient.Second,thestrongconnectionstoresearchmethodsanddataanalysisinthesefieldsalignwellwith the testmaker ’sgoalof increasingquestionson these topicsand theneed formedical students to design and critically analyze research, as part of evidence-based medicine.Finally,thecontentofpsychologyandsociologyisawelcomeadditiontoamedicalstudent’sfundofknowledge.

BIOLOGICALLYBASEDPASSAGESHowoftenhaveyouwondered toyourself—whilecrammingfor thatorganicchemistryorphysicsfinal—“WhydoIneedtoknowthisasadoctor?”ManypremedicalstudentsquestiontherelevanceofsomeofthematerialontheMCAT.

Thepresentationof thiscontent ischangingasananswer to thisquestionofrelevance.Rather thantestingthermodynamicsthroughagas-pistonsystem,whichfailstodemonstratewhyadoctorwouldactuallyneedtounderstandtheseprinciples,whynotpresentitinapassageonthepropertreatmentoffrostbite(slowrewarmingthroughaconvectioncurrentinarotatingwaterbathat40–42°C)?

Some schools are better than others at establishing these connections for students; integrated andclinicallybasedcoursesareextremelyhelpfulwiththisgoal.Butbymakingthisapplicationofhardscienceinabiologicalcontextatoppriority,MCAT2015canincreasethisexposureamongstudentsevenbeforetheyarrivefortheirwhite-coatceremony.

ALONG(ANDMOREPOWERED)EXAMThegreatlyincreasedlengthofMCATactuallyreflectsitsuseinadmissionsdecisions.Historically,the total score was the most important for admissions committees; section subscores in Physical

Page 24: Edited By Deeangelee Pooran-Kublall, MD/MPH · 9.5 Preparing for the MCAT: Biochemistry in the Chemical and Physical Foundations of Biological Systems Section 9.6 Preparing for the

Sciences,VerbalReasoning,andBiologicalSciencesmerelyshowedthebreakdowninthisscoresoschoolscouldpickuponstudentswhowerehighlylateralizedtowardonesection.

Thus,theincreasednumberofquestionsonthenewMCATrepresentsanumberlargeenoughtogivereliable,validdataforbothsectionscoresandanoverallscore—whilestillbeingmanageableforatesttakeninonlyoneday.

Page 25: Edited By Deeangelee Pooran-Kublall, MD/MPH · 9.5 Preparing for the MCAT: Biochemistry in the Chemical and Physical Foundations of Biological Systems Section 9.6 Preparing for the

CHAPTERTWO

ReadingtheKaplanWayMCATreadingisunlikeanyotherreadingthatyouhavedonein thepast. In thesamewaythatonereadsanoveldifferentlythanatextbook,MCATreadingrequiresitsownuniqueapproach.ReadingtheKaplanwayinvolvesnotingkeywords,properpassageoutlining,andanticipatingquestionswhilereading.Inpassages,keywordsarevitalcluesthatpointouttherelationshipsbetweenmajorthemes,highlighttheauthor ’sopinion,andelucidatethereasoninginatext.Passageoutliningisessentiallydoing thewriting process in reverse.Youwill learn how to extract an outline from each passage,whichwill serve as roadmap to assist in handling detail-oriented questions aswell as focus yourreading on the most testable passage content. And finally, anticipating questions is all aboutunderstandingpassagestructureandhowthisispredictiveofquestiontypes.

2.1HowtoReadStrategicallyUsingKeywordsTheMCATsectionsarepackedwithdenseacademicprose.Thereareseveraldistinctlevelsinwhichthetextshouldbeevaluated:content,organization,perspective,andreasoning.AddressingallfourmodesofreadingisessentialforTestDaysuccess.

ReadforContent–extracttheinformationfromthetext,discoveringpreciselywhatisbeingsaid.

Read for Organization–consider how the different ideas presented in the passage relate to oneanother.Iftheinformationalcontentisthewhatofthetext,thentheorganizationisthehow.

Read for Perspective–pay attention to the different perspectives contained in the passage. Manyauthors of passages, especially in the CARS section, do not state their intentions overtly. In thesecases,itiskeytoattendtotherhetoricalaspectsofthetext,especiallygoal,tone,andvoice.

ReadforReasoning–examinethestructureofthereasoningpresentedinapassage.Inotherwords,itisimportanttodeterminehowtheauthorstructureshisorherargument.

KeyConceptsAnypassagecanbeunderstoodinfourdifferentways,whichwecallthemodesofreading.Eachmodeanswersatleastonevitalquestion:•Content–Whatdoesthetextsay?•Organization–Howdosentencesconnect?Howdoideasrelate?•Perspective–Whydoestheauthorwrite?Howdoestheauthorfeel?Whoelsehasavoice?•Reasoning–Howareclaimssupported?Howareclaimschallenged?

RELATIONKEYWORDSWhentacklingapassage,itisessentialtobeabletounderstandhowwhatyou’rereadingnowfitsintothe whole. While there are many ways in which ideas might be related to one another, the vast

Page 26: Edited By Deeangelee Pooran-Kublall, MD/MPH · 9.5 Preparing for the MCAT: Biochemistry in the Chemical and Physical Foundations of Biological Systems Section 9.6 Preparing for the

majorityofRelationkeywordswillfallintooneoftwosubcategories:SimilarityorDifference.

Similaritykeywordsindicatethatmoreofthesameideaiscominginthetext.

Differencekeywordssignifyachangeintheauthor ’sfocus,oradirectcontrastbetweentwothings.

MoreComplexRelationships•Oppositionkeywordsindicateanoutrightconflictbetweenideas.•Sequencekeywordssuggestaseriesofeventsadvancingintime.•Comparisonkeywordsareusedtoevaluateideasandrankthemrelativetoothers.

Table2.1 lists examples ofRelationkeywords in each category.Note that somewords can fit intomore than one category; for example, not reveals a difference, and can also create a directopposition.

Similarity Difference Oppositionand but not/never/nonealso yet either...ormoreover however asopposedtofurthermore although onthecontrarylike (even)though versussame/similar rather(than) ononehand...ontheotherhandthatis incontrast otherwise

inotherwords ontheotherhand Sequenceforexample otherwise before/aftertakethecaseof nevertheless earlier/laterforinstance whereas previous/nextincluding while initially/subsequently/finallysuchas different first/second/third/lastinaddition unlike historically/traditionally/usedtoplus notwithstanding now/currently/modern

atthesametime another Comparison

aswellas instead better/bestequally still worse/worstthis/that/these/those despite less/least:[colon] alternatively more/most;[semicolon] unless –er/–est

Page 27: Edited By Deeangelee Pooran-Kublall, MD/MPH · 9.5 Preparing for the MCAT: Biochemistry in the Chemical and Physical Foundations of Biological Systems Section 9.6 Preparing for the

—[emdash] not primarily()[parentheses] conversely especially““[quotes] contrarily aboveall

Table2.1CommonRelationkeywords

AUTHORKEYWORDSAuthorkeywordsareverbs,nouns,adjectives,andadverbsthathintattheauthor ’sopinions.Authorkeywordshaveaconnotationofeitherapprovalordisapproval.Authorsmayusecharacteristicwordsandshortphrasestomaketheirclaimsmoreextreme,aswellasothersthatmoderatetheirclaims.

MCATExpertiseMostpassagesontheMCATcontainstrong,butnotextreme,opinions.Rarelywillanauthorbecompletelyneutral,becausethereislittlereasoningtotestiftheauthordoesnotexpressatleastamoderatelypositiveornegativeopinion.

Positivevs.Negative•Positive keywords–include nouns such asmasterpiece, genius, and triumph; verbs such asexcel,succeed, andknow; adjectives such as compelling, impressive, and elegant; and adverbssuchascorrectly,reasonably,andfortunately•Negativekeywords–includenounslikedisaster,farce,andlimitation;verbslikemiss,fail,andconfuse; adjectives like problematic, so-called, and deceptive; and adverbs like questionably,merely,andpurportedly

Notethatinadditiontopositive,negative,orneutral,anauthorcanalsobeambivalent.Ambivalenceliterallymeans“feelingbothways,”anditisdifferentfromimpartiality.

KeyConceptWhiletheyarebothoverallneutral,thesetwoattitudesareverydifferent:•Ambivalent=havingbothapositiveandnegativeopinion.•Impartial=havingneitherapositivenornegativeopinion.

EXTREMEExtremekeywordsarewordsthatenhancethechargeofwhattheauthorissaying,forcingtheauthorintooneortheotherextreme.

MODERATINGModeratingkeywordsarewords thatset limits toclaims inorder tomake themeasier tosupport,

Page 28: Edited By Deeangelee Pooran-Kublall, MD/MPH · 9.5 Preparing for the MCAT: Biochemistry in the Chemical and Physical Foundations of Biological Systems Section 9.6 Preparing for the

becauseastrongerstatementisalwaysmoredifficulttoprovethanaweakerone.

Table2.2listsexamplesofAuthorkeywordsineachcategory.

Positive

Negative

Extreme

Moderatingmasterpiece disaster must can/couldgenius farce need/necessary may/mighttriumph limitation always possiblyexcel miss every probablysucceed fail any sometimesknow confuse only onoccasioncompelling problematic should/ought oftenimpressive so-called indeed tendstoelegant deceptive very herecorrectly questionably especially nowreasonably merely obviously inthiscasefortunately purportedly aboveall insomesense

Table2.2CommonAuthorkeywords

LOGICKEYWORDSThefinallevelofreading(forreasoning)isperhapsthemostdifficult,becausethespecialone-wayrelationshipbetweenaconclusionanditsevidenceisamongthemostcomplexyou’llencounteronTestDay.Logickeywordstendtoberelativelyrare,occurringlessfrequentlythaneitherRelationorAuthorkeywordsinmostpassages.

EvidenceandConclusionAconclusionisaclaimthattheauthoristryingtoconvincetheaudiencetobelieve,whilepiecesofevidencearethereasonsthataregivenforbelievingit.

RefutationRefutation keywords are the opposite of evidence–countervailing reasons for rejecting aconclusion.

Table2.3listsexamplesofLogickeywordsineachcategory.

Page 29: Edited By Deeangelee Pooran-Kublall, MD/MPH · 9.5 Preparing for the MCAT: Biochemistry in the Chemical and Physical Foundations of Biological Systems Section 9.6 Preparing for the

Evidence

Conclusion

Refutation

because(of) therefore despitesince thus notwithstandingif then challengefor so underminedbywhy consequently objectthereasonis leadingto counter(argument)asaresultof resultingin critique/criticizedueto argue conflictasevidentin conclude doubtjustifiedby imply problemassuming infer weaknessafterall suggest calledintoquestionby

Table2.3CommonLogickeywords

Page 30: Edited By Deeangelee Pooran-Kublall, MD/MPH · 9.5 Preparing for the MCAT: Biochemistry in the Chemical and Physical Foundations of Biological Systems Section 9.6 Preparing for the

2.2HowtoCriticallyAnalyzePassagesTHEKAPLANMETHODFORPASSAGESTheKaplanMethod for critically reading passages consists of four steps:Scan,Read,Label andReflect.

• Scan–Scan for words that stand out because of capitalization, italics, quotation marks,parentheses,oranyotherdistinctivetextualfeaturestodecidewhichpassagestodofirst.Alwaysstartwithpassagesthatyou’remostcomfortablewith.•Read–Usekeywordsandthefourmodesofreadingtoreadstrategically.

•IdentifyRelationkeywords(toconnectdifferentideasinthetext),Logickeywords(to reveal thepassage’sarguments),andAuthorkeywords (toofferglimpsesof thewriter ’sintentions).•Don’trereadtextexcessively.•Evaluatetherolethateachparagraphplaysinthelargerwholeofthepassage.

•Label–Writedownthepurposeofeachparagraphofthepassage.•Labelparagraphsrightafteryoureadthem.•Usesymbols,abbreviations,andothershorthand.

•Reflect–Articulatetheauthor ’soverallgoalforwritingthepassage.•Beginthestatementwithaninfinitiveverb(theformofaverbstartingwithto).•Occasionally,authorsmayhavemultiplepurposes(usemultipleverbs).

There are several important differences between CARS passages and the science passages. CARSpassages are longer andmuchmore variable, both in their vast range of topics and their endlessdiversity of writing styles. They are usually written by authors who take sides and express theiropinions,althoughnotalwaysinastraightforwardmanner.

REVERSE-ENGINEERINGTHEAUTHOR’SOUTLINEInthissection,wediscusshowtooptimizeyourscratchpaperusage,whattoincludeinyouroutline,andwhichportionsofthepassageareworthhighlightingonthescreen.

ScratchPaperStrategyDuringtheScanstep,begintoconstructyouroutline.EachparagraphshouldbenumberedusingbriefnotationlikeP4.Besuretoincludeaspotattheendfortheauthor ’sgoal,whichwillalwaysstartwithan infinitiveverb,soyoucanwrite theword toasa reminder.For instance,youcouldsetupyourscratchpaperforasix-paragraphpassageasfollows:

P1.

P2.

P3.

P4.

P5.

Page 31: Edited By Deeangelee Pooran-Kublall, MD/MPH · 9.5 Preparing for the MCAT: Biochemistry in the Chemical and Physical Foundations of Biological Systems Section 9.6 Preparing for the

P6.

Goal:

Note that writing the goal is critical for CARS passages because most questions will require anoverallunderstandingofthepassage’sgoal.

Labeleachparagraphappropriatelyasyoureadthroughapassage.Besuretokeepslabelsconcise.Generally, five tosevenwordsare ideal,but ten to twelvewordsareacceptable formorecomplexpassages.

WhattoLabelThe four modes of reading discussed earlier in this chapter serve as guidelines for the kind ofmaterialtoincludeineachLabel.

•Content–Writedownthekeyideasofeachparagraph.• Organization–Write down how each paragraph functions within the larger whole of thepassage.•Perspectives–Noteinyourmapwheretheauthororotherprominentvoicesofferanopinion.Use‘+’or‘−’todesignatepositiveandnegativetone,respectively.•Reasoning–Identifywhethereachparagraphbolstersorobjectstoanargument.

HOWTOHIGHLIGHTHighlightingisneveranadequatesubstituteforOutlining,butitisanextremelyeffectivecomplementtotheKaplanMethodforCARSPassageswhenusedsparingly.Oneshouldneverrelyonhighlightingas the primary strategy for reading a passage because highlighting will actually disappear if youleave a passage and return to it later. Remember that highlightingworks through contrast: if youhighlightmostofthetext,theunmarkedportionswouldactuallystandoutmore.Consequently,strivetoadheretothefollowingguidelineswhenusingthisfeature.

•Double-Clickvs.Click-and-Drag•Highlightanentirewordbydouble-clickingonanypartofit.•Whenhighlightingaphrase,donotobsessoverhighlightingitcompletely.•Neverhighlightmorethanonelineoftext.

•FindtheFirstOccurrence• Highlight terms introduced by an author, especially when the author provides anovertdefinitionoroffersessentialbackgroundinformation.

•ProperNounsandNumbers•Names and proper nouns, aswell as dates and other numbers, have a tendency toshowupinquestionstems.

•Keywords•Donothighlightkeywords.

Page 32: Edited By Deeangelee Pooran-Kublall, MD/MPH · 9.5 Preparing for the MCAT: Biochemistry in the Chemical and Physical Foundations of Biological Systems Section 9.6 Preparing for the

2.3HowtoAttackDifferentPassageTypesSciencePassagesThesciencesectionsontheMCATgenerallypresentscontentintwotypesofpassages.Thefirsttypeis best described as information passages,where youwill need to pay attention to definitions andrelationships. The second, andmost common type, are experimental passages. For these passages,your job is topayattentiontowhytheexperimentwasdoneandwhat theoverall findingsare. It isimportanttonotethatanyfiguresortablesarealsoworthlabelingforthesciencepassagessincetheywillalsocontaintestableinformation.

VARIETIESOFCARSPASSAGESTheAAMCliststendifferentfieldsinthehumanitiesandadozendifferentsocialsciences,asshowninTable2.4.Ontheotherhand,mostCARSpassagesfit intooneofjustafewbasictypes.Quicklyidentifying the formof a passage during theScan step of theKaplanMethod can help shape yourexpectationsaboutwhatthepassagewillincludeandwhattheaccompanyingquestionswilltest.

MCATExpertiseAccordingtotheAAMC,approximately50percentofquestionsintheCriticalAnalysisandReasoningSkillssectionwillbefromthehumanitiesand50percentwillbefromthesocialsciences.

Humanities

SocialSciencesArchitecture AnthropologyArt ArchaeologyDance CulturalStudiesEthics EconomicsLiterature EducationMusic GeographyPhilosophy HistoryPopularCulture LinguisticsReligion PoliticalScienceTheater PopulationHealth

PsychologySociology

Table2.4HumanitiesandsocialsciencesdisciplinesintheCARSsection

Page 33: Edited By Deeangelee Pooran-Kublall, MD/MPH · 9.5 Preparing for the MCAT: Biochemistry in the Chemical and Physical Foundations of Biological Systems Section 9.6 Preparing for the

HUMANITIESPassagesinthehumanitiestendtofallintotwobroadcategories.Thefirstcategory,whichincludesmost of the passages from architecture, art, dance, literature, music, popular culture, and theater,couldbeconsideredArtspassages.The second category,Philosophical passages, includes ethics,philosophy,andmanyreligionpassages.

•Artspassagesoftenusequotationsfrombothartistsandcritics,includestrongopinions,andusedescriptivelanguagetoillustrateartisticexamples.•Philosophicalpassagestendtobeabstractandheavyonlogicaswellasfocusonconceptsandtherelationsbetweenthem.

Keep inmind that therewill be plenty of humanities passages thatmix characteristics ofArts andPhilosophicalpassages.

SOCIALSCIENCESWhenitcomestothesocialsciences,somepassagestakewhatmightbecalledaScientificform,suchas passages in anthropology, education, linguistics, population health, psychology, and sociology.The counterpart to Scientific would beHistorical passages, which include many instances fromarchaeology,culturalstudies,economics,geography,history,andpoliticalscience.

•Scientificpassagesincludeheavyreferencestoempiricalstudies.Usuallytheauthor ’sopinionismoresubtle.•Historicalpassagestendtodrawonhistoricaleventsandquotationsfromsourcesaliveatthetimeoftheeventstheydiscuss.Sometimesempiricalstudiesarereferenced.

SUPPORTINPASSAGESBecauseamajorityofquestionsintheCARSsectionwillhavesomeconnectiontologicalsupport,it’sessentialtounderstandthedifferentkindsofsupportthatcanbefoundinCARSpassages.

CategoriesofSupport• Unsupported Claims–Not every assertion in a passage is backed up with evidence.Unsupportedclaimslacklogicalconnectionstootherpartsofthepassage.•EmpiricalEvidence–Whenevertheauthorappealstoexperience,particularlyinthecontextofscientificstudies,heorsheisusingempiricalevidence.

•Historicalaccountsandcasestudiesdrawuponexperience,butare limited invaluebecausetheyonlyrepresentsinglecases.• Surveys, statistical analyses, and controlled experiments are more solid evidencesince variables can be isolated and evidence can be gathered by examining a wideswathofexperience.

•LogicalAppealsrefertotheusageoflogic,claims,orevidencetoargueforapoint.•Analogicalreasoning–Two things known to be alike are declared to be alike in adifferentrespectforwhichtheremaynotbedirectevidence.• Reduction to absurdity is supporting a position by elimination of alternativepossibilities.

•AppealingtoAuthorityisdrawinguponanotherpersonortesttosupportaclaim.•Thelevelofsupportdependsonthecredibilityoftheauthority.

Page 34: Edited By Deeangelee Pooran-Kublall, MD/MPH · 9.5 Preparing for the MCAT: Biochemistry in the Chemical and Physical Foundations of Biological Systems Section 9.6 Preparing for the

Primarysources provide themost support,whereas secondary sourcesaredubious invalueandvarybasedon theexpertiseof theauthoritybeingcited.

•Appeals to theReader–In this case, theauthoruses the reader tohelpgroundanargument.Sometimes the author will begin an argument with points that the reader is likely to agreewith.Another possibility is that the author uses charged language and colorful descriptions toevokeparticularresponsesfromthereader.•FaultySupportinvolvesbackingupacontroversialclaimwithanotherclaimthatissimilarlycontroversial.Thiskindofassertionisextremelyweakatbest.

ANTICIPATINGQUESTIONSWhile reading CARS passages, it is essential to anticipate the questions. Certain passagecharacteristics lend themselves to specific question types. The following are a few examples ofpassagetypesandwhattypesofquestionstypicallyaccompanyeach:

•Heavilyopinionatedpassageslendthemselvestoquestionsthatrequireunderstandingwhattheauthorwouldagreeordisagreewith.• Passages that are abundant in detail are likely to have questions that necessitate combingthroughthepassagesearchingforparticularbitsfromthetext.•Passageslackinginsupportwillprobablyhavequestionsthatincorporatenewinformation.• Passages thatuse numerousLogic keywords will tend to have questions that ask about theauthor ’sargumentativestructure.•Whentheauthorintroducesnewterminologyorconcepts, it is likely that thequestionswilltestforunderstandingofthenovelinformation.•Whenpassagesoffertwoopposingviewpoints,thenexpectquestionsthataskyoutocompareandcontrasttheviewpoints.

Page 35: Edited By Deeangelee Pooran-Kublall, MD/MPH · 9.5 Preparing for the MCAT: Biochemistry in the Chemical and Physical Foundations of Biological Systems Section 9.6 Preparing for the

CHAPTERTHREE

Kaplan’sQuestionandAnswerStrategyIn this chapter, we shift our strategic focus away from the passages to consider the treatment ofquestionstemsandanswerchoices.WebeginbyoutliningtheKaplanMethodforMCATQuestions.Subsequently,wewilllookattherecurringtrapsthatthetestmakerssetfortheunwarystudent,whichwecallWrongAnswerPathologies.Inthefinalportionofthechapter,we’llconsiderthecounterparttopathologies:patternscommonincorrectanswers.

3.1KaplanMethodforQuestionsInthepreviouschapter,wesawhowthegeneralKaplanMethodfortacklingMCATpassagescouldberefinedtotheneedsoftheCARSsection.Inthissection,we’lldothesamewithourquestionmethod,whichtakesthebasicformshowninFigure3.1.

Figure3.1KaplanQuestionMethod.

This same four-step approach should be used on all questions on theMCAT—in both theCriticalAnalysisandReasoningSkillssectionandthesciencesections.TheCARS-specificversionisshowninFigure3.2.

Page 36: Edited By Deeangelee Pooran-Kublall, MD/MPH · 9.5 Preparing for the MCAT: Biochemistry in the Chemical and Physical Foundations of Biological Systems Section 9.6 Preparing for the

Figure3.2DetailedStepsoftheKaplanQuestionMethod.

Itisimportanttonotethattheworked-outpracticepassagesinUnitIIIofthisbookallfocusonhowtoeffectivelyimplementthefourstepsofthisquestionstrategy.

STEPONE:ASSESSYoumightnoticethatthefirststepofthequestionmethodisinmanywayssimilartotheScanstepofthe Kaplan Method for Passages. This is not a coincidence, but is a consequence of the timingconstraintsposedbythesection.Becauseeveryquestionisworththesamenumberofpoints,there’snoreasontogetderailedbyanyoneofthem.Behonestwithyourself:atleastafewquestionsineachsectionaresodifficultthatyou’relikelytogetthemwrong,nomatterhowmanyminutesyouspendonthem.Wouldn’titbebettertorecognizewhichquestionsthosearerightaway,soyoucaninsteadusethatprecioustimewhereitwillactuallypayoff?

Towardthatend,yourfirsttaskwithanyquestionwillbetoreadthestem,andonlythestem,forthesakeofmakingthedecisiontoeitherworkonitnow,ortotriage(touseanaptmedicalmetaphor)andsaveitforlater.Assessingthedifficultywillbemadeeasierifyoucanidentifythequestiontype.Throughextensiveresearchofall releasedMCATmaterial,we’vediscoveredthatalmostallof theCARS section’s Foundations of Comprehension questions fall into one of four types, and itsReasoning Within the Text and Reasoning Beyond the Text categories can each be split into twopredominanttypeswithassortedOthersmakingrareappearances.Determiningthequestiontypewillmakedevisingandexecutingaplanmucheasier,whichiswhywedevotethenextthreechapterstothedifferenttypesofquestionsandtheiruniquestrategicconcerns.

Whyavoidlookingat theanswerchoicesintheCARSsection?Theprimaryreasonis thatmostof

Page 37: Edited By Deeangelee Pooran-Kublall, MD/MPH · 9.5 Preparing for the MCAT: Biochemistry in the Chemical and Physical Foundations of Biological Systems Section 9.6 Preparing for the

them arewrong. If you glance at just one of them, for instance, it’s three timesmore likely to beincorrectthancorrect,andcouldseriouslymisleadyouaboutthequestion.Inexperiencedtest-takersimmediately jump to the answers, and theAAMCpunishes them for it bywordingwrong optionsseductively. Selecting the first answer that looks good without really formulating a plan is just arecipeforfailure.Thus,untilyougetinthehabitofignoringtheanswersentirelyuntiltheAnswerstep,useyourhandorastickynotetocoverthemupwheneveryoustarttoworkonaquestion.

MCATExpertiseToavoidgettingseducedbywronganswerchoices,onlyscrolldownfarenoughtoreadthequestionstem,andleavetheanswerchoicesoff-screenuntilyou’rereadingfortheAnswerstep.Thiswillsaveyousignificanttimeandeffort,sothatyouavoidputtingactiveconsiderationtowards(andpossiblybeingmisledby!)thethreewronganswerchoices.

Ontheotherhand,youcanaffordaglanceattheanswerchoicesforthesciencequestions.Whileyoushouldnotreadthechoices,justlooktoseewhatformtheanswerchoiceisinsothatyouwillknowwhatformthepredictionshouldtake.

STEPTWO:PLANOnceyou’vedecidedtoattackaquestion,it’stimetoplanyourattack.First,beclearaboutwhatitisthatthequestionrequiresofyou—whatwecallthetask.Simplerquestiontypes likeMainIdeaandDefinition-in-Contextalways involveonespecific task (recognizing thebigpicture,andexplainingthemeaningofpartofthetextasusedinthepassage,respectively),andeventhemorecomplexonesonlyhaveasmallnumberofcommonobjectives,suchasApplyquestions,almostallofwhichwillfallintooneofthreetasks:gaugingtheauthor ’sresponse,predictingalikelyoutcome,orfindingagoodexample.

In the journey toacorrectanswer, the task isyourdestination.Toreachwhereyou’reheaded,youneedtoknowwhereyou’restartingfrom,yourpointofdeparture.Thus,theothermajoraspectofthePlanstepistodeterminewhereyou’llfindtheinformationthatyouneedtoanswerthequestion.Notethat thereareonlyfourviablesourcesof informationonTestDay: thequestionstem, thepassage,youroutline,andyourmemory.

It’s rare, but occasionally you’ll find that the stem gives you everything you need to answer thequestion. In other cases, the relevant information might be fresh in your mind, although humanmemoryisnotoriouslyfaulty,anditdoesn’thurttocheckanothersource.Thisisonereasonwhyit’ssovaluabletocreateanoutlineasyouread,toserveasamemoryaid,capturingimportantaspectsofthetextthatcouldslipyourmind.Infact,sometimestheoutlineistheonlyplaceyou’llneedtocheck,savingyouthetroubleofresearchingthepassage.Forinstance,withMainIdeaquestions,theauthor ’sGoalinyouroutlineisusuallyallyou’llneedtoexamine.NoteveryquestionontheCARSsectionrequires rereading the text: if a simple plan yields immediate success, accept it and don’t second-guessyourself.

MCATExpertise

Page 38: Edited By Deeangelee Pooran-Kublall, MD/MPH · 9.5 Preparing for the MCAT: Biochemistry in the Chemical and Physical Foundations of Biological Systems Section 9.6 Preparing for the

Ifyoueverfindyourselfscanningthepassagetofindananswerchoice,you’redoingitwrong!Theinformationyouneedfortheanswerisofteninyouroutline;evenifnot,usingyouroutlinetodeterminewhereinthepassagetolookfortheanswermeansthatyou’llusuallyscan—atmost—oneparagraphoftext.

Wheneveryou’refacinglessstraightforwardtasks,though,it’sadvisabletoyourereadatleastpartofthe passage. Stems almost always contain clues hinting at where to research, taking one of threeforms:

•Paragraphorsentencereferences•Directquotations•Paraphrases

Sometimes paragraph numbers will be listed in parentheses, or you’ll see a phrase like the finalsentenceofthepassage,whichmakeslocationabreeze.Theothercluesmaybehardertoworkwith.Keep in mind that not all quotations taken from the passage will be surrounded by quote marks,particularly if only a singleword or short phrase ismentioned—these brief terms are sometimesitalicizedbutmayappearwithoutanyembellishmentatall.Keepinmindthatshorterquotationscouldactually appear in multiple paragraphs. Although mercifully rare, paraphrases of passage claimswithout additional clues can be especially tricky: always remember that stems can use synonymsinstead of the author ’s original language. A good outline will help you find the paragraph thatcontainstheinformationforalessdirectclue.

Once you’ve figured out where to go in the passage, you’ll want to make sure you cast a largeenough net to findwhat you need.While the necessary information is often found directly in theportionofthepassagethatthestempointsto,notinfrequentlyyouwillfindthattheanswerisinthesurroundingtext,insentencesimmediatelybeforeorafter.Asageneralruleofthumb,plantostartrereadingonesentencebeforethereferenceandtostoponesentenceafterit.

What should your Plan be if you can’t locate the clues, or with vague questions that lack thementirely?Typically,suchquestionsarebesttotrylater,attheveryendofapassageset,afteryou’veresearchedtheotherquestionsandalreadyrereadmuchofthetext.Youmayfindthatbythetimeyoureturn to it, theeffortsyouput intoothersendeduprevealinganunhelpfulquestionstem’scorrectanswer.Whenyoudoattemptthesequestions,theprocessofeliminationwillusuallyendupbeingthebestPlan.

Ona similarnote, thePlan step for the sciencequestions iswhereyouwilldecidewhereyouwillhavetogotopredicttheanswer;thepassage,yourbackgroundknowledge,orboth.

STEPTHREE:EXECUTEEveryquestionhasexactlyonecorrectanswerbuttypicallywillhavemorethanonewaytoreachit.Indeed, the only difference between discovering the one correct answer and ruling out the threewrongonesisthatthelatterapproachusuallytakesmoretime;theresultisidentical.Consequently,youwanttostriveforthequickestapproach,butbeflexible,sothatifPlanAfails,youhavePlansBandCtofallbackon.

InthePlanstepformostquestions,you’llidentifywhereyouneedtoreread(inyouroutlineorinthepassageitself).KeepinmindthetaskoftheparticularquestiontypeasyounowExecutethePlanandlookoveryouroutlineorthetextagain.Here,unlikewithyourfirststrategicreadingofthepassage,

Page 39: Edited By Deeangelee Pooran-Kublall, MD/MPH · 9.5 Preparing for the MCAT: Biochemistry in the Chemical and Physical Foundations of Biological Systems Section 9.6 Preparing for the

it’sokaytorereadthecrucialsentencesafewtimesifyouneedto,becausepointsarenowontheline.Inmostcases,PlanA is touse the text topredicteverything thatyoucanabout thecorrectanswer,whilePlanBistosetanalogousexpectationsaboutwhatwouldruleoutchoices.Themorespecificthesedualexpectations,theeasieritwillbetoisolatetheanswer.

Where you look for information, aswell as theway inwhich you approach rereading (includingwhichkeywordsyoupayattentionto)willvarydependingonthetask.

STEPFOUR:ANSWERAllthatremainsnowisselectingtoselectthecorrectAnswer.Scanthechoices,andifyouseeanitemthatcloselyresembleswhatyouexpected,rereadeverywordofthatanswerchoicecarefullytomakesureitsayspreciselywhatyouthinkitdoes.Then,selectitandmoveontothenextquestion.Atthatpoint,readingtheotherchoiceswillnotbeworthyourtime—beconfident thatyou’veansweredthequestion correctly when you find a near perfect match. When a good match is not available,eliminationisalwaysanoption.

Thatsaid,don’tfeelthatyouimmediatelyneedtoresorttotheprocessofeliminationifyoureadoneanswerchoiceanddiscoverthatthecorrectanswerisgoingtotakeacompletelydifferentformthanyou first predicted. Part of being flexible is being able to revise your initial plan, to set newexpectationsiftheanswerchoicespointyouinthatdirection.Theanswerchoicescouldtechnicallybeconsideredafifthsourceofinformation,butkeepinmindthattheyincludealotofmisinformationandshouldbetreatedwithcaution.

For example, a question stem may ask for a claim that undermines the author ’s thesis, a classicStrengthen–Weaken (Within the Passage) task. For such a question, you may go into the answerchoices expecting that the answer is an objectionmentioned in the passage. As soon as you readchoice(A),though,youdiscoverit’ssomethingentirelynew,neverevenhintedatbytheauthor.SuchquestionsfallintotherelatedStrengthen–Weaken(BeyondthePassage)typeinstead.Beforemovingontoreadchoice(B),youshouldthinkaboutwhetherchoice(A)answersthequestioninawayyoudidn’tanticipate.Ifnot,useittohelpyoualteryourexpectations.Avoidthetemptationtoabandonthecritical thinking you performed during the Execute step by reading the remaining three answerchoices. Instead, formulate a new prediction and find amatch among the three remaining answerchoices.

Sometimesthequestionstemjustdoesn’tgiveyouverymuchtoworkwith,andonotheroccasionsyou’llsearch through theanswersbut findno likelymatch. In thesecases,youwillhave touse theprocessofelimination,whichmay requiremultiple returns to the textasyou researcheachchoiceindividually.IfyouwereabletosetexpectationsduringtheExecutestepforwrongchoices,however,lessadditionalresearchwillberequired.Whenevaluatingchoices,rememberthattheanswerchoiceswillsometimesvaryfromthelanguageusedinthepassage.Don’truleoutachoicejustbecausethewordingisnotquitewhatyouanticipated;conceptualconsistencymattersmorethanterminologicalexactness.Keepinmindthatanansweronlyrequiresonemajorflawforelimination,sotheWrongAnswerPathologiesdescribedbelowcangreatlyexpeditetheprocess.

Page 40: Edited By Deeangelee Pooran-Kublall, MD/MPH · 9.5 Preparing for the MCAT: Biochemistry in the Chemical and Physical Foundations of Biological Systems Section 9.6 Preparing for the

MCATExpertiseIfyoureadaquestionstemanditdoesn’tgiveyouverymuchtoworkwith,don’tjustsayIdon’tknowandjumpstraighttotheanswerchoices.Useyouroutlinetorememberthemainthemesofthepassage,andthenusethosethemestohelpwiththeprocessofelimination.Thiswillhelpyouavoidbeingdistractedbyanswerchoicesthatareseductivebutwhichdonotfitintothepassage.

Whenallelsefails,youcanfallbackoneducatedguessing.Eliminatewhateveryoucanandthengowithyourgutamongtheremainingoptions.Nevermakeablindguessunlessyou’recompletelyoutoftimeandneedtofillinananswerchoice.Evencrossingoffjustonewronganswerwillincreaseyourchancesofrandomlychoosingthecorrectoneby33percent.Crossingofftwowronganswerswilldoubleyourchances.Ifpossible,workonanyunansweredquestionsforthepassage,andseeifthatallowsyoutoreturntoruleoutadditionalwrongoptions.

MCATExpertiseShouldIcompareanswerchoices?Yourdefaultassumptionshouldbethatonlyoneanswerchoiceiscorrectandthattheotherscontainatleastoneflaweach,sufficientforrulingthemout.However,youmayoccasionallyfindquestionscontainingsuperlatives(strongestchallenge,mostsupported,bestexample,andsoon)inwhichyouneedtocomparetwoormoreanswersthathavethesameeffectbuttodifferentdegrees.Whenmakingsuchcomparisons,don’tassumethatanextremeanswerisnecessarilywrong,especiallyifthequestionstemincludesthewordsiftrueorsimilarlanguage.Astrongeranswerthatneverthelessproducesthedesiredoutcomewouldactuallybethecorrectchoice.

Page 41: Edited By Deeangelee Pooran-Kublall, MD/MPH · 9.5 Preparing for the MCAT: Biochemistry in the Chemical and Physical Foundations of Biological Systems Section 9.6 Preparing for the

3.2WrongAnswerPathologiesThe American Association of Medical Colleges (AAMC), the maker of the MCAT, has designedsectionsof theexam tobe fair testsofcritical thinkingskills.Theneed for fairness isgreatnews,becauseitmeansthatthequestionswon’tplaytricksonyou!Therewillneverbeaquestionwithtwocorrectanswerchoicesoroneinwhichalloftheoptionsarewrong.EachquestionyouencounteronTestDaywillhaveoneandonlyonerightanswerandthreethatareincorrectforatleastonereason.Evenbetter,therecanonlybesomanyofthesereasons:infact,afewofthemarefoundsofrequentlythatyoucantreatthemlikerecurringsignsandsymptomsofanswerchoiceillness.Naturally,wecallthemWrongAnswerPathologies.

KeyConceptWronganswerpathologiesarethemostfrequentpatternsfoundinincorrectanswerchoices.Theyaresocommonthatyou’llfindatleastoneinjustabouteveryCARSquestionandeveninmanyofthequestionsinthethreesciencesections!

Achoice only needs one fatal flaw to beworth eliminating, but oftenwrong answer options havemanyissues,sodon’tnecessarilybealarmedifyouruledoutawronganswerforadifferentreasonthan theonementioned in apracticequestion’s explanation. In addition tohaving someoccasionaloverlap, the following list ofpathologies isnotmeant tobe exhaustive,butonly includes the fourpatternswe’veidentifiedasthemostcommonthroughresearchingallofthereleasedMCATmaterial.IntheKaplanMethodforQuestionsjustdetailed,pathologiesfunctionasrecurringexpectationsforwrong answers, which you can assume fit for most of the questions you encounter (with a fewsignificantdeparturesnotedbelow).

FAULTYUSEOFDETAIL(FUD)Thetestmakerswilloftenincludeaccuratereflectionsofpassagedetailsinwronganswers,primarilytoappealtothosestudentswhojumpatthefamiliarlanguage.Whatmakestheuseofadetail“faulty”isthatitsimplydoesn’tanswerthequestionposed.Itmaybetoospecificforaquestionthatrequiresageneral answer, or itmay be that the detail comes from thewrong part of the passage. Even if achoicecomesfromtherightparagraph,thedetailcitedmightnotberelevanttothequestionposed,often the case in Strengthen–Weaken (Within the Passage) questions.A thorough predictionmakescatchingtheseFUDsmucheasier.

OUTOFSCOPE(OS)WiththenoteworthyexceptionofReasoningBeyondtheTextquestions(forwhichthispathologydoesnot apply), an answer choice that is outside the scope of the passage will inevitably be wrong.Typically,suchanswerswillbeon-topicbutwillbringinsomeelementthatthetextdoesnotdiscuss.Forinstance,ifanauthornevermakescomparisonswhendiscussingdifferentideas,anOutofScopeanswerchoicemightinvolvetheauthorrankingtwoormoreofthem.AnothercommonOSpatternisthesuggestion thataviewwas the firstof itskindor themost influential,when theauthorentirelyavoidsdiscussingitshistoricaloriginsorrelativepopularity.Keepinmind that informationcanbeunstatedbythepassagebutnotcountasoutofscope,aswillbethecasewiththecorrectanswersto

Page 42: Edited By Deeangelee Pooran-Kublall, MD/MPH · 9.5 Preparing for the MCAT: Biochemistry in the Chemical and Physical Foundations of Biological Systems Section 9.6 Preparing for the

manyReasoningWithintheTextquestions,sodon’tbetooquicktorejectachoiceasOSjustbecausetheauthordoesnotexplicitlysayit.

OPPOSITEWheneverananswerchoicecontainsinformationthatdirectlyconflictswiththepassage,wecallitanOpposite.Oftenthedifferenceisduesimplytothepresence(orabsence)ofasinglewordlikenotorexcept,aprefixlikeun–ora–,orevenasuffixlike–lessor–free.Beespeciallycarefulwhenstemsorchoicesinvolvedouble-(ortriple-)negatives;they’remuchlessdifficulttoassessifyourewordthemwithfewernegations.Moreover,don’tassumethatjustbecausetwoanswerchoicescontradict,that one of themhas to be correct. For example, suppose an author argues that it is impossible toprovewhetherornotadivinebeingexists,avariantofthereligiousviewknownasagnosticism.Ifaquestion accompanying the passagewas to ask for a claim the author agreedwith,God exists andThereisnoGodwouldbothbeOppositesofthecorrectanswer.

DISTORTIONExtremeanswersandothersthat“twist”theideasinthepassagefurtherthantheauthorwouldpreferare what we call Distortions. Although they do not automatically make a choice incorrect, thefollowingarecommonsignalsofdistortedclaims:

•Strongwordslikeall,always,none,never,impossible,oronly•Aprefixlikeany–orevery–•Asuffixlike–estor–less

MCATauthors typicallydonot takeradicalpositionson issues,so it’sworthnotingwhenever theydo.Inthoserarecases,extremechoiceswouldnotactuallybeDistortionsoftheauthor ’sview,andwouldactuallybemorelikelytobecorrect.Theothermajorcaseinwhichextremeanswerchoicesshouldnotbeimmediatelyruledoutiswhenthequestionstemtellsyouthatyoucantreattheanswerchoicesastrue,andyourtaskisonlytogaugewhichwouldhavethegreatestimpactonaparticularargument.ThisisoftenthecasewithStrengthen–Weaken(BeyondthePassage)questions.

Page 43: Edited By Deeangelee Pooran-Kublall, MD/MPH · 9.5 Preparing for the MCAT: Biochemistry in the Chemical and Physical Foundations of Biological Systems Section 9.6 Preparing for the

3.3SignsofaHealthyAnswerIfyou’relikemoststudentspreppingfortheMCAT.you’vehadadisputewithatleastonequestionexplanation.Hey,whataboutwhattheauthorsaysinthefirstparagraph?youmayhavewondered,orperhapsyou’vesaidtoyourself(oraloud!)Butcouldn’tyouthinkofitlikethisinstead?Whileyoumaybeinthehabitofarguingforpointswithcollegeprofessors,itdoesyounogoodtotrytoarguewith the MCAT. The testmakers are extremely deliberate about how they word correct answers,alwaystakingcaretoincludeexactlyoneperquestion.

Correctanswerchoicescanvarywidelyinappearance,buttherearepatternsinhowtheyarewrittenaswell. If the traps that can lead you astray on TestDay are appropriately calledWrongAnswerPathologies,thenthesecorrespondingtraitscanbethoughtofasindicationsofgoodhealth.Whilethefollowingsignsarenotenoughbythemselvestomakeananswerright,youcantreatthemasgeneralexpectationsforthecorrectchoicesinmosttypesofquestions.

APPROPRIATESCOPE

Youmight say correct answers follow the “Goldilocksprinciple”when it comes to scope: not toobroad,nottoospecific,butjustright.Thescopedefinesthelimitsofthediscussion,ortheparticularaspectsofthelargertopicthattheauthorreallycaresabout.Inyouroutline,theGoalthatyoujotteddown should give you an idea of the scope of the passage overall. As a general rule (with oneimportant exception), correct answers to MCAT questions will remain within the scope of thepassage,butyoucanformulateamorepreciseexpectationofwhatscopethecorrectanswerneedstohavebyidentifyingthequestion’stypeandtask.

MCATExpertiseThescopeofatextreferstotheparticularaspectsofatopicthattheauthoraddresses.EveryparagraphinaCARSpassagehasitsownscope,andtogetheryoucanthinkofthemasconstitutingthescopeofthewholepassage.Similarly,eachanswerchoicewillhaveitsownscope,whichcouldmimicanypartoftheauthor’sdiscussionordepartfromthepassageentirely.Itisessentialtonotethathavingthesamescopedoesn’tnecessarilymeanhavingidenticalcontent.Forinstance,unstatedassumptionsinanargumentaredefinitelywithinthescopeofthepassage,eventhoughtheinformationtheycontainisleftunsaidbytheauthor.

MainIdeaquestionswillalwayshavecorrectanswersthatmatchthescopeoftheentirepassage.Theywilltypicallyincludeatleastonewronganswerchoicethatistoofocused(FaultyUseofDetail)andatleastonethatgoesoutsidethepassageentirely(OutofScope).Incontrast,DetailandDefinition-in-Context questions usually requiremore refined scopes for their correct answer choices. If a cluedirectstoaparticularportionofthepassage,thenthecorrectanswermoreoftenthannotwillhavethesamescopeasthereferencedtext(orwhatimmediatelysurroundsit).

The important exception to the rule that answers must remain within the scope of the author ’sdiscussionappliestothecategoryofReasoningBeyondtheTextquestions,addressedinChapter11ofMCATCritical Analysis andReasoning Skills Review. Like their name suggests, these broaden thescopetonewcontexts,sometimesappearingtohavenoconnectiontothepassagewhatsoever.Note,however,thatsomeReasoningBeyondtheTextquestionswillpresentnewinformationinthestembuthaveanswers thatstick to thescopeof thepassageanyway.Sobesavvywith theanswerchoices in

Page 44: Edited By Deeangelee Pooran-Kublall, MD/MPH · 9.5 Preparing for the MCAT: Biochemistry in the Chemical and Physical Foundations of Biological Systems Section 9.6 Preparing for the

ReasoningBeyondtheTextquestions:whilethecorrectanswerchoicetendstomoveslightlyoutsidethescopeofthepassage,don’tautomaticallyruleoutananswerchoicejustbecauseithappenstobeinscope.

AUTHORAGREEMENTUnlessaquestionstemexplicitlyasksaboutanalternativeviewpointorachallengetotheinformationpresentedinthepassage,acorrectanswerchoicewillbeonethatisconsistentwithwhattheauthorsays. This is one reason why considerations of tone and voice (most clearly reflected by Authorkeywords)areusuallyimportantenoughtobeworthincludinginyourpassageoutline.Generally,acorrectanswershouldnotcontradictanythingthattheauthorsayselsewhereinthepassage,withthepossible exceptionof sentences that speak in adifferent voice than the author ’s (such asquotesorreferences to others’ opinions). In short, if it doesn’t sound like something the author would say,you’llmostlikelywanttoruleitout.

WEAKERISUSUALLYBETTEROnefinalconsiderationisaconsequenceofthefactthattheAAMCtendstoselectpassagesinwhichthe authors donot take extremeviews.Youmay findoneor twopassagesonTestDaywithmoreradical writers; for them, a stronger claim in the answer choices may actually be a good sign.However, for most of the passages you’ll encounter, authors tend to use numerous Moderatingkeywordstolimitthestrengthoftheirclaims.Becauseastrongerclaimhasahigherburdenofproof(thatis,strongerevidencemustbeprovidedtosupporttheclaim),mostauthorsavoidthemtomakewhat theywritemoreplausible.Thus,youshouldgenerallygivepreference toanswerchoices thatuseweaker languagesuchascan,could,may,might, ispossible,sometimes,often, likely,probably,and in some sense. Exceptions to this tendency were addressed previously in the discussion ofDistortions.

Page 45: Edited By Deeangelee Pooran-Kublall, MD/MPH · 9.5 Preparing for the MCAT: Biochemistry in the Chemical and Physical Foundations of Biological Systems Section 9.6 Preparing for the

3.4GettingtheEdgeUsingtheQuestionStrategyThischapterisonlyanintroductiontothequestionmethod;theworkedexamplesthatfollowinUnitIII are a necessary supplement for seeing how the method functions in practice. Specific strategysuggestionsandworkedexamplesareincludedforeachofthemostcommonquestiontasks,togetherconstitutingalargeproportionofwhatyou’llencounteronTestDay.TheexplanationsaccompanyingthesesamplequestionswillalsoidentifytheirWrongAnswerPathologies,givingyousomeconcreteexamplestogowiththeexplanationsprovidedhere.

Page 46: Edited By Deeangelee Pooran-Kublall, MD/MPH · 9.5 Preparing for the MCAT: Biochemistry in the Chemical and Physical Foundations of Biological Systems Section 9.6 Preparing for the

3.5ConceptandStrategySummary

KAPLANMETHODFORCARSQUESTIONS•Assess

•Readthequestion,NOT theanswers•Identifythequestiontypeanddifficulty•Decidetoattacknoworlater

•Plan•Establishthetasksetbythequestiontype•Findcluesinthestemonwheretoresearch•Navigatethepassageusingyouroutline

•Execute•Predictwhatyoucanabouttheanswer•Setexpectationsforwrongchoices•Beflexibleifyourfirstplanflops

•Answer•Findamatchforyourprediction,or•Eliminatethethreewrongoptions,or•Makeaneducatedguess

WRONGANSWERPATHOLOGIES•FaultyUseofDetail(FUD)answerchoicesmaybeaccuratestatements,butfailtoanswerthequestionposed.

•Theanswerchoicemaybetoospecificforaquestionthatrequiresageneralanswer.•Theanswerchoicemayuseadetailfromthewrongpartofthepassage.•Theanswerchoicemaybefromtherightparagraph,butstillnotberelevant to thequestionposed.

•OutofScope (OS) answerchoicesusuallybring in someelement that thepassagedoesnotdiscuss(andwhichcannotbeinferredfromthepassage).

• The answer choicemaymake connections or comparisons that the author did notdiscuss.•Theanswerchoicemaymakeastatementaboutthesignificanceofthehistoryofanideathattheauthordidnot.•Theanswerchoicemayotherwisebring in information thatdoesnot adhere to theconstraintsofthepassage.

•Oppositeanswerchoicescontaininformationthatdirectlyconflictswiththepassage.•Theanswerchoicemaycontain(oromit)asinglewordlikenotorexcept.•Theanswerchoicemaycontainaprefixlikeun–ora–orasuffixlike–lessor–free.•Theanswerchoicemaysaythatagivenclaimistrue,whentheauthorisambivalent.

•Distortionanswerchoicesareextremeortwisttheideasinthepassagefurtherthantheauthorwouldprefer.

•Theanswerchoicemayuseastrongwordlikeall,always,none,never, impossible,oronly.

Page 47: Edited By Deeangelee Pooran-Kublall, MD/MPH · 9.5 Preparing for the MCAT: Biochemistry in the Chemical and Physical Foundations of Biological Systems Section 9.6 Preparing for the

•Theanswerchoicemaycontainaprefixlikeany–orevery–orasuffixlike–estor–less.•TheanswerchoiceisusuallymoreradicalthantheauthorbecauseradicalpositionsarerareinMCATpassages.

SIGNSOFAHEALTHYANSWER•Correctanswerstendtohavetherightscope—nottoobroad,nottoospecific,butjustright.•Correctanswerstendtobeconsistentwithwhattheauthorsaid.•CorrectanswerstendtouseModeratingkeywords,suchascan,could,may,might,ispossible,sometimes,often,likely,probably,andinsomesense.

Page 48: Edited By Deeangelee Pooran-Kublall, MD/MPH · 9.5 Preparing for the MCAT: Biochemistry in the Chemical and Physical Foundations of Biological Systems Section 9.6 Preparing for the

II

StrategicInquiryandReasoningSkills

Page 49: Edited By Deeangelee Pooran-Kublall, MD/MPH · 9.5 Preparing for the MCAT: Biochemistry in the Chemical and Physical Foundations of Biological Systems Section 9.6 Preparing for the

CHAPTERFOUR

Skills1and2ThereisnoquestionthatahighscoreontheMCATrequiresextensivecontentknowledgeinbiology,biochemistry, general chemistry, organic chemistry, physics, psychology, and sociology.However,theMCATisnotacontenttest.Oneofthemostcommonmistakesthattest-takersmakewhenstudyingfortheMCATis treatingit likeacontent testandstudyingfor it likeanundergraduatecourse.TheMCATisactuallyacriticalthinkingtest,designedtotestyourabilitytoapplyyourknowledgetonewsituations.Inaddition,questionsontheMCAThavebeenexpandedtoincludeadditionalskills,suchasevaluation of experimental design and execution as well as interpretation of data and statisticalreasoning.TheAAMChasseparatedthesecategoriesintofourskills,appropriatelycalledSkills1,2,3,and4.Inthischapter,wewilldiscusshowtheMCATwilltestSkills1and2.

4.1WhatAreSkills1and2?Unlikemany tests administered to undergraduates, the questions are carefully crafted and tested toensure that the MCAT tests the specific skills and content areas that the AAMC believes is mostessential for success in medical school. To do this, the AAMC creates questions reflecting fourdifferent skills. Skills 1 and 2 are designed to evaluate your scientific knowledge and reasoningability.

SKILL1Skill 1 is the simplest of the skills, testing knowledge of scientific concepts and principles. Thesequestionswillbefairlystraightforward,askingyoutoidentifyrelationshipsbetweenrelatedconcepts.Essentially,Skill1questionstestyourcontentknowledgeinaveryspecificmanner.Skill1questionsaredesignedtotest:

•Knowledgeandrecognitionofscientificprinciples•Abilitytoidentifyrelationshipsbetweenrelatedconcepts• Identification of relationships between graphical, symbolic, and verbal representations ofinformation•Identificationofobservationsthatillustratespecificscientificprinciples•Theapplicationofmathematicalprinciplestosolveproblems

SKILL2Skill 2 is more complex than Skill 1 in that Skill 2 tests your scientific reasoning and problem-solving skills. Essentially, Skill 1 tests your content knowledge,while Skill 2 tests your ability toapplythatknowledge.Youwillbeaskedtoreasonaboutscientificprinciples,models,andtheoriesaswellastoevaluateandanalyzepredictionsandexplanationsrelatedtoscientificconcepts.

Overall,Skill2questionsaregoingtotestyourcriticalthinkingskills.Butwhatare“criticalthinkingskills?”Criticalthinkingisamethodofprocessingandusinginformation.Ithasbeendefinedas“theprocessofactivelyandskillfullyconceptualizing,applying,analyzing,synthesizing,andevaluating

Page 50: Edited By Deeangelee Pooran-Kublall, MD/MPH · 9.5 Preparing for the MCAT: Biochemistry in the Chemical and Physical Foundations of Biological Systems Section 9.6 Preparing for the

informationtoreachananswerorconclusion.”Criticalthinkingisnotjustmemorizinginformation;it is using this information through higher order processing. However, critical thinking requiressomelevelofknowledgeaboutatopic,beyondbasicmemorization.

Oneofthewaysthatwecantacklecriticalthinkingisbychanginghowwelearninformation.Manystudents use memorization as a primary method of learning, and are successful in passingundergraduatelevelsciencecourses.Butmedicalschoolisverydifferentfromundergraduatesciencecourses.Thevolumeofinformationisastounding,oftencoveringthesamevolumeofinformationinan entire semester of an undergraduate science course in twoweeks or less.Medical students areexpectedtolearnthisinformationandbeabletosystematicallyapplythisinformationtopatientcare.But,withoutthegiftofsuperhumanphotographicmemory,gettingthroughmedicalschoolbybruteforcememorization is just not realistic.But,more relevant to theMCAT, higher test scores resultfromtheabilitytothink,analyze,andapplyinformationyoualreadyknowtosituationsandconceptsthatyoumaynotknow.Thisparticularskill isessentialformedicine,asyouwillhavetoapplytheinformationyouknowaboutthehumanbodytoapatientaboutwhomyoumayknowlittle.

So,howcanyouimproveyourabilitytolearnandapplyinformation?Byfocusingonyourlearningbehavior.

One of the tools used in educational psychology for assessing learning is known as Bloom’staxonomy (See Figure 4.1). In this taxonomy, there are six levels of intellectual behavior that areimportantforlearning.Theseare:

•Remembering–canyourecalltheinformation?•Understanding–canyouexplaintheinformationtoaclassmate?•Applying–canyouapplytheinformationtoanewsituation?•Analyzing–canyoucompareandcontrastdifferencepartsoftheconcept?Canyoudifferentiatebetweenthedetailswithintheconcept?•Evaluating–canyouusetheinformationtosupportajudgmentbasedontheinformation?•Creating–canyouincorporatetheinformationtocreateanotherstructure/document/theory?

IneachcontentareaontheMCAT,itisBloom’staxonomythatisbeingtested,regardlessoftheskill.When evaluating your knowledge of a particular subject, you can use Bloom’s taxonomy todetermineyourleveloflearningandtoincreaseyourleveloflearning.Trythisstep-by-stepmethodfordeepeningyourlearningofaconcept:

•Doyouthinkyourememberandunderstandaparticularconcept?Then,explainittoafriendorclassmate.• Ifyoucando that, then try to findsituations thataredifferentbut towhich thesameconceptapplies.•Then,compareandcontrasttheconcepttoasimilartheoryorconcept.•Onceyoucanseethedifferencesandthesimilarities,workthroughapassageandquestions,explainingwhyeachanswerchoiceiscorrectorincorrect.•Finally,canyouwritesomethingaboutaconcept?TrytowriteapassagesimilartooneseenontheMCAT.

Whileit isunrealistictodothisforeverytopicontheMCAT,asitwouldbehighlyinefficient, thelevels of Bloom’s taxonomy represent the different levels of learning theMCAT requires. It alsoexplains why so many students spend thousands of hours studying for the MCAT, but are not

Page 51: Edited By Deeangelee Pooran-Kublall, MD/MPH · 9.5 Preparing for the MCAT: Biochemistry in the Chemical and Physical Foundations of Biological Systems Section 9.6 Preparing for the

successfulonTestDay.Manyfocusonlyonthe“remembering”partoflearning,andonlyalittlebitonthe“understanding”part.

ThesecrettoMCATsuccessonTestDayisnotjustinrememberingthecontent,itishavingahigherlevel of understanding of the content. If you can complete all levels ofBloom’s taxonomywith aconcept, thennotonlydoyoureallyknowthattopic,butyouarealsofarmorelikelytorecall thattopic on Test Day. Memorized information does not have nearly the same level of recall asinformationthatisdeeplyunderstoodandabletobesynthesizedandactivelyusedtosolveproblems.In other words, you are not going to remember everything you study for Test Day. In fact, it isunlikelythatyouwillbecomfortablewitheverysingletopiconTestDay.However,ifyoubuildyourknowledgetosuchadegreethatyoufocusonadeeperleveloflearning,youwillbeabletouselogictogofromwhatyouDOknowtodetermineanswerstoquestionsyoumaynotspecifically“know”theanswerto.

Figure4.1Bloom’staxonomy

Skill 2 questions specifically test the skillsmentioned inBloom’s taxonomy. Skill 2 questions aredesignedtotest:

•Abilitytoreasonaboutscientificmodels,principles,andtheories•Evaluationandanalysisofscientificpredictionsandexplanations•Interpretationandevaluationofargumentsregardingcausesandconsequences•Unificationofobservations,theories,andevidencetocometoconclusions• Recognition of scientific findings that pose a challenge or invalidate a scientific theory ormodel•Identificationanduseofscientificformulastosolveproblems

Skill1vs.Skill2

Skill1

Skill2

Thisskilltests:

Basicscientificknowledge

Scientificreasoningskills

Page 52: Edited By Deeangelee Pooran-Kublall, MD/MPH · 9.5 Preparing for the MCAT: Biochemistry in the Chemical and Physical Foundations of Biological Systems Section 9.6 Preparing for the

Questionswillrequire:

IdentificationofthetaskSelectionofthecorrectanswer

IdentificationofthetaskPlanningofanattackthatwillinvolveamultistepprocesstodetermineananswerSelectionofthecorrectanswer

Page 53: Edited By Deeangelee Pooran-Kublall, MD/MPH · 9.5 Preparing for the MCAT: Biochemistry in the Chemical and Physical Foundations of Biological Systems Section 9.6 Preparing for the

4.2HowWillSkills1and2BeTested?ThedifferencesbetweenSkills1and2requireadifferentapproachinorder toassesswhetheryouhave acquired the reasoning skills that will help you to be successful inmedical school and as aphysician.In thissection,wewillexplorehowtheskillswillbe testedoneachof thefourquestiontypes,aswellashowthesequestionsaregoingtoappearonTestDay.

SKILL1Skill1questionswillbefairlysimple.TheseareofteneasypointsonTestDay,aslongasyouhaveconducted a thorough and effective content review prior to Test Day. However, getting Skill 1questions correct on TestDay is only a small piece of earning a high score; youwill have to beproficientinallfourskills.

Discretequestions• Straightforward, direct questions, either addressing relationships between concepts oridentificationofasingledetailorcharacteristicsregardingaconcept.

Questionsthatstandalonefromthepassage•Verysimilartodiscretequestions,oftenthematicallyrelatedtothepassage.•Will require identification of a single concept or relationshipwithout amultistep reasoningprocess.

Questionsthatrequiredatafromthepassage•Willrequireminimalanalysisofdatafromthepassage,oftenconnectionofinformationfromthepassagetoscientificconcepts•Identificationofascientificconceptwithagraphortablemayberequired.•Amathematicalequationmaybepresentedandyoumaybe requested touse thisequation tosolveaproblem.

•Questionsthatrequirethegoalofthepassage•Skill1questionsareunlikelytorequirethislevelofanalysis.Whentheydo,thequestionswillbe simple and require identification of a relationship between a scientific concept and afundamentalpieceofinformationinthepassage.

SKILL2Skill 2 questionswill be extremely common on TestDay. In fact, thiswas an extremely commonquestiontypeonpreviousversionsoftheMCAT.

Discretequestions•Willrequireyoutounderstandaparticularconcept,andthenlinkthatconceptwiththetaskofthequestion.•Nopassagewillbeassociated,thusthescopeoftheanswerwillbelimitedtothescopeofthequestion.

Page 54: Edited By Deeangelee Pooran-Kublall, MD/MPH · 9.5 Preparing for the MCAT: Biochemistry in the Chemical and Physical Foundations of Biological Systems Section 9.6 Preparing for the

Questionsthatstandalonefromthepassage•Muchlikediscretequestions,butoftenthematicallyrelatedtothepassage.•Mayrequireapplicationofknowledgerelatedtothepassage,butnotspecificallymentionedinit.

Questionsthatrequiredatafromthepassage• Likely to require calculations, evaluation of data and connection of the data with scientificprinciples,anduseofscientificdataandknowledgetodrawconclusionsrelatedtoinformationpresentedinthepassage.•Mayrequireinterpretationoranalysisofgraphs,tables,ordiagrams.•Youmayhavetoapplyinformationobtainedfromthepassagetoanewsituationnotpresentedinthepassage.•Youmaybeaskedtoanalyzetherelationshipbetweentwovariables,intermsofcausationandcorrelation.

Questionsthatrequirethegoalofthepassage• Will require you to interpret the passage as a whole, and connect the passage with yourknowledgeaboutscientificprinciplesoryourknowledgeaboutnaturalorsocialphenomenon.•Mayrequireyoutoidentifyargumentsaboutcauseandeffectassupportedbythepassageornewevidenceprovidedinthequestionstem.• Information may be presented in the question stem that either strengthens or weakens theargumentmadeinthepassage.Youwillbeaskedtoidentifyhowthisnewinformationisrelatedtothepassageasawhole.•Youmaybeexpected todrawconclusionsfromthe informationpresented in thepassage,orevaluatethevalidityofaconclusionbasedonevidencefromthepassage.

Page 55: Edited By Deeangelee Pooran-Kublall, MD/MPH · 9.5 Preparing for the MCAT: Biochemistry in the Chemical and Physical Foundations of Biological Systems Section 9.6 Preparing for the

4.3GettingtheEdgewithSkills1and2Content knowledge is required for the MCAT, especially for Skill 1 questions. However, contentknowledgeisnotenoughforsuccessonTestDay.TheMCATusesscienceasavehicletotestcriticalthinkingskills,especiallyonSkill2questions.InordertogettheedgeonSkills1and2,athoroughand efficient content review must be coupled with a significant number of practice questions. AcommonmistakemadebystudentswhenstudyingfortheMCATisfocusingtooheavilyoncontentreviewandnotenoughonpracticequestionsandpassages.TheonlywaytobesuccessfulonTestDayis to integrate content review with practice questions and passages. Success on Skill 2 questionsrequires thatyouacquireadeeper levelof learning thanwhat is required inundergraduatesciencecourses. In order to do this, ensure that you review content and then immediately apply content topracticepassageandquestions.

Every question that you answer incorrectly on a practice passage or question is an opportunity toreview how you approached the question. Systematically reviewing questions to determineweaknessesinyourcriticalthinkingskillswillhelpyoutoidentifyandaddressthoseweaknesses.Ifyoufindthatyouarestuckatascoreplateauonpracticetests,itisessentialtotakeanhonestlookatyourcriticalthinkingskillstobreakthroughthatplateau.

When the AAMC released the specifications for the 2015 MCAT, it established the expecteddistributiontobe35%Skill1and45%Skill2questionsforeachofthesciencesections.

Page 56: Edited By Deeangelee Pooran-Kublall, MD/MPH · 9.5 Preparing for the MCAT: Biochemistry in the Chemical and Physical Foundations of Biological Systems Section 9.6 Preparing for the

CHAPTERFIVE

Skill3Modern medicine requires the practical application of research in the practice of evidence-basedmedicine.Asaphysician,youwillconstantlybeseekinganswersinresearchtodetermineprognoses,assesstheappropriatenessofatreatmentmodalityforagivenpatient,andanswerpatients’questions.Evaluation of research is critical to the progress of all fields of medicine and will be a keycomponentofyourlifeasaphysician.

Inordertopracticemedicineinthisway,certainbasicskillsarerequired,suchastheabilitytoreasonaboutthedesignandexecutionofresearch,otherwiseknownasSkill3.Inthissection,wewilldiscussthespecificcharacteristicsofSkill3aswellashowSkill3willbetestedontheMCAT.

5.1WhatIsSkill3?Skill3isdividedintotwomaincomponents:conceptsbehindscientificresearchandreasoningaboutethical issues in research.Thismeans thatyoumustunderstandhowstudiesaredesignedusing thescientificmethodasaguidingprinciple,aswellastheethicalissuespresentinresearch,especiallyintheuseofhumansubjects.

SKILL3QUESTIONSTEST:

•Identificationoftherolesofpastfindings,theory,andobservationsinscientificinquiry•Identificationoftestablehypothesesandresearchquestions•Ability to distinguish between samples and populations, and the identification of results thatsupportgeneralizationsaboutpopulations•Identificationofindependentandindependentvariables• Ability to reason about the features of research studies that suggest relationships amongvariables,includingcausality•Abilitytodrawconclusionsfromresultsproducedbyresearchstudies•Identificationofhowresultsfromresearchmayapplytoreal-worldsituations•Reasoningskillswithrespecttoethicalissuesinscientificresearch

Page 57: Edited By Deeangelee Pooran-Kublall, MD/MPH · 9.5 Preparing for the MCAT: Biochemistry in the Chemical and Physical Foundations of Biological Systems Section 9.6 Preparing for the

5.2FundamentalConceptsofSkill3Skill3questionswillaskyoutoapplyscientificconceptstoyourunderstandingofresearchinboththelifesciencesandbehavioralsciences.However,thisinformationisrarely,ifever,coveredinyourundergraduatescienceclasses.Thescientificmethodisusuallymentioned,butitisunlikelythatitiscoveredinthelevelofdetailthatyouwillneedonTestDay.Inthissection,wewilldiscussthebasicconceptsthatyouneedtobesuccessfulonSkill3questionsonTestDay.

THESCIENTIFICMETHODThescientificmethodisthebasicparadigmofallscientificinquiry.Itistheestablishedprotocolfortransitioningfromaquestiontoanewbodyofknowledge.Thestepsinthescientificmethodare:

Generateatestablequestion•Occursafterobservingsomethinganomalousinanotherscientificinquiryorindailylife

Gatherdataandresources•Phaseofjournalanddatabasesearchesandinformationcompilation•Lookatallinformation,notjustthoseconsistentwiththeopinionoftheinvestigator

Formahypothesis•Oftenintheformofanif-thenstatement,whichwillbetestedinsubsequentsteps

Collectnewdata• Collect data by experimentation (manipulation and control of variables of interest) or byobservation(usuallyinvolvesnochangesinthesubject’senvironment)

Analyzethedata•Lookfortrends•Performmathematicalmanipulationstosolidifytherelationship(s)betweenthevariables

Interpretthedataandexistinghypothesis•Considerwhetherthedataanalysisisconsistentwiththeoriginalhypothesis•Ifdataisinconsistent,consideralternatehypotheses

Publish•Provideanopportunityforpeerreview•Summarizewhatwasdoneduringthepreviousstepsinthepublication

Verifyresults•Repeattheexperimenttoverifyresultsundernewconditions

BASICCONCEPTSINSCIENTIFICRESEARCHBasic science research—the kind conducted in a laboratory, and not on people—is generally the

Page 58: Edited By Deeangelee Pooran-Kublall, MD/MPH · 9.5 Preparing for the MCAT: Biochemistry in the Chemical and Physical Foundations of Biological Systems Section 9.6 Preparing for the

easiesttodesignbecausetheexperimenterhasthemostcontrol.Oftenacausalrelationshipisbeingexamined because the hypothesis generally states a condition and an outcome. In order to makegeneralizations about our experiments, the outcome of interest must not be obscured. In addition,theremustalsobeamethodbywhichcausalitymaybedemonstrated,which is relativelysimple inbasic science research, but less so in other research areas. This requires the use of a control, orstandard,andanidentifiedsetofvariables.

ControlsInbasicscienceresearch,conditionsareappliedtomultipletrialsofthesameexperimentthatareasneartoidenticalaspossible.

•Acontrolorstandardisamethodofverifyingresults.•Controlscanalsobeusedtoseparateexperimentalconditionsaltogether.•Positiveandnegativecontrolsareusedaspointsofcomparisonoragroupofcontrolsthatcanbeusedtocreateacurveofknownvalues.

•Positivecontrolsarethosethatensureachangeinthedependentvariablewhenitisexpected.Forexample,ifanewassayisdevelopedforthedetectionofHIVinfection,anumberofbloodsamplesknowntocontainHIVviruscanactasapositivecontrol.• Negative controls ensure no change in the dependent variable when no change isexpected.Forexample,thesamenewHIVassaywouldbeusedtotestsamplesknowntobenormal. Inpharmaceutical trials,anegativecontrolcouldbeused toassess theplaceboeffect.Anobservedorreportedchangewhenanindividualisgivenaninactivesubstancesuchasasugarpillisanexampleoftheplaceboeffect.

CausalityBy manipulating all of the relevant experimental conditions, basic science researchers can oftenestablishcausality.Causalityisanif-thenrelationship,andisoftenthehypothesisbeingtested.

•Independentvariable–thevariablethatismanipulatedorchanged.•Dependentvariable–thevariablethatismeasuredorobserved.•Ifachangeintheindependentvariablealwayscausesachangeinthedependentvariable,andthechangeinthedependentvariabledoesnotchangewithoutchangeintheindependentvariable,thenarelationshipissaidtobecausal.

ErrorSourcesIn basic science research, experimental bias is usually minimal. The most likely way for anexperimenter ’spersonalopinionstobeincorporatedisthroughthegenerationofafaultyhypothesisfromincompleteearlydataandresourcecollection.Othersourcesoferrorincludemanipulationofresults by eliminating trials without appropriate background, or by failing to publish works thatcontradicttheexperimenter ’sownhypothesis.

Thelowlevelsofbiasintroducedbytheexperimenterdonoteliminateallerrorfrombasicscienceresearch.Measurementsareespeciallyimportantinthelaboratorysciences,andtheinstrumentsmaygivefaultyreadings.Instrumenterrormayaffectaccuracy,precision,orboth.Accuracy,alsocalledvalidity,istheabilityofaninstrumenttomeasureatruevalue.Precision,alsocalledreliability,istheability of the instrument to read consistently, orwithin a narrow range. Since bias is a systematicerror indata, only an inaccurate toolwill introducebias, but an imprecise toolwill still introduceerror.

Page 59: Edited By Deeangelee Pooran-Kublall, MD/MPH · 9.5 Preparing for the MCAT: Biochemistry in the Chemical and Physical Foundations of Biological Systems Section 9.6 Preparing for the

HUMANSUBJECTSRESEARCHResearchusinghumansubjectsisconsiderablymorecomplex,andthelevelofexperimentalcontrolis invariably lower than basic science research. In human subjects research, there are bothexperimentalandobservationalstudies.

EXPERIMENTALAPPROACHExperimental research, similar to basic science research, attempts to establish causality. Anindependentvariableismanipulatedandchangesinadependentvariableareidentifiedandquantified(if possible). Since subjects are in less-controlled conditions, the data analysis phase is morecomplicated than in laboratory studies.Twoof themost fundamental concepts of the experimentalapproacharerandomizationandblinding.

Randomization•Methodusedtocontrolfordifferencesbetweensubjectgroupsinbiomedicalresearch.•Usesanalgorithmtoplaceeachsubjectintoeitheracontrolgroupthatreceivesnotreatmentorashamtreatment,oroneormoretreatmentgroups.•Resultsaremeasuredinallgroups.•Ideally,eachgroupisperfectlymatchedonconditionssuchasageandgender.

Blinding• Many measures in biomedical research are subjective. Perception of the subject and theinvestigatormaybebiasedbyknowingthegrouptowhichthesubjecthasbeenassigned.•Whenastudyisblinded,thesubjectand/ortheinvestigatorarenotawareofthegroupinwhichthesubjecthasbeenplaced.•Single-blindexperiments–onlythepatientortheassessorisblinded•Double-blind experiments–neither the subject nor the assessor (or even the investigator) areawareofthegroupintowhichthesubjecthasbeenplaced•Lackofblindingresultsindiminishedplaceboeffectinthecontrolgroup,butthepresenceoftheplaceboeffectinthetreatmentgroup

In biomedical research, data analysis must account for variables outside the independent anddependentvariables.Mostoften, these includegenderandage; lifestylevariablessuchassmoking;bodymassindex;andotherfactorsthatmayaffectthemeasuredoutcomes.Confoundingvariables,orvariablesthatarenotcontrolledormeasured,mayalsoaffecttheoutcome.

OBSERVATIONALAPPROACHThe observational approach is often adopted to study certain causal relationships for which anexperimentiseitherimpracticalorunethical.Observationalstudiesinmedicinefitintothreedifferentcategories:

Cohortstudies•Subjectsaresortedintotwogroupsbasedondifferencesinriskfactors(exposures),andthenassessed at various intervals to determine how many subjects in each group had a certain

Page 60: Edited By Deeangelee Pooran-Kublall, MD/MPH · 9.5 Preparing for the MCAT: Biochemistry in the Chemical and Physical Foundations of Biological Systems Section 9.6 Preparing for the

outcome.

Cross-sectionalstudies•Patientsarecategorizedintodifferentgroupsatasinglepointintimebasedonthepresenceorabsenceofacharacteristic,suchasadisease.

Case-controlstudies•Subjectsareseparatedintotwogroupsbasedonthepresenceorabsenceofsomeoutcome.• The study looks backwards to assess how many subjects in each group had exposure to aparticularriskfactor.

Identifyingcausalityisn’tnecessarilysimple.Hill’scriteriadescribethecomponentsofanobservedrelationshipthatincreasethelikelihoodofcausalityinthatrelationship.Whileonlythefirstcriterionisnecessaryfortherelationshiptobecausal,itisnotsufficient.Increasedlikelihoodofcausalityissignifiedbyanincreasednumberofmetcriteria.Hill’scriteriadonotprovideanabsoluteguidelineoncausalityofarelationship.Thus,foranyobservationalstudy,therelationshipshouldbedescribedascorrelation.

Hill’sCriteria

Criterion

Description

TemporalityExposure(independentvariable)mustoccurbeforetheoutcome(dependentvariable)

Strength

Greaterchangesintheindependentvariablewillcauseasimilarchangeinthedependentvariableiftherelationshipiscausal

Dose-responseRelationship

Astheindependentvariableincreases,thereisaproportionalincreaseintheresponse(dependentvariable)

Consistency Therelationshipisfoundinmultiplesettings

Plausibility

Thepresenceofareasonablemechanismfortherelationshipbetweenthevariablesassupportedbyexistingliterature

ConsiderationofAlternateExplanations

Ifallotherplausibleexplanationshavebeeneliminated,theremainingexplanationismorelikely

Experiment Anexperimentcanconfirmcausality

Specificity

Changeintheoutcome(dependent)variableisonlyproducedbyanassociatedchangeintheindependentvariable

Newdataandhypothesesareconsistentwiththecurrentstateof

Page 61: Edited By Deeangelee Pooran-Kublall, MD/MPH · 9.5 Preparing for the MCAT: Biochemistry in the Chemical and Physical Foundations of Biological Systems Section 9.6 Preparing for the

Coherence scientificknowledge

ERRORSOURCESInadditiontothemeasurementerrorfoundinbasicscienceresearch,wemustbeawareofbiasanderror introduced by using human subjects as part of an experimental or observational model. Asmentionedearlier,biasisasystematicerror.Assuch,itgenerallydoesnotimpacttheprecisionofthedata, but rather skews the data in one direction or the other. Bias is a result of flaws in the datacollectionphaseofanexperimentalorobservationalstudy.Confoundingisanerrorduringanalysis.

SelectionBias•Mostprevalenttypeofbias•Occurswhensubjectsusedforthestudyarenotrepresentativeofthetargetpopulation•Mayalsoapplyincaseswhereonegenderismoreprevalent thananother,orwhentherearedifferencesintheageprofileoftheexperimentgroupandthepopulation•Measurementandassessmentofselectionbiasoccursbeforeanyintervention

DetectionBias•Resultsfromeducatedprofessionalsapplyingknowledgeinaninconsistentmanner• Often occurs when prior studies have indicated that there is a correlation between twovariables;whentheresearcherfindsoneofthevariables,thenheorsheismorelikelytosearchfor the second, possibly related variable. That makes the second variable more likely to befound,sincetheinvestigatorislookingforit.

ObservationBias•AlsoknownastheHawthorneeffect•Occurswhenthebehaviorofstudyparticipantsisalteredwhentheparticipantsareawarethattheyarebeingstudied•Systematicandoccurspriortodataanalysis

Confounding•Dataanalysiserror•Datamayormaynotbeflawed,butanincorrectrelationshipischaracterized•Variablesthatarenotcontrolledormeasured,butpresent

Page 62: Edited By Deeangelee Pooran-Kublall, MD/MPH · 9.5 Preparing for the MCAT: Biochemistry in the Chemical and Physical Foundations of Biological Systems Section 9.6 Preparing for the

ETHICALISSUESINRESEARCHInmedicine,therearefourcoreethicaltenets:beneficence,nonmaleficence,autonomy,andjustice.

•Beneficence:theobligationtoactinthepatient’sbestinterest•Nonmaleficence:theobligationtoavoidtreatmentsorinterventionsinwhichthepotentialforharmoutweighsthepotentialforbenefit•Patientautonomy:theresponsibilitytorespectpatients’decisionsandchoicesabouttheirownhealthcare•Justice:theresponsibilitytotreatsimilarpatientswithsimilarcare,andtodistributehealthcareresourcesfairly

Inresearch,theseprinciplesarereplacedbyaslightlymodifiedsetasdefinedbytheBelmontReport,alandmarkdocumentpublishedbytheNationalCommissionfortheProtectionofHumanSubjectsinBiomedicalandBehavioralResearch.AccordingtotheBelmontReport,thethreenecessarypillarsofresearchincluderespectforpersons,justice,andaslightlymoreinclusiveversionofbeneficence.

Respectforpersons• Includes the need for honesty between the subjects and the researcher andgenerally, but notalways,prohibitsdeception.•Alsoincludestheprocessofinformedconsent,inwhichapatientmustbeadequatelycounseledontheprocedures,risks,benefits,andgoalsofastudytomakeaknowledgeabledecisionaboutwhetherornottoparticipate.Consentmaybewithdrawnatanytime.•Prohibitsacoerciveinfluenceoverthesubjects.• Institutional review boards are in place to provide systematic protections against unethicalstudies.• Vulnerable persons, including children, pregnant women, and prisoners, require specialprotectionsaboveandbeyondthosetakenwiththegeneralpopulation.•Confidentialityisgenerallyconsideredtobepartofrespectforpersonsduringresearch.

Justice•Appliestoboththeselectionofaresearchtopicandtheexecutionoftheresearch.• Morally relevant differences are defined as those differences between individuals that areconsidered an appropriate reason to treat them differently. These differences include age andpopulation size. Race, ethnicity, sexual orientation, and financial status are NOT consideredmorallyrelevantdifferences.However,religionisaspecialcaseinthatcertaininterventionsareprohibitedbyagivenreligion.Thus,avoidanceofthattreatmentinindividualsofthatreligionisconsistentwithpatientautonomy.•Risks inherent in the studymust be distributed fairly so as not to impose undue harm on aparticulargroup.However,whenthereisapopulationthatislikelytoreceiveagreaterbenefitfromastudy,thisgroupmaynecessarilybearagreaterproportionoftherisk.Thus,likelihoodofbenefitmaybeamorallyrelevantdifferencebetweenindividualsincertainsituations.

Beneficence•Itmustbetheintentofastudytocauseanetpositivechangeforboththestudypopulationandthegeneralpopulation,anditmustbedoneinsuchawayastominimizeanypotentialharm.•Researchshouldbeconductedintheleast invasive,painful,or traumaticwaypossible.Whenchoosingbetweentwomethodsofmeasurement,theleastpainfulandinvasivemethodshouldbe

Page 63: Edited By Deeangelee Pooran-Kublall, MD/MPH · 9.5 Preparing for the MCAT: Biochemistry in the Chemical and Physical Foundations of Biological Systems Section 9.6 Preparing for the

employed.

Page 64: Edited By Deeangelee Pooran-Kublall, MD/MPH · 9.5 Preparing for the MCAT: Biochemistry in the Chemical and Physical Foundations of Biological Systems Section 9.6 Preparing for the

5.3HowWillSkill3BeTested?Skill3questionswillrequireaconsiderableamountofbackgroundinformation,asstudydesignandexecutionaregenerallynotcoverableinasinglequestionstem.Thus,themorecomplexandnuancedSkill3questionsarelikelytoeitherrequiredatafromthepassageorthegoalofthepassage.TheseSkill 3 questions are likely to also require the scientific reasoning skills tested in Skill 2. It isimportanttonotethatwhilethefourskillshavebeenseparatedneatlyintocategoriesbytheAAMC,the separationsmaynot be quite so distinct onTestDay, asmost questionswill still require somelevelofscientificreasoning.

Discretequestions•QuestionsthatareNOTassociatedwithadescriptivepassage•Likelytofocusonbasicresearchconcepts,includingidentificationofvariables;andconceptsofmeasurement,includingaccuracyandprecision

Questionsthatstandalonefromthepassage•Similartodiscretequestions•Likelytofocusonmorebasicconceptsofstudydesignandexecution

Questionsthatrequiredatafromthepassage•Bepreparedforquestions that requireyou tousedatafromthepassage to identifyvariablesandrelationshipsbetweenvariablesandtestyourtheabilitytodistinguishbetweencausationandcorrelation

Questionsthatrequirethegoalofthepassage•Thesequestionswillrequireanunderstandingofthepassageasawholeandidentificationofthegoalofthepassage•May ask for alternative explanations for the phenomena described in the passage aswell asexaminationofevidencepresentedinthepassage•Questionsmay require you to draw conclusions from evidence presented in the passage, aswellasidentifyconclusionsthatarenotsupportedbythepassage•Expecttoevaluatestudydesigninlightofresultsandconclusionsdrawnfromtheinformation• Identification of independent and dependent variables as well as confounding variables andtypesofbias

Page 65: Edited By Deeangelee Pooran-Kublall, MD/MPH · 9.5 Preparing for the MCAT: Biochemistry in the Chemical and Physical Foundations of Biological Systems Section 9.6 Preparing for the

5.4GettingtheEdgeinSkill3QuestionsMuchofwhatistestedinSkill3isnotnecessarilypresentedinintroductorylevelcourses.Sometest-takerswill have taken courseswithmore emphasis in research,while othersmay have little to noexperienceinthisarea.Youcanmaximizeyourscorebydevelopingathoroughunderstandingofthetopicsdiscussedin thischapter.TheMCATwillassumeacertainlevelofknowledgein theareaofresearch, and it is essential that you develop that knowledge by reading research articles andbecoming comfortable with the general format and discussion present in all research studies,including background, literature review, procedure, results, and discussion. As you read moreresearch studies, youwill gain increasing comfortwith research as a part of scientific inquiry.Besure to read research in all subject areas tested on the MCAT, including biology, biochemistry,general chemistry, organic chemistry, physics, psychology, and sociology. Most students arecomfortable with lab research, but are less comfortable with research involving human subjects.Focus your review of study design in the areas that are more likely to involve human subjects,includingbiology,psychology,andsociology.

When the AAMC released the specifications for the 2015 MCAT, it established the expecteddistributiontobe10%Skill3foreachofthesciencesections.

Page 66: Edited By Deeangelee Pooran-Kublall, MD/MPH · 9.5 Preparing for the MCAT: Biochemistry in the Chemical and Physical Foundations of Biological Systems Section 9.6 Preparing for the

CHAPTERSIX

Skill4Bynow,youhavenodoubtrealizedthattheMCATplacesahighlevelofimportanceonyourabilitytounderstandtheprocessbywhichnewscientificandmedicalknowledgeisacquiredbyresearch.Asdiscussedpreviously,Skill3questionswilltestyourabilitytoreasonaboutthedesignandexecutionofresearchinthelifesciencesandbehavioralsciences.However,itisSkill4questionsthatwilltestyourabilitytounderstandanddrawconclusionsusingthedatacollectedbyresearch.

6.1WhatIsSkill4?Academicpapersareextremelypredictable.Aresearchpapergenerallystartswithanabstract—afewshort paragraphs reflecting themajor points of the rest of the paper. The authors then provide anexpanded introduction, materials and methods, data, and discussion. The key to a high-qualityresearchpaper ismaking thisdiscussionunnecessary–anyscientists,whengiven thepriorsections,shouldbeledtothesameconclusionsasthosegivenbytheauthor.Thetestmakersarekeenlyawareof this fact.OnTestDay, youmaybepresentedwith research in the formof an experiment-basedpassage,andpartofyourtaskwillbeinferringtheimportantconclusionsthatcanbesupportedbythefindingsofthestudy.

Skill 4 questions will test your ability to draw conclusions using both raw data and graphicalrepresentationsofdata.Skill4canbedividedintotwomaincomponents:interpretationofpatternsintables,figures,andgraphs;anddrawingconclusionsfromthedatapresented.However,youwillbeexpected to use statistical reasoning skills in order to draw conclusions. While identification ofpatternsisimportant,youmustalsobeabletodetermineifthesepatternsarestatisticallysignificant.

SKILL4QUESTIONSTEST:•Use,analysis,andinterpretationofdataintables,graphs,andfigures•Determinationof themosteffectivewaytorepresentdataforspecificscientificobservationsanddatasets• Use of central tendency (mean, median, and mode) and dispersion measures (range,interquartilerange,andstandarddeviation)todescribedata•Identificationandreasoningregardingrandomandsystematicerror• Determination of the statistical significance and uncertainty of a data set using statisticalsignificancelevelsandconfidenceintervals•Identificationofrelationshipsandexplanationsofthoserelationshipsusingdata•Predictionofoutcomesusingdata•Abilitytodrawconclusionsandanswerresearchquestionsusingdata

Page 67: Edited By Deeangelee Pooran-Kublall, MD/MPH · 9.5 Preparing for the MCAT: Biochemistry in the Chemical and Physical Foundations of Biological Systems Section 9.6 Preparing for the

6.2FundamentalConceptsofSkill4Manystudentstakebiostatisticsorsomeotherstatisticsclassinpreparationforamajorinbiologyorthelifesciences.However,manystudentswhotaketheMCAThavenottakenastatisticscourse.Thus,thebasicinformationrequiredtoanswerthesequestionsisbrieflycoveredinthissection.

MEASURESOFCENTRALTENDENCYMeasuresofcentraltendencydescribeacentralvaluearoundwhichtheothervaluesinthedatasetareclustered.However,thiscentralvaluecanbedescribedinmultipleways,includingthemean,median,andmode.

MeanThemeanoraverageofasetofdataiscalculatedbyaddinguptheindividualvalueswithinthedataset and dividing the result by the number of values.Mean values are a good indicator of centraltendency when all of the values tend to be fairly close to one another. Having an outlier, or anextremely large or extremely small value compared to the other data values, can shift the meantowardoneendortheother.

MedianThemedianvalueforasetofdataisitsmidpoint,wherehalfofthedatapointsaregreaterthanthevalueandhalfaresmaller.Indatasetswithanoddnumberofvalues,themedianwillactuallybeoneofthedatapoints.Indatasetswithanevennumberofvalues,themedianwillbethemeanofthetwocentral datapoints.Tocalculate themedian, adata setmust first be listed in increasingorder.ThemedianpositioncanbecalculatedasshowninEquation6.1:

Equation6.1

wherenisthenumberofdatavalues.Inadatasetwithanevennumberofdatapoints,thisequationwillsolveforanumbersuchas9.5.Thismeansthatthemedianliesbetweentheninthandtenthvalueofthedataset,andistheaverageoftheninthandtenthvalues.

Themediantendstobetheleastsusceptibletooutliers,butmaynotbeusefulfordatasetswithverylargeranges(thedistancebetweenthelargestandsmallestdatapoints)ormultiplemodes.

If themean andmedian are far fromeachother, this implies thepresenceof outliers or a skeweddistribution,asdiscussedlaterinthischapter.Ifthemeanandthemedianareveryclose,thisimpliesasymmetricaldistribution.

ModeThemode,quitesimply, is thenumber thatappearsmostofteninadataset.Theremaybemultiplemodes inadata set,or—ifallnumbersappearequally—therecanevenbenomode for adata set.When we examine distributions, the peaks represent modes. The mode is not typically used as ameasure of central tendency for a data set, but the number ofmodes, and the distancebetween the

Page 68: Edited By Deeangelee Pooran-Kublall, MD/MPH · 9.5 Preparing for the MCAT: Biochemistry in the Chemical and Physical Foundations of Biological Systems Section 9.6 Preparing for the

modes,isofteninformative.Ifadatasethastwomodeswithasmallnumberofvaluesbetweenthemodes,itmaybeusefultoanalyzetheseportionsseparatelyortolookforconfoundingvariablesthatmayberesponsiblefordividingthedistributionintotwoparts.

DISTRIBUTIONSOftenasinglestatisticforadatasetisinsufficientforadetailedorrelevantanalysis.Inthiscase,itisusefultolookattheoverallshapeofthedistributionaswellasspecificsabouthowthatshapeimpactsour interpretation of data. The shape of a distribution will impact all of the measures of centraltendency,aswellassomemeasuresofdistribution.

NormalDistributionsInstatistics,normaldistributionsarethemostcommon.Evenwhenweknowthatthisisnotquitethecase,wecanusespecial techniquesso thatourdatawillapproximateanormaldistribution.This isveryimportantbecausethenormaldistributionhasbeen“solved”inthesensethatwecantransformanynormaldistribution to a standarddistributionwith ameanof zero and a standarddeviationofone,andthenusethenewlygeneratedcurvetoget informationaboutprobabilityorpercentagesofpopulations.ThenormaldistributionisalsothebasisforthebellcurveseeninmanyscenariosandinFigure6.1below,includingexamscoresontheMCAT.

Figure6.1ThenormaldistributionThemean,median,andmodeareatthecenterofthedistribution.Approximately68%ofthedistributioniswithinonestandarddeviationofthemean,95%withintwo,

and99%withinthree.

SkewedDistributionsDistributionsarenotalwayssymmetrical.Askeweddistributionisonethatcontainsatailononesideortheotherofthedataset(seeFigure6.2).OntheMCAT,skeweddistributionsaremostoftentestedby identificationof the typeof skeweddistribution.This isoftenanareaofconfusion for studentsbecause thevisual shift in thedataappearsopposite thedirectionof the skew.Anegatively skeweddistributionhasatailontheleft(ornegative)side,whereasapositivelyskeweddistributionhasatailontheright(orpositiveside).Insummary,usethetail(notthepeak)todeterminethedirectionoftheskew.

Because themeanismoresusceptible tooutliers than themedian, themeanofanegativelyskeweddistributionwillbelowerthanthemedian,whilethemeanofapositivelyskeweddistributionwillbe

Page 69: Edited By Deeangelee Pooran-Kublall, MD/MPH · 9.5 Preparing for the MCAT: Biochemistry in the Chemical and Physical Foundations of Biological Systems Section 9.6 Preparing for the

higherthanthemedian.

Figure6.2Skeweddistributions(a)Negativelyskeweddistribution,withmeanlowerthanmedian;(b)Positivelyskeweddistribution,withmeanhigherthanmedian.

BimodalDistributionsSome distributions have two ormore peaks. A distribution containing two peaks with a valley inbetweeniscalledbimodal(seeFigure6.3). It is important tonote thatabimodaldistributionmighthave only one mode if one peak is slightly higher than the other. The presence of two peaks ofdifferentsizesdoesnotdiscountadistributionfrombeingconsideredbimodal.Ifthereissufficientseparationofthetwopeaks,orasufficientlysmallamountofdatawithinthevalleyregion,bimodaldistributionscanoftenbeanalyzedastwoseparatedistributions.

Figure6.3Bimodaldistribution

MEASURESOFDISTRIBUTIONDistributionscanalsobecharacterizedbydistancebetweenthehighestandlowestvalues,aswellasdistancefromthemeanvalue.

RangeRange is an absolutemeasureof the spreadof a data set.The rangeof adata set is thedifferencebetweenthehighestandlowestvalues.Rangedoesnotconsiderthenumberofitemsinthedataset,nordoesitconsidertheplacementofanymeasuresofcentraltendency.Therefore,rangeisheavilyaffectedby the presenceof data outliers. In caseswhere it is not possible to calculate the standard

Page 70: Edited By Deeangelee Pooran-Kublall, MD/MPH · 9.5 Preparing for the MCAT: Biochemistry in the Chemical and Physical Foundations of Biological Systems Section 9.6 Preparing for the

deviation for a normal distribution because the entire data set is not provided, it is possible toapproximatethestandarddeviationasone-fourthoftherange.

StandardDeviationStandard deviation is the most informative measure of distribution, but it is also the mostmathematicallylaborious.It iscalculatedrelativetothemeanofthedata.OnTestDay,youmaybeasked to calculate a standard deviation.However, it ismore likely that youwill have to apply theconceptofthestandarddeviationtoidentifyinganoutlier.Inaddition,standarddeviationcanalsobeusedtodescribethedistancefromthemeanofaparticularvalue.

•Sixty-eightpercentofthedatapointsfallwithinonestandarddeviationofthemean.•Ninety-fivepercentofthedatapointsfallwithintwostandarddeviationsofthemean.•Ninety-ninepercentofthedatapointsfallwithinthreestandarddeviationsofthemean.•Valuesthataremorethanfourstandarddeviationsfromthemeancareconsideredoutliers.

OutliersOutlierstypicallyresultfromoneofthreecauses:

•Atruestatisticalanomaly•Ameasurementerror•Adistributionthatisnotapproximatedbythenormaldistribution

Whenanoutlierisfound,itshouldtriggeranautomaticinvestigationtodeterminewhichofthethreecausesapplies.ItislikelythatyouwillbeaskedtodeterminethemostlikelycauseofanoutlieronTestDay.Ifthereisameasurementerror,thedatapointshouldbeexcludedfromanalysis.However,theothertwosituationsarelessclear.

Ifanoutlieristheresultofatruemeasurement,butitisnotrepresentativeofthepopulation,itmaybeweighted to reflect its rarity, including normally, or excluded from the analysis depending on thepurposeofthestudyandpreselectedprotocols.Thedecisionshouldbemadebeforeastudybegins—not once an outlier has been found. When outliers are an indication that a data set may notapproximatethenormaldistribution,repeatedsamplesorlargersampleswillgenerallydemonstrateifthisistrue.

STATISTICALTESTINGHypothesistestingandconfidenceintervalsallowustodrawconclusionsaboutpopulationsbasedonoursampledata.Bothareinterpretedinthecontextofprobabilitiesandwhatwedeemanacceptableriskoferror.

HypothesisTestingHypothesistestingbeginswithanideaaboutwhatmaybedifferentbetweentwopopulations.Wehaveanull hypothesis,which is always ahypothesisof equivalence. Inotherwords, thenull hypothesissaysthattwopopulationsareequal,orthatasinglepopulationcanbedescribedbyaparameterequaltoagivenvalue.Thealternativehypothesismaybenondirectional(sayingthatthepopulationsarenotequal)ordirectional.

Page 71: Edited By Deeangelee Pooran-Kublall, MD/MPH · 9.5 Preparing for the MCAT: Biochemistry in the Chemical and Physical Foundations of Biological Systems Section 9.6 Preparing for the

Themost commonhypothesis tests are z- or t-tests,which rely on the standard distribution or thecloselyrelatedt-distribution.Fromthedatacollected,ateststatisticiscalculatedandcomparedtoatable to determine the likelihood that the statistic was obtained by random chance (under theassumptionthatournullhypothesis is true).This isourp-value.Wethencompareourp-value toasignificancelevel(α);0.05iscommonlyused.Foradirectionaltest,ifthep-valueisgreaterthanα,then we fail to reject the null hypothesis, which means that there is not a statistically significantdifferencebetweenthetwopopulations.Ifthep-valueislessthanα,thenwerejectthenullhypothesisand state that there is a difference between the two groups. If the alternative hypothesis is notdirectional,wecomparep-valueto instead.Again,whenthenullhypothesisisrejected,westatethatourresultsarestatisticallysignificant.

The value of α is a level of risk that we are willing to accept for incorrectly rejecting the nullhypothesis.ThisisalsocalledatypeIerror.Inotherwords,atypeIerroristhelikelihoodthatwereportadifferencebetweentwopopulationswhenonedoesnotactuallyexist.AtypeIIerroroccurswhenweincorrectlyfailtorejectthenullhypothesis.Inotherwords,atypeIIerroristhelikelihoodthatwereportnodifferencebetweentwopopulationswhenoneactuallyexists.Theprobabilityofatype II error is sometimes symbolized by β. The probability of correctly rejecting a false nullhypothesis(reportingadifferencebetweentwopopulationswhenoneactuallyexists)isreferredtoaspower,andisequalto1–β.Finally,theprobabilityofcorrectlyfailingtorejectatruenullhypothesis(reporting no difference between two populations when one does not exist) is referred to asconfidence.Table6.1summarizesallofthesescenariosforyourreview.

TruthAboutthePopulation

H0true(nodifference)

Hatrue(differenceexists)

ConclusionBasedonSample

RejectH0

TypeIerror(α)

Power(1–β)

FailtorejectH0

Confidence

TypeIIerror(β)

Table6.1Resultsofhypothesistesting

ConfidenceIntervalsConfidenceintervalsareessentiallythereverseofhypothesistesting.Withaconfidenceinterval,wedeterminearangeofvaluesfromthesamplemeanandstandarddeviation.Ratherthanfindingap-value,webeginwith a desired confidence level (95percent is standard) anduse a table to find itscorrespondingz-ort-score.Whenwemultiplythez-ort-scorebythestandarddeviation,andthenadd or subtract this number from themean,we create a range of values. For example, consider a

Page 72: Edited By Deeangelee Pooran-Kublall, MD/MPH · 9.5 Preparing for the MCAT: Biochemistry in the Chemical and Physical Foundations of Biological Systems Section 9.6 Preparing for the

populationforwhichwewishtoknowthemeanage.Wedrawasamplefromthatpopulationandfindthat themeanageofthesampleis30,withastandarddeviationof3.Ifwewishtohave95percentconfidence, the corresponding z-score (which would be provided on Test Day) is 1.96. Thus, therangeis30−(3)(1.96)=24.12to35.88.Wecanthenreportthatweare95percentconfidentthatthetruemeanageofthepopulationfromwhichthissampleisdrawnisbetween24.12and35.88.

APPLYINGDATAInanacademicpaper,thediscussionportioniswherethedatagatheredandinterpretedisappliedtotheoriginalproblem.Wecan thenbegindrawingconclusionsandcreatingnewquestionsbasedonourresults.

CorrelationandCausationCorrelationreferstoaconnection—directrelationship,inverserelationship,orotherwise—betweendata.Correlationdoesnotnecessarilyimplycausation;wemustavoidthisassumptionwhenthereisinsufficientevidencetodrawsuchaconclusion.Ifanexperimentcannotbeperformed,wemustrelyonHill’scriteria.Remember,theonlyoneofHill’scriteriathatisuniversallyrequiredforcausationistemporality.

IntheContextofScientificKnowledgeWheninterpretingdata, it is importantthatwenotonlystatetheapparentrelationshipbetweendata,butalsobegintodrawconnectionstootherconceptsinscienceandtoourbackgroundknowledge.Ataminimum,theimpactofthenewdatawouldbeintegratedintoallfutureinvestigationsonthetopic.Additionally,wemustdevelopaplausiblerationalefortheresults.Finally,wemustmakedecisionsaboutourdata’simpactontherealworld,anddeterminewhetherornotourevidenceissubstantialandimpactfulenoughtonecessitatechangesinunderstandingorpolicy.

Page 73: Edited By Deeangelee Pooran-Kublall, MD/MPH · 9.5 Preparing for the MCAT: Biochemistry in the Chemical and Physical Foundations of Biological Systems Section 9.6 Preparing for the

6.3HowWillSkill4BeTested?Skill 4 questions are likely to appear in a variety of scenarios onTestDay. This skill lends itselfeasilytoavarietyofscenariosandlevelsofinformation.

•Discretequestions•QuestionsNOTassociatedwithadescriptivepassage.•LikelytoaskfairlysimplequestionsrelatedtoSkill4.•Calculationsofamean,median,mode,orrangearelikelytoappear.•Mayalsoberelatedtofundamentalconcepts,includingtypesoferrororbias.

•Questionsthatstandalonefromthepassage•Verysimilartodiscretequestions.•Likelytobefairlysimpleandnotrequirealargeamountofanalysis.

•Questionsthatrequiredatafromthepassage•Mayrequireanalysisorinterpretationofachart,graph,ortable.•Calculationsmayberequired.•Comparisonsbetweenvariablesmayberequired.

•Questionsthatrequirethegoalofthepassage•Thesequestionsarethemostin-depthofalltheSkill4questiontypes.•Likelytorequireyoutoevaluatedatatodetermineifthedatasupporttheconclusionpresentedinthepassage.•Mayalsoaskyoutodrawconclusionsfromthedataandinformationpresentedinthepassage,requiringintegrationofthepassage,data,andstatisticalanalysis.• Expect that some Skill 4 questions in this category will require you to identifyrelationshipsbetweenvariablesorevaluatethestudyforerrororbiasbasedonyourstatisticalanalysis.

Page 74: Edited By Deeangelee Pooran-Kublall, MD/MPH · 9.5 Preparing for the MCAT: Biochemistry in the Chemical and Physical Foundations of Biological Systems Section 9.6 Preparing for the

6.4GettingtheEdgeinSkill4Skill 4 requires anunderstandingofhow researchdata is evaluated.Thebestway to identifyyourstrengthsandweaknessesinthisareaistoactuallyreadacademicpapers.However,justreadingtheacademicpaperisnotenough.Youhavetogothrougheachcalculationthattheyhaveperformedandinterpretthepaperinlightofyourownstatisticalanalysis.Thinkofyourselflikeateachergradingaresearch paper. Read critically, and identify any flaws in their analysis of data, includingmiscalculations.Finally,afterreadingthepaper,summarizethevalueoftheinformation.Ifyouhadapatientwhosediseaseortreatmentcoursecouldbeaffectedbytheinformationinthepaper,doyoubelievethattheinformationinthepaperisreliable?Isitstatisticallysignificant?Didthediscussioninthepaper actually address thequestions set forth at thebeginningof thepaper?Asyou readmorepapers,thesequestionswillbecomeeasiertoanswer.

Inadditiontoreadingpapers,itisessentialthatyoudoaconsiderablenumberofpracticequestionsinthisarea.Often,passagesthataresteepedinresearchwillalsopresentanumberofSkill3questions.Whilewepresenttheseskillsseparatelyhere,bothareessentialtounderstandingacademicresearchpapers. By seeking out practice passages and question sets that utilize both of these skills, thesequestiontypeswillbecomeeasier,andyouwillbeabletomaximizeyourscoreonTestDay.

When the AAMC released the specifications for the 2015 MCAT, it established the expecteddistributiontobe10%Skill4questionsforeachofthesciencesections.

Page 75: Edited By Deeangelee Pooran-Kublall, MD/MPH · 9.5 Preparing for the MCAT: Biochemistry in the Chemical and Physical Foundations of Biological Systems Section 9.6 Preparing for the

III

ScienceSubjectReview

Page 76: Edited By Deeangelee Pooran-Kublall, MD/MPH · 9.5 Preparing for the MCAT: Biochemistry in the Chemical and Physical Foundations of Biological Systems Section 9.6 Preparing for the

CHAPTERSEVEN

ScienceUnitOverviewNowthatyouaremorecomfortablewiththeMCATandtheKaplanMethodsfortheexam,itistimeforus todive intopracticemode.Thefollowingchapterssummarize themostsalient takeawaysofthe testmaker ’s, or the AAMC’s, content outlines with special emphasis on how you can use theinformationtosucceedonTestDay.

7.1TopicOverviewFor your convenience, each science discipline covered on the exam is presented for your reviewbeforethepractice.Thecontentreviewforeachsubjectincludes:

•Whatisuniqueaboutthattopicontheexam•Howyoushouldexpecttoseethecontentontheexam•HowyoucangettheedgebyusingtheKaplanMethods•Adetailed,annotatedoutlineofthetopicsandsubtopicscovered

Page 77: Edited By Deeangelee Pooran-Kublall, MD/MPH · 9.5 Preparing for the MCAT: Biochemistry in the Chemical and Physical Foundations of Biological Systems Section 9.6 Preparing for the

7.2PassagesasWorkedExamplesAfter the overview, youwill have the opportunity to see exactly how theKaplanMethod can helptackleachallengepassagewithpassagesasworkedexamples.Younotonlyhavetheopportunitytotrythepassageonyourown,butyoualsocanseetheKaplanMethodforquestionsdemonstratedforeachoftheworkedpassages

Page 78: Edited By Deeangelee Pooran-Kublall, MD/MPH · 9.5 Preparing for the MCAT: Biochemistry in the Chemical and Physical Foundations of Biological Systems Section 9.6 Preparing for the

7.3PassagesasPracticeProblemsFollowingthescienceoverviewandworkedexamples,youwillthenhavetheopportunitytopracticetheKaplanMethodsonyourownwithpracticeproblems(inthebookandonline).

In order to get the most out of your practice, be sure to note Question Types, Wrong AnswerPathologies,andCluestotheCorrectAnswer.

Page 79: Edited By Deeangelee Pooran-Kublall, MD/MPH · 9.5 Preparing for the MCAT: Biochemistry in the Chemical and Physical Foundations of Biological Systems Section 9.6 Preparing for the

CHAPTEREIGHT

BehavioralSciencesAsignificantfactorinfluencingapatient’sabilitytohealfromaninjuryormanageachronicdiseaseisthatperson’spsychiatricandsocioeconomicstatus.Inconsideringtreatmentoptionsforapatient,aphysicianmust take into account the patient’s ability to understand and adhere to a treatment plan.Patientswithapsychiatricdisorderorwhoareunabletoaffordtheirprescriptionsarenotlikelytotakemedicationsasprescribed,unlessthetreatmentregimenissimpleandaffordable.Inanefforttoemphasize thepsychological and social aspects ofmedicine, theAAMChas added a section to theMCAT known as the Psychological, Social, andBiological Foundations ofBehavior section. Thissectionwillbeapproximately65percentpsychology,30percentsociology,and5percentbiology.Inaddition,5percentofthepsychologyquestionswillcoverbiologicallyrelevanttopics.

8.1ReadingthePassageLiketheothersections,therewillbetwomainpassagetypes:informationandexperiment.However,thedistinctionisnotasclearasintheothersections.Behavioralsciencepassageswillalmostalwaysdiscussanexperimentorstudy,evenif thepassageseemstobemoreinformational.Manyof thesepassageswillbedesignedtotestyourabilitytoanalyzeexperimentaldataandevaluateexperimentaldesign.Theinformationalaspectsofthepassagewillprovidebackgroundordiscussafundamentalconcept,andtheattachedexperimentorstudywillberelatedtothepassagetopic.Assuch,youwillbeexpected touse thesamecritical readingskillsyouhavebeenusingallalong,but inamuchmoreintegratedway.

PASSAGETYPESAlmostallofthepassageswillcontainanexperiment.However,theexperimentitselfwilloftenbeintheformofalargestudy.Thedatafromthelargestudyismostlikelytobepresentedintheformofachartorgraph indicating theprobabilityofsomeoutcomeor the likelihoodofsomecharacteristicamongthestudypopulation.

However, the early parts of the passage will read like an information passage, much like thebackgroundportionofapeer-reviewedpaperwithinthescientificdisciplines.Thepassagewillthenoften change course to present the hypothesis, variables, and procedure of the study. Finally, theresultswillbepresented.Thus,theearlypartsofthepassagecontaininginformationshouldbetreatedlike an information passage, with reading aimed at getting the gist of the paragraph(s), withoutspending time analyzing the details. When discussion of the study begins, switch to a moreexperiment-based reading strategy, seeking out the hypothesis, procedure, and results. Finally, thechart,graph,ortablewillrequireanalysis,butthisanalysisneednotbedoneuntilaquestionrequiresit.

WhattoExpect:BehavioralSciences•Almostallpassageswillcontainanexperiment,especiallypsychologypassages.• Any passages that do not contain an experiment are more likely to be sociology passages.

Page 80: Edited By Deeangelee Pooran-Kublall, MD/MPH · 9.5 Preparing for the MCAT: Biochemistry in the Chemical and Physical Foundations of Biological Systems Section 9.6 Preparing for the

Thesepassageswillfocusprimarilyonsociologicaltheory.•Manypassageswillblendsubjectmatterandyouwillbeexpectedtounderstandtheconnectionsbetweenbiology,psychology,andsociology.

TheSciencesvs.BehavioralSciences

TheCoreSciences

BehavioralSciences

Topics

Biology,Biochemistry,GeneralChemistry,OrganicChemistry,Physics

Biology,Psychology,Sociology

PassageTypes

Informationandexperiment

Passageswillmainlyconsistofinformationandexperimentportions,butmanypassageswillnotfallneatlyintoeithercategory

Questions

Willrequireavarietyofskillsfrombasicrecallofinformation,applicationofinformationtoanewsituation,analysisofdata,andevaluationofexperimentdesign

Willrequireavarietyofskills,butdataanalysisandevaluationofexperimentdesignwillbeemphasized

OUTLININGTHEPASSAGERegardlessofhowthepassagesinthissectiondifferfromthosefoundintheothersections,thesameKaplanstrategyshouldbeapplied.Readthepassagequicklyandefficiently,andcreateanoutlinetohelpyoulocateinformationwhenyoustartansweringquestions.

ScanforStructure•Notethestructureofthepassage,notingthelocationoftheparagraphsandanyfiguressuchascharts,graphs,tables,ordiagrams.•Determinethetopicanddegreeofdifficulty.•Identifywhetherthispassagewillrequirealargetimeinvestment.•Decidewhetherthispassageisonetodonoworlater.

ReadStrategically• While reading the passage, decide whether the paragraph you are reading is presentinginformationordiscussinganexperiment.•If theparagraphispresentinginformation,readit likeaninformationpassagebygettingthegistoftheparagraphandidentifyingthelocationofdetails.• If the paragraph is describing an experiment, read it like an experiment passage by paying

Page 81: Edited By Deeangelee Pooran-Kublall, MD/MPH · 9.5 Preparing for the MCAT: Biochemistry in the Chemical and Physical Foundations of Biological Systems Section 9.6 Preparing for the

specialattentiontothehypothesis,procedure,andresults.•Identifythelocationofthedata,butdonotbeginyouranalysisuntilaquestionrequiresit.

LabelEachComponent•Forinformationparagraphs,notethepurposeormainideaoftheparagraph.•Ifthereislistofdetails,notehowthedetailsarerelatedtothemainideaoftheparagraph,andmoveon.•Ifanexperimentisdiscussed,notethehypothesis,procedure,andresults.•Notetherelationshipbetweenanyparagraphsandanycharts,graphs,ortables.•Foreachfigure,writedownwhatthefigureisdiscussing.

ReflectonYourOutline•At theendof thepassage,determine thegoalof thepassage.Whywas thispassagewritten?Whatwas theauthor trying todo?Another thing toconsider iswhat the testmaker is trying toachievewithpresentingthispassage.Remember,passagesinthebehavioralsciencessectionaredesignedtotestyourabilitytoanalyzeandapplyconceptsinalogicalfashion.

Page 82: Edited By Deeangelee Pooran-Kublall, MD/MPH · 9.5 Preparing for the MCAT: Biochemistry in the Chemical and Physical Foundations of Biological Systems Section 9.6 Preparing for the

8.2AnsweringtheQuestionsBehavioralsciencesquestionswillseektointegrateconcepts,especiallywithregardtodataanalysisandexperimentaldesign.However,thesamebasicfourquestionswillbepresent.

•Discretequestions•Questionsthatarenotconnectedtoapassage.• Will be preceded by a warning such as “Questions 12–15 are NOT based on adescriptivepassage.”•Oftenwillrequireyoutorecallorapplyabasicconcept,suchasatheoryorprocess.•Notlikelytoevaluateexperimentaldesignordataanalysis.

•Questionsthatstandalonefromthepassage•Questionsthataccompanyapassagebutdonotrequireinformationordatafromthepassagetoidentifythecorrectanswer.•Morelikelytorequireapplicationofabasicconcept,theory,orprocesstoasituationpresentedinthequestionstem.

•Questionsthatrequiredatafromthepassage•Questions that require apieceofdataor information from thepassage,butdonotrequirethegoalofthepassage.•These questions are likely to focus on data analysis or evaluation of experimentalprocedure.•Mayalsofocusontheinformationsectionofthepassage,andrequireyoutoidentifyadetailpresented.

•Questionsthatrequirethegoalofthepassage•Questionsthatrequirethegoalofthepassage,orafundamentalgraspofthepassageasawhole.•Thesequestionsarelikelytofocusontheresultsanddiscussionoftheresultsofanexperiment.•Mayalsoassessyourability toanalyzedataorevaluateanexperimentaldesignbutfromabroaderperspective,suchasachangethatcouldbemadeintheprocedurethatwouldaffecttheresultsinaparticularway.•Mayalsorequireyoutoevaluatetheresultsofanexperimentandapplythoseresultstotheinformationpresentedearlierinthepassage.• Likely to require application of basic concepts and theories to understanding thepassageasawhole.

DETERMININGTHEPURPOSEOFTHEQUESTIONEventhoughthebehavioralsciencespassagesandquestionsaresomewhatdifferentfromthoseseeninthesciences,thequestionsarestilldesignedtotestthesamebasicanalyticalskills.Thus,thesameKaplanquestionstrategyapplies.Inaddition,havingabasicstrategythatyougotoforeachandeveryquestioncombatsanynervousnessyoumayexperience,especiallyasyouencountertopicswithwhichyou are not as comfortable. The Kaplan question strategy provides a framework that aids in theidentificationof thecorrectanswer,preventshastyselectionof incorrectanswers,andcombats testanxiety.

1.AssesstheQuestion

Page 83: Edited By Deeangelee Pooran-Kublall, MD/MPH · 9.5 Preparing for the MCAT: Biochemistry in the Chemical and Physical Foundations of Biological Systems Section 9.6 Preparing for the

•Readthequestion,butavoidreadingtheanswerchoices.•Assessthetopicanddegreeofdifficulty.•Identifythelevelof timeinvolvement: is thisquestionlikelytotakea tremendousamountoftimetoidentifytheanswer?•Makeadecision–isthisquestiononetodonoworlater?• Good questions to do now in the behavioral sciences are those that stand alone from thepassage,asthesearegenerallyquickandeasypoints.

2.PlanYourAttack•Ifyoudecidetodothequestionnow,determineaplan.•Identifythetaskofthequestion.Whatareyoubeingaskedtodo?•Does thequestion require information from thepassage?Howmuch informationandwherecanthatinformationbefound?• Does this question focus on theory presented in the passage or analysis of data and/orexperimentalprocedure?

3.ExecuteYourPlan•Carryouttheplanasdeterminedinthepreviousstep.• Analyze the data, evaluate the experimental design, locate the information required, andconnecttheinformation,data,andexperimentaldesignwiththeinformationyoualreadyknow.

4.AnswertheQuestionbyMatching,Eliminating,orGuessing•Matchyouranswerwiththecorrectanswerchoice.•Ifthereisnoclearmatch,proceedtoeliminationofincorrectanswers.IncorrectanswersinthissectionwillresemblethoseinthesciencesandthoseinCARS.Someoftheanswerchoicesmaybeextremewithwordssuchasnever,always,andonly.Otherincorrectanswerchoiceswillnotmakesense,runningcountertoyourknowledgebase.• If you cannot come to a clear answer, get down to two probable answer choices.Make aneducatedguessbyatleasthavingareasonforchoosingananswerchoice.Yourreasonneednotbeconcreteorirrefutable,justareasonwhythatoneseemsmorecorrect.•Markyouranswerandmoveontothenextquestion.

Page 84: Edited By Deeangelee Pooran-Kublall, MD/MPH · 9.5 Preparing for the MCAT: Biochemistry in the Chemical and Physical Foundations of Biological Systems Section 9.6 Preparing for the

8.3GettingtheEdgeinBehavioralSciencesEarningahighscoreonTestDayinbehavioralsciencesrequirestheabilitytoreasonandapplywhatyoualreadyknowtonewsituations.Thisrequiresaleveloflearningthatisoftendifferentfromthatrequiredbyundergraduatecourses.Notonlydoyouneedtounderstandthedefinitionsandtheories,but you also need to know the real world applications of those theories. Getting the edge inbehavioral sciences requires practice byworking through a significant number of questions in thetopicinordertograspthelevelofthinkingrequiredforthistopic.

Inaddition,somelevelofflexibilityinapproachingthepassageswillberequired.Inthesciences,thepassageswillfallneatlyintotwocategories:experimentandinformation.Inthebehavioralsciences,thisdistinctionisnolongerpossibleonmanypassages.Thefirsthalfofmanypassageswillreadlikean information passage, and the second halfwill present an experiment related to the informationpresentedearlierinthepassage.Thisforcesyou,asthetest-taker,tohandlebothaspectswithinonepassage.Inyouroutline,noteiftheparagraphisrelatedtoinformationorexperiment—thiswillhelpyoufindtheinformationyouneedinatimelymanner.

Identificationofthetaskofthequestionwillensurethatyoulookforinformationinthecorrectplaceinthepassage.Somequestionswillrequiredataanalysisandevaluationofexperimentalprocedure,meaningthattheexperimentportionofthepassagewillberequired.Moretheoreticalquestionsarelikely to require the information portion. However, this may vary depending on the task of thequestion.Thetaskofthequestionwillguideyoutothecorrectlocationinthepassage.

Page 85: Edited By Deeangelee Pooran-Kublall, MD/MPH · 9.5 Preparing for the MCAT: Biochemistry in the Chemical and Physical Foundations of Biological Systems Section 9.6 Preparing for the

8.4Step-by-StepGuidetoBehavioralSciencesPassages

OUTLININGTHEPASSAGE

•Scanforstructure•Determinewhethertodothispassagenoworlater.•Identifythestructureofthepassage,includingcharts,graphs,tables,ordiagrams.

•Readstrategically• Identify theportionsof thepassage thataremore related to information,and thosethatdescribeanexperiment.•Identifythetopicareaofthepassage:sociologyorpsychology.•Payspecialattentiontotherelationshipsbetweenconcepts.•Determinethehypothesis,procedure,andresultsoftheexperiment.•Identifywhichinformationispresentedineachtable,graph,orchart.

•Labeleachcomponent•Writedownthepurposeofeachparagraphandfigure.• Note which portions of the passage are information and which are related to anexperimentorstudy.•Identifyanyconnectionsbetweenconceptswithinthepassage.

•Reflectonyouroutline•Determinethegoalofthepassageandwriteitdown.•Identifytheconceptswithinthepassageinanefforttoanticipatequestions.•Anticipatethetypeofanalysisthatmayberequired.

ANSWERINGTHEQUESTIONS

1.Assessthequestion•Determinewhetherthisquestionshouldbedonenoworlater.•Identifythetopicandthedegreeofdifficulty.•Goodquestionstodonowinbehavioralsciencesarethosethatdonotrequirethepassageordonotrequireextensiveanalysis.

2.Planyourattack•Determinewhatyoualreadyknow,andwhatinformationyouneed.•Identifythetaskofthequestion.•Identifywheretofindtherequiredinformation:thepassage,thequestion,youroutline,oryourownknowledge.• If youhave to goback to the passage, determinewhere to find the required informationbyusingyouroutline.• If data analysis is required, identify the correct data set, as there may be multiple datarepresentations.

Page 86: Edited By Deeangelee Pooran-Kublall, MD/MPH · 9.5 Preparing for the MCAT: Biochemistry in the Chemical and Physical Foundations of Biological Systems Section 9.6 Preparing for the

3.Executeyourplan•Analyzethedata,gobacktothepassage,andcarryoutyourplan.•Ifyougetstuckanalyzingdata,rememberthatthetrendofthedataisoftenenoughtoyieldacorrectanswerchoice.

4.Answerthequestionbymatching,eliminating,orguessing•Matchyouranswertotheanswerchoices.•Ifthereisnotmatch,eliminateincorrectanswerchoices.Someoftheanswerchoicesmaynotmakesense;eliminatethosefirst.•Ifeliminationdoesnotprovideaclearanswer,guessbetweentwoprobableanswers.

Page 87: Edited By Deeangelee Pooran-Kublall, MD/MPH · 9.5 Preparing for the MCAT: Biochemistry in the Chemical and Physical Foundations of Biological Systems Section 9.6 Preparing for the

8.5PreparingfortheMCAT:PsychologyThesearethepsychologytopicsyouarelikelytoseeonTestDay.

SENSORYPROCESSING• Familiarity with the fundamentals of sensation, including thresholds, Weber ’s law, signaldetectiontheory,andsensoryadaptation,andpsychophysics• Knowing how sensory information is received via sensory receptors, including sensorypathwaysandtypesofsensoryreceptors

VISION•Recognizingthestructureandfunctionoftheeye•Processingofvisualinformation,includingvisualpathwaysinthebrain,parallelprocessing,andfeaturedetection

HEARING•Recognizingthestructureandfunctionoftheear•Processingofauditoryinformation,includingauditorypathwaysinthebrain•Understandingtheroleofhaircellsinsensoryreception

OTHERSENSES•Perceptionofthebody(somatosensation)andpainperception•Taste,includingtastebudsaschemoreceptorsthatdetectspecificchemicals• Smell, including the olfactory cells/chemoreceptors that detect specific chemicals,pheromones,andolfactorypathwaysinthebrain•Determininghowthebrainsensesthelocationofthebodyinspace,includingkinestheticsenseandvestibularsense

PERCEPTION•Processingofinformation,includingbottom-upandtop-downprocessing•Organizationofperception,includingdepth,form,motion,andconstancy•ExplainingGestaltprinciples

ATTENTION•Recallingthedefinitionsofattention,includingselectiveattentionanddividedattention

COGNITION•Recallingthedefinitionandcharacteristicsoftheinformation-processingmodel

Page 88: Edited By Deeangelee Pooran-Kublall, MD/MPH · 9.5 Preparing for the MCAT: Biochemistry in the Chemical and Physical Foundations of Biological Systems Section 9.6 Preparing for the

• Knowing fundamental concepts of cognitive development, including Piaget’s stages ofcognitive development, cognitive changes in late adulthood, role of culture in cognitivedevelopment,andtheinfluenceofheredityandenvironmentoncognitivedevelopment•Identifyingbiologicalfactorsthataffectcognition•Modelsofproblem-solvinganddecision-making,includingtypesofproblem-solving,barrierstoeffectiveproblem-solving,andapproachestoproblem-solving• Fundamental concepts of problem-solving including heuristics and biases, biases, includingoverconfidenceandbeliefperseverance•Fundamental conceptsof intellectual functioning, includingmultiple theoriesof intelligence,influenceofheredityandenvironmentonintelligence,andvariationsinintellectualability

CONSCIOUSNESS• Identification of various states of consciousness, including alertness, sleep, hypnosis, andmeditation• Fundamental concepts of sleep, including stages of sleep, sleep cycles and changes to sleepcycles,sleepandcircadianrhythms,dreaming,andsleep-wakedisorders• Definition of a consciousness-altering drug as well as the types of consciousness-alteringdrugsandtheireffectsonthenervoussystemandbehavior,andthedrugaddictionandrewardpathwayinthebrain

MEMORY•Memory creation; the process of encoding information and processes that aid in encodinginformation• Memory storage; types of memory storage, including sensory, working, and long-termmemory;semanticnetworksandspreadingactivation•Memory retrieval; recall, recognition, and relearning; retrieval clues; and theprocesses thataidretrievalandtheroleofemotion•Memoryloss,agingandmemory,dysfunctionsofmemory(suchasAlzheimer ’sdiseaseandKorsakoff’ssyndrome),decay,interference,memoryconstruction,andsourcemonitoring• Definition of neural plasticity as a change in synaptic connections that aid in memory andlearning;long-termpotentiation

LANGUAGE•Languageacquisitiontheories,includinglearning,Nativist,andInteractionist•Theinfluenceoflanguageoncognitiveprocesses•Identificationthatlanguageandspeecharecontrolledbydifferentbrainareas

EMOTION•Identificationofcognitive,psychological,andbehavioral• Emotions that are universal to all humans, including fear, anger, happiness, surprise, joy,disgust,andsadness•Roleofemotionasanadaptivestrategy

Page 89: Edited By Deeangelee Pooran-Kublall, MD/MPH · 9.5 Preparing for the MCAT: Biochemistry in the Chemical and Physical Foundations of Biological Systems Section 9.6 Preparing for the

•Fundamentaltheoryofemotions,includingtheJames–Langetheory,Cannon–Bardtheory,andSchachter–Singertheory•Theroleofbiologicalprocessesinperceptionofemotion•Brainregionsinvolvedinthegenerationandexperienceofemotions•Identificationoftheroleplayedbythelimbicsysteminemotion• The nature of emotional experience as stored memories that may be recalled by similarexperiences•Theinteractionbetweenemotionandtheautonomicnervoussystem•Identificationoftherelationshipbetweenphysiologyandemotion,includingthephysiologicalmarkersofemotion(signaturesofemotion)

STRESS•Identificationofthenatureofstress,includingappraisal,differenttypesofstressors(includingcataclysmicandpersonalevents)andtheeffectsofstressonpsychologicalfunction•Physiological,emotional,andbehavioraloutcomesandresponsestostress•Strategiesformanagingstress,includingexercise,relaxationtechniques,andspirituality

BIOLOGICALBASESOFBEHAVIOR•General structure and function of the nervous system, including neurons, the reflex arc andneurotransmitters•Structuresandfunctionoftheperipheralnervoussystem• Structure and functions of the central nervous system including the forebrain, midbrain,hindbrain,andspinalcord•Lateralizationofcorticalfunctionstospecifichemispheres•Methodsusedtostudythebrain•Theinteractionandinfluenceofneuronsonbehavior•Influenceofneurotransmittersonbehavior•Therelationshipbetweentheendocrinesystemandthenervoussystem,includingcomponentsoftheendocrinesystemandtheeffectsoftheendocrinesystemonbehavior•Fundamentalconceptsofbehavioralgenetics,includinggenes,temperament,heredity,adaptivevalueoftraitsandbehaviors,andtheinteractionbetweenheredityandenvironmentalinfluences• The influence of genetics and environmental factors on the development of behaviors,including experience and behavior, and genetically based behavioral variation in naturalpopulations• Processes of human physiological development, including prenatal development, motordevelopment,anddevelopmentalchangesinadolescence

PERSONALITY• Fundamental theories of personality, including psychoanalytic, humanistic, trait, socialcognitive,biological,andbehaviorist•Situationalapproachtoexplainingbehavior

Page 90: Edited By Deeangelee Pooran-Kublall, MD/MPH · 9.5 Preparing for the MCAT: Biochemistry in the Chemical and Physical Foundations of Biological Systems Section 9.6 Preparing for the

PSYCHOLOGICALDISORDERS• Identification of approaches to psychological disorders, including biomedical andbiopsychosocialapproaches•Classificationandratesofpsychologicaldisorders• Identification of the major psychological disorders including anxiety disorders, obsessive-compulsivedisorder, traumatic and stress-relateddisorders,mood, schizophrenia, dissociativeandpersonalitydisorders• Identification of the role of biology in nervous system disorders, including schizophrenia,depressionAlzheimer ’sdiseaseandParkinson’sdisease

MOTIVATION•Factorsthatinfluencemotivation,includinginstinct,arousal,drives,andneeds•Identificationoftheroleofnegativefeedbacksystemsinmanagingdrives•Theories about the role ofmotivation in human behavior, including drive reduction theory,incentivetheory,andothercognitiveandneed-basedtheories•Theapplicationoftheoriesasawaytounderstandbehaviorssuchassocioculturaleating,sex,anddrugandalcoholuse, includingbiologicalfactors inregulationofmotivationalprocessesandsocioculturalfactorsinregulationofthesemotivationalprocesses

ATTITUDES•Cognitive,affective,andbehavioralcomponentsofattitudes• The interaction between attitudes and behavior, including the processes by which behaviorinfluencesattitudes(foot-in-the-doorphenomenon,role-playingeffects),theprocessesbywhichattitudesbehaviorinfluencebehavior,andcognitivedissonancetheory

HOWTHEPRESENCEOFOTHERSAFFECTSINDIVIDUALBEHAVIOR•Definitionandsignificanceofsocialfacilitation•Definitionandsignificanceofdeindividuation•Definitionandoutcomeofthebystandereffect•Definitionofsocialloafing•Definitionofsocialcontrol•Definitionandimpactofpeerpressure•Definitionandimpactofconformity•Definitionofobedience

GROUPDECISION-MAKINGPROCESSES•Definitionsandsignificanceofgrouppolarization

NORMATIVEANDNONNORMATIVEBEHAVIOR•Descriptionofsocialnorms

Page 91: Edited By Deeangelee Pooran-Kublall, MD/MPH · 9.5 Preparing for the MCAT: Biochemistry in the Chemical and Physical Foundations of Biological Systems Section 9.6 Preparing for the

SOCIALIZATION• Identification of the agents of socialization such as family, mass media, peers, and theworkplace

HABITUATIONANDDISHABITUATION•Fundamentalconceptsofhabituationanddishabituation

ASSOCIATIVELEARNING• Fundamental concepts of classical conditioning, including neutral, conditioned, andunconditioned stimuli; conditioned and unconditioned responses; processes of conditioning,includingacquisition,extinction,spontaneousrecovery,generalization,anddiscrimination•Fundamentalconceptsofoperantconditioning,includingprocessesofshapingandextinction;types of reinforcement, including positive, negative, primary, and conditional; reinforcementschedules, including fixed-ratio, variable-ratio, fixed-interval, and variable-interval;punishment;andescapeandavoidancelearning•Impactandidentityofcognitiveprocessesthataffectassociativelearning•Impactandidentityofbiological factors thataffectassociative learning, including instinctivedriftandbiologicalpredispositions

OBSERVATIONALLEARNING•Modelingasmodeoflearning•Impactofbiologicalprocessesonlearning,includingmirrorneuronsandtheroleofthebraininexperiencingvicariousemotions•Explanationofindividualbehaviorbyapplicationofobservationallearningprinciples

THEORIESOFATTITUDEANDBEHAVIORCHANGE• Concept of the elaboration likelihood model, including information processing routes topersuasion(centralandperipheralrouteprocessing)•Fundamentalconceptsofsocialcognitivetheory•Factorsthatcontributetoachangeinattitude,includingchangingbehavior,characteristicsofthemessageandtarget,andsocialfactors

SELF-CONCEPTANDIDENTITY• Fundamental factors in self-concept and self-identity, including the role of self-esteem, self-efficacy,andlocusofcontrol•Recognizing thedifferent typesof identitywith respect to race/ethnicity, gender, age, sexualorientation,andclass

FORMATIONOFIDENTITY

Page 92: Edited By Deeangelee Pooran-Kublall, MD/MPH · 9.5 Preparing for the MCAT: Biochemistry in the Chemical and Physical Foundations of Biological Systems Section 9.6 Preparing for the

• The process and stages of identity development, including theories of identity development(suchasgender,moral,psychosexual,andsocialdevelopment)•Recognizingtheinfluenceofsocialfactorsonidentityformation,specificallytheinfluenceofindividualsandgroups,respectively•Understandingtheinfluenceofcultureandsocializationonidentityformation

ATTRIBUTINGBEHAVIORTOPERSONSORSITUATIONS•Conceptsofattributiontheory,includingfundamentalattributionerrorandhowcultureaffectsattributions•Identificationofhowself-perceptionsaffecttheperceptionsofothers•Identificationofhowperceptionsoftheenvironmentshapeourperceptionsofothers

PREJUDICEANDBIAS•Delineationofprocessesthatcontributetoprejudice,suchastheroleofemotionandcognition•Definitionofstereotypesandstigma,

PROCESSESRELATEDTOSTEREOTYPES•Definitionandimpactofself-fulfillingprophecy•Definitionofstereotypethreat

SELF-PRESENTATION• Expression and detection of emotion, including the roles of gender and culture on theexpressionofemotion•Presentationofself,includingimpressionmanagementandthedramaturgicalapproach•Fundamentalconceptsofverbalandnonverbalcommunication•Identificationofhowanimalssignalandcommunicate

SOCIALBEHAVIOR•Fundamentalconceptsofattraction,aggression,attachment,altruism,andsocialsupport•Theroleofbiology in thesocialbehaviorsofanimals, includingforagingbehavior,matingbehaviorandmatingchoice,andaltruism

Page 93: Edited By Deeangelee Pooran-Kublall, MD/MPH · 9.5 Preparing for the MCAT: Biochemistry in the Chemical and Physical Foundations of Biological Systems Section 9.6 Preparing for the

8.6PreparingfortheMCAT:SociologyWhilesomeof thesociological topicsdescribedpreviouslyare integratedwith theirpsychologicalcounterparts,thesearethesociologytopicsyouarelikelytoseeonTestDayinisolationaswell:

HOWTHEPRESENCEOFOTHERSAFFECTSINDIVIDUALBEHAVIOR•Therolesplayedbysocialcontrol,peerpressure,conformity,andobedience

GROUPDECISION-MAKINGPROCESSES•Theroleplayedbygroupthink

NORMATIVEANDNON-NORMATIVEBEHAVIOR•Theroleofsocialnorms,includingsanctions,folkways,mores,taboos,andanomie• The role of deviance, including perspectives on deviance, such as differential association,labelingtheory,andstraintheory•Theaspectsofcollectivebehavior,suchasfads,riots,andmasshysteria

SOCIALIZATION•Theagentsofsocialization,includingfamilies,massmedia,peers,andworkplaces

SOCIALBEHAVIOR•Theroleofsocialsupportinsocialbehavior

DISCRIMINATION•Comparisonofindividualvs.institutionaldiscrimination•Relationshipbetweenprejudiceanddiscrimination•Theeffectofpower,prestige,andclassonfacilitatingdiscrimination

SELF-CONCEPTANDIDENTITY•Definitionsofself-concept,identity,andsocialidentity• Identificationof thedifferent typesof identities, including race/ethnicity,gender, age, sexualorientation,andclass

FORMATIONOFIDENTITY• Theories of identity development, including gender, moral, psychosexual, and socialdevelopment• The role of social factors on identity formation, including the influences of individuals

Page 94: Edited By Deeangelee Pooran-Kublall, MD/MPH · 9.5 Preparing for the MCAT: Biochemistry in the Chemical and Physical Foundations of Biological Systems Section 9.6 Preparing for the

(imitation,role-taking)andtheinfluencesofgroups(referencegroup)•Theroleofcultureandsocializationonidentityformation

PREJUDICEANDBIAS•Definitionofprejudice•Identificationoftheprocessesthatcontributetoprejudice,includingpower,prestige,andclass•Definitionandidentificationofstereotypesandstigmas• Definition and role of ethnocentrism in prejudice and bias, including in-group/out-group;ethnocentrismvs.culturalrelativism

ELEMENTSOFSOCIALINTERACTION•Thetypesofstatus,suchasachievedandascribedstatus•Theeffectsofroles,includingroleconflict,rolestrain,androleexit•Theeffectsofgroups,includingprimaryandsecondarygroups,in-groupsvs.out-groups,andgroupsize(includingdyadsandtriads)•Theeffectsofnetworks•Theeffectsoforganizations,includingformalorganization• The concept of bureaucracies, including characteristics of an ideal bureaucracy andperspectivesonbureaucracy(suchastheironlawofoligarchyandMcDonaldization)

SELF-PRESENTATIONANDINTERACTINGWITHOTHERS•Detectingandexpressingemotion,includingtheroleofgenderandculture•Management of impressions in the social context, including front stage vs. back stage self(dramaturgicalapproach)•Verbalandnonverbalcommunication

THEORETICALAPPROACHES• Fundamental characteristics of micro, and macrosociology, functionalism, conflict theory,symbolicinteractionism,socialconstructionism,exchange-rationalchoice,andfeministtheory

SOCIALINSTITUTIONS• The role of education including hidden curriculum, teacher expectancy, and educationalsegregationandstratification• The role of family, including forms of kinship, diversity of family forms, marriage anddivorce,violencewithinfamilies,religion• The role of religion, including the concept of religiosity, types of religious organizations(including churches, sects, and cults), and the role of religion in social change (such asmodernization,secularization,andfundamentalism)•Theroleofgovernmentandtheeconomy,includingpower,authority,anddivisionoflabor•Comparativeeconomicandpoliticalsystems

Page 95: Edited By Deeangelee Pooran-Kublall, MD/MPH · 9.5 Preparing for the MCAT: Biochemistry in the Chemical and Physical Foundations of Biological Systems Section 9.6 Preparing for the

• The role of health and medicine, including medicalization, the “sick role” and the illnessexperience,deliveryofhealthcare,andsocialepidemiology

CULTURE•Identificationandcharacteristicsofmaterialculture• Identification and characteristics of symbolic culture, including language, symbols, values,beliefs,norms,andrituals• The impact of culture lag, culture shock, assimilation, multiculturalism, sub- andcountercultures,massmediaandpopularculture,andthetransmissionanddiffusionofculture

DEMOGRAPHICSTRUCTUREOFSOCIETY• The role of aging, including aging and the life course, aging cohorts, and its socialsignificance•Theroleofgender,includingthedistinctionbetweensexandgender,thesocialconstructionofgender,andgendersegregation•Theroleofraceandethnicity, includingsocialconstructionofrace,racialization,andracialformation•Theconceptofpushandpullfactorsinmigration•Theconceptofimmigrationstatus,includingpatternsofimmigrationanditsintersectionswithraceandethnicity•Theroleofsexualorientation

DEMOGRAPHICSHIFTSANDSOCIALCHANGES•Theoriesofdemographicchange,includingMalthusiantheoryanddemographictransitions•Theconceptofpopulationgrowthanddecline,includingpopulationprojectionsandpopulationpyramids•Theconceptsoffertilityandmortality,includingratetypesandpatterns•Theconceptofpushandpullfactorsinmigration• The role of social movements, including relative deprivation, their organization, andmovementstrategiesandtactics•The roleofglobalization, includingcontributing factors suchas communication technologyandeconomic interdependence,perspectivesonglobalization,andsocialchangessuchascivilunrestandterrorism

SPATIALINEQUALITY• Identification and impact of residential segregation, including safety and violence inneighborhoods•Impactandroleofenvironmentaljusticeintermsoflocationandexposuretohealthrisks

SOCIALCLASS

Page 96: Edited By Deeangelee Pooran-Kublall, MD/MPH · 9.5 Preparing for the MCAT: Biochemistry in the Chemical and Physical Foundations of Biological Systems Section 9.6 Preparing for the

•Fundamentalaspectsofsocialstratification,includingclass,status,power,culturalandsocialcapital, social reproduction, privilege, prestige, socioeconomic gradients in health and globalinequalities•Relationshipsbetweensocialstratificationandrace,gender,andage•Socialmobilitypatterns,includingintergenerationalandintragenerationalmobility,downwardandupwardmobility,andmeritocracy•Definitions and impact of poverty, including relative and absolute poverty, social exclusion(segregationandisolation)

HEALTHDISPARITIES•Theimpactofrace,gender,andclassinequalitiesonhealth

HEALTHCAREDISPARITIES•Theimpactofrace,gender,andclassinequalitiesonhealthcare

Page 97: Edited By Deeangelee Pooran-Kublall, MD/MPH · 9.5 Preparing for the MCAT: Biochemistry in the Chemical and Physical Foundations of Biological Systems Section 9.6 Preparing for the

8.7BehavioralSciencesWorkedExamplesPASSAGEI:BIOLOGICALBASISOFBEHAVIORThe classic dopamine (DA) hypothesis is the most well-studied and longest-standing of theschizophrenia hypotheses.The hypothesis is centered upon themesolimbic pathwayof the brain—specifically,D2receptorslocatedinthesubcorticalregionofthebrain,whicharestronglyassociatedwith the positive symptoms of schizophrenia (e.g. hallucinations and delusions). Treatment ofschizophrenicpatientswithantipsychoticmedicationsknown toblockD2 receptors alleviates somesymptoms.Furthermore,DA-enhancingdrugsprovedpsychotogenic,thusestablishingtheconnectionbetweendopamineandschizophrenia.

The classic dopamine hypothesis has evolved to explain the enduring symptoms (e.g., apathy andantisocial behavior) and cognitive symptoms (e.g., memory loss and attention deficits). Thesefunctional deficits were only marginally treatable with D2 antagonists. Hypostimulation of D1receptors in the prefrontal cortex (PFC) has been implicated in the negative symptoms ofschizophrenia.

Research also suggests an association between the PFC and the subcortical region of the brain,includingtheventraltegmentalarea(VTA)andtheventralstriatum(VST).Mesolimbicdopaminergic(MLDA)neurons andmesocortical dopaminergic (MCDA)neuronsof theventral tegmental area(VTA)projecttotheventralstriatumandcortex,respectively.GlutamatergicneuronsfromthecortexprojecttotheVTAandupregulatetheMCDAneurons.Atthesametime,otherglutamatergicneuronsindirectlydownregulateMLDAneuronsthroughanintermediateGABAergicneuron.

Researchersconductedanimagingstudytoinvestigatetheinfluenceofglutamateonschizophrenia.Ketamine, a potential NMDA antagonist, was administered to four groups of patients: activeschizophrenics, schizophrenics in remission (no psychotic episode within sixmonths), abusers ofpsychostimulants, and healthy subjects. Dopamine levels weremeasured in the subcortical region.TheresultsofthestudyareillustratedinFigure1.

Figure1.Dopaminemeasurementsinvariousindividualstreatedwith0.5mgketamineperpoundofbodyweight

Page 98: Edited By Deeangelee Pooran-Kublall, MD/MPH · 9.5 Preparing for the MCAT: Biochemistry in the Chemical and Physical Foundations of Biological Systems Section 9.6 Preparing for the

P1.

P2.

P3.

P4.

FI.

1.Ecopipam,anantipsychoticdrug,isadministeredtoahealthypatientwho,followingtreatment,beginshallucinating.Whichofthefollowingcouldcorrectlycharacterizethefunctionofecopipam?

A.ItincreasesactivityoftheMCDAneurons.B.ItincreasesactivityoftheMLDAneurons.C.ItactsasaD2receptorantagonist.

D.ItactsasanNMDAreceptoragonist.

2.PatientswiththehighactivityallelecodingfortheenzymeCOMTexhibitnegativesymptomsofschizophrenia.Whichofthefollowingbestexplainsthisdiscovery?

A.COMTconvertsL-DOPAtodopamineinthePFC.B.COMTconvertsplasmadopaminetodopaminesulphate.C.COMToxidizesdopaminetodopamine-melanin.D.COMTdeactivatesdopaminethroughmethylation.

3.Whichofthefollowingwouldleadtobothpositiveandnegativesymptomsofschizophrenia?A.Administrationofasufficientamountofketamine.B.Administrationofacamprosate,aGABAreceptoragonist.C.Administrationoftalipexole,aD2receptoragonist.

D.Administrationofabenzazepinederivative,aD1receptorantagonist.

4.The“dopaminehypothesis”isusefulfortheclinicaltreatmentofschizophrenia.However,someresearchersbelievethenameismisleading.Whymighttheythinkthis?

A.GABAneuronsactasadeactivatingsystemtokeepsubcorticaldopaminelevelslow.B.Glutamatergicneuronsarelocatedupstreamofthedopaminergicneurons.C.Multipletypesofdopaminergicreceptorsexist,makingthenameambiguous.D.Psychostimulantsexacerbatethepositivesymptomsofschizophrenia.

5.Whichofthefollowingindividualswouldexperiencethemostpronouncednegativesymptomsaftertreatmentwithketamine?

A.Activepsychostimulantabusers

Page 99: Edited By Deeangelee Pooran-Kublall, MD/MPH · 9.5 Preparing for the MCAT: Biochemistry in the Chemical and Physical Foundations of Biological Systems Section 9.6 Preparing for the

B.HealthyindividualsC.SchizophrenicsinremissionD.Itisimpossibletodetermine

Page 100: Edited By Deeangelee Pooran-Kublall, MD/MPH · 9.5 Preparing for the MCAT: Biochemistry in the Chemical and Physical Foundations of Biological Systems Section 9.6 Preparing for the

BehavioralSciencesPassageIExplanation:

USINGTHEKAPLANMETHODS

P1.Dopaminehypothesisofschizophrenia;hyperstimulationofsubcorticalD2receptors

P2.Prefrontalcortex;hypostimulationofD1receptors

P3.LinkbetweenPFCandsubcorticalregionofbrain

P4.Experimentaldesign;ketamineadministration

FI.Dopaminelevelsinketaminetreatedpatients

KeyConceptsMechanismofNeurotransmitterFunction

1.Ecopipam,anantipsychoticdrug,isadministeredtoahealthypatientwho,followingtreatment,beginshallucinating.Whichofthefollowingcouldcorrectlycharacterizethefunctionofecopipam?

A.ItincreasesactivityoftheMCDAneurons.B.ItincreasesactivityoftheMLDAneurons.C.ItactsasaD2receptorantagonist.

D.ItactsasaNMDAreceptoragonist.

AssessthequestionTheantipsychoticpropertiesofecopipamarenotoutlined in thequestionstem,so it isnotagoodideatospeculateaboutitsuse.Inotherwords,itwouldnotbewisetoassumethatecopipamisusedtotreat schizophreniaasopposed toothermental illnesses.Thequestion stemexplains that ecopipaminduceshallucinations.Thispieceofevidence,alongwith information throughout thepassage,willbekeyinansweringthisquestion.

Planyourattack

Page 101: Edited By Deeangelee Pooran-Kublall, MD/MPH · 9.5 Preparing for the MCAT: Biochemistry in the Chemical and Physical Foundations of Biological Systems Section 9.6 Preparing for the

In the first paragraph, it’s implied that hyperstimulation of subcortical D2 receptors is a possiblecauseofhallucinations.Thecorrectanswerwilllikelyaddressthisfact.It’stoughtopredictbeyondthat(andmightbeawasteoftime),sothebestwaytotacklethisquestionisthroughtheprocessofelimination.

ExecutetheplanChoice(A) discussesMCDAneurons.Theseneuronsproject to the cortex,which is largely filledwithD1receptors,notD2receptors.SinceonlyD2receptorsarelinkedtothepositivesymptomsofschizophrenia,(A)isnotcorrect.

Choice (B) mentionsMLDA neurons, which project to the VST (part of the subcortical region).Upregulation of these neurons would increase dopamine concentration in the subcortical region,increasingthelikelihoodofhallucinations.Therefore,(B)islikelythecorrectanswer.

Choice (C) may be tempting because it directly mentions D2 receptors. However, it states thatecopipam is a D2 receptor antagonist. Because antagonists decrease receptor activity, ecopipamwouldreducehallucinations.

Choice (D) states that ecopipam is a NMDA receptor agonist. According to Figure 1, the NMDAantagonist, ketamine, increases dopamine concentration in the subcortical region. Therefore, ifecopipam were a NMDA receptor agonist, it would decrease dopamine concentration in thesubcortical region. A decrease in dopamine concentration would reduce the prevalence ofhallucinations,so(D)isalsoincorrect.

Answerbymatching,eliminating,orguessingTheprocessofeliminationquicklyledtotheselectionofthecorrectanswer,(B).

KeyConceptsReactionsofNeurotransmitters

2.PatientswiththehighactivityallelecodingfortheenzymeCOMTexhibitnegativesymptomsofschizophrenia.Whichofthefollowingbestexplainsthisdiscovery?

A.COMTconvertsL-DOPAtodopamineinthePFC.B.COMTconvertsplasmadopaminetodopaminesulphate.C.COMToxidizesdopaminetodopamine-melanin.D.COMTdeactivatesdopaminethroughmethylation.

Page 102: Edited By Deeangelee Pooran-Kublall, MD/MPH · 9.5 Preparing for the MCAT: Biochemistry in the Chemical and Physical Foundations of Biological Systems Section 9.6 Preparing for the

AssessthequestionNospecificinformationisprovidedaboutthehighactivityalleleofCOMTexceptthatitcausesthenegativesymptomsofschizophrenia.TheanswerchoicesalldescribepossiblefunctionsofCOMT.

PlanyourattackThesecondparagraphelaborateson thenegative symptomsof schizophrenia.Specifically, it statesthatthenegativesymptomsresultfromhypostimulationofD1receptorsintheprefrontalcortex.Theprocessofeliminationistheidealapproachtoansweringthisquestion.

ExecutetheplanChoice(A)statesthatCOMTwillcausetheconversionofL-DOPAtodopamineinthePFC,therebyincreasing dopamine levels. However, negative symptoms arise from hypostimulation of D1receptors,NOThyperstimulation,so(A)isout.

Choice(B)statesthatCOMTconvertsplasmadopaminetodopaminesulphate.Plasmadopamineisintheblood,notthebrain,sothisconversionwouldnotcausethenegativesymptoms.

Choice(C)statesthatCOMTconvertsdopaminetodopamine-melanin.Melaninisthecompoundthatdeterminesskincolor.Themoremelaninpresent,thedarkertheskinbecomes.Sincethisfunctionhasnothingtodowiththebrain,thiscannotbecorrect.

Finally,choice(D)describesamechanismthatisplausibleandwouldresultinhypostimulationofD1receptors.Hence,(D)iscorrect.

Answerbymatching,eliminating,orguessingPredictingthatthecorrectanswermustaddressthehypostimulationofD1receptorsintheprefrontalcortexallowedforefficienteliminationofanswerchoicesthatwereeitheroutofscopeoropposite.

KeyConceptsFunctionsofNeurotransmittersr

3.Whichofthefollowingwouldleadtobothpositiveandnegativesymptomsofschizophrenia?

Page 103: Edited By Deeangelee Pooran-Kublall, MD/MPH · 9.5 Preparing for the MCAT: Biochemistry in the Chemical and Physical Foundations of Biological Systems Section 9.6 Preparing for the

A.Administrationofasufficientamountofketamine.B.Administrationofacamprosate,aGABAreceptoragonist.C.Administrationoftalipexole,aD2receptoragonist.

D.Administrationofabenzazapinederivative,aD1receptorantagonist.

AssessthequestionTheanswerchoicesdiscussdifferentdrugsandtheirfunctions,sinceknowledgeofthedrugsisnotpertinent,focusonthefunctionasitrelatestopassageinformation.

PlanyourattackThe positive and negative symptoms of schizophrenia are discussed in paragraphs 1 and 2,respectively. Therefore, these are the paragraphs that must be referred to in order to make apredictionsufficientfortheprocessofelimination.

ExecutetheplanThe positive symptoms of schizophrenia arise from hyperstimulation of D2 receptors in thesubcorticalregionofthebrainwhereasthenegativesymptomsofschizophreniacomeaboutfromthehypostimulation of D1 receptors in the prefrontal cortex. The correct answer will fulfill bothconditions.

Answerbymatching,eliminating,orguessingChoice(B)proposesaGABAreceptoragonist.GABAinhibitsMLDAneuronalactivity.SinceMLDA neurons secrete dopamine into the subcortical region, inhibiting them would result in lessdopamine in this region.Less dopamine in the subcortical regionwill translate into less psychoticepisodesbecauseD2receptorswillbelessstimulated.

Choice(C)suggestsaD2receptoragonist.Althoughthiswouldleadtoincreaseddopaminereleaseinthesubcorticalregion,itdoesnotexplaintheabsenceofdopamineintheprefrontalcortex.

Choice(D) iswrongforthesamereasonthat(C) iswrong.AD1 receptorantagonistwilldecreasedopaminergic activity in the cortex. However, it does not explain the hyperstimulation of D2necessaryforpositivesymptomstooccur.

Page 104: Edited By Deeangelee Pooran-Kublall, MD/MPH · 9.5 Preparing for the MCAT: Biochemistry in the Chemical and Physical Foundations of Biological Systems Section 9.6 Preparing for the

Choice(A)istheonlychoiceremainingandthecorrectanswer.Lookingcloselyatparagraph3,itispossibletodeterminetheroleofNMDAreceptors(andtheassociatedneurotransmitter,glutamate)inschizophrenia.MLDAneuronsprojecttotheVST(subcorticalregion)andareindirectlyinhibitedbyglutamatewhiletheMCDAneuronsprojecttothecortexandareupregulatedbyglutamate.Limitingthe activity of the NMDA receptors would lead to disinhibition of the ML DA neurons (causinghyperstimulationofD2receptors)andinhibitionoftheMCDAneurons(causinghypostimulationofD1receptors).Therefore,ketamineadministrationwouldproducethedesiredresult.

KeyConceptsNeurotransmitterFunction

4.The“dopaminehypothesis”isusefulfortheclinicaltreatmentofschizophrenia.However,someresearchersbelievethenameismisleading.Whymighttheythinkthis?

A.GABAneuronsactasadeactivatingsystemtokeepsubcorticaldopaminelevelslow.B.Glutamatergicneuronsarelocatedupstreamofthedopaminergicneurons.C.Multipletypesofdopaminergicreceptorsexist,makingthenameambiguous.D.Psychostimulantsexacerbatethepositivesymptomsofschizophrenia.

AssessthequestionThequestionindicatesthatthename“dopaminehypothesis”ismisleading.Itcanbeinferredthatthedopamine hypothesis is either an oversimplification or incorrect characterization of the origin ofschizophrenia.

PlanyourattackIt is necessary to determine why abnormal dopamine concentration is probably not the bestexplanationofschizophrenia.Paragraphs3and4investigatetheotherneurotransmittersinvolvedinschizophrenia,sotheseareagoodplacetostartinthesearchforaprediction.

ExecutetheplanTaking a closer look at paragraph3, it is clear that glutamate is a keyplayer in schizophrenia. Its

Page 105: Edited By Deeangelee Pooran-Kublall, MD/MPH · 9.5 Preparing for the MCAT: Biochemistry in the Chemical and Physical Foundations of Biological Systems Section 9.6 Preparing for the

presence causes upregulation ofMCDAneurons and indirectly causes downregulation ofMLDAneurons.Inalllikelihood,thecorrectanswerwillincludeareferencetoglutamateortheassociatedNMDAreceptors.

Answerbymatching,eliminating,orguessingChoice(A)mentionsGABAergicneurons,whichdownregulateMLDAneurons.Thisisonlyasmallpieceofthepuzzleanddoesnotexplainwhy“dopaminehypothesis”isamisleadingtitle.

Choice(C)statesthatmultipledopaminergicreceptorsexist.Whileambiguitymightbeacauseforconcern, “the dopamine hypothesis of schizophrenia” is fairly clear and this is not the bestexplanationbasedontheprovidedpassageinformation.

Choice(D)bringsuppsychostimulants.Thisanswerchoice isoutof scopeand thereforedoesnotanswerthequestion.

Choice(B)matchesthepredictionthatthecorrectanswerwouldmentionglutamate.Inaddition,itistruethatglutamatergicneuronsarelocatedupstreamofthedopaminergicneurons.Researchersmightarguethatitwouldalmostmakesensetorenamethehypothesis,the“glutamatehypothesis”.

KeyConceptsBiologicalBasesofBehavior

5.Whichofthefollowingindividualswouldexperiencethemostpronouncednegativesymptomsaftertreatmentwithketamine?

A.ActivepsychostimulantabusersB.HealthyindividualsC.SchizophrenicsinremissionD.Itisimpossibletodetermine.

AssessthequestionTheanswerchoicesarethedifferentgroupsincludedinFigure1.

Planyourattack

Page 106: Edited By Deeangelee Pooran-Kublall, MD/MPH · 9.5 Preparing for the MCAT: Biochemistry in the Chemical and Physical Foundations of Biological Systems Section 9.6 Preparing for the

Thequestionstemisfocusedonthenegativesymptomsassociatedwithschizophrenia.Recallthatthenegativesymptomsare linked tohypostimulationof theD1 receptorsof theprefrontalcortex. NotethatFigure1illustratesthechangeindopamineconcentrationinthesubcorticalregionafterketamineadministration.ThereisnodataprovidedabouttherelativeimpactofketamineontheD1receptorsofthePFC.

ExecutetheplanSince there is no data available about ketamine’s impact on thePFC, it is impossible to determinewhichgroupwillexperiencethemostpronouncednegativesymptoms.

Answerbymatching,eliminating,orguessingThe prediction ismetwith amatch. It is impossible to determinewhich groupwill experience themostpronouncednegativesymptoms,therefore,choice(D)iscorrect.

BEHAVIORALSCIENCESPASSAGEII:MEMORYLong-termmemorystorageisultimatelytheresultofstrengthenedsynapticconnections.Theprocessof long-term memory formation utilizes cytoplasmic polyadenylation element binding (CPEB)proteinswhich, remarkably, resembleprions in that they act as a template for promoting the localformationofotherCPEBproteins.Theseproteins,onceformed,stacktogether toformoligomers.ThepresenceofstackedCPEBproteinsintheaxonterminalstrengthensthesynapticconnectionandhelpsmaintainmemory.Regulationof theseproteins, however, presents a biological problem: toomucholigomerization can lead to the formationof toomanymemories.While thismight initiallyseemdesirable,itisimportanttoconsiderthatforgettingnonessentialmemoriesstrengthensrecallofimportantones.

Searching for a mechanism of regulation, researchers studied drosophila. The CPEB protein indrosophila isOrb2,whichexists inAandB isoforms.Orb2Ais far lesscommon,butshowsmoreprion-likeactivity.TheresearchersobservedthatOrb2AbindswithTob,anotherproteinthatseemedtoregulateOrb2Alevels.TheyhypothesizedthatTobstabilizedOrb2A,whichisusuallylabile.TestsconfirmedthatincreasingTobreducedOrb2Adecaybyafactoroftwo,andthatneuronalstimulationwasshowntoincreaseTob-Orb2Abinding.TheotherOrb2protein,Orb2B,isfarmorecommon,butrequiresanOrb2A“seed”topromoteoligomerization.Oncetheseedisformed,Orb2Bcancontinuetostackwithitself.ThismechanismisdemonstratedinFigure1.

Page 107: Edited By Deeangelee Pooran-Kublall, MD/MPH · 9.5 Preparing for the MCAT: Biochemistry in the Chemical and Physical Foundations of Biological Systems Section 9.6 Preparing for the

Figure1.Tob-Orb2AbindingleadstoOrb2Boligomerization

TotestthebehavioraleffectsoftheTob-Orb2Acomplex,researchersexaminedcourtshipbehavior,awell-documentedapplicationoflong-termmemory,indrosophila.Typically,amaleflywilldisplaycourtship behavior towards a newly introduced female, but over time this behavior is prone toextinction if the female is unreceptive.UsingRNA interference techniques, researchers suppressedTobproductionandfoundthatTobdeficientfliescontinuedtotrytomatewithunreceptivefemalesafterrepeatedexposure.

Findingthemechanismfortimingoflong-termmemoryformationprovedmoredifficult,butreliedonthediscoverythatphosphatasePP2AiscontainedintheOrb2A-Tobcomplex.PP2AisanenzymethatremovesphosphatesfromOrb2A.Whendephosphorylationwaschemicallyblocked,thishadtheeffect of destabilizingTob but stabilizingOrb2A, aswell as causing an overall reduction inTob-Orb2Abinding.ResearchershypothesizedthattheprimaryfunctionofPP2AistoremovephosphatesfromOrb2A and destabilize it when the synapse is unstimulated, thus preventing oligomerization.StimulationpromotesTobbinding,whichstabilizesOrb2AandformstheseedrequiredforthemoreabundantOrb2Btocontinueoligomerization.

P1.

P2.

Fig1.

P3.

P4.

1.TheresultsofthestudyinthepassagesuggestthattheTob-Orb2Acomplexisamechanismforwhichofthefollowing?

A.NeuroplasticityB.SynapticpruningC.SpreadingactivationD.Potentiation

Page 108: Edited By Deeangelee Pooran-Kublall, MD/MPH · 9.5 Preparing for the MCAT: Biochemistry in the Chemical and Physical Foundations of Biological Systems Section 9.6 Preparing for the

2.IfOrb2BstackingdidnotrequireanOrb2Aseed,whichofthefollowingeffectsismostlikelytobeobserved?

A.IncreasedefficiencyofandhighersuccessrateforelaborativerehearsalB.AnincreaseintheratioofautomaticprocessingtoeffortfulprocessingC.AnincreasednecessityformaintenancerehearsalinformingnewpathwaysD.Strongerandmoreefficientsemanticnetworksthroughoutthecortex

3.Hyperthymesiaisaneurologicalconditionthatresultsinanexceptionallystrongautobiographicalmemory;individualswiththeconditionareabletorecallanabnormallylargenumberoftheirlifeexperiences.Whichofthefollowingdescribe(s)thetype(s)ofmemoryaffectedbythiscondition?

I.ImplicitII.EpisodicIII.Declarative

A.IonlyB.IIonlyC.IandIIIonlyD.IIandIIIonly

4.Basedonthestudycitedinthepassage,whichofthefollowingcanmostreasonablybeconcludedregardingcourtshipbehaviorindrosophila?

A.Tob-resistantmalefliesarelessresponsivethannormalfliestonegativereinforcementcuesfromfemales.B.Withouttheabilitytodifferentiatereceptivefromunreceptivefemales,courtshipbehaviorispronetoinstinctivedrift.C.LackingTobtoregulatelong-termmemoryformation,malefliesexhibitspontaneousrecoveryofextinctbehaviors.D.TheTob-Orb2Acomplexinmaledrosophilaaidsintheirlearningtodiscriminatebetweensimilarstimuli.

5.Supposethatitwerediscoveredthat,duringbinding,TobrecruitsLimK,akinasethatphosphorylatesOrb2A.Whateffectwouldthishaveontheresearcher ’sconclusions?

A.Theywouldbeweakened,becausephosphorylationdestabilizesTob,makingitlesslikelyforanOrb2Bseed.B.Theywouldbestrengthened,becausethefindingshowshowOrb2AconformationchangescancauseOrb2Bstacking.C.Theywouldbestrengthened,becausethefindingprovidesapotentialmechanismbywhichTobcanstabilizeOrb2A.D.ThesefindingsaresuperfluousandwouldhavenoeffectbecausePP2Aalreadyperformsthisfunctionintheaxon.

Page 109: Edited By Deeangelee Pooran-Kublall, MD/MPH · 9.5 Preparing for the MCAT: Biochemistry in the Chemical and Physical Foundations of Biological Systems Section 9.6 Preparing for the

BehavioralSciencesPassageIIExplanation:

USINGTHEKAPLANMETHODS

P1.CPEBproteinstackinghelpslong-termmemoryformation

P2.Mechanism:afterstimulation,TobbindstoOrb2A,formsaseedthatpromotesOrb2Bstacking

Fig1.Mechanismdiagram

P3.Applicationtobehavior:Tob-deficientmalefliesforgetfemaleunreceptiveness

P4.Timing:PP2AremovesphosphatesfromOrb2Acausingdegradationuntilneuronstimulated

KeyConceptsMemoryisahigh-yieldtopicintheBiologicalFoundationsofBehaviorsectionoftheMCAT.Makesuretoknowthebasicsofallfourmemoryprocesses:Encoding,Short-term,Long-termandRetrieval,aswellascommonfailuresofthesesystems.

1.TheresultsofthestudyinthepassagesuggestthattheTob-Orb2Acomplexisamechanismforwhichofthefollowing?

A.NeuroplasticityB.SynapticpruningC.SpreadingactivationD.Potentiation

AssessthequestionThispassageseemsverytechnical;followthebasicbiologicalprocessesdiscussedandyoushouldbeabletoincorporatethenewideaswiththerequiredknowledgeofmemorysystems.Thisquestionistestingknowledgeofmemoryvocabularyandusestheoveralltopicandscopeofthepassagetodoit.Attackthisquestionrightaway.

PlanyourattackStartwithananalysisoftheTob-Orb2Acomplex.Accordingtothepassage,theTob-Orb2Acomplexis a mechanism for long-term memory that helps strengthen the connections between synapses.

Page 110: Edited By Deeangelee Pooran-Kublall, MD/MPH · 9.5 Preparing for the MCAT: Biochemistry in the Chemical and Physical Foundations of Biological Systems Section 9.6 Preparing for the

Eliminateanyanswersthatdon’trefertolong-termmemory.

ExecutetheplanStartwithchoice(A);neuroplasticitydescribestheabilityofthebraintomolditsfunctiontoadapttochangesingeneral,andsoistoobroadtobeexplainedbythemechanismdescribedbythepassage.

Choice(B)istheprocessofremovingsynapticconnectionsovertime.Thisisnearlytheoppositeofthecorrectanswer.

Choice(C),spreadingactivation,istheprocessbywhichnodesthatareclosetogetherinasemanticnetwork stimulate each other. This sounds relevant, but refers to a process that occurs once thememoriesarealreadyformed.

Finallychoice(D),potentiation, is theprocessbywhichsynapticactivation isstrengthened throughrepeatedstimulation,turningashort-termmemoryintoalong-termone.

Answerbymatching,eliminating,orguessingExecuting the plan leads to the elimination of (A), (B), and (C), which did not agree with theassessmentoftheTob-Orb2Acomplex.Recallingthedefinitionofpotentiationallowsonetochoose(D)withconfidence.

TakeawaysSometimesthestudiesdescribedgetquitetechnical;ifso,yourgoalshouldbetofocusonprocedureandresults.Questionswillfocusontheapplicationofourpsychologyandsociologycontentknowledgetotheoverallstructureoftheexperimentsinthepassages.

ThingstoWatchOutForMemoryandLearningarebothsubjectsthatcontainquiteabitofvocabulary.Expectquestionsthattestcriticalthinkingtoalsorequirekeepingterminologystraight.

2.IfOrb2BstackingdidnotrequireanOrb2Aseed,whichofthefollowingeffectsismostlikelytobeobserved?

A.IncreasedefficiencyofandhighersuccessrateforelaborativerehearsalB.AnincreaseintheratioofautomaticprocessingtoeffortfulprocessingC.Anincreasednecessityformaintenancerehearsalinformingnewpathways

Page 111: Edited By Deeangelee Pooran-Kublall, MD/MPH · 9.5 Preparing for the MCAT: Biochemistry in the Chemical and Physical Foundations of Biological Systems Section 9.6 Preparing for the

D.Strongerandmoreefficientsemanticnetworksthroughoutthecortex

AssessthequestionThisisaninferencequestion,whichasksforextrapolationbasedonthepassage.Whilethisonecouldbe saved for later, the 2A/2B interactionwas discussed succinctly in the passage, so a little bit ofpassageresearchshouldbeenoughtoanswerthisonequickly.

PlanyourattackParagraph2mentionsOrb2Bexplicitly,andparagraph1describes theoverallrelationshipbetweentheoligomersinvolvedandlong-termmemory.Researchtheseparagraphs,andwithastrongoutline,thoroughinvestigationmaybeavoided.

ExecutetheplanParagraph 2 says that Orb2B can oligomerize with itself once it attaches to an Orb2A seed. TheparagraphalsomentionsthatOrb2Bisquitecommonintheaxonterminal.Itstandstoreason,then,that ifOrb2Bwereallowedtooligomerizemorefreely,synapticconnectionswouldbestronger ingeneral,andlong-termmemorieswouldbefareasiertoform,perhapsevenpathologicallyso.Lookforthecorrectanswertodescribethis,inotherwords.

Answerbymatching,eliminating,orguessingChoice(A) seems tomatch, after consideration ofmemory vocabulary.Elaboration is the kind ofrehearsal that links new concepts to preexisting memories. If Orb2B is able to stack freely, thissynapticconnectionshouldbefarlesseffortful.(A)matchestheprediction,andiscorrect.

Choice(B)describesautomaticandeffortfulprocessing,whichareimportantconceptsinencoding.Synapticconnectionstrengthshouldn’taffectthewayinwhichinformationisobtained.

Choice(C),like(A),describesrehearsal,butthistimementionsmaintenance.Thistypeofrehearsalis simply a way to keep information in short-termmemory and prevent it from being forgotten.Short-termmemorydoesnotrelyonsynapticconnectionstrength.

Choice (D) is tempting but extreme. The expectation is that long-termmemories will formmoreeasily,butthiswillnotnecessarilyhaveanybearingonhowthosememoriesareorganized.

Page 112: Edited By Deeangelee Pooran-Kublall, MD/MPH · 9.5 Preparing for the MCAT: Biochemistry in the Chemical and Physical Foundations of Biological Systems Section 9.6 Preparing for the

TakeawaysQuestionsthataskforamodificationofastudyarerelativelycommon.Attempttheseonlyafterobtainingpointsfrommorestraightforwardquestions.

3.Hyperthymesiaisaneurologicalconditionthatresultsinanexceptionallystrongautobiographicalmemory;individualswiththeconditionareabletorecallanabnormallylargenumberoftheirlifeexperiences.Whichofthefollowingdescribe(s)thetype(s)ofmemoryaffectedbythiscondition?

I.ImplicitII.EpisodicIII.Declarative

A.IonlyB.IIonlyC.IandIIIonlyD.IIandIIIonly

AssessthequestionThis is a Roman numeral question, and the third question in a row that will reward vocabularyknowledge.It’salsoapseudo-discrete,andsocanbedonerightaway.

PlanyourattackThestemdescribeshyperthymesia,solittleresearchshouldbenecessary.Thegoalshouldbetomakeapredictionbasedonthestemitself.AsaRomannumeralstrategy,eliminateuntilonlyoneanswerchoiceremains.

ExecutetheplanHyperthymesia is described as causing an overload of life-experience memory. This mightimmediatelybringtomindtheconceptofepisodicmemory,whichisachoice,butkeepinmindthatthediagramofthedifferenttypesofmemoryisabranchingtree,somorethanoneanswermaybecorrecthere.

Page 113: Edited By Deeangelee Pooran-Kublall, MD/MPH · 9.5 Preparing for the MCAT: Biochemistry in the Chemical and Physical Foundations of Biological Systems Section 9.6 Preparing for the

Answerbymatching,eliminating,orguessingBasedonourprediction,RomannumeralIIshouldbeapartofthecorrectanswer.Eliminate(A)and(C). Neither of the remaining choices includes Roman numeral I, so we need only to considerwhether III should be included. Declarative memory is the part of long-term memory that isconscious,andthatincludesbothepisodicandsemanticmemory.RomannumeralIIIshouldthereforebepartofouranswer,sochoice(D)iscorrect.

TakeawaysPredictingisahelpfultoolinRomannumeralquestions,butkeepanopenmindgiventhatmorethanonestatementcanbecorrect.

KeyConceptsIt’simportanttokeepstraightthedifferentkindsofmemoryandthewaysinwhichpsychologistsclassifyeach.

4.Basedonthestudycitedinthepassage,whichofthefollowingcanmostreasonablybeconcludedregardingcourtshipbehaviorindrosophila?

A.Tob-resistantmalefliesarelessresponsivethannormalfliestonegativereinforcementcuesfromfemales.B.Withouttheabilitytodifferentiatereceptivefromunreceptivefemales,courtshipbehaviorispronetoinstinctivedrift.C.LackingTobtoregulatelong-termmemoryformation,malefliesexhibitspontaneousrecoveryofextinctbehaviors.D.TheTob-Orb2Acomplexinmaledrosophilaaidesintheirlearningtodiscriminatebetweensimilarstimuli.

AssessthequestionThisquestion isaskingforan inference thatcanbedrawnfromthepassage,and thequestionstemindicatesexactlywheretogo,soapproachthisquestionimmediately.Ifthepassagewasreadactively,apredictionmayhavealreadybeenmade.

Planyourattack

Page 114: Edited By Deeangelee Pooran-Kublall, MD/MPH · 9.5 Preparing for the MCAT: Biochemistry in the Chemical and Physical Foundations of Biological Systems Section 9.6 Preparing for the

The study inquestion ispresented inparagraph3, so review theprocedureand results there.Youroutline will be invaluable for a question like this because you’ve likely summarized the basicsalready.

ExecutetheplanAccording to the passage, male flies that lacked Tob were not able to learn which females wereunreceptive, and continued to try to mate even after being rejected. The study supports the ideaspresentedearlierinthepassage,thatalackoftheTob-Orb2Acomplexpreventslong-termmemoryformation.Lookforananswerthatmatches.

Answerbymatching,eliminating,orguessingChoice(A)mentionsnegativereinforcement,whichreferstoanincreaseinbehaviorresultingfromremovalofanunwantedstimulus.Ifanything,thefemalefliesaresignalingthattheywouldlikethemalestodecreasecourtshipbehavior,so(A)seemsunlikely.

Choice (B) mentions instinctive drift, which is another learning phenomenon in which an animallearning a complicated behavior will revert to amore natural one. There is no such complicatedbehaviordescribedinthestudy.

Choice (C) only makes sense if the males’ courtship behavior became extinct; that is, the maleslearnednot topursueunresponsivefemales in thefirstplace.This isnotsupportedbythestudy,so(C)canlikelybeeliminated.

Byelimination,choice(D)mustbecorrect. Itdiscussesdiscrimination,which is aphenomenonbywhich an organism can differentiate between two similar, but distinct, stimuli. Here, the similarstimuliarethereceptiveandunreceptivefemales.Sincemalefliesareusuallyabletorememberthedifference and only exhibit courtship behavior toward receptive females (whereas Tob-deficientmalesarenot),itstandstoreasonthatTobisatleastpartlyresponsibleforthedifference.

TakeawaysWheneverapassagepresentstheresultsofastudywithoutaconclusion,expecttoformulatethatconclusiononyourown.Asyoucreateyouroutlineforthatportionofthepassage,takeamomenttosummarizetheresultssothatyoucanbereadyforsuchaquestion.

ThingstoWatchOutForAlwaysbewareofextremeanswerchoicesforquestionsthataskyoutomakeconclusionsbasedonastudy.Choicesthatmention“proof”or“causes”arelikelytobeincorrect.

Page 115: Edited By Deeangelee Pooran-Kublall, MD/MPH · 9.5 Preparing for the MCAT: Biochemistry in the Chemical and Physical Foundations of Biological Systems Section 9.6 Preparing for the

5.Supposethatitwerediscoveredthat,duringbinding,TobrecruitsLimK,akinasethatphosphorylatesOrb2A.Whateffectwouldthishaveontheresearcher ’sconclusions?

A.Theywouldbeweakened,becausephosphorylationdestabilizesTob,makingitlesslikelyforanOrb2Bseed.B.Theywouldbestrengthened,becausethefindingshowshowOrb2AconformationchangescancauseOrb2Bstacking.C.Theywouldbestrengthened,becausethefindingprovidesapotentialmechanismbywhichTobcanstabilizeOrb2A.D.ThesefindingsaresuperfluousandwouldhavenoeffectbecausePP2Aalreadyperformsthisfunctionintheaxon.

AssessthequestionThis question requires incorporation of new informationwith the passage information. Questionsliketheseshouldbesavedforlast.

PlanyourattackThequestionstemmentionsphosphorylation,aprocessdiscussedinparagraph4.It’slikelythatwe’llneedtounderstandtheroleofphosphorus,sostartwithsomeresearch.

ExecutetheplanAccordingtoparagraph4,PP2AremovesphosphorusfromOrb2A,andthatwhenthiswasblocked,Orb2Awas stabilized. If during binding, Tob causes phosphorus to be added to dephosphorylatedOrb2A,thisprovidesananswertothequestionofhowbindingstrengthensOrb2A.Armedwiththisprediction,lookforamatch.

Answerbymatching,eliminating,orguessingChoices(A)and(D)areoutimmediately,becausetheprocessmentionedinthequestionstemhelpstoqualify a previously unexplained portion of the hypothesis. Of the rest, choice (C) matches thepredictionperfectly.

Takeaways

Page 116: Edited By Deeangelee Pooran-Kublall, MD/MPH · 9.5 Preparing for the MCAT: Biochemistry in the Chemical and Physical Foundations of Biological Systems Section 9.6 Preparing for the

Whenanswerchoicesaregroupedastheyarehere,evenageneralpredictioncaneliminatehalfoftheanswerchoices.

Page 117: Edited By Deeangelee Pooran-Kublall, MD/MPH · 9.5 Preparing for the MCAT: Biochemistry in the Chemical and Physical Foundations of Biological Systems Section 9.6 Preparing for the

8.8BehavioralSciencesPracticePASSAGEIII(QUESTIONS1–6)Throughouttheday,humansexperienceinnumerablesensorystimuli.Thesestimulioriginatefromavarietyofsources,andareusuallycategorizedbasedon thefive traditionalhumansensesofsight,smell,hearing,taste,andtouch.Psychologistssortstimulibasedontheirphysicalconstituents,whicharereferredtoasstimulusmodalities.Thesemodalitiesmatchupfairlycloselywiththefivesenses,butare slightlymoregeneral.Somestimulusmodalitiesare light, sound, temperature, taste, smell,and pressure. Thesemodalities are also called thedistal stimuli, which refers to the fact that theydirectlystimulatethesensoryorgansofthebodyliketheeyes,nociceptors,orthebonesoftheinnerear.

The distal stimulus initiates neural processing. For the eye, the modality is the electromagneticradiation (in the visible spectrum) that reflects from an object and makes its way to the retina.Throughphototransductionthestimulusisturnedintonerveactionpotentials,theproximalstimulus.Fromtheneuralinformation,thebrain(re)createsanimageoftheobject.Theprocessisanalogousfortheothersensorymodalities,forinstance,thesoundofabarkingdogandtheperceptionofadog,or the scentof a rose and theperception thereof.Thebrain takesproximal stimuli and transformsthemintowhatwedetectasourworld.

There are hundreds of proximal stimuli running through the brain at any given time. How canhumans take all of this information and organize it into something meaningful and useful? Onetheoryisgestaltism,whichisexplainedasthetendencyofhumanstointegratepiecesofinformationinto meaningful wholes. For example, someone looking at a bookshelf would see a shelf full ofbooks,asawhole,beforenoticingthat therewereindividualbooks,orthat thebookshaddifferenttitles, or that the books weremade of paper, or that theywere arranged in a certain way. Gestalttheorists attempt to understand how top-down processing affects perception. To account for theseeminglyinnateorganizationthatoccursinthehumanbrain,Gestalttheoristdevisedtheprinciplesofgrouping.Theseprinciplesareorganizedintosixcategories:proximity,similarity,closure,goodcontinuation,commonfate,andgoodform.

Anotherorganizationalprinciplethatinfluenceshumanperceptionistheideaofperceptualsets.Alsoshapedbytop-downprocessing,theconceptofperceptualsetsimpliesthatstimulimaybeperceivedin different ways depending upon the perceiver. For instance, an individual may be exceptionallysensitivetothesoundoftheirname,especiallyiftheyarewaitingforittobecalled.Thethinkermayevenshapetheirperceptionssothatrealitycorrelatesmorecloselytotheirexpectations.Forinstance,anindividualwaitingtoheartheirnamemaybemoreapttomistakenlyidentifyasimilarsoundingnameastheirown.

P1.

P2.

P3.

P4.

Page 118: Edited By Deeangelee Pooran-Kublall, MD/MPH · 9.5 Preparing for the MCAT: Biochemistry in the Chemical and Physical Foundations of Biological Systems Section 9.6 Preparing for the

1.Supposethatanobservercandiscriminatebetweena50cdlightanda55cdlight,butnota50cdlightanda54cdlight.Assumingthesamesetup,whatisthefaintestintensityoflightthatcanbediscriminatedfroma300cdlight?

A.270cdB.305cdC.330cdD.355cd

2.Whichofthefollowingstatementswouldbestsupporttheideaofperceptivesets?A.Amotherreservingjudgmentforherchild’sperformanceatamusicalcontestuntilthejudgeshavereturnedtheirdecision.B.Asportsfancheeringfortheotherteamwhentheymakeaskillfulplay,evenifitresultsintheirownteamlosingpoints.C.Amotheradvocatingforherchildinacompetition,butremindingthechildtobeagoodsportwhethertheywinorlose.D.Asportsfanperceivingtherefereesinagametobeagainsttheirteamwhentherefereesmakeanycallsthathurttheirteam.

3.WhichofthefollowingwouldNOTbeconsideredadistalstimulus?A.LightbouncingoffofaflowerandintotheeyeB.Small,volatilemoleculesreleasedbyaflowerC.PollenreleasedbyaflowerintotheairD.Thesoundwavescreatedbyabee’swings

4.Whichofthefollowingmostaccuratelydescribesthebody’sreactiontoabeesting?A.Anefferentactionpotentialarrivesatarelayneuron,whichsendsanafferentactionpotentialtotheeffectormuscleafterwhichthesignalarrivesandisinterpretedatthebrain.B.Anafferentactionpotentialarrivesatarelayneuron,whichsendsanefferentactionpotentialtotheeffectormuscleafterwhichthesignalarrivesandisinterpretedatthebrain.C.Anafferentactionpotentialarrivesatarelayneuron,whichsendsasignaltothebrainwhichtheninterpretsthesignalandsendsanefferentactionpotentialtotheeffectormuscle.D.Anefferentactionpotentialarrivesatarelayneuron,whichsendsasignaltothebrainwhichtheninterpretsthesignalandsendsanafferentactionpotentialtotheeffectormuscle.

5.Phantosmiaisadisordercharacterizedbyolfactoryhallucinationsintheabsenceofanyphysicalodors.Whichofthefollowingismostlikelytocausephantosmia?

A.AnodormoleculestimulatingthewrongkindofchemoreceptorB.Seizuresoccurringinapatient’soccipitallobeoftheirbrainC.TemporallobeatrophyduetoAlzheimer ’sdiseaseD.Parietallobeactivationcausedbyschizophrenia

Page 119: Edited By Deeangelee Pooran-Kublall, MD/MPH · 9.5 Preparing for the MCAT: Biochemistry in the Chemical and Physical Foundations of Biological Systems Section 9.6 Preparing for the

6.Somechildren’sgamesinvolvesearchingforaknowncharacteramidothersimilarlydressedcharactersandsimilarlycoloredobjects.Thethoughtprocessinvolvedinthistypeofgamecouldbestbeconsidered:

A.top-downprocessingwithmultimodalstimuli.B.bottom-upprocessingwithunimodalstimuli.C.top-downprocessingwithunimodalstimuli.D.bottom-upprocessingwithmultimodalstimuli.

BehavioralSciencesPracticePassageExplanationsP1.Stimulusmodalities,distalstimuli

P2.Processingofdistalstimuliintoproximalstimuli

P3.Gestalttheory,top-downprocessing

P4.Perceptualsetsaffectourthinking

1.(A)

The just-noticeable difference for the discrimination of light (in this particular setup) can bedetermined from the question stem. According to Weber ’s law, the just-noticeable difference is

calculatedasa ratio.From thegiven information the JND is: or of the intensity of the light.Usingthisinformation,itcanbepredictedthattheobserverwouldbeabletodifferentiatebetweena300cdlightandanotherlightaslongastheirintensitiesvaryby10percent.Thequestionasksforthefaintest,whichmatches(A).A270cdlightcouldbedifferentiatedfroma(270×1.1)297cdlight,sotheobserverwouldbeabletodifferentiatebetweenthetwo.

2.(D)

Perceptivesetsaretheexpectationsthataffectone’sperception.Itmayresultintheorganizationandprocessingof information so that perceptionsmatchone’s expectationsormotivations.This is theessentialattributethatmustbefoundinthecorrectanswer.Inchoice(A),choice(B),andchoice(C)theperceptionisn’tbeingmodifiedinthisway.Choice(D),however,hasthesportsfan’sperceptionsbeingchangedbecauseoftheircommitmenttotheirownteamandisthecorrectresponse.

3.(C)

Distal stimuliare thephysical stimuli thatdirectly stimulate thesensoryorgans.Thisquestionaskswhichanswer isNOTadistalstimulus,so thecorrectanswerwillbeunlike theothers.Choice(A)wasdiscussedinthepassageasanexampleofdistalstimuli,soitisout.Choice(B)couldbealittleconfusing,butthesecriteriaallpointtowardsanodorantandanexampleofadistalstimulus.Choice(D) is stimulus thatwill travel to thesensorysystemandbeconverted intoanactionpotential, andthuscouldbeadistalstimulus.Thisleaveschoice(C);sincepollenmoleculesarelargeandarenotprocessed by the olfactory system, but rather create an immune response, they are not a distalstimulus.

Page 120: Edited By Deeangelee Pooran-Kublall, MD/MPH · 9.5 Preparing for the MCAT: Biochemistry in the Chemical and Physical Foundations of Biological Systems Section 9.6 Preparing for the

4.(B)

Whenthebodyexperiencesastimulusthatelicitsareflexarc, thereisaspecificorderforhowthesignaltravelsthroughthesystem.Theafferentneurontakesthesignalfromthesensoryorgantothecentralnervoussystem.Inreflexloops,thissignalistakeninbyarelayneuronandthensentdirectlybackoutthroughanefferentneurontotheeffectormuscletotryandmovethebodyawayfromthepainful stimulus. It isn’t until after this signal and reflex has happened that the brain realizes thestimulusortheactionatall,orchoice(B).

5.(C)

Lookingat theanswers, threehavetodowithdifferentpartsofthebrainwhiletheotherlooksatamix-upatthechemoreceptor.Choice(A)canbeeliminatedsincethequestionstemstatesthatthereisan absence of odor, thus nomix-up could occur since there is no odormolecule to stimulate thechemoreceptor. From there this question is a matter of remembering which area of the brain isresponsible for interpretingscent.Theanswer to thatquestion is the temporal lobeandphantosmiacanbecausedbyAlzheimer ’sdiseasewhenitatrophiesthetemporallobe,orchoice(C).

6.(C)

Taking a look at the answer choices reveals that this question is looking for the modality of thesensorystimulusandhowthechildren’sgameisbeingprocessed.Thequestionstates that thechildwillbelookingforsomethingamidafieldofotherobjects.Thismeansthechildalreadyhasanideaofwhatheislookingforandisparsingalargerimagelookingforaspecificthing.Thismeansthatthechild isusing top-downprocessing,whicheliminateschoices (B)and (D).Themodality of thestimulushastodowithwhatsensesarebeingused.Inthiscase,lightiscollectedandvisionisbeingusedexclusively–implyingaunimodalstimulusandchoice(C).

Page 121: Edited By Deeangelee Pooran-Kublall, MD/MPH · 9.5 Preparing for the MCAT: Biochemistry in the Chemical and Physical Foundations of Biological Systems Section 9.6 Preparing for the

CHAPTERNINE

BiochemistryTheMCATwillnowtest introductorybiochemistryasaportionofboth theChemicalandPhysicalFoundations of Biological Systems and the Biological and Biochemical Foundations of Livingsystems.Eachofthesesectionswillbeapproximately25percentbiochemistry.Theinformationtestedismaterial likely to be covered in an introductory biochemistry course for those studying the lifesciences.Therefore,itisimportantthatyoutakeyourpreparationforbiochemistryveryseriously,asa significantportionofyour scorewilldependonbiochemistry.Thatbeing said, thebiochemistrytestedineachofthesesectionsislikelytobeindifferentcontexts.Inthischapter,wewillexplorehowthe MCAT will be testing your knowledge of biochemistry in both the Chemical and PhysicalFoundations of Biological Systems and the Biological and Biochemical Foundations of LivingSystemssections.

9.1ReadingthePassageWhenglancingatabiochemistrypassagefor thefirst time, it is likely thatyouwillnotice images,data, and/or chemical equations. The images may represent a cellular structure, a biochemicalexperiment, or a graph representing experimental data. The images present in these passages arelikelytobeverydifferentfromthoseencounteredinphysics.Infact,thesepassageswillbedifferentin both scope and structure comparedwith those seen in physics. Thus, theway that biochemistrypassagesmustbereadisdifferentfromreadingpassagesinphysics.

PASSAGETYPESMuch like the physics passages, biochemistry passages can be either experiment or informationpassages.However,theexperimentpassagesinbiochemistryaremuchdifferentfromthosefoundintheothersciences.Inbiochemistry,experimentpassagesmayberelatedtoabiologicalprocess,anddatamayberepresentedvisuallyasachangeinstructureofacellortissueorasagraph,ratherthanrawnumerical datapoints.Questionson thesepassages are likely to assessyour ability to analyzevisualevidence.Experimentpassageswillrequirethetest-takertoputtogetheralloftheinformationin order to answer questions.While the passage presentationswill be similar in both sections, thescopeoftheinformationtestedwillbedifferent.

ChemicalandPhysicalFoundationsofBiologicalSystemsPassages in the Chemical and Physical Foundations of Biological Systems section will likely beaccompaniedbychemicalequations.Inaddition,thosebiochemistrypassagesarelikelytofallmorewithin the context of chemistry, both organic and inorganic (general) chemistry. Success onbiochemistry passages in this section will rely on your ability to connect the concepts inbiochemistry,organicchemistry,andgeneralchemistryandtoapplytheseconnectionstobiologicalsituations.Biochemistrypassagesinthissectionaremorelikelytoreflectchangesthatoccurat themolecularandtissuelevel.

• Experiment passages in this sectionmay include organic synthesis pathways andmolecular

Page 122: Edited By Deeangelee Pooran-Kublall, MD/MPH · 9.5 Preparing for the MCAT: Biochemistry in the Chemical and Physical Foundations of Biological Systems Section 9.6 Preparing for the

structures.•Experimentalresultswilllikelybesummarizedintheformofagraph,chart,orothervisualrepresentationofdata.• Experiment passages will require integration of biochemistry concepts at themolecularlevel.

•Likeinformationpassagesinphysics,informationpassagesinbiochemistrywillalsopresentinformation, but understanding this information will require a fundamental understanding ofbiology, even though this is not in the Biological and Biochemical Foundations of LivingSystemssection.

•Thesepassageswillintegratemultiplesubjectareas.•Connectionsbetween topics in thepassagemaybepresent but not explicitly stated.The test-taker will have to make these connections in order to fully understand thepassage.

Overall,biochemistrypassagesinthissectionareaperfectopportunityforthetestmakertopresentquestions and passages that require a thorough understanding in the areas of general chemistry,organicchemistry,biochemistry,andbiology.Thedivisionsbetween thesescienceswillbeblurredontheMCAT,suchthatasinglepassagewithquestionsislikelytotestmultipleareas,withindividualquestionsrequiringknowledgeinmultiplesubjects.

BiologicalandBiochemicalFoundationsofLivingSystemsBiochemistrypassagesinthissectionarelikelytofocusstronglyonbiology,andlessonthedetailsof organic and general chemistry. While knowledge of the fundamentals of chemistry will berequired,thebiochemistrypassagesinthissectionaremorelikelytoexplorehowthebiochemistryofanorganismwillaffecttheorganismanditsphysiologyasawhole.

•Experimentpassagesinthissectionwillfeatureanexperiment,butyouarelesslikelytohaveapassagewithanorganicsynthesispathway.

• The data will be less numerical, but you will have to analyze figures and data tounderstandhowaprocessinfluencestheoverallphysiologyoftheorganism.

•Informationpassageswillalsofocusmoreonthebiologicalsideofbiochemistry.•Expectthatinformationpassagesthatappeartobemorebiologyinscopemayalsobeaccompaniedbyquestionsregardingbiochemistryconcepts.

BiochemistryinitsDifferentSections

ChemicalandPhysical

FoundationsofBiologicalSystems

BiologyandBiochemical

FoundationsofLivingSystems

ExperimentPassages

Incorporateanorganicchemistrysynthesisorgeneralchemistryconcepts.Experimentislikelytofocusonbiochemistryatthemolecular,cellular,ortissuelevel

Heavyinbiologicalconcepts,oftenphysiology.Experimentislikelytofocusonhowabiochemicalprocess

Page 123: Edited By Deeangelee Pooran-Kublall, MD/MPH · 9.5 Preparing for the MCAT: Biochemistry in the Chemical and Physical Foundations of Biological Systems Section 9.6 Preparing for the

ratherthantheorganismasawhole.

affectstheorganismasawhole.

InformationPassages

Willbeacombinationofconceptsfromgeneralchemistry,organicchemistry,andbiochemistry,andagraspofthesesubjectareaswillberequiredinordertodeterminethegoalofthepassage.

Alsoheavyinbiologicalconcepts,passagesmaybedefinedasbothbiochemistryandbiology,withlittledistinctionbetweenthetwo.

Scope

Fromthemoleculartothetissuelevel.

Fromthemoleculartotheentireorganism.

Images

Mayinvolvemolecularstructures,chemicalequations,graphsandchartsrepresentingdata,andillustrationsofbiochemicalprocessesatthemolecularandcellularlevel.

Lesslikelytoinvolvemolecularstructure,butphysiologicalpathwaysmaypredominate.Chemicalequationswillrepresentphysiology.Illustrationswillfocusonbiochemicalprocessesattheleveloftheorganism.

OUTLININGTHEPASSAGERegardlessofthesection,biochemistrypassagesareapproachedinthesameway,usingtheKaplanway.

ScanforStructureThe structure of biochemistry passages will be somewhat dependent on the passage type and thesection.PassagesintheChemicalandPhysicalFoundationsofBiologicalSystemssectionaremorelikelytobeaccompaniedbymolecularstructuresandorganicsynthesispathways,whilepassagesinthe Biology and Biochemical Foundations of Living Systems section are more likely to beaccompaniedbyimagesoforganismsorcells.Asyouscanthepassage,notewhatispresent.Unlikethe physics section, data is more likely to be represented graphically or visually rather thannumerically.Afterscanninganddeterminingthesubjectarea,decidewhetherthispassageisonetodonoworonetodolater.

ReadStrategicallyReadingstrategicallyinthebiochemistrysectionrequiresthatyoutakenoticeofthetypeofpassageyouare reading.Likeallother sciences testedon theMCAT,experimentpassageswill require thatyou identify the hypothesis, procedure, and outcomes, as these are likely targets for questions.However, in the biochemistry section you should also be aware of the relationships between thechemistryandthebiology.In theBiologyandBiochemicalFoundationsofLivingSystemssection,take note of how the biochemistry affects the physiology. This topic is likely to be tested in the

Page 124: Edited By Deeangelee Pooran-Kublall, MD/MPH · 9.5 Preparing for the MCAT: Biochemistry in the Chemical and Physical Foundations of Biological Systems Section 9.6 Preparing for the

questions.

IntheChemicalandPhysicalFoundationsofBiologicalSystemssection,payspecialattentiontotheconnectionsbetweenthechemistryandthebiology/biochemistry.QuestionsontheMCAToftenfocusonhowwellyoucanmakeconnectionsbetweentopics.Ifyoureadsuchthatyoupayattentiontothesequestions,youwillbeabletoanticipatewherethequestionsaregoingtocomeafterapassage.

LabelEachComponentAfter you have read, it is time tomake your outline. Identify the purpose of each paragraph andfigure.Then,write it downonyouroutline.Forbiochemistry, it is also important tonote specificconceptswithinaparagraphandwhereelsetheseconceptsarediscussed.Ifafigureshowsthestepsinanorganic chemistry synthesis and thenextparagraph indicateshow theproductsof that synthesiswereusedtostudysomeaspectofbiochemistry,makeanoteaboutthatconnection.Whenyoucreateyouroutlineby labeling eachparagraph, it is alsohelpful to note the concept of theparagraphorfigure.Thiswillhelpyoutoidentifywheretofindinformationwhenyoustartansweringquestions.

ReflectonYourOutlineThis is,ofcourse,whereyoudetermine thegoalof thepassage.Whatprocesswasdescribed?If itwasanexperimentpassage,whatbiochemicalprocessisrepresentedandwhatwastheoutcome?Howdidtheoutcomeaffecttheorganismorchangethemolecularstructureofanenzyme?Jotthisdownon your outline, as it will come in handy as you further interpret the passage while answeringquestions. Remember, biochemistry passages on the MCAT are aiming to test how well you canextrapolate theeffectsofchangesat themolecular level topredicteffectsonthecell, tissue,and/ororganism.

Page 125: Edited By Deeangelee Pooran-Kublall, MD/MPH · 9.5 Preparing for the MCAT: Biochemistry in the Chemical and Physical Foundations of Biological Systems Section 9.6 Preparing for the

9.2AnsweringtheQuestionsLikeothersciencestestedontheMCAT,therewillbefourtypesofbiochemistryquestions:

Discretequestions•Likelytofocusonsingletopics,orintegrationoftwotopics

Questionsthatstandalonefromthepassage•Accompanyapassage,butthepassageisnotrequiredtoanswerthequestion•Likelytobeatopicrelatedtothepassage

Questionsthatrequiredatafromthepassage•Lesslikelytoinvolveacalculationonbiochemistrypassage•Datawilleitherbefoundinthepassageorrepresentedingraphorchartform•Thesequestionswilllikelyrequireyoutoanalyzethedatapresentedinagraphorchart

Questionsthatrequirethegoalofthepassage•Willrequireanunderstandingofthepassageasawholeaswellasthepurposeofthepassage•Willoftentestyourabilitytomakeconnectionsbetweendifferenttopicswithinthepassage•Mostlikelyrelatedtoyourabilitytounderstandbiochemistryonthetissueororganismallevel

DETERMININGTHEPURPOSEOFTHEQUESTIONBiochemistryquestionsontheMCATarelikelytotargetmultiple“skills.”Twooftheseskillsincludestatistical data analysis and researchdesign.Given thatbiochemistry is likely tobe themost testedtopic on Test Day, it is probable that some passages and questions will cover concepts related tobiochemistry research. However, questions like this can also be answered using a systematicapproach.WehavealreadyintroducedtheKaplanwaytoapproachquestions.Nowwewillfocusonhowtousethatstrategytoyouradvantageonbiochemistryquestions.

1.Assessthequestion•Readthequestion,butdonotreadtheanswers.•Determinewhetherthisisaquestiontodonowordolater.Ifitisaquestiontodonow,proceedtoplanningyourattack.

2.Planyourattack•Biochemistrypassages are less likely to involvemath,butmore likely to involve reasoningandanalysis.•Identifythetaskofthequestion.Thatis,whatisthequestionaskingyoutodo?•Tasksinbiochemistryarelikelytoinvolveanalysisofdataingraphorchartform,analysisofinformation in the passage, or identification of how a biochemical process will affect theorganismoranotherprocess.•Manyofthequestionswillrequireyoutoapplyyourknowledgeofothersubjectareassuchasbiology,organicchemistry,and/orgeneralchemistrytobiochemicalconcepts.

3.Executeyourplan

Page 126: Edited By Deeangelee Pooran-Kublall, MD/MPH · 9.5 Preparing for the MCAT: Biochemistry in the Chemical and Physical Foundations of Biological Systems Section 9.6 Preparing for the

•Carryoutyourplan,goingbacktothepassageandlocatingrequiredinformation,analyzingthedata,ordeterminingtheresultsandimpactofabiochemicalprocess.

4.Answerthequestionbymatching,eliminating,orguessing•Matchyouranswerdeterminedinthepreviousstepwiththeanswerchoices.• Some questions will give you graphics in the answer choices instead of words. Whenapproaching a set of answer choices like this, be sure to eliminate ones that simply are notpossible or sensible in the context of the question. For example, there may be imagesrepresentingstructures thatdonotexist.Donotdoubtyourknowledgeaboutwhat ispossible.Justbecausesomethingappearsasananswerchoicedoesnotmeanitispossible.•Ifguessingisrequired,thentrytogetdowntotwoanswersbeforemakingaguess.Then,makeyourbestguess. Justbecauseyouguessdoesn’tmean thatyoudon’tknow thematerial;manytest-takersachievingtopscoresdidasubstantialamountofguessingduringtheirtesttoo!

Page 127: Edited By Deeangelee Pooran-Kublall, MD/MPH · 9.5 Preparing for the MCAT: Biochemistry in the Chemical and Physical Foundations of Biological Systems Section 9.6 Preparing for the

9.3GettingtheEdgeinBiochemistryGettingahighscoreonTestDayrequiresaverysolidfoundationinbiochemistry,asthissubjectareaisheavily tested.However, it is important todevelopagraspofbiochemistry in the contextof theothersciences,especiallybiology,organicchemistry,andgeneralchemistry.Manyof thepassagesyou encounter on Test Day will require you to make connections between the other sciences andbiochemistry.Inaddition,biochemicalprocessesdonotoccurinisolation.Manyofthepassagesandquestionswillfocusonhowchangesatthemolecularlevelresultsinchangesintheentireorganism’sphysiology.

On Test Day, biochemistry passages will feature a variety of figures, including graphs, charts,molecular structures, chemical equations, and visual representations of biochemical concepts. Beprepared toanalyzeavarietyofdifferent images,asyouwillberepeatedlyasked todosoonTestDay.

Page 128: Edited By Deeangelee Pooran-Kublall, MD/MPH · 9.5 Preparing for the MCAT: Biochemistry in the Chemical and Physical Foundations of Biological Systems Section 9.6 Preparing for the

9.4Step-By-StepGuidetoBiochemistryPassages

OUTLININGTHEPASSAGE

•Scanforstructure•Determinewhethertodothispassagenoworlater.• Identify the structure of the passage, including charts, graphs, chemical equations,synthesispathways,metabolicpathways,andimages.

•Readstrategically•Identifythetypeofpassage•Payspecialattentiontotherelationshipsbetweenconcepts•Inanexperimentpassage,determinethehypothesis,procedure,andoutcome• In an information passage, identify how the information in each paragraph fitstogethertopresentaunifiedpicture•Identifywhatinformationispresentedineachfigureorimage

•Labeleachcomponent•Writedownthepurposeofeachparagraphandfigure•Identifyanyconnectionsbetweenconceptswithinthepassage

•Reflectonyouroutline•Determinethegoalofthepassageandwriteitdown•Identifytheconceptswithinthepassageinanefforttoanticipatequestions

ANSWERINGTHEQUESTIONS

1.Assessthequestion•Determinewhetherthisquestionshouldbedonenoworlater.•Identifythetopicandthedegreeofdifficulty.•Goodquestionstodonowinbiochemistryareonesthatstandalonefromthepassageordonotrequireextensivedataanalysisinordertodeterminethecorrectanswer.

2.Planyourattack•Determinewhatyoualreadyknow,andwhatinformationyouneed.•Identifywheretofindtherequiredinformation:thepassage,thequestion,youroutline,oryourownknowledge.• If youhave to goback to the passage, determinewhere to find the required informationbyusingyouroutline.• If data analysis is required, identify the correct data set, as there may be multiple datarepresentations.

3.Executeyourplan•Analyzethedata,gobacktothepassage,andcarryoutyourplan.•Ifyougetstuckanalyzingdata,rememberthatthetrendofthedataisoftenenoughtoyielda

Page 129: Edited By Deeangelee Pooran-Kublall, MD/MPH · 9.5 Preparing for the MCAT: Biochemistry in the Chemical and Physical Foundations of Biological Systems Section 9.6 Preparing for the

correctanswerchoice.

4.Answerthequestionbymatching,eliminating,orguessing•Matchyouranswertotheanswerchoices.•Ifthereisnotmatch,eliminateincorrectanswerchoices.Someoftheanswerchoicesmaynotmakesense;eliminatethosefirst.•Ifeliminationdoesnotprovideaclearanswer,guessbetweentwoprobableanswers.

Page 130: Edited By Deeangelee Pooran-Kublall, MD/MPH · 9.5 Preparing for the MCAT: Biochemistry in the Chemical and Physical Foundations of Biological Systems Section 9.6 Preparing for the

9.5PreparingfortheMCAT:BiochemistryintheChemicalandPhysicalFoundationsofBiologicalSystemsSectionThese are the biochemistry topics that you are likely to see on TestDaywithin theChemical andPhysicalFoundationsofBiologicalSystemssection.

ACID/BASEEQUILIBRIA•Definitionandidentificationofacids/bases,theirconjugates,andtheirstrengths•Descriptionofcommonbufferingsystemsandequilibriumconstants

IONSINSOLUTIONS•Defineanions,cations,andrecognizecommonions,includingammoniumandphosphate,etc.•Understandhydrationandthehydroniumion

SEPARATIONSANDPURIFICATIONS•Basicconceptsofseparationandpurificationofproteinsandpeptides•Theprocessofelectrophoresisandthesubstancesthatcanbeseparatedusingthismethod•Quantitativeanalysisofyieldsfromseparationsandpurifications•Themechanismofchromatographyandthespecifictypesofchromatography,includingsize-exclusion,ion-exchange,andaffinity

NUCLEOTIDESANDNUCLEICACIDS•Definitionsandcompositionsofnucleotidesandnucleosides•Thecharacteristicsandstructureofthesugarphosphatebackbone•Structuresandidentitiesofpyrimidinesandpurines•Basicchemistryofnucleotidesandnucleicacids•Additionalfunctionsofnucleotidesandnucleicacids

AMINOACIDS,PEPTIDES,ANDPROTEINS•Describingaminoacidsintermsofabsoluteconfigurationattheαposition•Thedefinitionofdipolarions•Classificationofaminoacidsasacidic,basic,hydrophilic,orhydrophobic•Reactionsofpeptidesandproteins•Identificationandpurposeofsulfurlinkagesforcysteineandcystine•Definitionandidentificationofpeptidelinkageswithinpolypeptidesandproteins•Definitionandoutcomeofhydrolysis•Structureofproteins,includingprimary,secondary,andtertiarystructures•Definitionanduseoftheisoelectricpoint

Page 131: Edited By Deeangelee Pooran-Kublall, MD/MPH · 9.5 Preparing for the MCAT: Biochemistry in the Chemical and Physical Foundations of Biological Systems Section 9.6 Preparing for the

THREE-DIMENSIONALPROTEINSTRUCTURE•Creationandmaintenanceofconformationalstability,includinghydrophobicinteractionsandsolvationlayers(entropy)•Definitionandcharacteristicsofquaternarystructure•Changesinproteinstructureasaresultofdenaturingandfolding

NON-ENZYMATICPROTEINFUNCTION• Protein functions that are not enzymatic in nature, including binding, immunity, andmotorfunctions

LIPIDS•Typesoflipids,includingthoseusedforstorage,structure,andsignaling/cofactors•Identificationofstoragelipids,includingtriacylglycerols•Identificationoffreefattyacids•Saponificationanditsuses•Identificationofstructurallipids,includingphospholipids,phosphatides,andsphingolipids•Identificationandpurposeofwaxes• Identification of signaling lipids and cofactors, including fat-soluble vitamins, steroids, andprostaglandins

CARBOHYDRATES•Definitionsandcharacteristicsofdisaccharidesandpolysaccharides

PHENOLS•Oxidationandreductionreactionsofphenols,includinghydroquinonesandubiquinones•Identificationofbiological2e–redoxcenters

POLYCYCLICHETEROCYCLICAROMATICCOMPOUNDS•Identificationandunderstandingofbiologicalaromaticheterocycles

ENZYMES•Classificationofenzymesbyreactiontype•Mechanismsof enzymes, including substrates and enzyme specificity, cofactors, coenzymes,andvitamins•Theoriesofenzymefunction,includingtheactivesitemodelandtheinduced-fitmodel•Kineticsofenzymes,includingcatalysis,Michaelis–Menten,enzymecooperativity,andeffectsoflocalconditionsonenzymefunction•Mechanismsofenzymeinhibition

Page 132: Edited By Deeangelee Pooran-Kublall, MD/MPH · 9.5 Preparing for the MCAT: Biochemistry in the Chemical and Physical Foundations of Biological Systems Section 9.6 Preparing for the

•Regulationofenzymesbyallostericbindingandcovalentmodification

PRINCIPLESOFBIOENERGETICS•Thermodynamicsandbioenergetics, includingfreeenergy,Keq,and therelationshipbetweenbioenergeticsandconcentration• Bioenergetics of phosphorylation and ATP, including identification of the ΔG of ATPhydrolysisasmuchlessthan0•ATPgrouptransfers• Identification of biological oxidation–reduction reactions, including half-reactions, solubleelectroncarriers,andflavoproteins

Page 133: Edited By Deeangelee Pooran-Kublall, MD/MPH · 9.5 Preparing for the MCAT: Biochemistry in the Chemical and Physical Foundations of Biological Systems Section 9.6 Preparing for the

9.6PreparingfortheMCAT:BiochemistryintheBiologicalandBiochemicalFoundationsofLivingSystemsSectionTheseare thebiochemistry topics thatyouare likely to seeonTestDaywithin theBiological andBiochemicalFoundationsofLivingSystemssection.

AMINOACIDS•Describingaminoacidsintermsofabsoluteconfigurationattheαposition•Thedefinitionofdipolarions•Classificationofaminoacidsasacidic,basic,hydrophilic,orhydrophobic•Reactionsofaminoacids,peptides,andproteins•Identificationandpurposeofsulfurlinkagesforcysteineandcystine•Definitionandidentificationofpeptidelinkageswithinpolypeptidesandproteins•DefinitionandProductsofhydrolysis

PROTEINSTRUCTURE•Definitionsandcomponentsofprimary,secondary,andtertiarystructuresofproteins•The roleofproline, cystine, andhydrophobicbonding in the formation andmaintenanceoftertiarystructure•Definitionandcharacteristicsofquaternarystructureofproteins•Factorsthatcontributetoconformationalstability•Changesinstructureasaresultofdenaturingandfolding•Theroleofhydrophobicinteractionsinthemaintenanceofconformationalstability•Definitionofthesolvationlayerandhowentropyrelatestoproteinconformation• Identification of techniques used to separate proteins, including isoelectric point andelectrophoresis

NON-ENZYMATICPROTEINFUNCTION•Proteinfunctionssuchasbinding,immunity,andmotors

ENZYMESTRUCTUREANDFUNCTION•Theroleofenzymesinbiologicalreactions•Classificationofenzymesbythetypeofreactioncatalyzed•Definitionofactivationenergyandidentificationofhowenzymeslowertheactivationenergy•Enzymespecificityforacertainsubstrateandhowthiscontributestobiologicalreactions•Modelsofenzymefunction,includingtheactivesitemodelandtheinduced-fitmodel•Theroleandfunctionofzymogenscofactors,coenzymes,andwater-solublevitamins•Howlocalconditionscanalterenzymefunction

Page 134: Edited By Deeangelee Pooran-Kublall, MD/MPH · 9.5 Preparing for the MCAT: Biochemistry in the Chemical and Physical Foundations of Biological Systems Section 9.6 Preparing for the

CONTROLOFENZYMEACTIVITY• Enzyme kinetics, including the role of enzymes in catalysis, Michaelis–Menten, andcooperativity•Definitionandroleoffeedbackinhibition• Definitions of the types of inhibition, including competitive, noncompetitive, mixed, anduncompetitiveinhibition• The role and function of regulatory enzymes, such as allosteric enzymes and covalentlymodifiedenzymes•Theroleandfunctionofzymogens

NUCLEICACIDSTRUCTUREANDFUNCTION•Overalldescriptionofnucleicacidsandhowthesesubstancesfunctioninlivingsystems•Definitionsofnucleotidesandnucleosides, including thesugarphosphatebackbone,purines,andpyrimidines•IdentificationofthestructureofDNA,includingthedoublehelixandtheWatson–CrickmodelofDNAstructure•Identificationofproperbasepairing;AwithT,GwithC•ChangesinDNAstructureasaresultofdenaturation,reannealing,andhybridization

PRINCIPLESOFBIOENERGETICS•Generalconceptsofbioenergeticsandthermodynamics•ThedefinitionoffreeenergyandKeq;theequilibriumconstant•HowtheequilibriumconstantisrelatedtotheGibbsfreeenergy•Theroleofconcentrationonbioenergeticsandthermodynamics,especiallywithregardstoLeChâtelier ’sprinciple•Identificationandcharacteristicsofendothermicandexothermicreactions•ThedefinitionandsignificanceofGibbsfreeenergy,ΔG•TheroleofΔG°indeterminingspontaneityofareaction•ThethermodynamicsofphosphorylgrouptransfersandATP•TheroleofΔGinATPhydrolysis,ΔGismuchlessthan0;ATPgrouptransfers•Identificationandunderstandingofbiologicaloxidation–reductionreactions,andtherolesofhalf-reactions,solubleelectroncarriers,andflavoproteins

CARBOHYDRATES•Propernomenclature,classification,andcommonnameswhendiscussingcarbohydrates•Identificationoftheabsoluteconfigurationofcarbohydrates•Theconformationsofhexoses,includingtheircyclicstructure•Identificationofanomersandepimers•Theglycosidicbondandhydrolysis•Definitionsandstructuresofmonosaccharides,disaccharides,andpolysaccharides

Page 135: Edited By Deeangelee Pooran-Kublall, MD/MPH · 9.5 Preparing for the MCAT: Biochemistry in the Chemical and Physical Foundations of Biological Systems Section 9.6 Preparing for the

GLYCOLYSIS,GLUCONEOGENESIS,ANDTHEPENTOSEPHOSPHATEPATHWAY

•Pathwaysofglycolysisunderaerobicconditionsincludingtheirsubstratesandproducts•Pathwaysthatfeedintoglycolysis,especiallyglycogenandstarchmetabolism•Fermentationunderanaerobicconditions•Gluconeogenesis•Pentosephosphatepathway•Theresultsofrespiration,includingthenetmoleculesformedandnetenergy

PRINCIPLESOFMETABOLICREGULATION•Identificationofhowmetabolicpathwaysareregulated,aswellashowthebodyismaintainedinadynamicsteadystate•Howthepathwaysofglycolysisandgluconeogenesisareregulated•Pathwayofglycogenmetabolism• How glycogen synthesis and breakdown is regulated based on conditions within the body,includingallostericandhormonalmechanisms•Definitionofmetaboliccontrolandanalysisofthemechanismsthatenablemetaboliccontrol

CITRICACIDCYCLE•Productionofacetyl-coAbyconversionofpyruvatebythepyruvatedehydrogenasecomplex•Thereactionsofthecitricacidcycle,includingsubstratesandproducts•Howthecycleisregulatedbyconditionswithinthebody•Moleculesrequiredforthecitricacidcycleaswellasnetenergyandproducts

METABOLISMOFFATTYACIDSANDPROTEINS•Identificationofthecomponentsandcharacteristicsoffattyacids•Fattransport,digestion,andmobilization•Fattyacidoxidationofbothsaturatedfatsandunsaturatedfattyacids•Productionanduseofketonebodies

OXIDATIVEPHOSPHORYLATION•Thestructureandroleoftheelectrontransportchaininoxidativephosphorylation•Substratesandproductsoftheoxidativephosphorylationpathway•Generalfeaturesofoxidativephosphorylation• The process of electron transfer in mitochondria and the use of electron carriers (NADH,NADPH),flavoproteins,andcytochromes• Function and structure ofATP synthase and the role of chemiosmotic coupling in oxidativephosphorylation•Howoxidativephosphorylationisregulated•Theroleofmitochondriainapoptosisandoxidativestress

Page 136: Edited By Deeangelee Pooran-Kublall, MD/MPH · 9.5 Preparing for the MCAT: Biochemistry in the Chemical and Physical Foundations of Biological Systems Section 9.6 Preparing for the

HORMONALREGULATIONANDINTEGRATIONOFMETABOLISM•Howhormonestructureandfunctionareintegratedatahigherlevel•Metabolismspecifictocertaintissues•Regulationoffuelmetabolismbyhormones•Bodymassregulationandobesity

PLASMAMEMBRANE•Generalfunctionandcomposition•Identificationandroleoflipidsandproteinsintheplasmamembrane•Lipidcomponents,includingphospholipids,phosphatides,steroids,andwaxes•Fluidmosaicmodelofplasmamembranestructure•Dynamicsoftheplasmamembrane•Transport of solutes across theplasmamembrane, including thermodynamic considerations,osmosis,passivetransport,andactivetransport•Functionandroleofthesodium–potassiumpump•Descriptionofmembranechannels,membranepotential,andmembranereceptors•Therolesofexocytosisandendocytosisinmembranetraffic

MEMBRANE-BOUNDORGANELLESANDDEFININGCHARACTERISTICSOFEUKARYOTICCELLS

•Thestructureofthemitochondrialinnerandoutermembrane

BIOSIGNALLING•Theroleofoncogenes•Theprocessofapoptosis•Typesofgated-ionchannels,includingvoltage-gatedandligand-gatedchannels•IdentificationandfunctionofG-protein-coupledreceptors•Identificationandrolesofreceptorenzymes

LIPIDS•Structureanddescriptionoflipids,includingsteroids,terpenes,andterpenoids

Page 137: Edited By Deeangelee Pooran-Kublall, MD/MPH · 9.5 Preparing for the MCAT: Biochemistry in the Chemical and Physical Foundations of Biological Systems Section 9.6 Preparing for the

9.7BiochemistryWorkedExamplesPASSAGEI:OXIDATIVEPHOSPHORYLATIONMitochondrial function isheavily regulatedbycalcium ions.Whencytosolic [Ca2+] levels increase,that increaseis transmittedtothemitochondria,whereCa2+stimulatestheactivityofα-ketoglutaratedehydrogenase, isocitrate dehydrogenase, and pyruvate dehydrogenase. Excessive mitochondrial[Ca2+],however,cantriggermitochondrialreleaseofcytochromec.Derangementsofmitochondrial[Ca2+]arebelievedtoplayaroleinthedamagetoheartmusclethatresultsfromthechangesinbloodflowthatoccurinheartattacks.

Several proteins are involved in regulating mitochondrial calcium levels. One of these is themitochondrialcalciumuniporter,ormCU.mCU,whichhaslowaffinityforCa2+butahighcapacity,allowspassive transportofCa2+ into themitochondrialmatrix.Thisoccursbecause there isa largevoltagegradient across the innermitochondrialmembrane (approximately–200mVon thematrixside).

Anotherprotein that regulatesmitochondrial [Ca2+] is thesodium–calciumexchanger, orNCX.Thisantiportproteinisfoundbothintheinnermitochondrialmembraneandintheplasmamembraneofcardiacmyocytes.Initsnormal“forward”mode,NCXremovesoneCa2+ionfromthemitochondrialmatrixandallowsthreeNa+toenter.Inits“reverse”mode,NCXbringsoneCa2+ionintothematrixandremovesthreeNa+.

Theflowofsodiumionsacrosstheinnermitochondrialmembraneisintimatelylinkedtotheflowofprotonsbythesodium–protonexchanger(NHE),whichcanalsoruninreverse.Therefore,theflowofcalcium also affects the flow of protons across the inner mitochondrial membrane, as depictedschematically in Figure 1. Note that because of pores in the outer mitochondrial membrane, ionconcentrationsintheintermembranespaceareessentiallyidenticalwiththoseinthecytosol.

Figure1.Ioncarriersintheinnermitochondrialmembrane

P1.

P2.

P3.

P4.

Page 138: Edited By Deeangelee Pooran-Kublall, MD/MPH · 9.5 Preparing for the MCAT: Biochemistry in the Chemical and Physical Foundations of Biological Systems Section 9.6 Preparing for the

F1.

1.Basedoninformationinthepassage,whichofthefollowingisthemostlikelyexplanationforwhytheforwardmodeofNCXtransportsthreesodiumionsintothematrixforeveryonecalciumionittransportsout?

A.Asaresult,theantiporterdoesnotconsumeATP.B.NCXevolvedfromtheNa+/K+ATPase.C.ItreducesthetendencyofCa2+tore-enterthematrix.D.ItenhancesATPsynthase’sabilitytoproduceATP.

2.Basedoninformationinthepassage,mCUmostlikelyhas:A.arelativelyhighKmvalueandarelativelyhighvmax.

B.arelativelyhighKmvalueandarelativelylowvmax.

C.arelativelylowKmvalueandarelativelyhighvmax.

D.arelativelylowKmvalueandarelativelylowvmax.

3.ArecentlydiscoveredproteinintheintermembranespaceofmitochondriainratcardiacmyocytesbindsbothCa2+andATPsynthase.Abiochemiststudyingitsfunctioninisolatedmitochondriafoundthefollowing:

IntermembraneCa/ATPase-bindingproteinconcentration,μM

IntermembraneCa2+

concentration,μMATPsynthaseactivity

(arbitraryunits)0.0 0.0 12.50.1 0.0 9.40.5 0.0 1.50.5 2.0 11.9

Whichofthefollowingconclusionsregardingthisproteinismostreasonable?A.Whentheproteinbindscalciumions,itbindstoATPsynthase,increasingATPsynthaseactivity.B.Theprotein,whichnormallybindsATPsynthase,losesitsabilitytobindATPsynthasewhenitbindstocalcium.C.TheproteinhasahigheraffinityforcalciumthanitdoesforATPsynthase.D.Theproteinisoneofthecalcium-activateddehydrogenasesmentionedinthepassage.

4.AbiochemiststudyingmCUhastwosamplesofmitochondria.Toone,thebiochemistaddsenoughrutheniumred,aknownmCUinhibitor,tocompletelyinhibitmCU.Bothsamplesaregivenradiolabeledglucose.Thebiochemistcouldmostreasonablypredictthattherutheniumredsample,comparedtothecontrol,would:

A.exhibitcompletecessationofATPsynthesis.B.haveincreasedproductionofradiolabeledcarbondioxide.

Page 139: Edited By Deeangelee Pooran-Kublall, MD/MPH · 9.5 Preparing for the MCAT: Biochemistry in the Chemical and Physical Foundations of Biological Systems Section 9.6 Preparing for the

C.exhibitalong-termdecreaseinATPsynthesis.D.haveincreasedproductionofradiolabeledATP.

5.Duringaheartattack,cytosolicCa2+levelsoftenincrease.AnincreaseinCa2+intheintermembranespaceofmitochondriawouldmostlikelycause:

A.increasedATPsynthesisasaresultofH+flowintothemitochondrialmatrix.B.increasedATPsynthesisasaresultofH+flowoutofthemitochondrialmatrix.C.decreasedATPsynthesisasaresultofH+flowintothemitochondrialmatrix.D.decreasedATPsynthesisasaresultofH+flowoutofthemitochondrialmatrix

Page 140: Edited By Deeangelee Pooran-Kublall, MD/MPH · 9.5 Preparing for the MCAT: Biochemistry in the Chemical and Physical Foundations of Biological Systems Section 9.6 Preparing for the

BiochemistryPassageIExplanation:

USINGTHEKAPLANMETHODS

P1.Calciumaffectsmitochondrialactivity

P2.mCUmovescalciumintomatrix

P3.NCXantiporter:3Na+for1Ca2+

P4.NHEantiporter:1Na+for1H+

Fig.1Ioncarriersininnermitochondrialmembrane

KeyConceptsSkill2—ScientificReasoningandProblem-SolvingMembranePotentialOxidativePhosphorylation

1.Basedoninformationinthepassage,whichofthefollowingisthemostlikelyexplanationforwhytheforwardmodeofNCXtransportsthreesodiumionsintothematrixforeveryonecalciumionittransportsout?

A.Asaresult,theantiporterdoesnotconsumeATP.B.NCXevolvedfromtheNa+/K+ATPase.C.ItreducesthetendencyofCa2+tore-enterthematrix.D.ItenhancesATPsynthase’sabilitytoproduceATP.

AssessthequestionThewordsmostlikelyinthequestionstemareasignthatweareexpectedtoreasonouttheanswertothisquestion.ThisquestionwantstoknowthereasonwhyNCXexhibitsa3:1chargeratio.Whenweseeawhyquestion,weshouldalwaysaskourselvestwoquestionsaboutthecorrectanswer:Isittrue?Isitrelevant?Iftheanswertoeitherquestionisno,wecaneliminatethatanswerchoice.

Planyourattack

Page 141: Edited By Deeangelee Pooran-Kublall, MD/MPH · 9.5 Preparing for the MCAT: Biochemistry in the Chemical and Physical Foundations of Biological Systems Section 9.6 Preparing for the

Asthequestionstemnotes,NCXnormallyejectsoneCa2+ionforeverythreeNa+ionsthatenterthematrix.ThenetresultofthisisthateachtimeNCXoperatesintheforwardmode,itmovesachargeintothematrix.

Paragraph 2 of the passage states that there is large gradient across the inner mitochondrialmembrane, with thematrix side at –200mV, and that this drives the passive flow of Ca2+ into thematrixthroughthemCU.

ExecutetheplanCombiningthesetwofacts,wecanpredictthateachtimetheNCXfiresinitsforwardmode,theinnermitochondrialmembrane isdepolarized: the gradient becomes smaller.Our correct answer shoulddeal,insomeway,withthisdecreaseinthemembranepotential.

Answerbymatching,eliminating,orguessingLet’scycle through theanswerchoices. (A) is true,but it is irrelevant; thequestionaskswhy NCXworks the way it does. (B) says that NCX evolved from the sodium–potassium pump. Thismightsound tempting—after all, the sodium–potassiumpumpmoves three sodium ionsoutof thecell tomovetwopotassiumionsintothecell.It’sastretch,though,toreachtheconclusionthatNCXevolvedfromit.

Choice(C),however,fitsthebill.Ifwe’reejectingCa2+fromthematrix,wedon’twantittojustre-enter thematrix again.Depolarizing themembranewould reduce the tendency ofCa2+ to enter thematrixviathemCU,whichwouldpreventthatfromhappening.

(D),ontheotherhand,issimplyfalse.ThechemiosmotichypothesisstatesthatATPsynthesisdependsontheexistenceofagradientacrosstheinnermitochondrialmembrane.DepolarizingthatmembranewouldtendtoreduceATPsynthase’sabilitytomakeATP,notincreaseit.

TakeawaysWhenasciencequestionasksforthemostlikelyexplanation,expecttocombinefacts.

ThingstoWatchOutForWatchoutforsuperficialsimilarities.Whilethesodium–potassiumpumphasthesamechargebalanceasNCX,that’snotproofthattheNCXevolvedfromit.

KeyConcepts

Page 142: Edited By Deeangelee Pooran-Kublall, MD/MPH · 9.5 Preparing for the MCAT: Biochemistry in the Chemical and Physical Foundations of Biological Systems Section 9.6 Preparing for the

Skill1—ScientificReasoningandProblem-SolvingEnzymeKinetics

2.Basedoninformationinthepassage,mCUmostlikelyhas:A.arelativelyhighKmvalueandarelativelyhighvmax.

B.arelativelyhighKmvalueandarelativelylowvmax.

C.arelativelylowKmvalueandarelativelyhighvmax.

D.arelativelylowKmvalueandarelativelylowvmax.

AssessthequestionThisisaSkill1question,askingustosimplyidentifytwofactsaboutmCU.First,doesmCUhavearelativelyhighvalueforKm (theMichaelisconstant),orarelativelylowvalue?Second,doesmCUhavearelativelyhighvmax(maximumvelocity),orarelativelylowvmax?

PlanyourattackmCUisdiscussedinparagraph2,wherewe’retoldthatmCUhasalowaffinityforcalciumions,butahighcapacity.

Thepassagedoesn’t tellus, though,howthatcorrelates toKmandvmax, sowe’llneed topull thosefrommemory.

ExecutetheplanOfthetwo,vmaxissimpler,solet’sstartthere.Inenzymekinetics,vmax isameasureofhowfastanenzymecanconvert itssubstratetoproduct.Sincewe’retoldthatmCUhasahighcapacity,wecanassumethatitcancarryoutitsfunctionquickly,whichmeansitshouldhaveahighvalueforvmax.

Km, theMichaelis constant, represents the substrate concentrationatwhich theenzyme’svelocity isone-halfofvmax.SincethepassagesaysthatmCUhasa lowaffinity forcalciumions, thatmeans itdoesnotbindCa2+verywell.WewouldexpectthatitwouldtakearelativelyhighconcentrationofCa2+

toreachvmax,soKmshouldalsoberelativelyhigh.

Page 143: Edited By Deeangelee Pooran-Kublall, MD/MPH · 9.5 Preparing for the MCAT: Biochemistry in the Chemical and Physical Foundations of Biological Systems Section 9.6 Preparing for the

Answerbymatching,eliminating,orguessingOnceweknow thatvmax should be relatively high,we can eliminate (B) and (D). The fact thatKmshouldalsoberelativelyhigheliminatestheoneremainingwronganswerchoice,(C),leavingchoice(A)asthecorrectanswer.

TakeawaysWhenquestionshavemultipartanswers,youshouldstarteliminatingwronganswerschoiceassoonasyou’vefiguredoutonepart.

KeyConceptsSkill4—Data-BasedandStatisticalReasoningEnzymeBindingEnzymeInhibition

3.ArecentlydiscoveredproteinintheintermembranespaceofmitochondriainratcardiacmyocytesbindsbothCa2+andATPsynthase.Abiochemiststudyingitsfunctioninisolatedmitochondriafoundthefollowing:

IntermembraneCa/ATPase-binding

proteinconcentration,μMIntermembraneCa2+

concentration,μMATPsynthaseactivity

(arbitraryunits)0.0 0.0 12.50.1 0.0 9.40.5 0.0 1.50.5 2.0 11.9

Whichofthefollowingconclusionsregardingthisproteinismostreasonable?A.Whentheproteinbindscalciumions,itbindstoATPsynthase,increasingATPsynthaseactivity.B.Theprotein,whichnormallybindsATPsynthase,losesitsabilitytobindATPsynthasewhenitbindstocalcium.C.TheproteinhasahigheraffinityforcalciumthanitdoesforATPsynthase.D.Theproteinisoneofthecalcium-activateddehydrogenasesmentionedinthepassage.

Page 144: Edited By Deeangelee Pooran-Kublall, MD/MPH · 9.5 Preparing for the MCAT: Biochemistry in the Chemical and Physical Foundations of Biological Systems Section 9.6 Preparing for the

AssessthequestionThepresenceofatableinthequestionstemshouldimmediatelytellyouthatthisisaSkill4question,asking you for data-based reasoning. The question stem is asking us to draw themost reasonableconclusionamong the four listed; theremightbebetterones,but those fourare theonlyones thatmatteronTestDay.

Moreover, thisquestiontalksaboutacompletelynewexperimentthat isonlytangentiallyrelatedtothe topic of the passage, making it a good candidate for triaging. Finally, looking at the answerchoices,wecanseethatourconclusionsaddressthefunctionoftheproteinthatbindsCa2+ ionsandATPsynthase.

PlanyourattackTheredoesn’tappeartobeanyinformationinthepassagethatwillhelpusanswerthisquestion.Toanswerit,we’llneedtoexaminethetableandseehowchangesincalciumandproteinconcentrationsaffect theactivityofATPsynthase.Thenwe’llneedtouseourunderstandingofenzymebindingtodrawaconclusion.

ExecutetheplanThefirsttrialisourcontrol;withzeroproteinandzerocalcium,ATPsynthasehasarelativelyhighactivity.Inthesecondandthirdtrials,thebiochemistincreasestheconcentrationoftheATPsynthase-binding protein, but leaves out calcium.What happens?ATP synthase activity drops, and themoreprotein we add, the more it drops. So we can reasonably conclude that the protein inhibits ATPsynthase.

Looking at the last trial, though, the biochemist adds a lot of calcium to the intermembrane spacewhilealsoaddinga lotof theprotein.Whathappensnow?ATPsynthaseactivity isnearlynormal!This suggests that calcium acts as anantagonist to this protein, preventing it from inhibitingATPsynthaseactivity.Nowthatweknowwhathappensinthistrial,wecanlookforananswerchoicethatmatchesourfindings.

Answerbymatching,eliminating,orguessingScanningtheanswers,onlyone—choice(B)—agreeswithouranalysis.Alloftheothersfailinsomesignificantway.

(A)mightbetempting,sinceATPsynthaseactivitydoesrisewhencalciumisaddedinthefourthtrial,butitdoesn’texplainwhyATPsynthaseactivitydecreaseswhentheproteinisaddedintheabsenceofcalcium.

Page 145: Edited By Deeangelee Pooran-Kublall, MD/MPH · 9.5 Preparing for the MCAT: Biochemistry in the Chemical and Physical Foundations of Biological Systems Section 9.6 Preparing for the

Wedon’thaveenoughinformationtoadequatelydecideon(C).Weknowthat2.0μMofcalciumionsisenoughtomore-or-lesscompletelyrestoreATPsynthaseactivity,but,forallweknow,thatcouldbeanenormousexcess.Moreover,wecan’tbesurewhat theprotein’saffinityforATPsynthaseis;evenwhenweadd0.5μMofprotein,that’snotenoughtocompletelyinhibitATPsynthase.

(D)suffersfromthesameproblemas(A):whilethatmightbeareasonableinferencefromthelasttwo trials, it doesn’t explain the overall data. Even worse, the dehydrogenases mentioned in thepassagearepartoftheKrebscycle,whichtakesplaceinthemitochondrialmatrix.Thequestionstemclearlystatesthatthisproteinwasdiscoveredintheintermembranespace,notthematrix.

ThingstoWatchOutForWhenaquestionasksWhichofthefollowingismostlikely?,rememberthatyou’relimitedtothefourchoicesintheanswerquestion.Don’twastetimetryingtocomeupwiththe“best”possibleconclusion!

KeyConceptsSkill3—ReasoningabouttheDesignandExecutionofResearchKrebsCycleElectronTransportChainOxidativePhosphorylation

4.AbiochemiststudyingmCUhastwosamplesofmitochondria.Toone,thebiochemistaddsenoughrutheniumred,aknownmCUinhibitor,tocompletelyinhibitmCU.Bothsamplesaregivenradiolabeledglucose.Thebiochemistcouldmostreasonablypredictthattherutheniumredsample,comparedtothecontrol,would:

A.exhibitcompletecessationofATPsynthesis.B.haveincreasedproductionofradiolabeledcarbondioxide.C.exhibitalong-termdecreaseinATPsynthesis.D.haveincreasedproductionofradiolabeledATP.

AssessthequestionThisisanexampleofaSkill3question,whichgivesyoudetailsofanewexperiment,andthenaskswhattheresearchercouldpredict.TwooftheanswersdealwithdecreasesinATPsynthesis,whiletheothertwodealwiththeradiolabeledcarbon.

Page 146: Edited By Deeangelee Pooran-Kublall, MD/MPH · 9.5 Preparing for the MCAT: Biochemistry in the Chemical and Physical Foundations of Biological Systems Section 9.6 Preparing for the

PlanyourattackThe first step here is to figure out what effect completely inhibiting mCU would have on themitochondrion.Thatinformationisinparagraph2,whichsaysthatmCUtransportscalciumintothematrix.Butwhateffectdoesthathaveonthemitochondrion?Toanswerthatquestion,weneedtolookatparagraph1,whichsaysthatcalciumionsupregulatetheactivityofdehydrogenasesintheKrebscycle.

So now we need to consider how the Krebs cycle feeds into ATP synthesis in oxidativephosphorylation.Thatinformationisnotinthepassage,soit’llneedtocomefromourknowledgeofaerobicrespiration.WemayalsoneedtouseourknowledgeofhowATPsynthasemakesATP.

ExecutetheplanIfweinhibitmCUcompletely,thatwouldreduceCa2+levelsinthemitochondrialmatrix.Accordingtoparagraph 1, that would reduce flux through the Krebs cycle, whichwouldmean less NADH andFADH2wouldbeproduced.IfwehavelessNADHandFADH2, thatnecessarilymeansthatwehavelesssubstrateavailablefortheelectrontransportchain,andthereforewewouldexpectadecrease inATPsynthesis.

Answerbymatching,eliminating,orguessingScanning the answer choices,we can eliminate two answers immediately. (B) can’t be the answer,because it would require increased flux through theKrebs cycle. (D) also doesn’t work: not onlywoulditrequireATPsynthesis,butthecarbonsfromglucosearenotdirectlyincorporatedintoATPduringATPsynthesis.

Thatleavestwochoices:(A),whichsaysthatATPsynthesisstopscompletely,and(C),whichsaysitdecreases.Todecidewhichiscorrect,weneedtolookatparagraph1,whichsaysthatCa2+stimulatesenzymesintheKrebscycle.Stimulatesmeansthatcalciumincreasestheiractivity,butisnotrequiredfor theiractivity.Thus,wecanpredict thata lackofcalciumshouldslowdowntheseenzymes,notshut themdown.ATPsynthesisshould thereforedecrease,notstop,so thecorrectanswer ischoice(C).

MCATExpertiseForTestDay,youareresponsibleforunderstandingATPsynthesisatthelevelofdetailpresentedinanintroductorybiochemistrycourse,whichwouldincludethefactthatcarbonsfromglucosearenotdirectlyincorporatedintoATP.Youarenotrequired,however,toknowhowadenosineandothernucleotidesareactuallysynthesized.

Page 147: Edited By Deeangelee Pooran-Kublall, MD/MPH · 9.5 Preparing for the MCAT: Biochemistry in the Chemical and Physical Foundations of Biological Systems Section 9.6 Preparing for the

TakeawaysWhenanswerchoicescovertwo(ormore)differenttopics,youmayneedtoconsidermultipleissuestodeterminethecorrectanswer.

KeyConceptsSkill2–ScientificReasoningandProblem-SolvingElectronTransportChainOxidativePhosphorylation

5.Duringaheartattack,cytosolicCa2+levelsoftenincrease.AnincreaseinCa2+intheintermembranespaceofmitochondriawouldmostlikelycause:

A.increasedATPsynthesisasaresultofH+flowintothemitochondrialmatrix.B.increasedATPsynthesisasaresultofH+flowoutofthemitochondrialmatrix.C.decreasedATPsynthesisasaresultofH+flowintothemitochondrialmatrix.D.decreasedATPsynthesisasaresultofH+flowoutofthemitochondrialmatrix.

AssessthequestionThisisaclassicSkill2question,askingwhatwouldhappenifcytosolicCa+ increases.Theanswersask about whether ATP synthesis will increase or decrease, and the direction of flow of protonsacrosstheinnermitochondrialmembrane.

PlanyourattackToanswerthisquestion,we’llneedtwopiecesofinformation.First,weneedtofigureoutwhateffectincreasedCa2+ levelshave in the intermembranespace.Theanswer to thatquestionwillcomefromFigure1.

Thesecondpieceof informationis theeffectof thatflowonATPsynthesis.We’llneedtopull thatfromourknowledgeoftheelectrontransportchainandoxidativephosphorylation.

ExecutetheplanAccordingtoFigure1, themCUbringsCa2+ in the intermembranespace into thematrix.WhenCa2+

Page 148: Edited By Deeangelee Pooran-Kublall, MD/MPH · 9.5 Preparing for the MCAT: Biochemistry in the Chemical and Physical Foundations of Biological Systems Section 9.6 Preparing for the

buildsupinthematrix,NCXexchangestheextracalciumforsodiumions,soNa+willaccumulateinthematrix.Accordingtoparagraph4,theNHEexchangesNa+forH+.SowhenCa2+buildsupintheintermembranespace,thenetresultistheflowofprotonsintothemitochondrialmatrix.

Butdoes thathelporhurtATPsynthesis?Accordingto thechemiosmotichypothesis,ATPsynthaseutilizestheprotongradientacrosstheinnermitochondrialmembraneastheenergysourceforATPsynthesis,andsendsprotons intothematrix todoso. (Theelectron transportchainuses theenergyfromoxidationofNADHandFADH2 tomoveprotons into the intermembranespace.)Theflowofprotonsasa resultofcalciumbuildup is thesameas theflowofprotons inATPsynthesis.Wecanconclude that the two processes compete for the same pool of protons, so ATP synthesis shoulddecline.

Answerbymatching,eliminating,orguessingOnce we predict that protons should flow into the mitochondrial matrix, we can immediatelyeliminate (B) and (D). Knowledge of the chemiosmotic hypothesis allows us to confidently pickchoice(C)asthecorrectanswer.

MCATExpertiseManyMCATquestionshavetwo-partanswers.Onceyoupredicthalftheanswer,it’sworthcheckingtheanswerchoices.Sometimes,onlyoneanswerchoicehasthatpartcorrect!

BIOCHEMISTRYPASSAGEII:METABOLICREGULATIONInhumans,theenzymeisocitratedehydrogenase(IDH)existsinmultipleforms;oneform,IDH1,isfound in the cytosol, while two others, IDH2 and IDH3, are expressed in themitochondria. IDH3existsasaheterotrimerofthreesubunits.InE.coli,afacultativeanaerobe,onlyoneIDHexists:itisadimerof416-subunitresidues.ThepercentageofresiduesinE.coliIDHthatareequivalenttothosein any form of human IDH is less than 15 percent. Unlike all human IDH forms,E. coli IDH isinhibitedbyphosphorylation.Moreover,unlikemostenzymesregulatedbyphosphorylation,E.coliIDHundergoesphosphorylationatitsactivesite.

IDHphosphorylationiscriticalinE.colibecausedecreasedIDHactivityincreasesflowthroughtheglyoxylatecycle,analternativemetabolicpathwayshownschematically inFigure1.Theglyoxylatecycle,whichisalsofoundinplants,isusedtobypasspartsofthecitricacidcyclewhentheorganismisdependingonfattyacidsand/oracetyl-CoAasitsprimarysourceofenergy.Intheglyoxylatecycle,isocitrate lyase converts isocitrate to glyoxylate and succinate. Malate synthase then combinesglyoxylateandamoleculeofacetyl-CoAtoformmalate.Isocitratelyaseandmalatesynthasearenotknown to exist in humans, although recent evidence suggests that at least some vertebrates mayproducethem.

Page 149: Edited By Deeangelee Pooran-Kublall, MD/MPH · 9.5 Preparing for the MCAT: Biochemistry in the Chemical and Physical Foundations of Biological Systems Section 9.6 Preparing for the

Figure1.Glyoxylatecycle

To study IDH regulation, researchers cultured cells, as shown in Table 1, inmedia containing 31P.After lysing the cells, SDS-PAGEwas runon an aliquot fromeach sample.AWesternblot,whichgivesaspot(apositiveresult)whenaproteincapableofbindingaparticularantibodyispresent,wasperformedonSamples1–4usinganantibodythatdetectsE.coliIDH.

Sample Cellsused Carbonsourceused1 E.coli 0.4%glucose2 E.coli 0.4%sodiumacetate3 Humanhepatocytes 0.4%glucose4 Humanhepatocytes 0.4%sodiumacetate5 E.coli 0.2%sodiumacetate6 E.coli 0.4%sodiumacetate+0.4%glucose

Table1.CellculturesusedtostudyIDHactivity

P1.

P2.

F1.

P3.

T1.

1.WhichofthefollowingmostlikelyshowstheresultoftheWesternblotexperimentdescribedin

Page 150: Edited By Deeangelee Pooran-Kublall, MD/MPH · 9.5 Preparing for the MCAT: Biochemistry in the Chemical and Physical Foundations of Biological Systems Section 9.6 Preparing for the

thepassage?(Note:thenumbers1through4correspondtothesamplesinTable1.)A.

B.

C.

D.

2.InE.coli,whichofthefollowingwouldbeexpectedtoincreaseasaconsequenceofadropinIDHactivity?

I.PyruvatekinaseactivityII.Acetyl-CoAlevelsIII.CO2production

A.IonlyB.IIonlyC.IandIIIonlyD.I,II,andIII

Page 151: Edited By Deeangelee Pooran-Kublall, MD/MPH · 9.5 Preparing for the MCAT: Biochemistry in the Chemical and Physical Foundations of Biological Systems Section 9.6 Preparing for the

3.Abiochemistdecidestoperformautoradiography,whichcanquantifytheamountofaradioactiveisotopepresent,onthesamplesdescribedinTable1.Basedoninformationinthepassage,hewouldpredictthatIDHfrom:

A.onlySample1wouldexhibit31Pactivity.B.onlySamples2and5wouldexhibit31Pactivity.C.onlySamples2,5,and6wouldexhibit31Pactivity.D.alltheIDHsamplesinTable1wouldexhibit31Pactivity.

4.Basedoninformationinthepassage,whichofthefollowingisthemostlikelyexplanationforhowphosphorylationinhibitsIDHinE.coli?

A.Phosphorylationcausesallostericchangestotheactivesitethatpreventisocitratefrombinding.B.Phosphorylationcauseselectrostaticrepulsionofthenegativelychargedsocitrate.C.PhosphorylationtargetsIDHfordestructionbylysosomes.D.PhosphorylationresultsinthedissociationoftheIDHdimer.

5.Inhumans,IDH3isNAD+-dependent,whileIDH1andIDH2aredependentonNADP+.WhichofthefollowingwouldbeLEASTlikelytobetrue?

A.ExpressionofIDH1isregulatedbymechanismsthathavenoeffectonIDH3expression.B.ThevaluesofKmandVmaxforIDH1andIDH2arelikelytodiffersignificantlyfromthoseforIDH3.C.ThegeneforIDH1andthegeneforIDH3arelocatedondifferentchromosomes.D.Invitro,theequilibriumratioofisocitratetoα-ketoglutarateusingIDH1equalstheratioforIDH3.

Page 152: Edited By Deeangelee Pooran-Kublall, MD/MPH · 9.5 Preparing for the MCAT: Biochemistry in the Chemical and Physical Foundations of Biological Systems Section 9.6 Preparing for the

BiochemistryPassageIIExplanation:

USINGTHEKAPLANMETHOD

P1.Isocitratedehydrogenasedifferentinhumans,E.coli

P2.Glyoxylatecycle

F1.GlyoxylatecycleandKrebscycle

P3.IDHregulationexperiment

T1.CellculturesusedinIDHregulationexperiment

KeyConceptsSkill4—Data-BasedandStatisticalReasoningNon-EnzymaticProteinFunction

1.WhichofthefollowingmostlikelyshowstheresultoftheWesternblotexperimentdescribedinthepassage?(Note:thenumbers1through4correspondtothesamplesinTable1.)

A.

B.

C.

Page 153: Edited By Deeangelee Pooran-Kublall, MD/MPH · 9.5 Preparing for the MCAT: Biochemistry in the Chemical and Physical Foundations of Biological Systems Section 9.6 Preparing for the

D.

AssessthequestionTheAAMChassaidthenewMCATwillhaveanincreasedfocusontestingyourabilitytointerpretdata.ThisSkill4question illustratesonewayofdoing that—givingyou figures to interpretas theanswerchoices.Thequestionstemisaskingwhichofthesefourgelsbestcorrespondstotheresultsof theWestern blot experiment in the passage.Note, though, that in every case, Sample 2 gives apositiveresult.

PlanyourattackToanswerthisquestion,weneedthebriefdescriptionofhowWesternblotsworkgiveninparagraph3.WealsoneedtoknowwhetherhumanIDHwillrespondtoanantibodyspecificforE.coliIDH;toanswerthatquestion,we’llneedinformationinparagraph1.

ExecutetheplanFirst, basedonparagraph3, aWesternblot produces a spotwhenever there’s a protein capable ofbindingtotheantibody.Inthiscase,we’reusinganantibodyforE.coliIDH.

Toanswerthesecondquestion,weneedtoknowifthehumanIDHwillbindtheantibody.Accordingtoparagraph1,there’slessthana15percentmatchbetweenthesequencesofE.coliIDHandhumanIDH.Ifthere’sthatlittlesimilarity,thenit’sreasonabletoconcludethattheantibodyforE.coli IDHwillnotbeabletobindhumanIDH.ThatmeansSamples3and4shouldgivenoresponse.

Page 154: Edited By Deeangelee Pooran-Kublall, MD/MPH · 9.5 Preparing for the MCAT: Biochemistry in the Chemical and Physical Foundations of Biological Systems Section 9.6 Preparing for the

Answerbymatching,eliminating,orguessingIfwelookattheanswerchoices,wecanimmediatelyeliminateboth(A)and(B):theyshowspotsforlanes3and4,wherethereshouldn’tbeany.

Thatleavesuswithtwochoices,(C)and(D).Todecidewhichoneiscorrect,weneedtoask:DoesE.coli always produce IDH? The answer to this question is yes. Paragraph 1 tells us E. coli is afacultative anaerobe, which means it is capable of performing aerobic respiration if oxygen ispresent.Inordertodothat,itneedstohavetheenzymesoftheKrebscycleavailable.Bothlanes1and2shouldthusyieldaspotonaWesternblot,sotheanswerischoice(C).

TakeawaysDon’tfallintothetrapof“compartmentalizing”knowledgeonTestDay.SomeMCATquestionsintegrateknowledgefrommultipleareas.

KeyConceptsSkill2—ScientificReasoningandProblem-SolvingPrinciplesofMetabolicRegulationCitricAcidCycle

2.InE.coli,whichofthefollowingwouldbeexpectedtoincreaseasaconsequenceofadropinIDHactivity?

I.PyruvatekinaseactivityII.Acetyl-CoAlevelsIII.CO2production

A.IonlyB.IIonlyC.IandIIIonlyD.I,II,andIII

AssessthequestionThisisaRomannumeralquestionthatasksuswhichofthesethreethingsincreaseswhenIDHactivitydecreases.

Page 155: Edited By Deeangelee Pooran-Kublall, MD/MPH · 9.5 Preparing for the MCAT: Biochemistry in the Chemical and Physical Foundations of Biological Systems Section 9.6 Preparing for the

PlanyourattackToanswerthisquestion,we’llneedtouseFigure1andourknowledgeoftheKrebscycletofigureoutwhathappenswhenIDHactivitydrops.SincethisisaRomannumeralquestion,though,weshouldchecktheanswerchoicesbeforewestartassessingthestatements.

Since itemIshowsup three times,weshouldstart there: if it’swrong,we’redone.Todetermine ifitemIistrue,weneedtofigureoutwhathappenstopyruvatekinaseactivity.Todothat,we’llneedourknowledgeofwhatpyruvatekinasedoesandhowit’sregulated.

If item I is true, then we should look at III, CO2 production. Answering that will require ourknowledgeoftheKrebscycle.Finally,wemightneedtoconsideritemII,acetyl-CoAlevels.Acetyl-CoAshowsupinFigure1,sowe’llneedtolookthere.

ExecutetheplanAccordingtoFigure1,ifIDHdoesn’tconvertisocitratetoα-ketoglutarate,it’llentertheglyoxylatecycle,whereit’llbeturnedintoglyoxylateandsuccinate.Thatmeanswebypasstheformationofα-ketoglutarateandsuccinyl-CoA.ThisisimportantbecausebothofthosestepsproduceNADH,whichisusedtoproduceATP.SolessIDHactivitymeanslessATP.

Pyruvate kinase, the last enzyme in glycolysis, produces ATP as it converts phosphoenolpyruvate(PEP) into pyruvate. SinceATP is a product of the reaction,LeChâtelier ’s principle says that lowlevelsofitshouldincreasepyruvatekinase’sactivity.ItemIisthereforetrue.

LookingatitemIII,CO2production,isocitratehassixcarbons,whilesuccinatehasfourcarbons.SowheredotheothertwocarbonsgointheKrebscycle?ToCO2!LessIDHactivitymeanslessCO2isproduced,soitemIIIisfalse.

Answerbymatching,eliminating,orguessingBecauseitemIistrue,wecaneliminate(B).ItemIIIisfalse,whichallowsustoeliminate(C)and(D).Sothecorrectanswermustbechoice(A),andwedon’tevenneed to lookat itemII.Thatsaid, it’sworth understanding why item II is false. According to the passage, when IDH activity drops,glyoxylate forms. Glyoxylate combines with acetyl-CoA to form oxaloacetate. So each time amolecule of isocitrate enters the glyoxylate cycle, a molecule of acetyl-CoA is consumed, notproduced.

TakeawaysWithRomannumeralquestions,checkingtheanswerchoicesbeforeconsideringthestatementscansavevaluabletime.

Page 156: Edited By Deeangelee Pooran-Kublall, MD/MPH · 9.5 Preparing for the MCAT: Biochemistry in the Chemical and Physical Foundations of Biological Systems Section 9.6 Preparing for the

KeyConceptsSkill3—ReasoningabouttheDesignandExecutionofResearchCitricAcidCycle

3.Abiochemistdecidestoperformautoradiography,whichcanquantifytheamountofaradioactiveisotopepresent,onthesamplesdescribedinTable1.Basedoninformationinthepassage,hewouldpredictthatIDHfrom:

A.onlySample1wouldexhibit31Pactivity.B.onlySamples2and5wouldexhibit31Pactivity.C.onlySamples2,5,and6wouldexhibit31Pactivity.D.alltheIDHsamplesinTable1wouldexhibit31Pactivity.

AssessthequestionYoumighthavebeenwonderingwhythepassagementioned 31Pbutthendidn’tmentionitagain.OryoumighthavewonderedwhyTable1includessixsamples,whiletheWesternblotexperimentonlyusedthefirstfour.ThefirstthingthatshouldcometoyourmindinsuchcasesisThere’sprobablyaquestion coming up on that! In this case, we have a Skill 3 question, asking us to formulate ahypothesisaboutincorporationof31PintoIDH.

PlanyourattackTo answer this question, we’ll need to synthesize the information in the passage on IDHphosphorylation—which is in paragraphs 1 and 2—with our understanding of how organismsregulatemetabolicpathways.

ExecutetheplanParagraph2statesthatIDHisphosphorylatedwhenE.coliuseseitherfattyacidsoracetyl-CoAasitsprimary fuel source.Paragraph1 tellsus thathumanIDHdoesn’tundergophosphorylation.Soweshouldn’texpectany31PactivityfromSamples3and4.

Whatdoes“primaryfuelsource”mean?Practicallyspeaking, itmeansthatnoglucoseisavailable.SowewouldpredictthatE.coliwillphosphorylateIDHintheabsenceofglucose.LookingatTable1,that’sSamples2and5.

Page 157: Edited By Deeangelee Pooran-Kublall, MD/MPH · 9.5 Preparing for the MCAT: Biochemistry in the Chemical and Physical Foundations of Biological Systems Section 9.6 Preparing for the

Answerbymatching,eliminating,orguessingChoice(B)matchesourpredictionexactly.

(D) is wrong because human IDH is not regulated by phosphorylation. (A) doesn’t work becauseSample1reliesonglucoseasitsprimarysource.Similarly,(C)iswrongbecauseSample6containsbothacetateandglucose.Glucose is thepreferred fuel source,andcellswillalmost invariablyusethatfirstwhenit’savailable.

ThingstoWatchOutForExperimentswithseveraltrialgroupsareagreatsourceofquestionsonTestDay.

KeyConceptsSkill2—ScientificReasoningandProblem-SolvingControlofEnzymeActivity

4.Basedoninformationinthepassage,whichofthefollowingisthemostlikelyexplanationforhowphosphorylationinhibitsIDHinE.coli?

A.Phosphorylationcausesallostericchangestotheactivesitethatpreventisocitratefrombinding.B.Phosphorylationcauseselectrostaticrepulsionofthenegativelychargedsocitrate.C.PhosphorylationtargetsIDHfordestructionbylysosomes.D.PhosphorylationresultsinthedissociationoftheIDHdimer.

AssessthequestionThisisaSkill2question,askingustofigureouthowphosphorylationcausesinhibitionofIDHinE.coli.

PlanyourattackToanswerthisquestion,weneedtodrawonourknowledgeofenzymefunctionandinhibition.We’llalsoneedtoknowwhatthepassagetellsusaboutphosphorylationofIDHinE.coli;thatinformation

Page 158: Edited By Deeangelee Pooran-Kublall, MD/MPH · 9.5 Preparing for the MCAT: Biochemistry in the Chemical and Physical Foundations of Biological Systems Section 9.6 Preparing for the

isinparagraph1.

ExecutetheplanEnzymeinhibitionmeansthattheactivityoftheenzymedrops.Paragraph1tellsusthatinE.coli,IDHundergoes phosphorylation at its active site. That tells us thatwhatever happens, itmust involve achangeattheactivesite.

Answerbymatching,eliminating,orguessingCyclingthroughtheanswerchoices,theonlyonethatcouldplausiblyinvolvetheactivesiteischoice(B).

(A) iswhatwemightpredict ifwedidn’t lookatparagraph1;mostenzymes thatare regulatedbyphosphorylationundergo allosteric changes. (C) can’t be true becausewe’re talking aboutE. coli,whichisabacterium—andbacteriadon’thaveorganellessuchaslysosomes.(D)couldbe true,butwe’relookingforthemostlikelyexplanation,and thepassagedoesn’tgiveusany information thatsuggeststhatIDHmustbedimerizedtobeactive.

SimilarQuestionBasedoninformationinthepassage,theinhibitionofIDHinE.colimostcloselyresembles:

A.reversiblecompetitiveinhibition.B.irreversiblecompetitiveinhibition.C.reversiblenoncompetitiveinhibition.D.irreversiblenoncompetitiveinhibition.

Sincethepassagestatesthatthephosphategroupbindstotheactivesite,we’redealingwithaformofcompetitive inhibition.Sincephosphorylation involves formationof acovalent bond, rather than atemporaryelectrostaticassociationbetweenthesubstrateandactivesite,theinhibitionisconsideredirreversible.Thecorrectanswerischoice(B).

ThingstoWatchOutForOnTestDay,ifanMCATpassagenotesthataparticularphenomenonisunusual,payattentiontoit.Thisisoftenasignalthatyou’llbetestedonthatconcept!

KeyConcepts

Page 159: Edited By Deeangelee Pooran-Kublall, MD/MPH · 9.5 Preparing for the MCAT: Biochemistry in the Chemical and Physical Foundations of Biological Systems Section 9.6 Preparing for the

Skill2–ScientificReasoningandProblem-SolvingPrinciplesofMetabolicRegulationEnzymeActivity

5.Inhumans,IDH3isNAD+-dependent,whileIDH1andIDH2aredependentonNADP+.WhichofthefollowingwouldbeLEASTlikelytobetrue?

A.ExpressionofIDH1isregulatedbymechanismsthathavenoeffectonIDH3expression.B.ThevaluesofKmandVmaxforIDH1andIDH2arelikelytodiffersignificantlyfromthoseforIDH3.C.ThegeneforIDH1andthegeneforIDH3arelocatedondifferentchromosomes.D.Invitro,theequilibriumratioofisocitratetoα-ketoglutarateusingIDH1equalstheratioforIDH3.

AssessthequestionThis question tells us something new: IDH1 in humans uses NADP+, while IDH3 uses NAD+. Thequestionthenasksus,asaSkill2questionoftendoes,todrawaconclusion.Butthisquestionismoredifficult because it asks us to identify the conclusion that is least likely to be true. Looking at theanswerchoices,there’snocommonthemerunningamongthem.

PlanyourattackQuestionsthataskwhatisLEASTlikelytobetrueareoftenmoredifficultbecauseit’shardtopredictwhat thecorrect answerwillbe.Ourbestbetwillbe to lookatparagraph1 to figureoutwhat thepassage tells us about IDH1, IDH2, and IDH3.We’ll also need to use our knowledge of enzymeactivitytoanswerthisquestion.

ExecutetheplanParagraph 1 tells us that IDH1 is expressed in the cytosol, while IDH3 is expressed in themitochondria, and that all three forms of IDH carry out the conversion of isocitrate to α-ketoglutarate.But,sinceIDH3usesNAD+whileIDH1andIDH2useNADP+,wecanconcludethatthemechanismusedbyIDH3isdifferent fromthemechanismusedbyIDH1andIDH2.(Note:wecan’tsayifIDH1andIDH2havethesamemechanismornot.)

Atthispoint,it’susefultolookthroughtheanswerchoicesoneatatime.Sincewe’relookingforthestatementthatisLEASTlikely,anythingthatwecanreasonablyassumetobetruecanbeeliminated;

Page 160: Edited By Deeangelee Pooran-Kublall, MD/MPH · 9.5 Preparing for the MCAT: Biochemistry in the Chemical and Physical Foundations of Biological Systems Section 9.6 Preparing for the

wewanttoselecttheanswerthatiseitherimplausibleoroutrightimpossible.

Answerbymatching,eliminating,orguessingLet’s startwith (A). Can we think of a plausible scenario where IDH1 and IDH3 are regulated indifferentmechanisms?Absolutely.There’sagreatanalogyinglucokinaseandhexokinase.Bothcarryoutthefirststepinglycolysis,buttheyareregulatedindifferentways.So(A)islikelytobetrue,andwecaneliminateit.

(B) testsourunderstandingofenzymekinetics.Shouldweexpect IDH1andIDH3 tohavedifferentvaluesforvmax,themaximumvelocityfortheenzyme,andKm,thesubstrateconcentrationatwhichitishalf-saturated?Continuingourglucokinase/hexokinasediscussion, absolutely.Glucokinasehas alower affinity for glucose thanhexokinase, and therefore different values forKm andvmax. So it’sreasonabletoexpectthesamethingtobetruehere.

(C)isactuallytheeasiestofthefour;nothinginthepassagegivesusanyreasontobelievethatIDH1andIDH3shouldhavetheirgenesonthesamechromosome,so(C)isalsolikelytobetrue.Eventhegenesfortheαandβsubunitsofhemoglobinarelocatedondifferentchromosomes!

Byprocessofelimination,thatleaves(D).Atfirstblush,thismightseemtobetrue:enzymesspeedupreactions,buttheydonotaffecttheequilibriumyield,sinceKeqdependsonlyontherelativestabilityof the reactants and products.But do IDH1 and IDH3 catalyze the same reaction?No! Since IDH1dependsonNADP+,whileIDH3requiresNAD+,theydon’t.ThustheyhavetwodifferentexpressionsforKeq,andwecan’treasonablyexpectthattheratioofisocitratetoα-ketoglutaratewillbethesame.Sincechoice(D)islikelytobefalse,it’stheanswerthatearnsusapoint.

MCATExpertiseTheinterfaceonTestDayallowsyoutohighlighttextinthepassage,butitdoesnotletyouhighlightquestionstems.Ifyouwanttomakenotesonquestions,you’llneedtowritethemdownonyourscratchpaper.

Page 161: Edited By Deeangelee Pooran-Kublall, MD/MPH · 9.5 Preparing for the MCAT: Biochemistry in the Chemical and Physical Foundations of Biological Systems Section 9.6 Preparing for the

9.8BiochemistryPracticePASSAGEIII(QUESTIONS1–5)Peptide synthesis is a common target for antimicrobial compounds. Many such drugs, includingchloramphenicol, work by directly inhibiting peptide bond synthesis. Another category, themacrolides,donotinhibitpeptidebondformation,butinsteadblockthechannelthroughwhichnewlysynthesizedpeptidesexittheribosome,thenascentpeptideexittunnel(NPET).

Macrolidesaremoleculeswithlactoneringscontaining14to16atoms,andincludeerythromycin;anewer,morepotentsubclasscalledketolidesincludestelrithromycin.Macrolideswerelongthoughttoact likea“plug” in theNPET,blockingsomuchof the tunnel thatpeptidescannotpass through.Morerecentevidence,however,suggeststhistheoryisincorrect.

First,neitherdrugcompletelyinhibitsproteinsynthesis.AtleastsomeproteinscanexittheNPET,andundergocomplete translation,evenin theirpresence.ThelikelihoodofaproteinbeingsynthesizeddependsonitsN-terminalsequence.MostsequencescauseblockageoftheNPETandanearlyendtoproteinsynthesis,whilesomesequencespermitcompletesynthesisofaprotein.Asequenceallowingfull synthesis is shown inFigure1; proteinswith relativelyhomologous chains are alsoknown toallowfullsynthesis.Moreover,ahybridmRNAinwhichthesequenceinFigure1precedesachainthatwouldnormallybeblockedalsotendstobefullytranslated.

Figure1.Apeptidesequencecapableofescapingfromamacrolide-boundNPET,alongwithitscorrespondingmRNAsequence

Second, telrithromycin is actually less effective than erythromycin at inhibiting synthesis: cellsexposed to telrithromycin synthesizemore proteins than do those exposed to erythromycin. Someproteinsarefullyexpressed,whilesynthesisofsomeendsafterjustafewresidues;insuchcases,thenascentpeptidefallsoutoftheribosomebeforetheribosomeeverreachesastopcodon.(ThehumangeneticcodeisdepictedinFigure2.)

Page 162: Edited By Deeangelee Pooran-Kublall, MD/MPH · 9.5 Preparing for the MCAT: Biochemistry in the Chemical and Physical Foundations of Biological Systems Section 9.6 Preparing for the

Figure2.ThehumangeneticcodeforRNAmolecules

P1.

P2.

P3.

F1.

P2.

F2.

1.Aresearcherculturesabacteriumsensitivetobotherythromycinandtelrithromycinonthreeagarplates.Plate1haserythromycin,plate2hastelrithromycin,andplate3isacontrol.Basedoninformationinthepassage,theresearcherwouldpredictthatthepercentageofpartiallytranslatedproteinswouldbe:

A.highestonplate1andlowestonplate2.B.highestonplate2andlowestonplate1.C.highestonplate1andlowestonplate3.D.highestonplate2andlowestonplate3.

2.WhichofthefollowingchangesinthemRNAsequenceshowninFigure2wouldmostlikelyleadtothegreatestinhibitioninproteinsynthesis?

A.ChangingAGCtoAGUincodon2B.ChangingGAAtoGAGincodon3C.ChangingCUGtoUUGincodon8D.ChangingAACtoAGCincodon10

Page 163: Edited By Deeangelee Pooran-Kublall, MD/MPH · 9.5 Preparing for the MCAT: Biochemistry in the Chemical and Physical Foundations of Biological Systems Section 9.6 Preparing for the

3.Basedoninformationinthepassage,whichofthefollowingchangesismostlikelytobeobservedinthefirstfewminutesaftererythromycinadministrationinabacteriumsusceptibletoit?

A.AdecreaseintheconcentrationofchargedtRNAsB.Covalentbindingof30Sand50SribosomalsubunitsC.MutationsattachingthesequenceinFigure1toothergenesD.Anincreaseintheconcentrationofpeptidyl-tRNAs

4.ThegraphbelowshowsproteinsynthesisinanE.colicellcultureintheabsenceofantibiotics.

Whichofthefollowinggraphsmostlikelyrepresentstheresultsobtainedinthepresenceoftelrithromycin?(Note:Allverticalscalesarethesameasthegraphabove;peptides<20kDawerenotmeasured.)A.

B.

Page 164: Edited By Deeangelee Pooran-Kublall, MD/MPH · 9.5 Preparing for the MCAT: Biochemistry in the Chemical and Physical Foundations of Biological Systems Section 9.6 Preparing for the

C.

D.

5.AnE.colicellisgivenmethionineradiolabeledwith31S.Fiveminuteslater,whichofthefollowingismostlikelytobetrue?

A.Mostproteinssynthesizedinthatfive-minuteperiodwillexpresstheradiolabelattheirN-terminus.B.Mostproteinssynthesizedinthatfive-minuteperiodwillexpresstheradiolabel,butnotattheirN-terminus.

Page 165: Edited By Deeangelee Pooran-Kublall, MD/MPH · 9.5 Preparing for the MCAT: Biochemistry in the Chemical and Physical Foundations of Biological Systems Section 9.6 Preparing for the

C.Mostproteinssynthesizedinthatfive-minuteperiodwillexpresstheradiolabelatboththeirN-terminusandatinteriorpositions.D.Bothnewlysynthesizedandalready-existingproteinswillexhibittheradiolabel.

BiochemistryPracticePassageExplanationsP1.Macrolideantibiotics

P2.“Pluginabottle”mechanismprobablywrong

P3.Stillhaveproteinsynthesis

F1.Sequencethatcanevademacrolides,exitribosome

P4.Erythromycininhibitsproteinsynthesismorethantelrithromycin

F2.Humangeneticcode

MCATExpertiseWiththeexceptionofthestartandstopcodons,youarenotexpectedtomemorizethegeneticcodeforTestDay.Ontheotherhand,theMCATwillnotaskaquestionthatpunishessuperiorknowledge.Ifyouwereaskedthissamequestionwithoutbeinggiventhegeneticcode,theAAMCwouldnotuseCUG/UUGinananswerchoice.

1.(C)

Thekeytothisquestionisacarefulreadingofthequestionstemandthepassage.Thequestionstemasks us which plate will have the highest and lowest percentages of partially translated proteins.Paragraph 2 tells us that telrithromycin is more potent than erythromycin, so wemight be led topredict that the order is plate 2 (telrithromycin) > plate 1 (erythromycin) > plate 3 (control). Thatwouldleadusto(D)—whichisatrap.Paragraph3 tellsus thatwhile telrithromycin ismorepotent asanantibiotic, it isactuallyworse atinhibitingsynthesisthanerythromycinis!Sothecorrectanswerisactuallyplate1>plate2>plate3,whichmatcheschoice(C).

2.(D)

Paragraph3statesthatthemRNAsequenceshowninFigure2makesapeptidemacrolide-resistant:itcanescapethetunneleveniferythromycinortelrithromycinisbound.Ifwewantthegreatestincreasein inhibition, then we need a significant change in the sequence of that peptide. To answer thisquestion,we’llneedtousethegeneticcodeinFigure2,andlookforamutationthatwouldleadtoadifferentaminoacid.Cyclingthroughthechoices,wefindthatthecorrectanswerischoice(D):AACcodesforasparagine,whileAGCcodesforserine.(A) Proximity to the start codon does not, by itself, increase the likelihood that amutationwouldaffectinhibitionbymacrolides.(B) The third nucleotide in a codon is in thewobble position, which tends to be the most likely

Page 166: Edited By Deeangelee Pooran-Kublall, MD/MPH · 9.5 Preparing for the MCAT: Biochemistry in the Chemical and Physical Foundations of Biological Systems Section 9.6 Preparing for the

nucleotidetoresultinasilentmutationwhenaltered.(C)might lookpromising: it changes the first nucleotide in thecodon.A lookat thegenetic code,though,showsusthisisoneofthefewcaseswheresuchamutationisactuallysilent;bothCUGandUUGcodeforleucine.

3.(D)

We’re looking for a change likely to happen in the presence of erythromycin. According toparagraph4,erythromycincancausethenascentpeptidetofalloutoftheribosome.However,sincethishappenswithoutastopcodon, thenascentpeptide isnot released from the tRNA,sowewouldexpectabuildupofpeptidyl-tRNA,orpeptidesstillboundto tRNAmolecules.Thismatcheschoice(D).(A) Sinceprotein synthesisdecreases,wewouldnot expect the concentrationof charged tRNAs todrop;ifanything,theywouldincrease.(B)Thiswouldbetrueiferythromycinboundthetworibosomalsubunitstogether,butnowheredoesthepassageimplythishappens.(C) Paragraph 4 states that this kind of mutation results in the expression of proteins that wouldnormally be stopped by erythromycin. However, it is unlikely that such amutationwould happenspontaneouslyinthefirstfewminutesaftererythromycinadministration.

4.(B)

Thegraph in thequestionstemrepresents theproductsofproteinsynthesis innormalE.coli cells.The graphs in the answer choices represent possible protein synthesis inE. coli after exposure totelrithromycin.Accordingtoparagraph3,theinhibitionofsynthesisisdeterminedonlybythenatureoftheN-terminalsequenceofthepeptide.SincewehavenoreasontobelievethatthereisaspecificcorrelationbetweenN-terminal sequence andprotein size,wewould expect an overall decrease inproteinsynthesisatallmolecularmasses.Thismatcheschoice(B).(A) This would be correct if the inhibition were based solely on molecular size (in this case,inhibitingsynthesisofproteins>150kDainmass).(C)The totalamountofproteinproduced is the same;according toparagraph4,protein synthesisdropswhenmacrolidesaregiven.(D) This choice shows partial inhibition, as the passage states, but only for proteins with highmolecular weights; as with (A), we have no reason to believe the inhibition is limited to certainmolecularweights.

5.(B)

Wherewouldwe find radiolabeledmethionine after fiveminutes?Every peptide chainbeginswithmethionine—theonlycodonforMetisalsothestartcodon.Butthemajorityoffinishedpeptidesdonot have Met at their N-terminus; in most proteins, that initial Met, along with other N-terminalresidues,areremovedinpost-translationalprocessing.Amongotherthings,thatN-terminussequenceisofteninvolvedinsignalingthedestinationofaprotein(forexample,whetheritshouldendupinthecellmembrane).There’salsonomechanismforexchangingmethionineinexistingproteins,sothecorrectanswerischoice(B).(A)Mostinitialmethioninesarelostinpost-translationalprocessing.(C)Mostproteinshaveinternalmethionines,butmostoftheinitialmethioninesareremoved.

Page 167: Edited By Deeangelee Pooran-Kublall, MD/MPH · 9.5 Preparing for the MCAT: Biochemistry in the Chemical and Physical Foundations of Biological Systems Section 9.6 Preparing for the

(D)Thereisnomechanismforexchangingaminoacidsinexistingproteinswithnewaminoacids.

Page 168: Edited By Deeangelee Pooran-Kublall, MD/MPH · 9.5 Preparing for the MCAT: Biochemistry in the Chemical and Physical Foundations of Biological Systems Section 9.6 Preparing for the

CHAPTERTEN

BiologyMCATtest-takersusuallyviewbiologyastheirstrongestarea,asmanystudentstakingtheMCATaremajoringinorhaveearneddegreesinthebiologicalsciences.However,theMCATdoesnotseektotest biological knowledge; it aims to test the scientific thinking and reasoning skills required forsuccessinmedicalschool.Whilebiologyknowledgeisimportant,itishowwellyouareabletousethatknowledgethatearnspointsonTestDay.

WhenencounteringanMCATbiologypassage,manystudents try toapply thesamestrategies theyused when studying for undergraduate biology courses, including reading the passage forunderstandingandfocusingondetailsinthepassage.However,theMCATismorelikeanopen-booktestandmemorizingthedetailswastestimeanddoesn’tnecessarilyresultincorrectanswers.Inthischapter,wewilldiscusstheKaplanapproachtobiologypassagesandquestions.

10.1ReadingthePassageOne of the trademarks ofMCAT biology passages is that the topic itself will be familiar, but thecontextwillbeunfamiliar.Forexample,apassageonactionpotentialtransmissionmaydiscussrarediseases associatedwith derangements in neural transmission.The job of the test-taker is to applywhatheorshealreadyknowsaboutthetopictonewsituations.Theabilitytousewhatyouknowinnewwaysisasignificantpartofsucceedingasamedicalstudentandaphysician.

Oneofthebiggestmovementsinmodernmedicineisthepracticeofevidence-basedmedicine.Whatthis means is that a physician uses current studies and information as a guide to diagnosis andtreatment.Thus,theMCAThasshiftedtotesttheseskillsbypresentinginformationinthecontextofastudy, and asking questions about experimental design. In the Biological and BiochemicalFoundationsofLivingSystemssection,theareastestedincludeorganicchemistry,biochemistry,andbiology,withasmallamountofgeneralchemistry.Sinceexperimentsinchemistrydonottypicallyinvolve control groups, population studies, or statistical analysis, you can expect that passages inbiologywillseektotestyourskillsinanalyzingstatisticaldataandevaluatingexperimentdesign.Tothatend,biologicalpassageswillrequireyoutoapplyyourknowledgeintheseareas.

PASSAGETYPESThepassage typesrelated tobiologyon theMCATare thesameas in theothersciences.However,therearespecificdifferencesrelatedtothefieldofbiology.

InformationPassages•Containprosesimilartothatfoundinatextbook.• A typical passage might discuss a particular disease, describe its symptoms, and include adiscussionaboutthepathophysiology(howadiseaseoccurs),includinggenetics,environmentalfactors,andderangementsatthemolecularlevel.•Inadditiontoarecountingofinformation,thesepassagesmayalsodescribepreviousstudies

Page 169: Edited By Deeangelee Pooran-Kublall, MD/MPH · 9.5 Preparing for the MCAT: Biochemistry in the Chemical and Physical Foundations of Biological Systems Section 9.6 Preparing for the

withinthisfield,providingstatisticaldatafromaparticularstudy.•Averylargeamountofdetailmaybepresent.Notethatthedetailsarepresent,andthenmoveon.Avoidgettingcaughtupinunderstandingeverydetail.

ExperimentPassages•Includeoneormoreexperimentsandadescriptionoftheresultsintheformofachart,graph,table,ordiagram.•Backgroundinformationisgenerallypresentedtoprovidecontext.•Experimentsmaybelaboratory-basedorpopulation-based.•Payspecialattentiontothehypothesis,experimentaldesign,andresults.

Remember, your goal is to get through a passage as quickly as possible but still read the passagecriticallyenoughtoallowyoutoanswerquestions.Becarefulnottospendsomuchtimedigestingapassagethatyourunoutoftimewhenansweringquestions.MCATbiologypassagesmaycontainatremendousamountofinformationanddata.However, it is importanttorememberthatthepassagewillalwaysbetheretogobackto,ifneeded.

PassageTypesinBiology

InformationPassages

ExperimentPassages

Scope

Discussaparticularphenomenon,disease,orstudy.

Discussanexperiment.

HowtoReadIt

Readtogetthegistofeachparagraph;avoidgettingcaughtupinthedetails.

Readtoobtainthehypothesis,experimentaldesign,andresults.

CommonPitfalls

Spendingtoomuchtimetryingtounderstandandanalyzethedetails.

Spendingtoomuchtimetryingtounderstandtheprocedureandresultsbeforethequestionsaskforit.

OUTLININGTHEPASSAGEBiologypassagesareoftenhighlydetailed,andintegratemultipleconceptswithinbiology.WiththeadditionofbiochemistryontheMCATin2015,thescopewillalsoincludebiochemicalconceptsaswellasorganicandgeneralchemistry.Withtheintegrationofalargenumberofpossibletopicsandcontent areas, MCAT biology passages have the potential to be exceptionally detailed. While thedetails are important, there arenopoints awarded forone’s ability tomemorizeand recalldetails.However,somesenseofthedetailsisrequiredinordertoanswerthequestionscorrectly.Thekeytoa high score on Test Day is to find a balance between investing enough time into the passage todevelop a solid understanding and reading quickly enough so that all of the questions may be

Page 170: Edited By Deeangelee Pooran-Kublall, MD/MPH · 9.5 Preparing for the MCAT: Biochemistry in the Chemical and Physical Foundations of Biological Systems Section 9.6 Preparing for the

answeredinatimelymanner.Themostefficientwaytodothisistoreadquickly,butcritically,usingtheKaplanway.

ScanforStructure•Lookatthepassage,takingnoteofthepresenceofcharts,graphs,tables,ordiagrams.

•Determinethetopicanddegreeofdifficulty.•Triagethepassage,decidingwhethertodoitnoworlater.

ReadStrategically•Identifythetypeofpassage.• Actively read every paragraph, asking, “What is being said here?Why is this informationhere?”•Ifyouseelistsofdetailsorthepassagebecomesexceptionallydetailed,movequickly,takingnoteofthepresenceofdetails,nottheactualidentityofthosedetails.•Donotstoptoanalyzehowthedetailsfitorwhatthedetailsmean.•Keepmoving,butgetthegistofeachparagraph.

LabelEachComponent•Writedownthepurposeofeachparagraph,chart,graph,table,ordiagram.•Ifanimagehasarelationshipwithaparagraph,besuretonotethat.•Ifitwasanexperimentpassage,besuretonotethehypothesis,procedure,andoutcome.•Ifitwasaninformationpassage,identifythegeneralsubjectarea.•Noteanylistsofdetailssothatthesecanbeidentifiedquickly.•Keepyouroutlinesimple,justenoughsothatyoucanfindrequiredinformation.

ReflectonYourOutline•Identifythegoalofthepassage.Whywasthispassagewritten?

Page 171: Edited By Deeangelee Pooran-Kublall, MD/MPH · 9.5 Preparing for the MCAT: Biochemistry in the Chemical and Physical Foundations of Biological Systems Section 9.6 Preparing for the

10.2AnsweringtheQuestionsLiketheothersciences,biologyquestionsfollowthefourbasictypesofquestions.

Discretequestions•Questionsthatarenotbasedonapassage.• Often preceded by a warning such as, “Questions 12–15 are not based on a descriptivepassage.”•Most likely toaskaquestionabouteitherapieceof information thatyoualreadyknoworaconceptthatwillrequireyoutoapplyyourknowledge.

Questionsthatstandalonefromthepassage•Questions that arewithin a block of passage-based questions but do not actually require thepassagetodeterminetheanswer.•Usuallyarerelatedtothetopicofthepassage,butwilllikelyaskyoutoapplyyourknowledgetoanewsituation.

Questionsthatrequiredatafromthepassage•Thesequestionsmayrequireinformationordatafromthepassage.•Verylikelytoaskyoutoapplyyourknowledgetoanewsituation.• May involve statistical analysis, evaluation of experimental design, interpretation of dataand/orinformation,oridentificationofarelationshipbetweenconcepts.

Questionsthatrequirethegoalofthepassage• Common goals of these questions include evaluation of the validity of a hypothesis,identification of supporting or refuting evidence, comparison of related topics, orinterpretation/analysisofatheorygivenanewpieceofevidence.•Interpretationofdatafromatable,chart,orgraphmayalsoberequired.

DETERMININGTHEPURPOSEOFTHEQUESTIONMCAT biology covers a tremendous amount of topics. In addition, an understanding of severalbackground concepts may be required in order to answer the question. Answering the questionsquickly,efficiently,andcorrectlyrequiresasolidpassageoutlineandasystematicapproachtoensurethatessentialdetailsarenotmissed.

1.AssesstheQuestion•Readthequestionbutavoidreadingtheanswers.•Determinethetypeofquestionandthedegreeofdifficulty.•Makeajudgmentregardinghowmuchworkthequestionwillrequire.Doesitappeartorequireextensiveinformationfromthepassageoralargeamountofanalysis?•Decidewhethertodothequestionnoworlater.

2.PlanYourAttack•Identifythequestiontask.Whatisthisquestionaskingyoutodo?Analyzedataorexperimentdesign?Evaluateconceptsordatawithrespecttoanewpieceofinformation?

Page 172: Edited By Deeangelee Pooran-Kublall, MD/MPH · 9.5 Preparing for the MCAT: Biochemistry in the Chemical and Physical Foundations of Biological Systems Section 9.6 Preparing for the

•Using the taskof thequestion,determinewhatmustbedone inorder toarriveatananswer.Doesthequestionrequiredataorinformationfromthepassage?

3.ExecuteYourPlan•Carryouttheplanasdeterminedinthepreviousstep.•Useyouroutlinetofindrequiredinformationortoremindyourselfofthepassagegoal.•Lookuptherequiredinformation,analyzethedata,evaluatenewinformationpresented.

4.AnswertheQuestionbyMatching,Eliminating,orGuessing•Matchyouranswertotheanswerchoices.•Ifthereisnoclearmatch,startbyeliminatingtheincorrectanswers.•Incorrectanswersoftencomeintheformofanswersthatsimplydonotmakesense.Criticallyreadtheanswerchoicesanddeterminewhichanswerchoicesdonotmakesense.• Ifaclearanswerdoesnotemerge,doyourbest toeliminateat least twoanswers,andguessbetweentheremainingtwoanswers.•Avoidblindguessing:thebestguessesareonesthatareeducated,withareasonforchoosingaparticularanswer.

Page 173: Edited By Deeangelee Pooran-Kublall, MD/MPH · 9.5 Preparing for the MCAT: Biochemistry in the Chemical and Physical Foundations of Biological Systems Section 9.6 Preparing for the

10.3GettingtheEdgeinBiologySuccessonMCATbiologypassagesrequiresaquick,butthorough,readingofthepassagecoupledwithanexcellentpassageoutline.MCATbiologypassageswilloftencoverawiderangeof topics,withalargeamountofdetail.LearningtoidentifythelocationofthedetailwithoutinvestingtimeintheinterpretationofthosedetailsisanessentialreadingskillforTestDaysuccess.Thisisoftenverydifficult for students to learn how to do as it runs counter to how most students study for theirundergraduateclasses.But,theMCATisnotlikethetestsadministeredinundergraduateclasses,asitrequiresmuchmorethanmemorizationofconceptsanddetails;theMCATrequiresinterpretationandevaluationoftheconceptsanddetails,butonlywhenrequiredbyaquestion.

Answering questions on MCAT biology passages requires quick location of information using apassageoutline.Inaddition,thequestionsrequirestudentstothinkcriticallyaboutboththequestionstemandtheanswerchoices.Approachingthequestionstemandanswerchoiceswithconfidenceanda critical eye will allow you to identify the correct question task and eliminate incorrect answerchoices.

Page 174: Edited By Deeangelee Pooran-Kublall, MD/MPH · 9.5 Preparing for the MCAT: Biochemistry in the Chemical and Physical Foundations of Biological Systems Section 9.6 Preparing for the

10.4Step-By-StepGuidetoBiologyPassages

OUTLININGTHEPASSAGE•Scanforstructure

•Determinewhethertodothispassagenoworlater.• Identify the structure of the passage, including charts, graphs, tables, diagrams,and/orlargeblocksoftext.

•Readstrategically•Identifythetypeofpassage.•Payspecialattentiontotherelationshipsbetweenconcepts.• In an information passage, identify how the information in each paragraph fitstogethertopresentaunifiedpicture.•Identifywhatinformationispresentedineachfigureorimage.

•Labeleachcomponent•Writedownthepurposeofeachparagraph,chart,table,graph,ordiagram.• If a paragraph and image are related, be sure to note this, as paragraphs may beentirelydevotedtodescribingadiagramorchart.• If a paragraph discusses experiment procedure or study design, be sure tomake anoteofit,asbiologypassagesareripeforevaluationoftheseresearchconcepts.•Identifyanyconnectionsbetweenconceptswithinthepassage.•Makesureeachpartofyouroutlineidentifiesthetopic,asyouroutlinewillhelpyoutomakesenseofapotentiallywiderangeofconceptswithinabiologypassage.

•Reflectonyouroutline•Determinethegoalofthepassageandwriteitdown.•Identifytheconceptswithinthepassageinanefforttoanticipatequestions.•Identifyanyrelationshipsbetweentheconceptswithinthepassage.

ANSWERINGTHEQUESTIONS

1.AssesstheQuestion•Determinewhetherthisquestionshouldbedonenoworlater.•Identifythetopicandthedegreeofdifficulty.•Good questions to do now in biology are those that do not require extensive integration ofinformationordata from thepassage.These areoften thequestions that standalone from thepassage.

2.PlanYourAttack•Determinewhatyoualreadyknow,andwhatinformationyouneed.•Identifywheretofindtherequiredinformation:thepassage,thequestion,youroutline,oryourownknowledge.• If youhave to goback to the passage, determinewhere to find the required informationbyusingyouroutline.•Ifdataanalysisisrequired,identifythedataset.•Ifanalysisofexperimentalprocedureordataisrequired,identifywherethatinformationcan

Page 175: Edited By Deeangelee Pooran-Kublall, MD/MPH · 9.5 Preparing for the MCAT: Biochemistry in the Chemical and Physical Foundations of Biological Systems Section 9.6 Preparing for the

befound.

3.ExecuteYourPlan•Analyzethedata,gobacktothepassage,andcarryoutyourplan.•Ifyougetstuckanalyzingdata,rememberthatthetrendofthedataisoftenenoughtoyieldacorrectanswerchoice.• Ifyouareanalyzingstudydesign,besure to identify thevariablesand thecontrolgroup,aswellasthehypothesisorreasonforthestudy.

4.AnswertheQuestionbyMatching,Eliminating,orGuessing•Matchyouranswertotheanswerchoices.•Ifthereisnomatch,eliminateincorrectanswerchoices.Someoftheanswerchoicesmaynotmakesense;eliminatethosefirst.•Ifeliminationdoesnotprovideaclearanswer,guessbetweentwoprobableanswers.

Page 176: Edited By Deeangelee Pooran-Kublall, MD/MPH · 9.5 Preparing for the MCAT: Biochemistry in the Chemical and Physical Foundations of Biological Systems Section 9.6 Preparing for the

10.5PreparingfortheMCAT:BiologyTheseare thebiology topics thatyouare likely to seeonTestDay.Note this list doesnot includetopicsforwhichbiochemistryknowledgeisalsorequired;thosetopicsarelistedinSection9.6.

PROTEINSTRUCTURE•Recognitionofthedifferentlevelsofstructureandconformationalstability

NONENZYMATICPROTEINFUNCTION•Withintheimmunesystemandasmotors

ENZYMESTRUCTUREANDFUNCTION• Applying the different models of enzymatic structure and function and defining control ofenzymeactivity

NUCLEICACIDSTRUCTUREANDFUNCTION•Theroleofnucleicacidsinthetransmissionofgeneticinformation

DNAREPLICATION•TheprocessandmechanismofDNAreplication,includingstrandseparation,andcouplingofnucleicacids•Definitionofreplicationasasemi-conservativeprocess•Identificationofspecificenzymesinvolvedinreplication• Definition and identification of origins of replication; the existence of multiple origins ofreplicationineukaryotes•TheprocessandsignificanceofreplicationoftheendsofDNAmolecules

REPAIROFDNA•MechanismsofDNArepairduringreplication•Mechanismsofmutationrepair

GENETICCODE• The Central Dogma of molecular biology, the flow of information from DNA to RNA toprotein•Thegeneticcodeastriplets•Therelationshipbetweencodonsandanticodons•Theconceptofadegeneratecodeandtheroleofwobblepairing•Thedefinitionandsignificanceofmissenseandnonsensecodons

Page 177: Edited By Deeangelee Pooran-Kublall, MD/MPH · 9.5 Preparing for the MCAT: Biochemistry in the Chemical and Physical Foundations of Biological Systems Section 9.6 Preparing for the

•Thedefinition,role,andstructureofmRNA(messengerRNA)•Theuseofinitiationandterminationcodonsinthegeneticcode

TRANSCRIPTION•ThedefinitionandroleoftransferRNA(tRNA)•ThedefinitionandroleofribosomalRNA(rRNA)•Themechanismoftranscription•TheprocessandsignificanceofmRNAprocessingineukaryotes,includingintronsandexons• The roles of ribozymes, spliceosomes, small nuclear ribonucleoproteins, and small nuclearRNA(snRNAs)•Thedefinitionandfunctionofintrons,andtheirevolutionarysignificance

TRANSLATION•DefinitionsandrolesofmRNA,tRNA,andrRNA•Structureandroleofribosomes•Cofactorsrequiredforinitiationandtermination•Modificationofproteinsaftertranslation

EUKARYOTICCHROMOSOMEORGANIZATION•Proteinsassociatedwithchromosomes•Definitions,significance,anddifferencesbetweensingle-copyvs.repetitiveDNA•Definitionandsignificanceofsupercoiling•Differencesbetweenheterochromatinandeuchromatin•Identificationandrolesofcentromeresandtelomeres

CONTROLOFGENEEXPRESSIONINPROKARYOTES•TheconceptofoperonsandtheJacob–Monodmodelofoperonfunction•Mechanismsandfunctionofgenerepressioninbacteria•Mechanismsofpositivecontrolinbacteria

CONTROLOFGENESEXPRESSIONINEUKARYOTES•Regulationofgenesattheleveloftranscription•TheroleandfunctionofDNA-bindingproteinsandtranscriptionfactors•Amplificationandduplicationofgenes•Conceptofgenesplicing,includingintronsandexons•Controlofgeneexpressionatthepost-transcriptionallevel•Processbywhichafailureofnormalcellularcontrolsresultsincancer,includingtherolesoftumor-suppressorgenesandoncogenes•Chromatinstructureregulation•MethylationofDNA

Page 178: Edited By Deeangelee Pooran-Kublall, MD/MPH · 9.5 Preparing for the MCAT: Biochemistry in the Chemical and Physical Foundations of Biological Systems Section 9.6 Preparing for the

•DefinitionandroleofnoncodingRNAs

RECOMBINANTDNAANDBIOTECHNOLOGY•Theprocessofgenecloning•Thedefinitionanduseofrestrictionenzymes•CharacteristicsandformationofDNAlibraries•ProcessusedforthegenerationofcDNA•Theprocessofhybridization•Theprocessbywhichclonedgenesareexpressed•Theprocessandgeneraluseofthepolymerasechainreaction•TheproceduresanduseofgelelectrophoresisandSouthernblotting•MechanismofDNAsequencing•Theprocessofgeneexpressionanalysis•Howgenefunctionsaredetermined•Thedefinition,role,andsignificanceofstemcells• DNA technology for practical use, including medical applications, human gene therapy,pharmaceuticals,forensicevidence,environmentalcleanup,andagriculture•SafetyandethicalconsiderationsinDNAtechnology

EVIDENCETHATDNAISGENETICMATERIAL•UnderstandingofexperimentsthathaveshownthatDNAisinheritedandexpressedtocreateaparticularphenotype.

MENDELIANCONCEPTS•Definitionsofphenotypeandgenotypeandhowtheconceptsarerelated•Definitionsofgeneandlocus•Theroleofalleles;definitionofsinglealleleandmultiplealleles•Definitionsofhomozygosityandheterozygosity•Definitionofwildtype• Concepts of recessiveness, complete dominance, codominance, incomplete dominance,leakage,penetrance,andexpressivity•Thedefinitionofhybridizationandtheconceptofhybridviability•Definitionandsignificanceofthegenepool

MEIOSISANDOTHERFACTORSAFFECTINGGENETICVARIABILITY•Theprocessandsignificanceofmeiosis•Comparisonofmeiosisandmitosis,includingsignificantdifferences•Theprocessofgenesegregation,includingindependentassortmentandlinkage• Gene recombination, including single crossovers, the double crossovers, synaptonemalcomplex,andtetradformation•Concept,identification,anddefinitionofsex-linkedcharacteristics•TheYchromosomeanditslackofgeneticinformation

Page 179: Edited By Deeangelee Pooran-Kublall, MD/MPH · 9.5 Preparing for the MCAT: Biochemistry in the Chemical and Physical Foundations of Biological Systems Section 9.6 Preparing for the

•Howsexisdetermined•Thedefinition,patterns,andprocessofcytoplasmic/extranuclearinheritance•Mutations,includingthegeneralconceptofmutationsasanerrorsinDNAsequence•Typesofmutationsandconsequences,includingrandom,translationerror,transcriptionerror,basesubstitution,inversion,addition,deletion,translocation,andmispairing•Thedefinitionsandrolesofadvantageousanddeleteriousmutations•Definitionandsignificanceofinbornerrorsofmetabolism•Therelationshipbetweenmutagensandcarcinogens•Definitionandconsequencesofgeneticdrift•Themechanismofcrossingoverorsynapsisforincreasinggeneticdiversity

ANALYTICMETHODSINGENETICS•DefinitionandapplicationoftheHardy–Weinbergprinciple•Useand roleofa testcrossorbackcross, including theconceptsof theparental,F1,andF2generations•Usingcrossoverfrequenciestocreateagenemap•Statisticalmethodsusedtostudybiologicalphenomena;biometry

EVOLUTION•Theconceptofnaturalselectionasrelatedtofitnessandselectionbydifferentialreproduction•Theconceptsofnaturalandgroupselection•Measuringevolutionary success as an increase in thepercent representationof alleles in thegenepoolofthenextgeneration•Speciationasaresultofpolymorphism,adaptation,andspecialization•Definitions,roles,andcharacteristicsofoutbreeding,inbreeding,andbottlenecks•Measuringevolutionarytimebygradualrandomchangesinthegenome

METABOLISMOFFATTYACIDSANDPROTEINS•Fatanabolism•Biosynthesisoflipidsandpolysaccharidesasanon-templatesynthesismechanism•Proteinmetabolism

PLASMAMEMBRANE•Junctionsbetweencells,includinggapjunctions,tightjunctions,anddesmosomes

MEMBRANE-BOUNDORGANELLESANDDEFININGCHARACTERISTICSOFEUKARYOTICCELLS

• Characteristics of eukaryotic cells, including membrane-bound nucleus, presence oforganelles,andmitoticdivision• Structure and function of the nucleus, including compartmentalization, storage of genetic

Page 180: Edited By Deeangelee Pooran-Kublall, MD/MPH · 9.5 Preparing for the MCAT: Biochemistry in the Chemical and Physical Foundations of Biological Systems Section 9.6 Preparing for the

information,thenucleolus,nuclearenvelope,andnuclearpores•Locationandfunctionofthenucleolus•MitochondriaasthesiteofATPproduction•Structureoftheinnerandoutermembraneofmitochondria•Mitochondrialself-replication•Characteristicsoflysosomesasamembrane-boundvesiclecontaininghydrolyticenzymes• Structure and role of the endoplasmic reticulum, including rough and smooth components;ribosomerelocationtotheER;doublemembranestructure,roleinmembranebiosynthesis;androleinsecretionofproteins•StructureandfunctionoftheGolgiapparatus,especiallyinpackagingandsecretion•Functionofperoxisomesasahydrogen-peroxidecontainingorganelle

CYTOSKELETON•Generalfunctionofthecytoskeletonincellsupportandmovement•Compositionofmicrofilaments,andtheirroleincleavageandcontractility•Compositionofmicrotubulesandtheirroleinandtransport•Roleofintermediatefilamentsinsupport•Functionandcompositionofflagellaandcilia•Microtubuleorganizingcentersandcentrioles

TISSUESFORMEDFROMEUKARYOTICCELLS•Connectivetissuecells•Epithelialcells

CELLTHEORY•Thehistoryanddevelopmentofcelltheory•Howcelltheorychangedbiology

CLASSIFICATIONANDSTRUCTUREOFPROKARYOTICCELLS•Prokaryoticdomains,includingArchaeaandBacteria•Classificationofbacteriabyshape,includingbacilli(rod-shaped),spirilli(spiral-shaped),andcocci(spherical)•Characteristicsandstructureofprokaryoticcells, including the lackofanuclearmembrane,mitoticapparatus,andtypicaleukaryoticorganelles•Compositionandpresenceofcellwallinbacteria•Characteristicsandmechanismofflagellarpropulsion

GROWTHANDPHYSIOLOGYOFPROKARYOTICCELLS•Prokaryoticmechanismofreproductionbyfission•Prokaryoticgeneticadaptabilityandtherelationshipwithacquisitionofantibioticresistance

Page 181: Edited By Deeangelee Pooran-Kublall, MD/MPH · 9.5 Preparing for the MCAT: Biochemistry in the Chemical and Physical Foundations of Biological Systems Section 9.6 Preparing for the

•Growthcharacteristics,includingexponentialgrowth•Definitionandfundamentalcharacteristicsofanaerobesandaerobes•Definitionsofparasiticandsymbioticrelationships•Bacterialmovementandchemotaxis

GENETICSOFPROKARYOTICCELLS•ExtragenomicDNAandplasmids•DefinitionandsignificanceoftransformationasamechanismbywhichDNAfragmentsfromtheexternalmediumareincorporatedintothebacterialgenome•Definitionandsignificanceofconjugation•Theroleandfunctionoftransposons

VIRALSTRUCTURE• General structure of viruses, including nucleic acid, protein, presence or absence of anenvelope,andthelackoforganellesoranucleus•Structureofatypicalbacteriophage•Variationsinthecontentsoftheviralgenome(RNAorDNA)•Therelativesizeofvirusescomparedtobacteriaandeukaryoticcells

VIRALLIFECYCLE•Viral reproductionas a self-replicatingbiologicalunit that requires themachineryof ahostcellforreproduction•Thegenerallifecyclesofphagesandanimalviruses,includingattachmenttohost,penetrationofcellmembraneorcellwall,entryofviralgeneticmaterial, replicationofviralcomponentsusingthemachineryofthehostcell,andself-assemblyandreleaseofnewviralparticles•Definitionandsignificanceoftransductionasatransferofgeneticmaterialbyviruses• The life cycle of retroviruses, including integration into hostDNA; the purpose of reversetranscriptase•TheHIVlifecycle•Subviralparticles,includingprionsandviroids

MITOSIS•Theprocessofmitosis,includingthestagesofprophase,metaphase,anaphase,telophase,andinterphase•Identificationandroleofstructuresinvolvedinmitosis,includingcentrioles,asters,spindles,chromatids, centromeres, kinetochores, nuclear membrane (both breakdown andreorganization),andchromosomes(especiallymovementpatterns)•Identificationandsignificanceofeachportionofthecellcycle,includingG0,G1,S,G2,andMphases•Mechanismandpurposeofgrowtharrest•Mechanismsandroleofcell-cyclecontrol•Significanceoflossofcell-cyclecontrolsincancercells

Page 182: Edited By Deeangelee Pooran-Kublall, MD/MPH · 9.5 Preparing for the MCAT: Biochemistry in the Chemical and Physical Foundations of Biological Systems Section 9.6 Preparing for the

REPRODUCTIVESYSTEM•Theprocessofgametogenesisbymeiosis•Compareandcontrastovumandspermwithregardtodifferencesinformation,morphology,andrelativecontributiontothenextgeneration•Identificationofthereproductivesequence,includingfertilization,implantation,development,andbirth

EMBRYOGENESIS• Identification and processes occurring during early embryological development, includingfertilization,cleavage,blastulaformation,gastrulation,andneurulation•Theprocessofgastrulation,includingfirstcellmovementsandtheformationoftheprimarygermlayers(endoderm,mesoderm,andectoderm)•Identificationofmajorstructuresarisingfromtheprimarygermlayers•Identificationandsignificanceoftheneuralcrest•Interactionsbetweentheenvironmentandgenesduringdevelopment

MECHANISMSOFDEVELOPMENT•Mechanismsofcellspecialization, includingdeterminationanddifferentiationandhowtheseprocessesresultindifferenttissuetypes•Identificationofcell–cellcommunicationmechanismsduringdevelopment•Patternsofcellmigration•Thedefinitionofpluripotencyandhowitrelatestostemcells•Regulationofgenesduringdevelopment•Theroleofprogrammedcelldeath(apoptosis)duringdevelopment•Definitionofregenerativecapacityanditsexistenceincertainspecies•Definitionofsenescenceandhowitappliestoaging

NERVOUSSYSTEM:STRUCTUREANDFUNCTION• Identificationof themajor functionsof thenervoussystem, includinghigh levelcontrolandintegrationofbodysystemsandadaptivecapabilitytoexternalinfluences•Identificationofthebasicorganizationofthenervoussystem•Therolesandrelationshipbetweensensorandeffectorneurons• Roles, functions, and antagonistic control of the sympathetic and parasympathetic nervoussystem•Identificationofreflexes,includingthefeedbackloopandreflexarcaswellastheroleofthespinalcordandsupraspinalcircuits• Identification of feedback control and integration of the nervous systemwith the endocrinesystem

NERVECELL• Identification of the cell body and the structures it contains, including the nucleus and

Page 183: Edited By Deeangelee Pooran-Kublall, MD/MPH · 9.5 Preparing for the MCAT: Biochemistry in the Chemical and Physical Foundations of Biological Systems Section 9.6 Preparing for the

organelles•Structureandroleofthedendritesasbranchedextensionsofthecellbody•Structureandfunctionoftheaxon•IdentificationofthemyelinsheathasproducedbySchwanncellsanditsfunctionasaninsulatoroftheaxon**•TheprocessbywhichsaltatoryconductionoccurstopropagatethenerveimpulseandtheroleofnodesofRanvier**•Identificationofthesynapseasthesiteofimpulsepropagationbetweencells•Theroleoftransmittermoleculesinsynapticactivity•Identificationandroleoftherestingpotentialandthefunctionoftheelectrochemicalgradient•Transmissionofnerveimpulsesbytheactionpotential, includingthethresholdpotential,all-or-nothingconductionofimpulses,andtheroleofthesodium–potassiumpump• The purpose and relationship between excitatory and inhibitory nerve fibers; frequency offiring•Thedefinitionofsummation•Thepurposeofglialcellsandneuroglia

ENDOCRINESYSTEM:HORMONESANDTHEIRSOURCES•The functionof theendocrinesystemandhow itexerts specificchemicalcontrolat thecell,tissue,andorganlevel•Definitionsofendocrineglandandhormone•Thenames,locations,andproductsofmajorendocrineglands•Majortypesofhormones•Neuroendocrinologyasthestudyoftherelationshipbetweenneuronsandhormonalsystems

ENDOCRINESYSTEM:MECHANISMSOFHORMONEACTION•Mechanismsofhormoneactionatthecellularlevel•Bloodsupplyandthetransportofhormonestotargettissues•Hormonespecificityanditsrelationshiptothetargettissue•Nervoussystemintegrationandfeedbackcontrolregulationbysecondmessengers

RESPIRATORYSYSTEM• General functions of the respiratory system, including gas exchange, thermoregulation,protectionagainstdisease,andparticulatematterfiltration•Identificationofthestructureoflungsandalveoli•Mechanismsofbreathing,includingtherolesofthediaphragm,ribcage,differentialpressure,resiliency,andsurfacetensioneffects•Functionof the respiratory system in thermoregulation, including the roles of thenasal andtrachealcapillarybeds,evaporation,andpanting•Functionof the respiratory system inparticulate filtration, including the rolesofnasalhairsandthemucus/ciliasysteminlungs•Mechanismofalveolargasexchange,includingdiffusionanddifferentialpartialpressure•ControlofpHusingrespiration

Page 184: Edited By Deeangelee Pooran-Kublall, MD/MPH · 9.5 Preparing for the MCAT: Biochemistry in the Chemical and Physical Foundations of Biological Systems Section 9.6 Preparing for the

•Regulationofrespirationbynervouscontrol,includingCO2sensitivity

CIRCULATORYSYSTEM•Functionsofthecirculatorysystem,includingcirculationofoxygen,nutrients,hormones,ions,andfluidsandtheremovalofmetabolicwaste•Theroleofthecirculatorysysteminthermoregulation•Structureandfunctionofthefour-chamberedheart•Identificationandroleofendothelialcells• Definitions of systolic and diastolic pressures and which parts of the circulatory cycle arerepresentedbyeach•Circulation,includingthepulmonaryandsystemiccirculations•Structureandfunctionaldifferencesbetweenthearterialandvenoussystems**•Structuresofthearterial(arteries,arterioles)andvenous(venules,veins)systems•Differencesbetweenthepressureandflowcharacteristicsofthearterialandvenoussystems•Themechanismsofgasandsoluteexchangeandheatexchangeinthecapillarybeds•Thecapillarybedsasasourceofperipheralresistance•Thecompositionofblood,includingplasma,chemicals,andbloodcells•Productionoferythrocytesinthebonemarrowanddestructionoferythrocytesinthespleen•Regulationofplasmavolume•Mechanismsofcoagulationandclotting• Transport of oxygen by blood, including the role of hemoglobin and the definition ofhematocrit•Oxygencontent,affinity,andtransportofblood;modificationofoxygenaffinityasrelatedtoconditionsandlocation(tissuesorlungs)•Transportandlevelofcarbondioxideinblood•Controlofthecirculatorysystembythenervousandendocrinesystems

LYMPHATICSYSTEM•Generalstructureofthelymphaticsystem• Functions of the lymphatic system, including equalization of fluid distribution, transport ofproteins and large glycerides, production of lymphocytes involved in immune reactions, andreturnofmaterialstotheblood

IMMUNESYSTEM•Definitionsandtargetsoftheinnate(nonspecific)andadaptive(specific)immunity•Cellsoftheadaptiveimmunesystems,includingT-lymphocytesandB-lymphocytes•Cellsoftheinnateimmunesystem,includingmacrophagesandphagocytes•Major tissues of the immune system including the spleen, bonemarrow, thymus, and lymphnodes•Definitionofantigenandantibody•Howantigensarepresentedintheimmunesystem•Clonalselection•Antigen-antibodyrecognition

Page 185: Edited By Deeangelee Pooran-Kublall, MD/MPH · 9.5 Preparing for the MCAT: Biochemistry in the Chemical and Physical Foundations of Biological Systems Section 9.6 Preparing for the

•Thestructureofantibodies•Autoimmunediseaseandtheprocessbywhichtheimmunesystemdistinguishesselffromnon-self•Theroleofthemajorhistocompatibilitycomplex

DIGESTIVESYSTEM•Theprocessofingestion,includingsalivaaslubricantandsourceofenzymes,andtheroleoftheesophagusandesophagealtransport•Structureandroleofthestomach,includingstorage,churningoffood,productionofdigestiveenzymes,andsiteofdigestion• The role of low pH and gastric juice in digestion, and mucosal protection against self-destruction•Thestructural relationshipof the liverwithin thegastrointestinalsystemand the rolesof theliver,includingproductionofbile,bloodglucoseregulation,anddetoxification•Functionofbileanditsstorageinthegallbladder• The role of the pancreas in digestion, including production of enzymes and transport ofenzymestothesmallintestine• The structure (including divisions) and role of the small intestine in digestion, includingabsorption of food molecules and water, production of enzymes, site of digestion, andneutralizationofstomachacid•Functionandstructureofvilliwithinthesmallintestine• The gross structure and function of the large intestine, including absorption of water andlocationofbacterialflora•Theroleoftherectuminstorageandeliminationofwasteandfeces•Muscularcontrolofthegastrointestinalsystem,includingtheroleandnatureofperistalsis•Endocrinecontrolofthegastrointestinalsystembyhormonesandtargettissues• The enteric nervous system and how it exerts nervous control over functions of thegastrointestinalsystem

EXCRETORYSYSTEM• Roles of the excretory system in homeostasis, including blood pressure regulation,osmoregulation,acid–basebalance,andremovalofsolublenitrogenouswaste•Structureofthekidney,includingthecortexandmedulla•Structureof thenephron, including theglomerulus,Bowman’scapsule,proximalconvolutedtubule,loopofHenle,distalconvolutedtubule,andcollectingduct•Urineformationincludingtheconceptsofglomerulusfiltration,secretionandreabsorptionofsolutes,concentrationofurine,andthecounter-currentmultipliersystem•Storageandeliminationofurinebytheureter,bladder,andurethra•OsmoregulationandtheroleofcapillaryresorptionofH2O,aminoacids,glucose,andions•Muscularcontrolofurination,includingthesphinctermuscle

REPRODUCTIVESYSTEM• Structure and function of the male and female reproductive systems, including gonads and

Page 186: Edited By Deeangelee Pooran-Kublall, MD/MPH · 9.5 Preparing for the MCAT: Biochemistry in the Chemical and Physical Foundations of Biological Systems Section 9.6 Preparing for the

genitalia•Differencesbetweenmaleandfemalestructures•Controlofreproductionbyhormones,includingmaleandfemalesexualdevelopment,femalereproductivecycle,pregnancy,parturition(birth),lactation,andintegrationwithnervouscontrol

MUSCLESYSTEM• Functions of the muscle system, including support for mobility, peripheral circulatoryassistance,andthermoregulationbytheshiveringreflex• Identification and structure of the three basic muscle types, including striated, smooth, andcardiacmuscle• Structure of muscle, including the T-tubule system, contractile apparatus, sarcoplasmicreticulum,andfibertype;controlofcontraction•Contractilevelocityofdifferentmuscletypes•Cardiacmusclecontractionandregulation•Fatigueandoxygendebt•Nervouscontrolwithinthemusclesystem,includingmotorneurons,neuromuscularjunction,motor endplates, sympathetic andparasympathetic innervation, andvoluntary and involuntarymuscles

SPECIALIZEDCELL–MUSCLECELL•Structureandcharacteristicsofstriated,smooth,andcardiacmusclecells•SignificantquantitiesofmitochondriainredmusclecellstoproduceareadyATPsource• Identification and organization of contractile elements within muscles, including actin andmyosinfilaments,crossbridges,andtheslidingfilamentmodels•Structureswithinsarcomeres,includingthe“I”and“A”bands,“M”and“Z”lines,andthe“H”zone•Roleandpresenceoftroponinandtropomyosin•Regulationofcontractionbycalcium

SKELETALSYSTEM•Functionsoftheskeletalsystem,includingstructuralrigidityandsupport,calciumstorage,andphysicalprotection•Structureoftheskeleton,includingspecializationandstructuresofbonetypes,jointstructures,andendoskeletonvs.exoskeleton•Structureofbones,includingthecalcium/proteinmatrixandthecellularcompositionofbone•Structureandfunctionofcartilage•Definitionsandrolesofligamentsandtendons•Endocrinecontroloftheskeletalsystem

SKINSYSTEM• Structure of the skin system, including layer differentiation, cell types, and relativeimpermeabilitytowater

Page 187: Edited By Deeangelee Pooran-Kublall, MD/MPH · 9.5 Preparing for the MCAT: Biochemistry in the Chemical and Physical Foundations of Biological Systems Section 9.6 Preparing for the

•Functionsoftheskinsysteminhomeostasisandosmoregulation•Functionsoftheskinsysteminthermoregulation,includinghairanderectilemusculature,fatlayer for insulation, sweat glands located in the dermis, and vasoconstriction/vasodilation insurfacecapillaries• Functions of the skeletal system in physical protection, including nails, calluses, hair, andprotectionagainstabrasionanddiseaseorganisms•Hormonalcontroloftheskinsystem,includingsweating,vasodilation,andvasoconstriction

Page 188: Edited By Deeangelee Pooran-Kublall, MD/MPH · 9.5 Preparing for the MCAT: Biochemistry in the Chemical and Physical Foundations of Biological Systems Section 9.6 Preparing for the

10.6BiologyWorkedExamplesPASSAGEI:THENERVOUSSYSTEMDuringdevelopment, oligodendrocyteprogenitor cells are formed,which furtherdifferentiate intotheglialcells responsible forcreating themyelinsheathsof thenervoussystem.Multiplesclerosis(MS)ischaracterizedbydamageto,ordestructionof,myelinsheathsinthecentralnervoussystem(CNS).Thisresultsinsymptomsthatvaryfromphysicaltomentalmanifestations.Thereiscurrentlynocureforthisdiseaseanditscauseisstillnotcompletelyclear.

ThemostwidelyacceptedtheoryfortheonsetofMShasconsistentlybeentheclassicalautoimmunehypothesis. Recent research reports evidence that nonspecific T-cells in the cervical lymph nodes,which drain theCNS, become exposed to cellswithmyelin-like characteristics. TheseT-cells thenbecomespecificagainstthemyelinatingoligodendrocytes.AfterentryintotheCNS,aninflammatoryresponseisinduced,signalingotherleukocytesandmacrophagestomigratethroughtheblood–brainbarrier. The attackmounted by these cells effectively kills the oligodendrocytes and degrades themyelinsheaths.Figure1outlinesthisprocess.

Another theory is the oligodendropathy hypothesis. According to the researchers supporting thispostulation, unexplained reasons cause oligodendrocytes to undergo apoptosis (spontaneous selfdestruction). Death of the glial cells then leads tomyelin degradation. The inflammatory immuneresponse by macrophages is simply a means to clean away the already destroyed myelin debris.Figure2outlinesthisprocess.

Figure1.Autoimmunehypothesis

Page 189: Edited By Deeangelee Pooran-Kublall, MD/MPH · 9.5 Preparing for the MCAT: Biochemistry in the Chemical and Physical Foundations of Biological Systems Section 9.6 Preparing for the

Figure2.Oligodendropathyhypothesis

P1.

P2.

P3.

F1.

1.ThepropagationofactionpotentialsintheCNSofapatientwithmultiplesclerosiscomparedtoacontrolsubjectwouldbe:

A.faster,becausethereislessmattertoinhibititspathdowntheaxon.B.faster,becausethelackofmyelinallowsionstomoreeasilycrossthecellmembrane.C.slower,becauseofthedecreasednumberofSchwanncells.D.slower,becauseconductionisnolongersaltatory.

2.ThesequenceinFigure2wouldbebestsupportedoverthatofFigure1bywhichofthefollowing?A.TheCNSofapatientwithmultiplesclerosiswasobservedtohavemanyscarsandlesions.B.T-cellsfromamousewithmultiplesclerosisaretransferredtoanormalmousethatovertimebeginstodisplaysymptomsofmultiplesclerosis.C.AstudywasconductedthatshowedlargenumbersofimmunecellswereobservedtofirstentertheCNSonlyaftersignsofmyelindegradationhadbeennoted.D.UnusuallyhighnumbersofmacrophagesarefoundintheCNSofpatientswithmultiplesclerosis.

3.InGuillain–Barrésyndrome,demyelinationoccursintheperipheralnervoussystemonly.ApatientwiththisconditionwouldbeLEASTlikelytoexhibitadecreasein:

A.motorcoordination.

Page 190: Edited By Deeangelee Pooran-Kublall, MD/MPH · 9.5 Preparing for the MCAT: Biochemistry in the Chemical and Physical Foundations of Biological Systems Section 9.6 Preparing for the

B.autonomicfunction.C.efferentmotorresponses.D.afferentsensoryresponses.

4.ResearchersdevelopadrugthatslowstheprogressionofmultiplesclerosisbystimulatingproductionofmyelinintheCNS.Thistreatmentaddressestheprimarycauseoftheconditiondescribedby:

A.theautoimmunehypothesisonly.B.theoligodendropathyhypothesisonly.C.bothhypotheses.D.neitherhypothesis.

5.Thediagrambelowdepictsasomaticreflexarc.

Inapatientwithmultiplesclerosis,thereflexarcwouldmostlikely:A.remainunchanged.B.exhibitadelayatneuronA.C.exhibitadelayatinterneuronB.D.exhibitadelayatneuronC.

Page 191: Edited By Deeangelee Pooran-Kublall, MD/MPH · 9.5 Preparing for the MCAT: Biochemistry in the Chemical and Physical Foundations of Biological Systems Section 9.6 Preparing for the

BiologyPassageIExplanation:SamplePassageMapP1:Multiplesclerosis

P2:autoimmune;T-cellskilloligodendrocytes

P3:oligodendropathic;apoptosisthenmacrophages

F1:Autoimmunehypothesis

F2:Oligodendropathyhypothesis

KeyConceptsMyelinsheath,Actionpotentials

1.ThepropagationofactionpotentialsintheCNSofapatientwithmultiplesclerosiscomparedtoacontrolsubjectwouldbe:

A.faster,becausethereislessmattertoinhibititspathdowntheaxon.B.faster,becausethelackofmyelinallowsionstomoreeasilycrossthecellmembrane.C.slower,becauseofthedecreasednumberofSchwanncells.D.slower,becauseconductionisnolongersaltatory.

AssessthequestionThequestionwantstoknowwhethermultiplesclerosisspeedsuporslowsdownactionpotentials,aswellaswhy.

PlanyourattackWe’ll need to look at the description of how multiple sclerosis damages the nervous system inparagraph1,andthenuseourknowledgeofthenervoussystemtofindthecorrectanswer.

Page 192: Edited By Deeangelee Pooran-Kublall, MD/MPH · 9.5 Preparing for the MCAT: Biochemistry in the Chemical and Physical Foundations of Biological Systems Section 9.6 Preparing for the

ExecutetheplanParagraph 1 tells us that inMS, neurons in theCNS lose theirmyelin sheaths. Themyelin sheathservesanumberoffunctions,butitsmostnotableoneistospeedupconductionofactionpotentials.Without the myelin sheath, conduction speed decreases dramatically, so we would predict thatconductionshouldslowdowninitsabsence.

Specifically, the myelin sheath speeds up conduction because conduction becomes saltatory—ionexchangeacrossthemembranebecomesnecessaryonlyatthenodesofRanvier,theperiodicgapsinthemyelinsheath.Soalossofmyelinshouldresultinalossofsaltatoryconduction.

Answerbymatching,eliminating,orguessingChoice(D)matchesbothpartsofourprediction,andisthecorrectanswer.(C)mightseemtempting,butreaditcarefully:itreferstoadecreaseinSchwanncells,whicharefoundonlyintheperipheralnervoussystem.SinceMSinvolvesthecentralnervoussystem,thatanswerisatrap.

ThingstoWatchOutForSometrapanswersonTestDayaredesignedtobesotemptingthattest-takerswillpickthemandmoveonbeforeeverreadingthecorrectanswer.Criticalthinkingwillhelpyouavoidthosetraps!

KeyConceptsNervecell,Myelinsheath

2.ThesequenceinFigure2wouldbebestsupportedoverthatofFigure1bywhichofthefollowing?

A.TheCNSofapatientwithmultiplesclerosiswasobservedtohavemanyscarsandlesions.B.T-cellsfromamousewithmultiplesclerosisaretransferredtoanormalmousethatovertimebeginstodisplaysymptomsofmultiplesclerosis.C.AstudywasconductedthatshowedlargenumbersofimmunecellswereobservedtofirstentertheCNSonlyaftersignsofmyelindegradationhadbeennoted.D.UnusuallyhighnumbersofmacrophagesarefoundintheCNSofpatientswithmultiplesclerosis.

Page 193: Edited By Deeangelee Pooran-Kublall, MD/MPH · 9.5 Preparing for the MCAT: Biochemistry in the Chemical and Physical Foundations of Biological Systems Section 9.6 Preparing for the

AssessthequestionThisquestionasksusforanewfindingthatwouldsupporttheoligodendropathytheorydepictedinFigure2over theautoimmunehypothesis inFigure1.Thatmeans the correct answermust be trueonlyoftheoligodendropathytheory.

PlanyourattackWe’llneedtoexaminethetwofigurestofindthekeydifferences.We’llthenneedtofindananswerchoicethatillustratesoneofthosedifferences.

ExecutetheplanLooking at the two figures, the key difference is how the cells die. In the autoimmune hypothesis(Figure1), theT-cellskill theoligodendrocytes; intheoligodendropathyhypothesis(Figure2), thecellsundergoapoptosisfirst(forunknownreasons),andthentheT-cellsattack.Ourcorrectanswershouldillustratethelattertheoryonly.

Answerbymatching,eliminating,orguessingFor (A), the presence of scars and lesions in the central nervous system could fit with eitherhypothesis—itissimplyaneffectofdemyelinationingeneral,howeveritoccurred.(A)isincorrect.

Next,lookat(B).Here,apreviouslynormalmousedevelopsMSafteritisexposedtoT-cellsfromanaffectedmouse.ThissuggeststhattheT-cellscausedtheonsetofmultiplesclerosis.Figure1showsthisisthefirststepoftheautoimmunehypothesis,so(B)isout.

Choice (C) describes a scenario inwhich demyelination has already occurred before the immunecellsarrive.ThismatcheswhatFigure2showsus,andcontradictsFigure1.Choice(C)mustbeouranswer.

Choice(D)istoogeneral.BothfiguresshowmacrophagesintheCNS,sothiscannotbetheanswer.

KeyConceptsNervoussystemstructureandfunction

3.InGuillain–Barrésyndrome,demyelinationoccursintheperipheralnervoussystemonly.A

Page 194: Edited By Deeangelee Pooran-Kublall, MD/MPH · 9.5 Preparing for the MCAT: Biochemistry in the Chemical and Physical Foundations of Biological Systems Section 9.6 Preparing for the

patientwiththisconditionwouldbeLEASTlikelytoexhibitadecreasein:A.motorcoordination.B.autonomicfunction.C.efferentmotorresponses.D.afferentsensoryresponses.

AssessthequestionThisquestion is testing the functionsof theperipheralandcentralnervoussystems.Note that sincethisisaLEASTquestion,we’relookingfortheoneanswerthatdoesn’tfit.

PlanyourattackThisisapseudodiscretequestion;wedon’tneedanyinformationfromthepassage,butwewillneedour knowledge of the functions of the central and peripheral nervous systems. In particular,we’relookingfortheonefunctionthatisnotpartoftheperipheralnervoussystem.

ExecutetheplanThecentralnervoussystemincludesthebrainandspinalcord,whiletheperipheralnervoussystemcontainsthesensoryandmotorsystems.Sowe’relookingforananswerinvolvingthebrainand/orspinalcord.

Answerbymatching,eliminating,orguessingMotorcoordinationtakesplaceinthecerebellumofthebrain,whichispartoftheCNS.Thuschoice(A)iscorrect,andwecanstop.

Forcompleteness’sake,let’slookattheotheranswers.AutonomicfunctioniscontrolledbyboththeCNSandPNS, so (B) iswrong.Efferentmotor responses and afferent sensory responses arebothrelayedby theperipheralnervoussystem(andprocessed in theCNS),sowecaneliminate (C) and(D).

ThingstoWatchOutForReadquestionscarefullytocatchkeywords,suchas“NOT”or“EXCEPT.”

Page 195: Edited By Deeangelee Pooran-Kublall, MD/MPH · 9.5 Preparing for the MCAT: Biochemistry in the Chemical and Physical Foundations of Biological Systems Section 9.6 Preparing for the

KeyConceptsNervecell,Myelinsheath

4.ResearchersdevelopadrugthatslowstheprogressionofmultiplesclerosisbystimulatingproductionofmyelinintheCNS.Thistreatmentaddressestheprimarycauseoftheconditiondescribedby:

A.theautoimmunehypothesisonly.B.theoligodendropathyhypothesisonly.C.bothhypotheses.D.neitherhypothesis.

AssessthequestionThisquestionasksustodeterminewhetherthephenomenondescribedinthequestionstemsupportsthetheoriesdescribedinthepassage.

PlanyourattackTo answer this question, we’ll need to see if stimulating CNS production of myelin supports theautoimmune hypothesis, by checking paragraph 2; and/or the oligodendropathy hypothesis, bycheckingparagraph3.

ExecutetheplanParagraph2tellsusthat,accordingtotheautoimmunehypothesis,multiplesclerosisistriggeredbyT-cells attacking myelin-producing oligodendrocytes. Would creating more myelin prevent thisattack?No.Sothedrugdoesnottreatthecause,accordingtothistheory.

Paragraph 3 describes the oligodendropathy hypothesis, which argues that MS is triggered byapoptosis,orspontaneousself-destruction,ofoligodendrocytes.Wouldcreatingmoremyelinpreventthisfromhappening?No;myelindoesnotplayaroleinapoptosis.Sothenewdrugwouldn’ttreatthecauseinthistheory,either.

Answerbymatching,eliminating,orguessing

Page 196: Edited By Deeangelee Pooran-Kublall, MD/MPH · 9.5 Preparing for the MCAT: Biochemistry in the Chemical and Physical Foundations of Biological Systems Section 9.6 Preparing for the

Onceweknowthatthedrug’sactiondoesn’ttreatthecauseaccordingtotheautoimmunehypothesis,we can eliminate (A) and (C). Knowing that it doesn’t treat the cause in the oligodendropathyhypothesiseithermeanswecanalsoeliminate(B),leavingchoice(D)asthecorrectanswer.

TakeawaysWheneverapassagediscussesmultipleprocessesortheories,besuretounderstandthekeyfeaturesofeach,aswellastheirkeysimilaritiesanddifferences.

KeyConceptsNervoussystemstructureandfunction,Reflexes

5.Thediagrambelowdepictsasomaticreflexarc.

Inapatientwithmultiplesclerosis,thereflexarcwouldmostlikely:A.remainunchanged.B.exhibitadelayatneuronA.C.exhibitadelayatinterneuronB.D.exhibitadelayatneuronC.

AssessthequestionThisquestiongivesusadiagramofareflexarc,andthenasksushowMSwouldaffectthediagram.Notethatoneofthechoicesis“nochange,”soweshouldn’tignorethatpossibility.

Planyourattack

Page 197: Edited By Deeangelee Pooran-Kublall, MD/MPH · 9.5 Preparing for the MCAT: Biochemistry in the Chemical and Physical Foundations of Biological Systems Section 9.6 Preparing for the

We’llneedthedescriptionofmultiplesclerosisinparagraph1,aswellasourknowledgeofreflexarcs,toanswerthisquestion.

ExecutetheplanMultiple sclerosis only affects the central nervous system—that is, the brain and spinal cord—accordingtoparagraph1.Thatmeansitdoesnotaffecttheperipheralnervoussystem,withitsmotorandsensorycomponents.Asaresult,wecanpredictthatMSwouldaffectaneuronthatispartofthecentralnervoussystem,ifthereisone.Ifthereisn’t,thenthereshouldbenochange.

Answerbymatching,eliminating,orguessingInthediagram,neuronAisasensoryneuron,whileneuronCisamotorneuron.Bothofthesearepart of theperipheral nervous system, sowe can eliminate (B) and (D).NeuronB, however, is aninterneuron,whichispartofthecentralnervoussystem.Thereforewewouldexpectittobeinvolved,sochoice(D)isthecorrectanswer.

BIOLOGYPASSAGEII:THEENDOCRINESYSTEMAldosterone release from the adrenal cortex plays a key role in controlling blood pressure. ThisimportanthormoneisregulatedasshowninFigure1:

Figure1.Therenin-angiotensinfeedbacksystem

Low blood volume through the kidneys triggers juxtaglomerular cells to secrete renin intocirculation. Angiotensinogen released from the liver is converted by this enzyme into the peptideangiotensinI.Angiotensin-convertingenzyme(ACE)thenconvertsangiotensinIintoangiotensinII,which stimulates the adrenal cortex to release aldosterone. The ultimate increase in blood volumecausedbyaldosteroneshutsoffthesecretionofreninfromthejuxtaglomerularcells.AngiotensinIIalsoincreasesarteriolarvasoconstriction,sympatheticnervousactivity,andreabsorptionofsodiumandchloride.Aldosterone,inadditiontoitseffectsonsodium,alsoincreasespotassiumexcretion.

Researchersatapharmaceuticalcompanyareinvestigatingtheuseofcompoundsthoughttoinhibit

Page 198: Edited By Deeangelee Pooran-Kublall, MD/MPH · 9.5 Preparing for the MCAT: Biochemistry in the Chemical and Physical Foundations of Biological Systems Section 9.6 Preparing for the

stepsintherenin-angiotensin-aldosteronesystemasameansoftreatinghypertension.Aclinicaltrialhas been approved to test the effects of four such drugs. Eighty subjects, each exhibiting chronichypertensionwithsystolicpressuresabove160mmHganddiastolicpressuresabove100mmHg,wererecruited;thesubjectsweredividedintofourgroupsof20patientseach.Eachgroupreceiveda10mgdailydoseofoneofthefourdrugs.Bloodpressure,urinevolume,andconcentrationofurinepotassiumweremeasuredeverydayforeachpatientineachgroup.Attheendofthetrial,theaverageresultsofthesemeasurementsoverthelastweekofthetrialwerecalculated;theresultsareshowninTable1.

AverageSystolic/DiastolicBloodPressure(mmHg)

AverageDailyUrineVolume(L)

AverageDailyUrinePotassiumConcentration(mEq/L)

DrugA

118/75

2.5

175

DrugB

145/95

2.2

50

DrugC

116/80

2.5

40

DrugD

122/78

1.1

135

Normalranges

0.8–2.0

25–125

Table1.Resultsofaclinicaltrialoffourdrugstotreathypertension

P1.

F1.

P2.

P3.

Table1.

1.BasedontheresultsinTable1,whichofthefollowingconclusionsaboutDrugAandDrugCismostreasonable?

A.DrugAinhibitsreninreleaseandDrugCinhibitsADHrelease.B.DrugAinhibitsADHreleaseandDrugCinhibitsACErelease.

Page 199: Edited By Deeangelee Pooran-Kublall, MD/MPH · 9.5 Preparing for the MCAT: Biochemistry in the Chemical and Physical Foundations of Biological Systems Section 9.6 Preparing for the

C.DrugAinhibitsACEreleaseandDrugCinhibitsreninrelease.D.DrugAinhibitsaldosteronereleaseandDrugCinhibitsADHrelease.

2.Supposethatacertainsubstanceiscapableofpreventingbindingofallhormonestoreceptorsonthecellmembrane.Suchadrugwouldinfluencethefunctionof:

A.ADH,butnotaldosterone.B.aldosterone,butnotADH.C.bothADHandaldosterone.D.neitherADHnoraldosterone.

3.Asuccessfulinhibitoroftherenin-angiotensin-aldosteronesystemcouldreasonablybeexpectedtocauseallofthefollowingEXCEPT:

A.increaseddilationofarterioles.B.decreasedurinesodiumconcentration.C.decreasedpupildilation.D.increasedurinevolume.

4.WhileDrugBwastheleastsuccessfulfortreatinghypertension,theresultsobtainedsuggestitwouldbemostusefulintreating:

A.hypotension.B.dehydration.C.edema.D.hyperkalemia(highplasmaK+levels).

5.AresearcherbelievessheknowsthemechanismbywhichDrugDlowersbloodpressure.Toconfirmherhypothesis,sheshouldperformanexperiment,withappropriatecontrols,tomeasure:

A.angiotensinogenreleasefromtheliver.B.reninreleasefromthekidneys.C.arteriolarpressureincreasescausedbyangiotensinII.D.concentrationofACEintheplasma.

Page 200: Edited By Deeangelee Pooran-Kublall, MD/MPH · 9.5 Preparing for the MCAT: Biochemistry in the Chemical and Physical Foundations of Biological Systems Section 9.6 Preparing for the

BiologyPassageIIExplanation:USINGTHEKAPLANMETHODSP1:Aldosterone

F1:Renin-angiotensin-aldosteronesystem

P2:Hormonefunctions

P3:Exp:inhibitRAApathwaytolowerbloodpressure

T1:Expresults;4drugs

TakeawaysWhenapassagepresentsanexperiment,expectquestionstestingyourabilitytointerprettheresults,especiallyifthepassagedoesn’tdrawanyconclusionsforyou.

KeyConceptsOsmoregulation,Secretionandreabsorptionofsolutes

1.BasedontheresultsinTable1,whichofthefollowingconclusionsaboutDrugAandDrugCismostreasonable?

A.DrugAinhibitsreninreleaseandDrugCinhibitsADHrelease.B.DrugAinhibitsADHreleaseandDrugCinhibitsACErelease.C.DrugAinhibitsACEreleaseandDrugCinhibitsreninrelease.D.DrugAinhibitsaldosteronereleaseandDrugCinhibitsADHrelease.

AssessthequestionAll theanswerchoicesdealwith inhibiting releaseofhormones, soour job is to figureoutwhichhormonesDrugAandDrugCinhibit,outofrenin,ADH,ACE,andaldosterone.

Planyourattack

Page 201: Edited By Deeangelee Pooran-Kublall, MD/MPH · 9.5 Preparing for the MCAT: Biochemistry in the Chemical and Physical Foundations of Biological Systems Section 9.6 Preparing for the

Toanswerthisquestion,we’llneedtoexaminethedifferencesinTable1betweenDrugAandDrugC; we’ll also need to use Figure 1, which shows how the various hormones are regulated, andparagraph 3, which gives us the normal urine output and potassium content. We may also needparagraph2,whichdescribesthefunctionsofsomeofthesehormones.

ExecutetheplanLookingatTable1,theaveragebloodpressureforpatientstakingDrugAandpatientstakingDrugCisapproximately thesame.Theaverageurineoutput is the same forbothdrugs,and iselevated inbothcases.TheonlydifferenceinthetableistheurineconcentrationofK+ions;theurineofDrugApatientshasanelevatedpotassiumlevel,whileDrugCpatientshaveapotassiumlevelatthelowendof theaveragerange.Ourcorrectanswer thusneeds toexplainhowDrugAcausesK+excretion torise,butDrugCdoesnot.

Thekey toanswering thisquestion is tucked into theendofparagraph2: increasedK+excretion iscausedbyaldosterone.Therefore,DrugA’seffectsshouldnotincludeinhibitingaldosterone.

Answerbymatching,eliminating,orguessingWecanimmediatelyeliminate(D),sincewe’veestablishedthatwouldnotresultinelevatedurineK+.Renin (A) andACE (C) are both needed to stimulate aldosterone release. Inhibiting either of themshouldresultinreducedaldosteroneactivity,whichwouldmakeitunlikelythatDrugAwouldcauseelevatedurinepotassiumlevels.Eliminatingthoseanswersleaveschoice(B)asthecorrectanswer.

Notethatwedidn’tevenneedtoconsiderDrugChere,butifitwereinhibitingACE,we’dexpecttoseedecreasedaldosteroneactivity,whichdoesn’tcontradicttheinformationinthetable.

ThingstoWatchOutForWatchoutwhenanexperimentalpassagepresentsresultsformultiplevariables.Oddsaretherewillbeatleastoneproblemthatrequiresaconclusiontobedrawnfromacomparisonofthoseresults.

KeyConceptsCellularmechanismsofhormoneaction

2.Supposethatacertainsubstanceiscapableofpreventingbindingofallhormonestoreceptorsonthecellmembrane.Suchadrugwouldinfluencethefunctionof:

A.ADH,butnotaldosterone.

Page 202: Edited By Deeangelee Pooran-Kublall, MD/MPH · 9.5 Preparing for the MCAT: Biochemistry in the Chemical and Physical Foundations of Biological Systems Section 9.6 Preparing for the

B.aldosterone,butnotADH.C.bothADHandaldosterone.D.neitherADHnoraldosterone.

AssessthequestionWhileit’shardtoenvisionthatasubstancesuchastheonesuggestedbythequestionstemcouldevenexist,thatdoesn’tmatter.Weassumeit’strueandgoonfromthere.TheanswerchoicesaskwhetherthissubstancewouldaffectADH,aldosterone,both,orneither.

PlanyourattackThis is actually a pseudodiscrete question, as we don’t need any information directly from thepassage.Infact,weonlyneedtwopiecesofinformation:anunderstandingofwhathormonesbindtocellmembranereceptors,andthehormoneclassestowhichADHandaldosteronebelong.Thenwecanapplythatinformationtothenewscenariointhequestionstem.

ExecutetheplanThere are twomajor classes of hormones: steroid hormones and peptide (amino acid) hormones.Steroid hormones have intracellular receptors, because steroids are permeable and cross the cellmembrane.Peptidehormones,however,havemembrane-bound receptors,becauseaminoacidsaretoopolartocrossthecellmembrane.

Soifourhypotheticalcompoundcanblockallhormonebindingatthecellmembrane,thenitwouldaffect peptide hormones, but it should not affect steroid hormones. Aldosterone, like all adrenalcortexhormones, isasteroid;ADH, likeallhormonesproduced in thehypothalamus, isapeptide.Keepingthisinmind,wewouldexpectthedrugtoaffectonlyADH,andnotaldosterone.

Answerbymatching,eliminating,orguessingChoice(A)matchesourprediction.

KeyConceptsOsmoregulation,Secretionandreabsorptionofsolutes,Sympatheticandparasympatheticnervoussystems

Page 203: Edited By Deeangelee Pooran-Kublall, MD/MPH · 9.5 Preparing for the MCAT: Biochemistry in the Chemical and Physical Foundations of Biological Systems Section 9.6 Preparing for the

3.Asuccessfulinhibitoroftherenin-angiotensin-aldosteronesystemcouldreasonablybeexpectedtocauseallofthefollowingEXCEPT:

A.increaseddilationofarterioles.B.decreasedurinesodiumconcentration.C.decreasedpupildilation.D.increasedurinevolume.

AssessthequestionWe are being asked to consider the effects of blocking the renin-angiotensin-aldosterone (RAA)pathwayandtoidentifytheanswerthatdoesnotfitthatpicture.

PlanyourattackFigure 1 shows the various steps of the pathway, while paragraph 2 describes the effects of thehormones.

ExecutetheplanSincewe’relookingforsomethingthatblockingthepathwaywillnotcause,thewronganswersarethings that inhibiting the pathwaywill cause.The correct answerwill therefore be either a normaleffectoftherenin-angiotensin-aldosteronepathway,orsomethingthatisnotaffectedbyitatall.

Answerbymatching,eliminating,orguessingParagraph2tellsusthatangiotensinIIincreasesarteriolevasoconstriction.So,inhibitingRAAwouldmeanalackofconstriction—inotherwords,weexpectvasodilation.Thatmeans(A)wouldhappen,andisthereforeincorrect.

TheendeffectoftheRAApathwayisreleaseofaldosterone,whichincreasessodiumreabsorptioninthekidney. Inhibiting thepathwaywouldmeandecreased sodiumreaborption,and thereforehigherexcretion.Choice(B)sayswe’dexpectlowersodiumexcretion;thisisthe“normal”RAAeffectwepredictedwe’dseeasthecorrectanswer.

Paragraph 2 tells us that angiotensin II increases sympathetic nervous activity. Pupil dilation is asympatheticeffect,soinhibitingRAAwoulddecreasedilation.Ruleoutchoice(C).

Page 204: Edited By Deeangelee Pooran-Kublall, MD/MPH · 9.5 Preparing for the MCAT: Biochemistry in the Chemical and Physical Foundations of Biological Systems Section 9.6 Preparing for the

Becausealdosterone increases sodiumreabsorption, it also indirectly increaseswater reabsorption.WewouldthusexpecttheRAApathwaytodecreaseurinevolume.Choice(D)isalsoincorrect.

ThingstoWatchOutForWatchoutforwhenaquestionstemusesthewordEXCEPT.Thisbecomesevenmoreimportantwhenaquestioneffectivelycontainsadoublenegative,asthisonedoes.Whenselectingyouranswerchoice,makesureitsatisfiesthequestion.

KeyConceptsOsmoregulation,Mechanismsofhormoneaction,Feedbackcontrolregulation

4.WhileDrugBwastheleastsuccessfulfortreatinghypertension,theresultsobtainedsuggestitwouldbemostusefulintreating:

A.hypotensionB.dehydration.C.edema.D.hyperkalemia(highplasmaK+levels).

AssessthequestionThisquestionisfairlystraightforward:whichoftheseconditionswouldDrugBbebestattreating?

PlanyourattackToanswer thisquestion,we’ll need to lookat the results forDrugB inTable1, and thenuseourknowledgeoftheconditionsintheanswerchoices.

ExecutetheplanLooking at Table 1, patients taking Drug C had slightly elevated urine output, but normal urinepotassiumlevels.So thecorrectanswerwillbeacondition thatcanbe treatedsolelyby increasingurineoutput.

Page 205: Edited By Deeangelee Pooran-Kublall, MD/MPH · 9.5 Preparing for the MCAT: Biochemistry in the Chemical and Physical Foundations of Biological Systems Section 9.6 Preparing for the

Answerbymatching,eliminating,orguessingLookingat theanswerchoices, theonlyonethatcanbetreatedsolelybyincreasingurineoutput isedema,whichisanaccumulationoffluidininterstitialspaces.Thecorrectanswerischoice(C).

(A)might sound tempting; ifDrugBdoesn’t reducebloodpressure,maybe itcanbeused to raiseblood pressure. However, the passage told us that the patients involved in the trial have bloodpressuresabove160/100,soDrugBstillmanagedtoreducebloodpressure,andthiscannotbetheanswer.To treatdehydration, (B),wewouldwantadrug that increaseswater retention in thebody.DrugBresultedinhighurinevolume,soitwouldnotincreasewaterretention.

Inhyperkalemia, (D)plasmapotassiumlevelsareelevated.DrugBdidnotelevateurinepotassiumlevels,soitisunlikelythatitwouldbeeffectiveinriddingthebodyofexcesspotassium.

TakeawaysIfyou’renotfamiliarwithaconceptinananswerchoice,don’treflexivelyselectorrejectthatanswer.

KeyConceptsMechanismsofhormoneaction,Feedbackcontrolregulation

5.AresearcherbelievessheknowsthemechanismbywhichDrugDlowersbloodpressure.Toconfirmherhypothesis,sheshouldperformanexperiment,withappropriatecontrols,tomeasure:

A.angiotensinogenreleasefromtheliver.B.reninreleasefromthekidneys.C.arteriolarpressureincreasescausedbyangiotensinII.D.concentrationofACEintheplasma.

AssessthequestionThisisaSkill3question,askingwhattestwouldconfirmthemechanismofactionforDrugD.AllofthechoicesinvolvehormonesshowninFigure1.

Page 206: Edited By Deeangelee Pooran-Kublall, MD/MPH · 9.5 Preparing for the MCAT: Biochemistry in the Chemical and Physical Foundations of Biological Systems Section 9.6 Preparing for the

PlanyourattackToanswerthisquestion,weneedtoknowhowDrugDworks.Thatinformationisn’texplicitlygiventousinthepassage,sowe’llneedtousethedatainTable1tohelpusfigureouthowDrugDworks.Thenwe can use our knowledge of the hormones in the answer choices to determine the correctanswer.

ExecutetheplanInTable1,patientsonDrugDhadsignificantlyreducedbloodpressure,buttheyalsohadhighurinepotassium levels and normal urine volume. We’re therefore looking for an answer that doesn’tinvolveexcessurineexcretion.

Answerbymatching,eliminating,orguessingLooking at the answer choices, the most promising answer is choice (C), which doesn’t directlyinvolve the kidneys at all. Angiotensin II, according to paragraph 2, causes vasoconstriction;inhibitingangiotensinIIcoulddirectlyresultinvasodilation.

Changesinangiotensinogenrelease(A)andreninrelease(B)shouldbothalteraldosteronelevels,soitwouldnotbelikelythatthesewouldproduceadecreaseinbloodpressurewithoutincreasingurinevolume. Finally, while (D) sounds tempting, the actual concentration of ACE is irrelevant; whatmatters is itsactivity. It is possible that there are large amounts ofACE in the blood, but the vastmajorityofitcouldbeinhibited.

Page 207: Edited By Deeangelee Pooran-Kublall, MD/MPH · 9.5 Preparing for the MCAT: Biochemistry in the Chemical and Physical Foundations of Biological Systems Section 9.6 Preparing for the

10.7BiologyPracticeBIOLOGYPASSAGEIII(QUESTIONS1–6)Yeasts,whichareunicellularfungi,canreproducebybudding,orbyperformingmitosisandmeiosis.S. cerevisiae, for example, will undergo mitosis when resources are plentiful, but can undergomeiosis to form haploid spores when the primary fuel sources support fermentation rather thanaerobicrespiration.

Bothmitosisandmeiosisdependonformationofacomplex,Cdk1-Clb,betweentheproteinsCdk1andcyclinBtoactivatespindleformation.Inmitosis,Cdk1-Clbisactivatedwithinminutes,whileinmeiosis it typically takes at least threehours.Thehuman analogofCdk1,Cdc2, can substitute forCdk1inyeast.

TheentryofyeastintomeiosisIdependsontheexpressionofNdt80,atranscriptionfactorexpressedonly during meiosis, which triggers formation of the meiotic spindle and disassembly ofsynaptonemalcomplexes.Itisregulatedinpartbypositivefeedback:Ndt80actsasanactivatorforitsown gene. Another required protein is Ama1, which activates the anaphase-promoting complex(APC/C),whichisrequiredforthecelltoenteranaphaseI.DuringprophaseI,APC/Csuppressesthesignalsthatcauseyeastcellstoundergomitosis;italsohelpsinhibitentryintoSphase.

Inanexperiment,a researcherculturedS.cerevisiae cells inglucose-richmedium.Onepopulationwas mutated to lack the gene for Ndt80; the other lacked genes for both Ndt80 and Ama1. Twohundred cells from each sample were then transferred to media designed to induce meiosis. Theresearcherthencountedthenumberofcells ineachsamplethatexhibitedthepresenceofameioticspindle;theresultsareshowninFigure1.

Figure1.Developmentofthemeioticspindlein–Ndt80and–Ndt80–Ama1cells

P1.

P2.

P3.

Page 208: Edited By Deeangelee Pooran-Kublall, MD/MPH · 9.5 Preparing for the MCAT: Biochemistry in the Chemical and Physical Foundations of Biological Systems Section 9.6 Preparing for the

P4.

F1.

1.SupposeresearchersdiscoveracompoundthatpreventsbindingofCdk1tocyclinB.Whiletheyusethedrugintheirresearch,theydonotpursueusingitasatreatmentforfungalinfections.Whichofthefollowingwouldbethemostlikelyreasonforthatdecision?

A.Thedrugmightnotquantitativelykillfungalcells.B.Thedrugmightbetooexpensivetomassproduce.C.Thedrugmightposetoogreatarisktosubjects.D.Thedrugmightcausedangerousgeneticmutationsintheyeastcells.

2.Basedoninformationinthepassage,whichofthefollowingismostlikelytobetrueofmeioticcellslackingNdt80andAma1?

A.CellslackingNdt80onlywillarrestinprophaseI;cellslackingbothproteinswillarrestinanaphaseI.B.CellslackingNdt80onlywillarrestinanaphaseI;cellslackingbothproteinswillarrestinmetaphaseI.C.CellslackingNdt80onlywillarrestinprophaseI;cellslackingbothproteinswillarrestinmetaphaseI.D.CellslackingNdt80onlywillarrestinmetaphaseI;cellslackingbothproteinswillarrestinanaphaseI.

3.Whichofthefollowing,iftrue,wouldbethebestexplanationforhowAma1isregulatedpriortoentryintoanaphase?

A.Thecellcandetectchromosomesnotattachedtospindlefibers,andproducesasignalthatinhibitsAma1whenitdoes.B.Thecellcandetectchromosomesnotattachedtospindlefibers,andproducesasignalthatactivatesAma1whenitdoes.C.Thecellcandetecttheattachmentofchromosomestospindlefibers,andproducesasignalthatactivatesAma1whenitdoes.D.Thecellcandetecttheattachmentofchromosomestospindlefibers,andproducesasignalthatinhibitsAma1whenitdoes.

4.AsampleofS.cerevisiaegrowninamediumcontainingonlyacetateandnoglucosewouldmostlikelyexhibit:

A.formationofCdk1-Clbinlessthananhour.B.activationofAPC/CinprophaseI.C.exclusivelymitosis.D.decreasedexpressionofNdt80.

5.AcertainyeastcellisunabletoproceedtometaphaseIofmeiosisandunabletoproceedto

Page 209: Edited By Deeangelee Pooran-Kublall, MD/MPH · 9.5 Preparing for the MCAT: Biochemistry in the Chemical and Physical Foundations of Biological Systems Section 9.6 Preparing for the

metaphaseIofmitosis.Itmostlikelycontainsamutationinthegenefor:A.Ama1.B.Ndt80.C.tubulin.D.Cdc2.

6.Basedoninformationinthepassage,isitreasonabletoconcludethataftertelophaseI:I.APC/CremainsactiveintomeiosisII.II.Ndt80remainsactiveintomeiosisII.III.AdditionalAma1mustbesynthesized.A.IIonlyB.IandIIonlyC.IandIIIonlyD.I,II,andIIIonly

BiologyPracticePassageExplanationsP1.Yeastdobothmitosisandmeiosis(nutrientshortage)

P2.TimingofCdk1-Clbcomplexcritical

P3.Proteinsthatinducemeiosis

P4.Ndt80/Ama1knockoutexperiment

Fig1.Resultsofexperiment

MCATExpertiseTheAAMChassaidthatSkill3questionscantestethicalissuesinresearch.Don’tbesurprisedifyouseequestionssuchasthisoneonTestDay!

1.(C)

Oneofthekeystoantimicrobialtherapy,ingeneral,isthatitmustberelativelysafeforthehost,andattackamechanismthattheinfectedorganism’scellsdonotuse.Forexample,thebacterialribosomeis not identical to the human ribosome, so drugs that target the bacterial ribosomegenerally poselittlethreattohumans.Here,however,paragraph2tellsusthatthehumananalogofCdk1,Cdc2,canwork likeCdk1 in yeast cells. That implies there is a high degree of homology between the yeastpathwayand themitoticpathway inhumansandothermammals.Asa result, suchadrugwouldbelikely to affect both the yeast cells and human cells; it could cause damage to organs that requirefrequentcelldivisions(suchasthestomach).Choice(C)isthebestanswer.(A)Thismightbetrue,butevenifitcouldkilltheyeastcells(andperhapsespeciallyifitcouldkill

Page 210: Edited By Deeangelee Pooran-Kublall, MD/MPH · 9.5 Preparing for the MCAT: Biochemistry in the Chemical and Physical Foundations of Biological Systems Section 9.6 Preparing for the

theyeastcells!),itwouldstillposearisktothesubjects.(B)Evenifthisweretrue,itwouldnotbethebestreasontoavoidtestingthedrug.(D)FormationofthemitoticspindleisindependentofDNAsynthesis,andthereisnoinformationinthepassagethatsuggeststhedrugwouldbecapableofcausingmutationsintheyeastcells.

2.(C)

First,notethatalloftheanswersstatethatthecellswillarrestatsomestageofmeiosis.Toanswerthisquestion,then,weneedtoknowthekeyeventsofprophaseIandmetaphaseI.ToexitprophaseI,chromosomesmustcondense,thenuclearenvelopeneedstodisintegrate,andthespindlebodiesneedtoform.AccordingtoFigure1,incellslackingNdt80only,thespindleneverforms;incellslackingbothNdt80andAma1,though,itdoesform.Therefore,wecanconcludethatNdt80cellsshouldarrestin prophase I. According to paragraph 3, Ama1 activates APC/C, which is needed for entry intoanaphase I.Therefore, it ismost reasonable toconclude that in theabsenceofAma1, thecellswillarrestinmetaphaseI.Thismatcheschoice(C).(A)Ndt80cellswillarrestinprophaseI,butthelackofAma1preventsactivationofAPC/C,whichisneededforentryintoanaphaseI.(B)WithoutNdt80,thespindleneverforms,socellscannotleaveprophaseI.(D)Without Ndt80, the spindle never forms, so cells cannot leave prophase I; moreover, withoutAma1,cellscannotenteranaphaseI.

3.(A)

During anaphase, the cell needs to avoidnondisjunction, the incorrect separationof chromosomesduringcelldivision.Todothis,thecellneedstomakesurethateverychromosomeisattachedtoaspindle fiber. Since the number of chromosomes in cells varieswidely, the easiestway to do thiswouldbetodependonasignalthatdisappearswhenallchromosomeshavebeenattached,ratherthanone that appearswhenchromosomesareattached; as a result,wecaneliminate (C) and (D). Sinceparagraph3tellsusthatAma1activatesAPC/C,whichinitiatesanaphase,thenwewouldexpectthatthesignalshouldbeaninhibitoryone;onceallthechromosomeshaveattached,theinhibitorysignaldisappears,andAma1willbecomeactive.Thismatcheschoice(A).

4.(B)

Paragraph 1 states thatwhen fermentable fuel sources predominate, yeast cells aremore likely toundergosporulation.Acetatewouldbeanexampleofsuchafuelsource,especiallyintheabsenceofglucose. Thus, we would expect the yeast to undergo meiosis, and the only answer choice thatcorrespondstoinductionofmeiosisischoice(B).(A)Accordingtoparagraph2,Cdk1-Clbformswithinminutesinmitosis,notmeiosis.(C)Wewouldexpecttoseeexclusivelymeiosis,notmitosis.(D)Ndt80isrequiredformeiosis,sothiswouldbeasignalformitosis.

5.(C)

AccordingtotheinformationinFigure1,neitherAma1norNdt80isnecessaryforformationofthemeioticspindle.Thatrulesoutboth(A)and(B).Aclosereadingofparagraph2showsthatCdc2isagene found in humans, not in yeast, which eliminates (D). That leaves choice (C), tubulin, as the

Page 211: Edited By Deeangelee Pooran-Kublall, MD/MPH · 9.5 Preparing for the MCAT: Biochemistry in the Chemical and Physical Foundations of Biological Systems Section 9.6 Preparing for the

correctanswer;tubulinistheproteinrequiredforactualformationofthespindlefibers.

6.(B)

ThisquestiontestsyourunderstandingofmeiosisII.Inparticular,toanswerthiscorrectly,youneedtoknowthatthereisnoSphaseprecedingmeiosisII,asthereisprecedingmeiosisI.Therefore,wewouldexpectthatAPC/C(itemI)wouldremainactivetopreventthecellfromundergoingSphase.Thateliminates(A).WewouldalsoexpectthatNdt80(itemII)wouldbeactive,sinceasecondmeioticspindle isneededformeiosisII,sowecaneliminate(C).On theotherhand,wehavenoreason toconcludethatadditionalAma1 isneeded toactivate theAPC/C(itemIII); it ispossible thatAma1’sactivationofAPC/Cduringprophase I lasts throughoutmeiosis Iandmeiosis II.Choice(B) is thebestanswer.

ThingstoWatchOutForItemIIIisincorrectherebecausethequestionasksifitisreasonabletoconcludethatitistrue.Ifthequestionhadaskedwhichofthefollowingcouldbetrue,thenitwouldbecorrect,sinceit’satleastpossible.

**ThistopiccanalsobetestedontheChemical&PhysicalFoundationsofBiologicalSystemssection.**ThistopiccanalsobetestedontheChemical&PhysicalFoundationsofBiologicalSystemssection.

Page 212: Edited By Deeangelee Pooran-Kublall, MD/MPH · 9.5 Preparing for the MCAT: Biochemistry in the Chemical and Physical Foundations of Biological Systems Section 9.6 Preparing for the

CHAPTERELEVEN

GeneralChemistryOneofthemaincharacteristicsoftheMCATisthatallofthequestions,eveninthephysicalsciences,will be related to living systems. Thus, passages presenting an inorganic chemical reaction withstoichiometryquestionswillbefar less likely.Anyquestionsrelated togeneralchemistrywillalsoinvolvebiochemistryorbiology.Inaddition,generalchemistrywillappearinboththeChemicalandPhysical Foundations of Biological Systems and the Biological and Biochemical Foundations ofLivingSystemssections.Approximately5percentofthequestionsintheBiologicalandBiochemicalFoundations of Living Systems sectionwill be strictly general chemistry, while approximately 30percentofthequestionsintheChemicalandPhysicalFoundationsofBiologicalSystemssectionwillbe general chemistry. However, it is important to remember that knowledge in general chemistryconcepts isessential tosuccess inorganicchemistryandbiochemistry.Assuch, fewerquestions ingeneralchemistrydoesnotnecessarilymeanthatlessknowledgeisrequired;itsimplymeansthatthetestmakerislikelytobemorecreativeintestinggeneralchemistryconceptsinthecontextoforganicchemistryorbiochemistry.

11.1ReadingthePassageOneofthemostdifficultaspectsoftheMCATisreadingthepassages.Test-takerstendtowastealotoftimereadingpassagesthesamewayasonewouldreadatextbookornewspaper;byattemptingtocompletelyunderstandeveryargumentmadeandeverydetailgiven.However,thegoaloftheMCATisNOTreadingthepassages:itisansweringquestionsbyusingthepassageasapieceofreferencematerial. MCAT science passages are full of data that will always be there if required to answerquestions.Whenyoureadapassage,takeabroadlookatwhatishappeningandthinkaboutthebasicscienceconceptsthatarebeingappliedtosituationspresentedwithinthepassage.Nomorethantwominutesshouldbespentonasinglepassage;infact,mostefficienttest-takersreadMCATpassagesin90secondsorless.GeneralchemistrypassagesontheMCATwillrequiretheabilitytoreadquickly,focusingonthegoalofthepassage,andthenansweringthequestionswithakeeneyetowardthetaskinthequestion.

PASSAGETYPESAsintheothersciences,therearetwotypesofgeneralchemistrypassagesontheMCAT:informationand experiment.However, the content of the passages has changed drastically on themost currentrevisionoftheMCATin2015.Previously,passagesingeneralchemistrywouldfocusoninorganicchemistryalone;formationofcompounds,balancingequations,andthebasicgeneralchemistryonemight see in an introductory level course. However, the new MCAT will be presenting generalchemistryinthecontextoflivingsystems.Thismeansthateachpassagewillrequireknowledgeandthe ability to apply the fundamental concepts of biology and biochemistry within the context ofgeneral chemistry. Since only 5 percent of the questions in the Biological and BiochemicalFoundations of Living Systems sectionwill be related to general chemistry, it is unlikely that anysingle passage in that sectionwill be strictly general chemistry. That being said, passages that arestrictlygeneralchemistryaremorelikelytobefoundinthePhysicalandChemicalFoundationsof

Page 213: Edited By Deeangelee Pooran-Kublall, MD/MPH · 9.5 Preparing for the MCAT: Biochemistry in the Chemical and Physical Foundations of Biological Systems Section 9.6 Preparing for the

BiologicalSystemssection.

Informationpassages•Readlikeatextbookorjournalarticle.•Usuallyexplainthegeneralchemistrybehindanaturalphenomenon.•Maybeassociatedwithoneormorefigures,suchasareaction,equation,table,chart,graph,ordiagram.Thetextofthepassageusuallydescribesthesefigures.

Experiment•Consistsofbriefbackgroundinformation,mostlikelylinkingthechemicalprocesspresentedintheexperimentwithabiologicalprocess.•Theprocedurewillbedescribed,andresultswillbepresented.•Thehypothesisoftheexperimentisusuallystatedfairlyearlyinthepassage.•Numericalresultsmaybepresentedinatableoragraph.

GeneralChemistryintheBiologyandBiochemicalFoundationsofLivingSystemssectionandtheChemicalandPhysicalFoundationsofBiologicalSystemssection

BiologyandBiochemicalFoundationsofLivingSystems

ChemicalandPhysicalFoundationsofBiologicalSystems

Percentofquestionsingeneralchemistry

5%

33%

Passages

Possiblyone,butunlikelytoseeapassagethatisstrictlygeneralchemistry

Morethanone,generalchemistrytopicsarealsolikelytobemixedwithbiochemistryandphysics

Questions

Veryfewquestionsthatwillrequireonlygeneralchemistryknowledge.Manymorequestionsmayrequireyoutointegrategeneralchemistryconceptswithothersciences.

Somequestionswillonlyrequiregeneralchemistryknowledge,butallwilldiscusslivingsystems.Expecttointegrategeneralchemistrywithbiochemistryandorganicchemistry.

OUTLININGTHEPASSAGEAlmostallgeneralchemistrypassages,regardlessofthesection,willinvolveoneormoregraphicalrepresentations,especiallygraphsandtables.Thus,youroutlinemustreflecttheseimagesinorderto

Page 214: Edited By Deeangelee Pooran-Kublall, MD/MPH · 9.5 Preparing for the MCAT: Biochemistry in the Chemical and Physical Foundations of Biological Systems Section 9.6 Preparing for the

accuratelyrepresentthepassage.However,thisonlyrequiresveryslightchangestotheKaplanwayofreadingandoutlininggeneralchemistrypassages.

ScanforStructure•Scanthepassageforstructure,notingthepresenceofcharts,graphs,tables,diagrams,andtext.•Determinethedegreeofdifficultyandtheleveloftimecommitmentrequired.•Decidewhethertodothispassagenoworlater.

ReadStrategically• Read each paragraph as a critic, asking, “What is this paragraph doing here?What is themessagethatisbeingconveyed?”• Identify each chart, graph, table, or diagramanddeterminewhat is represented in eachone.Thereisnoneedtointerprettheimage—simplyidentifytherelationshipbetweentheimageandthetextwithinthepassage.•Ifaparagraphexplainsanimageorprovidesdefinitionsforalistofvariables,thisisacuethatyoumayskimthisinformationbecauseitappearsintwoplaceswithinthepassage.•Oftenthepurposeofachartorgraphisspecificallystatedinthetitleofthefigure.

LabelEachComponent•Outlineeachparagraph,statingthepurposeofeachparagraph.•Writedownthepurposeofeachimagepresentedwiththepassage.•Identifyanyrelationshipsbetweenthetextandtheimage.Forexample,anentireparagraphmaybedevoted toexplainingvariablesor resultsofanexperiment.Thesesame ideasmayalsoberepresentedvisually,meaningthatthereisadirectrelationship.

ReflectonYourOutline•Determinethegoalofthepassage.•Writeitdown,andmoveontothequestions.

Page 215: Edited By Deeangelee Pooran-Kublall, MD/MPH · 9.5 Preparing for the MCAT: Biochemistry in the Chemical and Physical Foundations of Biological Systems Section 9.6 Preparing for the

11.2AnsweringtheQuestionsSimilartotheothersciences,generalchemistryquestionswillfallintooneoffourcategories.

Discretequestions•Questionsnotassociatedwithadescriptivepassage.•Will be preceded by awarning such as, “Questions 12–15 areNOT based on a descriptivepassage.”•Likelytoaskforyoutorecallaspecificpieceof informationorapplyyourknowledgetoanewsituation.•NOTlikelytorequireanalysisofanexperiment.

Questionsthatstandalonefromthepassage•Questionsassociatedwithapassage,butthepassageisnotrequiredtoanswerthequestion.•Oftenthematicallyrelatedtothepassage,butoftendesignedtotestanadditionalaspectofthetopicthatisnotmentionedinthepassage.

Questionsthatrequiredatafromthepassage•Oftenwillrequiredataanalysisorconceptualunderstandingofideaspresentedinthepassage.•Youwillberequiredtoapplyyourknowledgetoinformationwithinthepassage.•Doesnotrequirethegoalofthepassage.

Questionsthatrequirethegoalofthepassage•Willrequireadeeperunderstandingofthepassageasawhole,especiallytheoverallgoalofthepassage.•Usuallythemosttime-consumingofthequestiontypesingeneralchemistry.

DETERMININGTHEPURPOSEOFTHEQUESTIONEachquestionisdesignedto testaspecificskillandinformationset.TheMCATisdesignedto testyourabilitytoapplyyourknowledgeinalogicalmannerinordertoanswerquestions.TheKaplanwayprovidesyouwitha logicalmethodforapplyingyourknowledgetomaximizeyourpointsonTestDay.

1.AssesstheQuestion•Identifythetopicanddegreeofdifficultyofthequestion.•Decidewhetherthisquestionisonetodonoworlater.

2.PlanYourAttack•Determinewhatinformationisrequiredtoanswerthequestion.• If a specific image is required to answer the question, identifywhat kind of information issummarizedwithintheimage.Then,identifythetypeofanalysisrequired.Istheanalysislargebasedontheresultsofanexperiment?Isacalculationrequired?Areyoubeingaskedtoidentifyatrend?•Locatetheinformationrequiredtoanswerthequestion.•Identifythetaskofthequestion.Whatarebeingaskedtodo?Themorespecificyouarewith

Page 216: Edited By Deeangelee Pooran-Kublall, MD/MPH · 9.5 Preparing for the MCAT: Biochemistry in the Chemical and Physical Foundations of Biological Systems Section 9.6 Preparing for the

identifyingthetask,theeasieritwillbetocarryoutyourplan.

3.ExecuteYourPlan•Gobacktothepassage,figure,orquestionstemtofindrequiredinformation.•Analyze,calculate,recalltheinformation—carryoutyourplan.

4.AnswertheQuestionbyMatching,Eliminating,orGuessing•Matchyouranswerwiththeanswerchoices.•Ifthereisnoclearanswer,startbyeliminatinganswerchoicesthatareobviouslyincorrect.• After obviously incorrect answer choices are eliminated, identify any answer choices thatsimplydonotmakesense.• If you cannot get down to a single answer choice, choose between two probable answerchoices.

Page 217: Edited By Deeangelee Pooran-Kublall, MD/MPH · 9.5 Preparing for the MCAT: Biochemistry in the Chemical and Physical Foundations of Biological Systems Section 9.6 Preparing for the

11.3GettingtheEdgeinGeneralChemistryOver the years, the amount of general chemistry on theMCAT has declined and shifted in scope.Currently,theonlygeneralchemistrytestedontheMCATisthatwhichhasadirectapplicationtothelife sciences. Thus, performing calculations using the Henderson–Hasselbalch equation is stillimportant,butonlyas it isapplied todetermininghowchanges in theconcentrationofbicarbonateandcarbonicacidaffectthepHofblood.OnTestDay,youcanexpectthatgeneralchemistryconceptswillbetestedinthecontextofbiologicalsystems.Thisisoftenvastlydifferentfromwhatisseeninundergraduategeneralchemistrycourses,asthepurposeofthesecoursesistoprovideafoundationingeneralchemistrythatcanbeappliedtomorespecifictypesofchemistry.Therefore, thetypeoflearningrequiredtosucceedongeneralchemistrypassagesisdifferent.

Generalchemistrypassageswilloftenfeaturemultipleimages,andyouwillberequiredtointerprettheseimages.However,interpretationisonlynecessaryifaquestionrequiresit.Notetheimage,whatit summarizes, andmove on.Noneed to interpret itwhen reading the passage.The questionswilllikely ask you to perform a calculation or analysis in the context of biology. For example, LeChâtelier ’sprinciple is likely tobe tested alongwithknowledgeof the carbonic acidbloodbuffersystem,ratherthanasatopicinwhichsubstancesareaddedtoabeaker.

Mastering general chemistry on the MCAT means being able to apply the principles of generalchemistrytolivingsystems.Thisrequirespractice,andtopscoringtest-takersoftenrefinetheirskillsbyreadingpassagesandansweringquestionsaboutthetopicathand.Readingatextbookorreviewbookishelpful,buttheMCATrequiresyoutoapplyknowledge,nottomemorizeit.

Page 218: Edited By Deeangelee Pooran-Kublall, MD/MPH · 9.5 Preparing for the MCAT: Biochemistry in the Chemical and Physical Foundations of Biological Systems Section 9.6 Preparing for the

11.4Step-By-StepGuidetoGeneralChemistryPassages

OUTLININGTHEPASSAGE

Scanforstructure•Determinewhethertodothispassagenoworlater.• Identify the structure of the passage, including charts, graphs, chemical equations, synthesispathways,metabolicpathways,andimages.

Readstrategically•Identifythetypeofpassage.•Payspecialattentiontotherelationshipsbetweenconcepts.•Inanexperimentpassage,determinethehypothesis,procedure,andoutcome.•Identifyrelationshipsbetweenthetextandthefigurespresentedwiththepassage.

Labeleachcomponent•Writedownthepurposeofeachparagraphandfigure•Determinethetypeofinformationpresentedinallofthefiguresandtheconnectionswiththeinformationwithinthetextofthepassage.

Reflectonyouroutline•Determinethegoalofthepassageandwriteitdown.•Identifytheconceptswithinthepassageinanefforttoanticipatequestions.

ANSWERINGTHEQUESTIONS

1.Assessthequestion•Determinewhetherthisquestionshouldbedonenoworlater.•Identifythetopicandthedegreeofdifficulty.•Goodquestions todonowingeneralchemistryareones thatdonot requirea lotofdataorexperimentalanalysisfromthepassage.

2.Planyourattack•Determinewhatyoualreadyknow,andwhatinformationyouneed.•Identifywheretofindtherequiredinformation:thepassage,thequestion,youroutline,oryourownknowledge• If youhave to goback to the passage, determinewhere to find the required informationbyusingyouroutline• If data analysis is required, identify the correct data set, as there may be multiple datarepresentations.•Determinethetaskofthequestion.Paraphrasethequestion,ifrequired,inordertoensurethatyourplanisdesignedtoactuallyanswerthequestion.

Page 219: Edited By Deeangelee Pooran-Kublall, MD/MPH · 9.5 Preparing for the MCAT: Biochemistry in the Chemical and Physical Foundations of Biological Systems Section 9.6 Preparing for the

3.Executeyourplan•Analyzethedata,gobacktothepassage,andcarryoutyourplan.•Ifyougetstuckanalyzingdata,rememberthatthetrendofthedataisoftenenoughtoyieldacorrectanswerchoice.

4.Answerthequestionbymatching,eliminating,orguessing•Matchyouranswertotheanswerchoices.•Ifthereisnotamatch,eliminateincorrectanswerchoices.Someoftheanswerchoicesmaynotmakesense;eliminatethosefirst.•Ifeliminationdoesnotprovideaclearanswer,guessbetweentwoprobableanswers.

Page 220: Edited By Deeangelee Pooran-Kublall, MD/MPH · 9.5 Preparing for the MCAT: Biochemistry in the Chemical and Physical Foundations of Biological Systems Section 9.6 Preparing for the

11.5PreparingfortheMCAT:GeneralChemistryTheseare thegeneralchemistry topics thatyouare likely to seeonTestDay.Allof the followingtopicsappear in theChemicalandPhysicalFoundationsofBiologicalSystemssectionexcept thoselabeledotherwise.

PRINCIPLESOFBIOENERGETICS•TransfersofphosphorylgroupsandATP,includingtheconceptthatΔGofATPhydrolysisismuchlessthan0

GASPHASE•DefinitionofabsolutetemperatureandtheapplicationoftheKelvin(K)scale•Definitionofpressureandthefunctionofasimplemercurybarometer•Identificationofthatthemolarvolumeofanidealgasat0°Cand1atmis22.4L/mol•Definitionofanidealgasandtheapplicationoftheidealgaslaw(PV=nRT),Boyle’slaw(PV=constant),Charles’sLaw(V/T=constant),andAvogadro’slaw(V/n=constant)•Fundamentalconceptsofthekineticmoleculartheoryofgases•Concept thatbehaviorofarealgasmaydeviatefromthe idealgas law, includingqualitativeandquantitative(VanderWaalsequation)analysis•Identificationandcalculationofpartialpressureandmolefraction•ApplicationofDalton’slawtocalculatecompositionfrompartialpressures

ELECTROCHEMISTRY•Definitionandcharacteristicsofanelectrolyticcell,includingelectrolysis,anode,cathode,andelectrolyte•ApplicationofFaraday’slawrelatedtotheamountofelementsdepositedorgasliberatedatanelectrodetocurrent•Identificationofelectronflow,oxidation,andreductionattheelectrodesofanelectrolyticcell•Definitionandcharacteristicsofgalvanicorvoltaiccells, includinghalf-reactions, reductionpotentials,cellpotential,anddirectionofelectronflow•Characteristicsofaconcentrationcell**•ThedirectionofelectronflowandapplicationoftheNernstequationinconcentrationcells*•Fundamentalcharacteristicsofbatteries, includingelectromotive force,voltage, lead-storagebatteries,andnickel-cadmiumbatteries

MOLECULARSTRUCTUREANDABSORPTIONSPECTRA•Absorptionofvisiblelightbycompounds,includingtheconceptofthecolorreflectedandtheeffectsofstructuralchangesonabsorption(asinindicators)

ATOMICNUCLEUS

Page 221: Edited By Deeangelee Pooran-Kublall, MD/MPH · 9.5 Preparing for the MCAT: Biochemistry in the Chemical and Physical Foundations of Biological Systems Section 9.6 Preparing for the

•Definitionsandapplicationsofatomicnumberandatomicweight•Characteristicsofneutrons,protons,andisotopes•Identificationofnuclearforcesandbindingenergy• Concepts of radioactive decay, including α, β, and γ decay as well as half-life, exponentialdecay,andsemi-logplots•Functionofamassspectrometer

ELECTRONICSTRUCTURE•Generalorbital structureof thehydrogenatom,definitionof theprincipalquantumnumber,andthenumberofelectronsperorbital•Definitionsandcharacteristicsofgroundstateandexcitedstates•Definitionandapplicationofabsorptionandemissionlinespectra•ApplicationofthePauliexclusionprinciple•Conceptsofparamagnetismanddiamagnetism•CharacteristicsoftheBohratom•DefinitionandconceptsrelatedtotheHeisenberguncertaintyprinciple•Calculationofeffectivenuclearcharge•Identificationofthephotoelectriceffect

THEPERIODICTABLE–CLASSIFICATIONOFELEMENTSINTOGROUPSBYELECTRONICSTRUCTURE

• Identification and characteristics of specific groups on the periodic table, including alkalimetals, alkaline earth metals, halogens. Identification and characteristics of representativeelements,transitionmetals,metals,andnon-metals

THEPERIODICTABLE–VARIATIONSOFCHEMICALPROPERTIESWITHGROUPANDROW

•Fundamentalconceptsofvalenceelectrons• Definition of the first and second ionization energy, as well as predicting first and secondionizationenergyfromelectronicstructureforelementsindifferentgroupsorrows•Definitionofelectronaffinityandvariationwithgroupandrow•Definitionof electronegativityandcomparativevalues for some representativeelementsandimportantgroups•Relationshipbetweenelectronshellsandsizesofatomsandions

STOICHIOMETRY•Definitionandcalculationofmolecularweight•Thedifferencebetweentheempiricalandmolecularformula•Identificationofmetricunitscommonlyusedinchemistry•Usingpercentmasstodeterminecompositionofacompound•ApplicationofthemoleconceptandAvogadro’snumberNA

Page 222: Edited By Deeangelee Pooran-Kublall, MD/MPH · 9.5 Preparing for the MCAT: Biochemistry in the Chemical and Physical Foundations of Biological Systems Section 9.6 Preparing for the

•Definitionandapplicationofdensity• Identification and application of the oxidation number, including common oxidizing andreducingagents;disproportionationreactions•Useofchemicalequations todescribe reactions, includingconventions forwritingchemicalequations;balancingequations, including redoxequations;determining limiting reactants; andcalculatingtheoreticalyields

ACID/BASEEQUILIBRIA•DefinitionofanacidandabaseaccordingtoBrønsted–Lowry•Characteristicsoftheionizationofwater,includingtheapplicationandvalueofKw(Kw=[H+][OH−]=10−14at25°Cand1atm)•DefinitionandapplicationofpH,includingthepHofpurewater•Identificationofconjugateacidsandbases•Identificationofstrongacidsandbases•Identificationandcharacteristicsofweakacidsandbases,includingdissociationofweakacidsandbaseswithorwithoutaddedsalt;hydrolysisofsaltsofweakacidsorbases;calculationofpHofsolutionsofsaltsofweakacidsorbases(Henderson–Hasselbalchequation)•ApplicationoftheequilibriumconstantsKaandKb;applicationofpKa,pKb•Definition and fundamental concepts of a buffer, including common buffer systems and theinfluenceofabuffersystemonatitrationcurve

IONSINSOLUTION•Definitionsofanionandcation•Identificationofcommonnames,formulas,andchargesforfamiliarionssuchasammonium,phosphate,andsulfate•Theconceptofhydrationandthehydroniumion

SOLUBILITY•Identificationoftheunitsofconcentrationsuchasmolarity•ApplicationofthesolubilityproductconstantandtheequilibriumexpressionKsp• Characteristics of the common ion effect and its use in laboratory separation; complex ionformation•Relationshipsbetweencomplexionsandsolubility;solubilityandpH•Theroleofcolligativepropertiesandosmoticpressureinthetransferofsubstancesacrosstheplasmamembrane*•ApplicationofHenry’slawofgassolubility,especiallyintermsofalveolargasexchange*

TITRATION•Useandidentificationofcommonindicators•Fundamentalconceptsofneutralization•Interpretationoftitrationcurves

Page 223: Edited By Deeangelee Pooran-Kublall, MD/MPH · 9.5 Preparing for the MCAT: Biochemistry in the Chemical and Physical Foundations of Biological Systems Section 9.6 Preparing for the

•Characteristicsofredoxtitrations

COVALENTBOND•FundamentalconceptsofLewiselectrondotformulas,includingresonancestructures,formalcharge,andLewisacidsandbases• Definition of partial ionic character, including the role of electronegativity in determiningchargedistributionanddipolemoment•Definitions and characteristics ofσ andπ bonds, including hybrid orbitals (sp3, sp2, sp) andrespective geometries; valence shell electron pair repulsion and prediction of shapes ofmoleculessuchasNH3,H2O,andCO2;structuralformulasformoleculesinvolvingH,C,N,O,F,S,P,Si,andCl;delocalizedelectronsandresonanceinionsandmolecules•Conceptsofmultiplebonding,includingeffectonbondlength,bondenergies,andrigidityofmolecularstructure

LIQUIDPHASE–INTERMOLECULARFORCES•Roleofhydrogenbonding,dipoleinteractions,andVanderWaalsforces(Londondispersionforces)inintermolecularinteractionsintheliquidphase

ENERGYCHANGESINCHEMICALREACTIONS–THERMOCHEMISTRY,THERMODYNAMICS

•Theconceptofathermodynamicsystemandstatefunction•Thezerothlawofthermodynamicsandtheconceptoftemperature•Firstlawofthermodynamics:ΔE=Q−W(conservationofenergy)• Second law of thermodynamics: concept of entropy as a measure of disorder, and theidentificationofrelativeentropyforgas,liquid,andcrystalstates•Measurementofheatchanges(calorimetry),heatcapacity,andspecificheat• Fundamental differences between endothermic and exothermic reactions, including enthalpy(H),standardheatsofreactionandformation,andHess’slawofheatsummation•Relationshipbetweenbonddissociationenergyandheatsofformation•Theconceptoffreeenergy:G**•RelationshipbetweenΔG°andspontaneityofreaction•Definitionsandapplicationoftheheatoffusionandheatofvaporization•Fundamentalconceptsofthephasediagram,includingpressureandtemperature

RATEPROCESSESINCHEMICALREACTIONS–KINETICSANDEQUILIBRIUM

• Concept of the reaction rate and its dependence upon reactant concentration, rate law, rateconstant,andreactionorder•Conceptanddeterminationoftherate-limitingstep• Impact of temperature on the reaction rate, including activation energy and use of theArrheniusequation

Page 224: Edited By Deeangelee Pooran-Kublall, MD/MPH · 9.5 Preparing for the MCAT: Biochemistry in the Chemical and Physical Foundations of Biological Systems Section 9.6 Preparing for the

•Identificationoftheactivatedcomplexortransitionstateandinterpretationofenergyprofilesshowingenergiesofreactants,products,activationenergy,andΔHforthereaction•Comparisonofkineticcontrolandthermodynamiccontrolofareaction•Useandpurposeofcatalysts•Reversiblechemical reactionsand the roleofequilibrium, including the lawofmassaction,equilibriumconstant,andapplicationofLeChâtelier ’sprinciple**•RelationshipbetweentheequilibriumconstantandΔG°**

Page 225: Edited By Deeangelee Pooran-Kublall, MD/MPH · 9.5 Preparing for the MCAT: Biochemistry in the Chemical and Physical Foundations of Biological Systems Section 9.6 Preparing for the

11.6GeneralChemistryWorkedExamplesPASSAGEI:ACIDSANDBASESCarbon dioxide gas is a by-product of metabolism in the human body. After diffusing out ofmetabolically active tissue, the majority of carbon dioxide will enter red blood cells, where it israpidlyhydratedbytheenzymecarbonicanhydrase,asshowninReaction1.

Reaction1The apparent equilibrium constant for the dissociation of carbonic acid is and theconcentrationofcarbonicacidisproportionaltotheamountofcarbondioxide,asshownbelow.

WherekHistheHenry’slawconstantforcarbondioxidesolubility,approximately ,andpCO2 is thepartialpressureofCO2 in theblood.The latter varieswith location in the circulatorysystem, but typically measures between 35–44 mm Hg for arterial blood and 39–52 mm Hg forvenous.

BicarbonateionsaretransportedoutofredbloodcellswheretheyhelpmaintainaconstantserumpH.During transport,bicarbonate ionsareexchangedforchloride ions.Equilibriumkeepsbicarbonateandchlorideionserumlevelsatapproximately25mMand100mM,respectively.Thisbuffersystem,alongwiththebody’scompensatorymechanisms,resultsinanormalarterialpHstabilizedat7.40.

Acidosis isacondition inwhich thebody’sarterialpHdropsbelow7.35.Respiratoryacidosis isaspecificcondition thatoccurswhenpCO2 is toogreat.Metabolic acidosis is a similar condition inwhichanacidisbeingeitheroverproducedbythebodyorinsufficientlyexcretedbyit.

Conversely,alkalosisisaconditioninwhichthebody’spHrisesabove7.45.Respiratoryalkalosisisa specific condition that can developwhen carbon dioxide is removed too quickly from the body,such as hyperventilation during periods of significant stress—which commonly exacerbates thiscondition.Metabolicalkalosisalsohasmanycausativeagents,includingbicarbonateionandchlorideionimbalances.Ingeneral,metabolic imbalancescanbecompensatorymechanismsforrespiratoryabnormalities.

P1.

R1.

P2.

E1.

P3.

Page 226: Edited By Deeangelee Pooran-Kublall, MD/MPH · 9.5 Preparing for the MCAT: Biochemistry in the Chemical and Physical Foundations of Biological Systems Section 9.6 Preparing for the

P4.

P5.

1.WhichofthefollowingprovidesthebestapproximationforthepHofvenousbloodinaphysiologicallynormalindividual?

A.

B.

C.

D.

2.WhyisthechlorideionrequiredbythecellforReaction1tocontinue?A.Chlorideionsactascatalystsforthesystem.B.Anionexchangepreventstheaccumulationofproducts.C.Chlorideionsrelievetheexcessivechargebuildup.D.Thechlorideionisabase,whichhelpsneutralizeacidicproducts.

3.Thearterialpartialpressureofcarbondioxideinbloodisapproximately:A.0.0052kPa.B.0.052kPa.C.5.3kPa.D.40kPa.

4.Overthelastcentury,carbondioxideemissionshavenoticeablyincreasedthecarbondioxideconcentrationintheatmosphere.Whateffecthasthishadontheoceans?

A.Marinelifedependenton haveproliferated.B.Theoceanshavebecomemorebasic.C.Therehasbeenanincreaseinthebicarbonatelevels.D.Therisein[CO2]atmhasbeenexacerbatedbyglobalwarming.

5.Givenatypicalplasmavolumeof5L,atypicaladulthasanamountofbicarbonatewithabufferingcapabilityequivalenttohowmany500mgcalciumcarbonateantacidtablets?

A.12.5B.25

Page 227: Edited By Deeangelee Pooran-Kublall, MD/MPH · 9.5 Preparing for the MCAT: Biochemistry in the Chemical and Physical Foundations of Biological Systems Section 9.6 Preparing for the

C.125D.6×103

6.Metabolicacidosisischaracterizedbya:A.highplasma ,alowplasma[H+],andincreasedbreathingrate.B.highplasma ,ahighplasma[H+],anddecreasedbreathingrate.C.lowplasma ,highplasma[H+],andincreasedbreathingrate.D.lowplasma ,highplasma[H+],anddecreasedbreathingrate.

Page 228: Edited By Deeangelee Pooran-Kublall, MD/MPH · 9.5 Preparing for the MCAT: Biochemistry in the Chemical and Physical Foundations of Biological Systems Section 9.6 Preparing for the

GeneralChemistryPassageIExplanation:USINGTHEKAPLANMETHODSP1.CO2metabolizedtobicarbonate

R1.Reversiblereaction,carbonicacidasintermediate

P2.pHvariesthroughoutcirculation

E1.Henry’slawrelationshipfor[H2CO3]

P3.Cl−exchangedtoallowR1tocontinue,normalvaluesofbuffersystem

P4.Acidosisdescribed

P5.Alkalosisdescribed

1.WhichofthefollowingprovidesthebestapproximationforthepHofvenousbloodinaphysiologicallynormalindividual?

A.

B.

C.

D.

AssessthequestionIt isnecessary tosetup the calculations forpH.While itmightbepossible tomake aquickguess(slightlylessthan7.4), that is insufficientbecauseour logarithmiccalculationswillnotbeaccurateenough to differentiate between the answers. Notice the similarities within the answers: they allcontain“−log(7.9×10−7)”—thisnumberisfromthepassageandcanbeusedtohelpplananattack.

Planyourattack

Page 229: Edited By Deeangelee Pooran-Kublall, MD/MPH · 9.5 Preparing for the MCAT: Biochemistry in the Chemical and Physical Foundations of Biological Systems Section 9.6 Preparing for the

ThequestionrequiresacalculationofpH.Theanswerchoicesareallsomewhatsimilar;it’samatterof the numerator and denominator of the second term. An attack should begin by identifying theequationbeingusedandthenusingthepassagetofillinthenecessaryterms.

ExecutetheplanAll of the answers include the same term; that term is the pK′a for carbonic acid. The equationincludespH,pKaforanacidandthelogofaratio.Thisinformationandthecontextofthepassage(buffering)leadsonetotheHenderson–Hasselbalchequation.Theequationbecomes:

.Thepassagegivesthebicarbonateconcentrationas25mM,butthecarbonicacid concentration is more elusive. In order to solve for it, it is necessary to combine

andthegivenvaluesforpCO2.

Answerbymatching,eliminating,orguessingExecutingtheplanquicklyleadstotheeliminationof(B)and(D)becausetheyhavetheconcentrationofbicarbonate in thedenominator.Thedifferencebetween (A)and (C) is thepCO2—which canbededucedfromthepassage;itiseither39mmHgor44mmHg.Usingthereferencevalues,35mmHgistoolowforvenousblood,sothecorrectanswerischoice(C).

2.WhyisthechlorideionrequiredbythecellforReaction1tocontinue?A.Chlorideionsactascatalystsforthesystem.B.Anionexchangepreventstheaccumulationofproducts.C.Chlorideionsrelievetheexcessivechargebuildup.D.Thechlorideionisabase,whichhelpsneutralizeacidicproducts.

AssessthequestionThequestionasksaboutreasoning.Thecorrectanswermustmatchtheroleofthechlorideionandmakesense,basedonafundamentalchemicalunderstanding.

Planyourattack

Page 230: Edited By Deeangelee Pooran-Kublall, MD/MPH · 9.5 Preparing for the MCAT: Biochemistry in the Chemical and Physical Foundations of Biological Systems Section 9.6 Preparing for the

First identify the function of the chloride ion. Paragraph 3 explicitly states that an exchange ofchlorideionsoccursduringtransportofthebicarbonateionoutofthecell.Thecorrectanswerwillreasonablyexplainwhytheremovalofbicarbonateand/ortheadditionofchlorideionsisnecessaryforthereactiontocontinue.Usewhatisknowntoattackeachanswerchoice.

ExecutetheplanStartingwithchoice(A),chlorideionasacatalyst isnot implausible,however, there isnotenoughinformation to confirm this.Becauseof that, and the statement that chloride is “exchanged,” this asuspectanswer.

Forchoice(B),sincebicarbonateisaproductofReaction1andthechlorideiondoeshelpremoveit,thisanswerisplausiblebasedonLeChâtelier ’sprinciple.

Choice(C)canbeeliminatedbecausethereisnodifferenceelectrically.

Finally,inchoice(D),thechlorideionistheconjugatebaseofastrongacid(HCl),whichmakesitaveryweakbase—tothepointthatitisinert,sothisisnotcorrect.

Answerbymatching,eliminating,orguessingExecutingtheplanledtotheeliminationof(A),(C),and(D),indicatingthat(B)iscorrect.Sincethereaction is reversible it isplausible that the lawofmassactionwould inhibit furtherproductionofbicarbonate—ifitremainsinthecell.Thisreasoningmatcheschoice(B).

3.Thearterialpartialpressureofcarbondioxideinbloodisapproximately:A.0.0052kPa.B.0.052kPa.C.5.3kPa.D.40kPa.

MCATExpertiseEvenrelativelystraightforwardcalculationscanvexmanyMCATtest-takers.TheabilitytomakequickworkofquestionslikethisonTestDaywillgiveyouanadvantageoveryourcompetition!

Page 231: Edited By Deeangelee Pooran-Kublall, MD/MPH · 9.5 Preparing for the MCAT: Biochemistry in the Chemical and Physical Foundations of Biological Systems Section 9.6 Preparing for the

AssessthequestionThisquestionasksforpCO2,thepartialpressureofCO2intheblood.Theanswersarenumbersthatdifferinorderofmagnitude,soweshouldpaymoreattentiontothatthantheexactvalue.Also,theanswersareinkilopascals,notmmHgasinthepassage.

PlanyourattackToanswerthisquestion,we’llneedtofindinformationonthepressure;thatinformationappearsattheendofparagraph2.We’llalsoneedtoconverttheanswertokilopascals.Todothat,weneedtoknowtheconversionfactorsforpressureunits.

ExecutetheplanParagraph 2 states that pCO2 is between 35 and 44 mm Hg. Given the spacing between answerchoices,wecanjustpickanumberthatiswithintherangeandeasytouse;40mmHgwilldo.

All that’s left is to convertmmHg tokilopascals; since1 atm=760mmHg=101.3kPa,weget

,whichgiveschoice(C).

Answerbymatching,eliminating,orguessingTheonlyanswercloseto5kPaischoice(C).Notethat(D)assumesthat1kPa=1mmHg,whichisincorrect.(A)and(B)arebothbasedonusingtheconversionfactor1atm=760mmHg.

4.Overthelastcentury,carbondioxideemissionshavenoticeablyincreasedthecarbondioxideconcentrationintheatmosphere.Whateffecthasthishadontheoceans?

A.Marinelifedependenton haveproliferated.B.Theoceanshavebecomemorebasic.C.Therehasbeenanincreaseinthebicarbonatelevels.D.Therisein[CO2]atmhasbeenexacerbatedbyglobalwarming.

Page 232: Edited By Deeangelee Pooran-Kublall, MD/MPH · 9.5 Preparing for the MCAT: Biochemistry in the Chemical and Physical Foundations of Biological Systems Section 9.6 Preparing for the

AssessthequestionThis question relies on your ability to relate the partial pressure of a gas above a fluid to the pHwithinthefluid.ThisistestingHenry’slawaswellastheconsequencesgiventhisparticularsystem.

PlanyourattackOneofthemostimportantconceptsofHenry’slawistheideathatamountofgasdissolvedinaliquidis proportional to the partial pressure of the gas above the liquid. Furthermore, gases becomeincreasingly soluble at lower temperatures. Using outside knowledgewill be sufficient tomake aprediction.

ExecutetheplanIf the atmospheric concentration of carbon dioxide increases, then the amount of carbon dioxidedissolvedintheoceanswouldalsoincrease.Iftheamountofdissolvedcarbondioxideincreases,then(accordingtoReaction1),sodoestheamountofcarbonicacidandultimatelybicarbonateionsandacidity.Iftheacidityoftheoceansincreases,thenthepHdecreases.

Answerbymatching,eliminating,orguessingExecutingtheplanledtotheeliminationof(B)andtheselectionof(C).(A)canbeeliminatedafterfurtherconsideration,becauseanincreaseintheacidityoftheoceanswouldpromotetheformationof from andH+,renderinglessof thecarbonate ionavailable formarine life. (D) relatessolubility to temperature, andwhile the two are related, if there were to be an increase in globaltemperatures, then that would reduce the solubility of carbon dioxide and raise atmosphericconcentrations; however this does not address the effect on the oceans, so it cannot be the correctanswer.

5.Givenatypicalplasmavolumeof5L,atypicaladulthasanamountofbicarbonatewithabufferingcapabilityequivalenttohowmany500mgcalciumcarbonateantacidtablets?

A.12.5B.25C.125D.6×103

Page 233: Edited By Deeangelee Pooran-Kublall, MD/MPH · 9.5 Preparing for the MCAT: Biochemistry in the Chemical and Physical Foundations of Biological Systems Section 9.6 Preparing for the

AssessthequestionThequestionisaskingaboutthelimitofabuffersystem.Theanswerchoicesandthecontextofthepassageindicateyou’llprobablyneedtodoastoichiometrycalculationtosolvethisproblem.

PlanyourattackFirst determine howmuch bicarbonate would be found in a typical adult. This will require someinformation from the passage aswell as a conversion from volume and concentration to amount.Thenusestoichiometrytodeterminetheequivalentamountofantacid.

ExecutetheplanParagraph 3 indicates that the typical level of bicarbonate is 25mM and the question provides anaveragevolumeof5L,sothereare12.5Lofbicarbonate.Sincebicarbonate’sformulais onemolecanneutralizeonemoleofacid,and1mmol=1mEq.Therefore,wehave125mEqofbase.

Tocalculatethenumberofcalciumcarbonatetabletsneeded,weneedtoknowthatcarbonateis ,so1mmol=2mEq.Nowwecandoastoichiometrycalculation:

Answerbymatching,eliminating,orguessingOurpredictionof12.5tabletsmatcheschoice(A).Noticecommonerrorsthatmayleadonetopickawrong answer: (B) from incorrectly assuming that onemole of calciumcarbonate neutralizes oneequivalent of acid. (C) is the number of milliequivalents of bicarbonate in blood, and (D) is themilligramsofcalciumcarbonaterequired,notthenumberof500-mgtablets.

6.Metabolicacidosisischaracterizedbya:A.highplasma ,alowplasma[H+],andincreasedbreathingrate.B.highplasma ,ahighplasma[H+],anddecreasedbreathingrate.C.lowplasma ,highplasma[H+],andincreasedbreathingrate.D.lowplasma ,highplasma[H+],anddecreasedbreathingrate.

Page 234: Edited By Deeangelee Pooran-Kublall, MD/MPH · 9.5 Preparing for the MCAT: Biochemistry in the Chemical and Physical Foundations of Biological Systems Section 9.6 Preparing for the

AssessthequestionFor this question it is necessary to reason through theprovided information andmakepredictionsabouteffectsonplasmabicarbonateconcentrations,pH,andbreathingrate.

PlanyourattackUsepassageinformationpertainingto“metabolicacidosis,”aswellastherelationshipsinReaction1,tomakeaprediction.Onceaconclusionisreached,looktoeliminateanswers—thisisastrategicmove when there is significant repetition in the answer choices. Continue until only one answerremains.

ExecutetheplanTheterm“acidosis”isenoughtopredictadecreaseinpHandanincreasein[H+]—eliminate(A).Thepassage indicates that “metabolic acidosis is a similar condition in which an acid is being eitheroverproduced by the body or insufficiently excreted by it.” Use the equilibrium in Reaction 1 todeterminetheeffectofincreasedacidproductiononbicarbonateconcentrations.Anincreasein[H+]would shift the equilibriumback towardswater and carbon dioxide, so bicarbonate concentrationswoulddecrease—eliminate(B).

Answerbymatching,eliminating,orguessingWith two answer choices left, it’s necessary to discern the differences between them and make aprediction. The difference is in the breathing rate. If the body experiences an increase in carbondioxide levels, it would be expected that the ventilation rate would increase to compensate.Furthermore,thepassageindicatesthathyperventilationexacerbatesalkalosis(presumablyloweringpH,whichisdesired)—withapredictioninhand,lookforamatch,(C).

GENERALCHEMISTRYPASSAGEII:SOLUBILITYAdequatesolubilityofionsandmacromolecules,alongwiththeirpermeabilityinvarioussectionsofthedigestiveandexcretorysystems,helpspreventmalnutritionandotherdangerousconditions.

The human body needs to absorb regular doses of certain vitamins and minerals. Vitamins areorganicmoleculesthatanorganismcannotsufficientlysynthesize,andthatarenotbrokendownforenergy,butareneededfornormalcellularoperation.Somevitaminshavehormone-likefunctionality(vitaminD for example),while others act as antioxidants (vitaminC) or as precursors to enzymecofactors(vitaminB12).

Dietaryminerals are elements (other thanC,H,N, andO) required for living; traceminerals are

Page 235: Edited By Deeangelee Pooran-Kublall, MD/MPH · 9.5 Preparing for the MCAT: Biochemistry in the Chemical and Physical Foundations of Biological Systems Section 9.6 Preparing for the

dietary minerals essential only in minute quantities. Most of them must be taken as parts ofcompounds,becausemanyfreeelementscanreactwithsubstancesinthedigestivetracttoformby-products, some of which are toxic. Approximately 30 elements are known, or suspected to be,essential tonormalbiological function.Mostmineralsarenaturallyoccurring in thediet,althoughfoodsmaybefortified.

Thechemicalandphysicalnatureofrequirednutrientsaffectstheirsolubilityandpermeability.Itcanalso alter their bioavailability, the fraction of the ingested dose that reaches systemic circulationunaltered.Anyprocessthataltersthecompoundorhindersitsabsorptionwillreducebioavailability.Forexample,certain foodadditivescanalterbioavailability; forexample, the fat substituteOlestra(molecularformulaC13H14O11)reducesvitaminKabsorption.EachgramofOlestrainfoodcanreduceabsorption of vitaminK by 8micrograms. Table 1 displays some of the nutritional and chemicalcharacteristicsof importantvitaminsandminerals. (Note: theaverageadulthasabloodvolumeof∼5liters.)

NutrientSource

OralBioavailability

RecommendedDailyIntake(RDI)

Molecularmass(g/mol)

VitaminAAcetate 0.99 900μg 328.49

VitaminB6 0.85 1.7mg 123.11VitaminC 0.99 90.0mg 176.12VitaminsD2andD3

0.99 10.0μg 396.65

VitaminEAcetate 0.67 15.0mg 472.74

VitaminK 0.90 120μg 444.65PotassiumChloride 0.90 4700mg 74.55

SodiumChloride 0.99 1500mg 58.44

CalciumCarbonate 0.35 1300mg 100.09

ZincSulfate 0.37 5.0mg 161.47

Table1.Nutritionalandchemicalcharacteristics

P1.

P2.

Page 236: Edited By Deeangelee Pooran-Kublall, MD/MPH · 9.5 Preparing for the MCAT: Biochemistry in the Chemical and Physical Foundations of Biological Systems Section 9.6 Preparing for the

P3.

P4.

T1.

1.VitaminCformulationsareadministeredinavarietyofways,includingpureascorbicacidadministeredorallyandascorbylpalmitate(vitaminCester)administeredtopically.HowdoestheadditionofthepalmitategroupaffectoralandtopicalbioavailabilityofvitaminC?

A.Thepalmitategroupdecreasesoralbioavailabilityviareducedsolubilityinthedigestivetract,butincreasestopicalbioavailabilitybyminimizingfreeradicalsintheepidermis.B.Thepalmitategroupincreasesoralbioavailabilityviaincreasedpermeabilityinthedigestivetract,andincreasestopicalbioavailabilitybyincreasingvitaminCstabilityintopicalsolutions.C.Thepalmitategroupdecreasesoralbioavailabilityviadecreasedpermeabilityinthedigestivetract,andhaslittleeffectontopicalbioavailabilitybecauseascorbylpalmitatepenetratestheskinaseffectivelyaspurevitaminC.D.Thepalmitategrouphaslittleeffectonoralbioavailabilitybecausethepalmitategroupisremovedbyhydrolysisinthedigestivetract,andincreasestopicalbioavailabilitybyincreasingthestabilityofvitaminCintopicalsolutions.

2.AcompanywouldliketodevelopanoralvitaminEsupplementfordiet-restrictedpatients.WhatistheminimumamountofvitaminEacetate(Ksp=1.6×10−7)neededinthesupplementinordertoreachserumsaturationlevelsforvitaminE?(Ignoreioncontributionsfromothercompounds.)

A.0.006gB.0.015gC.0.3gD.1.5g

3.WhyistheobservedbioavailabilityofmineralsgenerallyloweventhoughtheyhaverelativelyhighKspvalues?

A.Somemineralionsareinsolubleinaqueoussolutionandthustheionscannotbedigestedandabsorbed.B.Othermoleculesareinteractingwiththeionsintheoralroute,inhibitingabsorption.C.Mineralionsdonothaveenoughvalenceelectronstosufficientlyinteractwithintestinalionchannels.D.Ionsareimmediatelydepositedintheboneandothertissue,thusmeasuredbioavailabilityislow.

Page 237: Edited By Deeangelee Pooran-Kublall, MD/MPH · 9.5 Preparing for the MCAT: Biochemistry in the Chemical and Physical Foundations of Biological Systems Section 9.6 Preparing for the

4.HyperoxaluriacausesCaC2O4todepositinurine.A0.75literurinesamplefreeofoxalatewascollected.Thecalciumconcentrationoftheurinewasdeterminedtobe1mM,andthenoxalatewasaddeduntilcrystalsformed.If0.15mgwasadded,whatistheKspofcalciumoxalate?

A.Ksp=2.9×10−7

B.Ksp=2.4×10−9

C.Ksp=2.9×10−9

D.Ksp=2.4×10−11

5.Olestra(C13H14O11),onceapopularfoodadditive,hasbeenfoundtoreducetheabsorptionofcertainnutrients.Forexample,eachmoleofOlestraeffectivelyremoves8molesofvitaminKfromthedigestivetract.HowmuchvitaminKmustbeingestedtoachieveserumlevelsequivalenttotheRDI,if0.34gofOlestraistakenconcurrently?

A.3.18gB.3.31gC.3.49gD.3.81g

Page 238: Edited By Deeangelee Pooran-Kublall, MD/MPH · 9.5 Preparing for the MCAT: Biochemistry in the Chemical and Physical Foundations of Biological Systems Section 9.6 Preparing for the

GeneralChemistryPassageIIExplanation:USINGTHEKAPLANMETHODSP1.Solubility=excretory/digestivetractsectiondependence

P2.Vitamins=variousfunctions;examples

P3.Minerals≠C,H,O,N;description

P4.Chemicalcompositiondictatesbioavailability/absorptionrates

T1.Valuesformineralsandvitamins

KEYConceptsStoichiometryisahigh-yieldgeneralchemistryskillontheMCAT.Thisskillcanalsobeutilizedinphysicsandbiochemistry.Becomfortablerelatingdimensionalanalysistotopicssuchassolubilityproducts.

1.VitaminCformulationsareadministeredinavarietyofways,includingpureascorbicacidadministeredorallyandascorbylpalmitate(vitaminCester)administeredtopically.HowdoestheadditionofthepalmitategroupaffectoralandtopicalbioavailabilityofvitaminC?

A.Thepalmitategroupdecreasesoralbioavailabilityviareducedsolubilityinthedigestivetract,butincreasestopicalbioavailabilitybyminimizingfreeradicalsintheepidermis.B.Thepalmitategroupincreasesoralbioavailabilityviaincreasedpermeabilityinthedigestivetract,andincreasestopicalbioavailabilitybyincreasingvitaminCstabilityintopicalsolutions.C.Thepalmitategroupdecreasesoralbioavailabilityviadecreasedpermeabilityinthedigestivetract,andhaslittleeffectontopicalbioavailabilitybecauseascorbylpalmitatepenetratestheskinaseffectivelyaspurevitaminC.D.Thepalmitategrouphaslittleeffectonoralbioavailabilitybecausethepalmitategroupisremovedbyhydrolysisinthedigestivetract,andincreasestopicalbioavailabilitybyincreasingthestabilityofvitaminCintopicalsolutions.

Page 239: Edited By Deeangelee Pooran-Kublall, MD/MPH · 9.5 Preparing for the MCAT: Biochemistry in the Chemical and Physical Foundations of Biological Systems Section 9.6 Preparing for the

AssessthequestionThisquestionhaslonganswerchoicesthatdealwithchangesinoralandtopicalbioavailability.Eachanswerchoicehasatotaloffourparts(changeinoralbioavailability,explanation,changeintopicalbioavailability, explanation). In answering this question,we’ll probablywant to startwith just onepart,ratherthantrytopredicteverythingatonce.

PlanyourattackThepassageonlymentionsvitaminCinpassing,inparagraph2,whereitstatesthatvitaminCisanantioxidant.We alsoget data on it inTable1; in particular, its bioavailability is a very high 99%.Beyond that, we’ll need to rely on our knowledge of nutrient absorption and even a little bit oforganicchemistry—namely,thebehaviorofesters—toanswerthisquestion.

ExecutetheplanWhat will happen when the ester reaches the stomach? It’s in an aqueous solution that containssignificant amounts of acid. That’s the perfect recipe for hydrolysis of the ester, which results invitaminCandpalmiticacid.Asaresult,vitaminCshouldbeavailable in thestomachas it is in itspureform,soitsbioavailabilityshouldremainapproximatelythesame.

Answerbymatching,eliminating,orguessingScanning the first part of each answer, only choice (D) matches this prediction. Looking at theremainderof the answer, the explanationmatchesourprediction.Moreover, the secondhalfof thestatementiscorrectaswell:thepalmitategroupshouldincreasethestabilityofvitaminCintopicalsolutions.

Let’squicklylookatthewronganswers.(B)wouldbeunlikelyongeneralprinciple;sincetheoralbioavailabilityisalready99percent, it’sunlikelythatpalmitate—oranythingelse,forthatmatter—couldsignificantlyincreasethatpercentage.Ontheotherhand,(A)and(C)bothclaimasignificantdecrease. (A) claims this occurs because the ester is less soluble in water; as stated above, estersundergohydrolysisinwater,sothisisincorrect.(C),ontheotherhand,claimsthatit isbecauseofdecreasedpermeabilityoftheester.Infact,thereverseistrue:ifanything,thevitaminCester,becauseofitslargehydrophobicgroup,shouldbemoresoluble,notless.

TakeawaysForpredictionsofpermeabilityandsolubility,usethechemicalstructures,butbemindfulofchemicalreactionsthatcouldalterthephysicalproperties.

Page 240: Edited By Deeangelee Pooran-Kublall, MD/MPH · 9.5 Preparing for the MCAT: Biochemistry in the Chemical and Physical Foundations of Biological Systems Section 9.6 Preparing for the

ThingstoWatchOutForEvenafteryou’vefoundaprediction,besuretocheckalloftheanswers—theymaycontaincluesthatcouldrevealanunconsideredfactor.

KeyConceptsBiologicalsystemshaveevolvedtointeractwiththeinorganicworld.Normalfunctioningreliesonaharmonybetweenthetwo.

2.ApharmaceuticalcompanyisdevelopinganoralvitaminEsupplement.WhatistheminimumoraldoseofvitaminEacetate(Ksp=1.6×10−7)neededtosaturatethebloodwithvitaminE?(Assumenoothersourcesofeitherion,andthattheacetate'sbioavailabilityequalsvitaminE’s.)

A.0.15gramsB.0.3gramsC.1.0gramsD.1.5grams

AssessthequestionThis calculation question asks for the minimummass of vitamin E acetate needed to saturate theblood. The answer choices are relatively far apart, so approximations should be OK. That said,stoichiometryquestionsareoftengoodcandidatesfortriage.

PlanyourattackTo answer this question, we’ll need several pieces of information. First, we need Ksp from thequestionstem.We’llalsoneedthemolecularmassfromTable1,aswellasthebioavailability,sinceparagraph4tellsusthatnotallofanoraldosereachessystemiccirculation.We’llneedtocombinethatwithourknowledgeofmolarsolubilityandstoichiometrycalculations.

ExecutetheplanVitaminEestershoulddissociateinwatertogivevitaminEandacetateions.Sincethequestionstemtellsuswecanassumetherearenoothersourcesofeither ion,wecanassumethatxmolesof theesterwillyieldxmolesofvitaminEandxmolesofacetate,sinceacetateisa−1ion.

Page 241: Edited By Deeangelee Pooran-Kublall, MD/MPH · 9.5 Preparing for the MCAT: Biochemistry in the Chemical and Physical Foundations of Biological Systems Section 9.6 Preparing for the

Ksp=[VitaminE][Acetate].Sincethequestionindicatesioncontributionsfromothercompoundscanbe ignored, the only contributor of either ion is vitaminE acetate. Predicting a 1:1 stoichiometricratio(theacetateioncarriesa-1charge),gives: .

At thispoint, round theKspvalue toaworkablenumbersuchas1.6×10−7whichallows thesquareroot calculation to become: . Notice that “workable” (whentakingthesquarerootofanumberinscientificnotation)meansthesignificandisaperfectsquareandtheexponentiseven.

ThisisthetheoreticalmaximumconcentrationofvitaminE(fromvitaminEacetate)thatcansaturatethe blood. In order to determine the actual amount necessary to reach these levels, the volume isrequired.Usethevolumeofbloodgiveninthepassagetomakeanestimate.

(4×10−4mol/L)(5L)=2×10−3mol.Thenusethemolarmasstodeterminetheequivalentamountingrams:(2×10−3mol)(4.75×102g/mol)=∼10×10−1g=∼1g.

Lastly,considerthebioavailability(0.67).Only67percentoftheingestedvitaminEacetateisactually

absorbed. Therefore, in order to absorb 1 gram of vitamin E acetate ,approximately1.5gramsmustbeingested.

Answerbymatching,eliminating,orguessingOnly one answer, choice(D), is greater than 1 gram, so itmust be our correct answer.We didn’tactuallyneedtoperformthefinalbioavailabilitycalculation,butwedidneedtoconsiderit.

Thewronganswersall reflect reasoningerrors test takersmightmake. (A) results fromtaking thesquarerootincorrectly(getting4×10−3insteadof4×10−4).Ifyouforgottomultiplybytheaveragebloodvolumeof5L,you’dget(B).Finally,(C)resultsfromneglectingthebioavailability.

TakeawaysLookattheanswerchoicesandtheinformationgiven.Alwaysensurethatyourcalculationsincludeunitstoensurethatyouhaveachievedtheanswerasdesired.

ThingstoWatchOutForScientificnotationmakesmathmuchsimpler.Dimensionalanalysisinthesesituationsiscriticalandawarenessofcommonions(acetate)willensureyouutilizetheKspequationproperly.

Page 242: Edited By Deeangelee Pooran-Kublall, MD/MPH · 9.5 Preparing for the MCAT: Biochemistry in the Chemical and Physical Foundations of Biological Systems Section 9.6 Preparing for the

KeyConceptsStoichiometryanddimensionalanalysesareskillsutilizedinvarioustopics.

3.Whichofthefollowingwouldbestexplaintherelativelylowbioavailabilityofzincsulfate?A.Zincsulfateisinsolubleinaqueoussolutionandthusitsionscannotbedigestedandabsorbed.B.Othersubstancesinteractwiththeionsintheoralroute,inhibitingabsorption.C.Zn2+ionsdonothaveenoughvalenceelectronstosufficientlyinteractwithintestinalionchannels.D.Zn2+ionsareimmediatelydepositedintheboneandothertissue,thusloweringthebioavailability.

AssessthequestionThis question asks us why zinc sulfate has a low bioavailability. The answers are reasons, whichmeansweneedananswerthatisbothscientificallyplausibleandrelevanttothequestionathand.

PlanyourattackBioavailabilityisdefinedinparagraph4.Zincsulfateisconsideredamineral,whichisdiscussedinthethirdparagraph.Table1confirmsthatzincsulfatedoes,infact,havelowbioavailability.

ExecutetheplanParagraph4 states that processes that hinder absorptionor alter a substance reducebioavailability.Paragraph3notes thatelementscanreactwithmolecules in thedigestive tract, sometimesformingtoxic by-products. It’s reasonable to believe that the samemight happenwith ions as well as freeelements.Therefore,wecanpredictthatthecorrectanswershouldindicateeitherareactioninvolvingzincionsorsomefactorthathindersabsorption.

Answerbymatching,eliminating,orguessingConsideringtheanswerchoicesoneatatime,(A)isfalse:sulfatesaregenerallysoluble.Choice(B)matchesourprediction:itgivesusareasonwhyZn2+wouldbepoorlyabsorbed.

Page 243: Edited By Deeangelee Pooran-Kublall, MD/MPH · 9.5 Preparing for the MCAT: Biochemistry in the Chemical and Physical Foundations of Biological Systems Section 9.6 Preparing for the

(C) is incorrect; cations aredrawnby electrostatic forces as a result of their positive charges, nottheirelectrons.Finally,(D)isalsoincorrect;bonedoesnotactasastoragepoolforzinc.

TakeawaysWhenleftwithfourconfusingchoices,eliminatetheonesthatarecertainlywrong.

ThingstoWatchOutForSolubilityrulesareoften,butnotalways,necessarytosolvesolutionproblems.Whenindoubt,startwithwhatyouaremostconfidentwithandworkfromthere.

KeyConceptsQuestionsthattaskyouwithexplainingdiscrepanciesrequirecriticalthinkingandafirmunderstandingofthepassage.Ifunsurehowtoapproachthese,markthemforreviewandreturntothemwhenyouhavemoretime.

4.HyperoxaluriacausesCaC2O4todepositinurine.A0.75literurinesamplefreeofoxalatewascollected.Thecalciumconcentrationoftheurinewasdeterminedtobe1mM,andthenoxalatewasaddeduntilcrystalsformed.If0.15mgwasadded,whatistheKspofcalciumoxalate?

A.Ksp=2.9×10−7

B.Ksp=2.4×10−9

C.Ksp=2.9×10−9

D.Ksp=2.4×10−11

AssessthequestionThisisclearlyacalculationquestion.InordertosolvefortheKspitwillbenecessarytodeterminetheionproductat the timeofprecipitation.Thiswill involvecalculatingtheconcentrationsofbothionspresent.Thequestionprovidesthecalciumconcentration,andtheconcentrationofoxalatecanbecalculatedfromtheinformationinthequestionstem.

Planyourattack

Page 244: Edited By Deeangelee Pooran-Kublall, MD/MPH · 9.5 Preparing for the MCAT: Biochemistry in the Chemical and Physical Foundations of Biological Systems Section 9.6 Preparing for the

ItwillbenecessarytohavetheKspexpressionforCaC2O4(whichcanbederivedfromthebalancedreaction)andtheconcentrationsofeachoftheionsintheequation.

Thecalciumconcentrationisgivenandtheconcentrationofoxalatecanbecalculatedfromthemassandthevolumeofthesample.

Executetheplan

Answerbymatching,eliminating,orguessingExecutingtheplanledtoapredictionof2.3×10−9.Fortunately,thereisanearmatchwith(B).

TakeawaysBemethodicalandorganizedinyourscratchwork.

ThingstoWatchOutForBesuretousetheproperdata.Forinstance,massandmolecularweightarecommonlymixedup.ThatisaneasywaytoarriveatawronganswerontheMCAT.

KeyConceptsSolubilityisanimportantbiologicalmechanismfornutrienttransfer;befamiliarwiththedifferencesbetweenIPandKspandhowtousethemtodetermineionconcentrations.

5.Olestra(C13H14O11),onceapopularfoodadditive,hasbeenfoundtoreducetheabsorptionofcertainnutrients.Forexample,eachmoleofOlestraeffectivelyremoves8molesofvitaminKfromthedigestivetract.HowmuchvitaminKmustbeingestedtoachieveserumlevelsequivalenttotheRDI,if0.34gofOlestraistakenconcurrently?

A.3.18gB.3.31g

Page 245: Edited By Deeangelee Pooran-Kublall, MD/MPH · 9.5 Preparing for the MCAT: Biochemistry in the Chemical and Physical Foundations of Biological Systems Section 9.6 Preparing for the

C.3.49gD.3.81g

AssessthequestionThisquestionasksforthenumberofmolesofvitaminKrequired,butallthenumericalinformationinthepassageandquestionstemdealwithgrams.Thisisaclassicstoichiometrysetup,andshouldtipusofftowatchourunits.Fortunatelyforus,theanswersarerelativelyfarapart,soapproximationsshouldn’tcauseaproblem.

PlanyourattackWe’llneedtheamountofvitaminKthatOlestraremoves,giveninparagraph4.Then,we’llneedtofigureoutthetotalamountofvitaminKthebagneeds,usingtheinformationinTable1.Finally,we’llneedtoconvertgramstomoles.

ExecutetheplanBeginwith0.34gOlestraandusetheperiodictabletofindthemolecularmass(roughly346g/mol)andthenmoles.Simplifyingthemath:0.34gOlestra/340g/mol=∼0.001molOlestra

Thenusetheconversionfactorof8to1: vitaminK—lost.AtthispointnomatterhowmuchvitaminKisrequired,atleast0.008molesmustbeconsumed

(becausethat’showmuchwillbelostwithOlestra).Thiscorrespondsto ;anyanswerssignificantlylessthan3.56canbeeliminated.

Theamountdesiredintheserum(equivalenttotheRDI)is120μg—anominalamount(whichshouldallowonetoconfidentlychooseananswer).Takingintoaccountthebioavailability,inordertoreachserumlevelsof120μg,120μg/0.9=∼133μgmustbeavailableforabsorption.Sincethefirst3.56gramswillbeunavailable,thetotalamountthatmustbeingestedis3.56g+133;since133μgisaninsignificantamount,agoodapproximationis3.56g.

Answerbymatching,eliminating,orguessingOurpredictionof5×10−7molmatcheschoice(C).Note that twoof thewronganswersstemfromusingthewrongnumberinthestoichiometrycalculation.(A)comesfromsubtracting80μgfromthe

Page 246: Edited By Deeangelee Pooran-Kublall, MD/MPH · 9.5 Preparing for the MCAT: Biochemistry in the Chemical and Physical Foundations of Biological Systems Section 9.6 Preparing for the

RDI,while(B)resultsfromusingjustthe80μgremovedbyOlestra.(D)isaclassictrapanswerinmulti-stepcalculations:weneed200microgramsofvitaminK,not200micromoles.

TakeawaysConversionfactorsmaysometimesbegiveninapassageorquestionstem;besuretoutilizethemappropriately(usinglabelswhenevernecessary).

ThingstoWatchOutForRoundingerrorwillneedtobetakenintoaccountattimes.It’sbesttokeepnotes(mentaloronyourscratchpaper)abouttheapproximationsthathavebeenmade,sothatadjustmentscanbemadequicklyandaccurately.

ThingstoWatchOutForDon’trushthroughstoichiometrycalculations.Makesurethatyou’reusingthecorrectratios,andthattheunitscanceloutineachstep.

Page 247: Edited By Deeangelee Pooran-Kublall, MD/MPH · 9.5 Preparing for the MCAT: Biochemistry in the Chemical and Physical Foundations of Biological Systems Section 9.6 Preparing for the

11.7GeneralChemistryPracticeGENERALCHEMISTRYPASSAGEIII(QUESTIONS1–6)Theconversionofreactantstoproductscanbemediatedbyanumberoffactors.Inareactionthathasa high free energy of activation for the conversion of reactants to an intermediate, the observedreaction rate is based on the concentration of the intermediate in solution. Take the followingexample:

Reactants Intermediate→Products

where there is an equilibrium constant, KI, for the equilibrium between the reactants and theintermediate.Therateoftheoverallreaction,V,isdictatedby:

V=v[I]

where is the rate constant, and [I] is the concentration of the intermediate. The activationenergyrequired tocomplete this reaction is thedifferencebetween the freeenergyof the reactantsandthatofthetransitionstate(betweenthereactantsandtheintermediate).Dependingonthereaction,this free energy of activation may be very large. A large free energy of activation implies thatrelatively fewof the reactantswill have sufficient energy to overcome thebarrier and [ultimately]becomeproducts.Thefreeactivationenergyandtheequilibriumconstant(for thereactantsandtheintermediate)arerelatedby:

ΔGI=−RTlnKI

This equation, combinedwith the equation for theoverall reaction rate, shows that the rate canbeaffectedbychangingtheactivationenergyofthesystem;thelowertheactivationenergy,thequickerthereactionwillproceed.

Inbiological systems, enzymeswork to lower activationenergies and therefore allow reactions toreachequilibriummorequickly.Reactionratesandenzymeactivityalikedependonthetemperatureofthesystem.Forreactionrates,thetemperatureaffectsKeq,therebyestablishinganewequilibrium.Foroptimalenzymeactivity,onlyacertainrangeoftemperaturesissuitable.Astheenzymeisheated,theenzymedenaturesandtheconformationoftheactivesiteiscompromised.

P1.

E1.

E2.

P2.

1.Whichofthefollowingequationsrelatestherateofthereactiontotheactivationenergy?A.v[I]RTlnKeqB.v[I]e−(ΔGI/RT)

Page 248: Edited By Deeangelee Pooran-Kublall, MD/MPH · 9.5 Preparing for the MCAT: Biochemistry in the Chemical and Physical Foundations of Biological Systems Section 9.6 Preparing for the

C.v[R]e(ΔGI/RT)

D.v[R]10−(Δ GI/2.3RT)

2.Ascientiststudyingthekineticsofatwo-substratereactionobservesratesintheratio4:1:16.Afterthefirsttrial,theonlychangewasareductionintheconcentrationofA(byafactoroffour).Forthethirdtrial,theconcentrationofAwasunchangedfromthesecondtrial,andtheconcentrationofBwasquadrupled.Whichofthefollowingbestrepresentstheratelawforthisreaction?

A.k[A][B]B.k[A]2[B]C.k[A][B]2

D.k[A]2[B]2

3.AllofthefollowingquantitiesdoNOTincreaseaftertheadditionoftheappropriateenzyme,EXCEPT:

A.Keq.

B.ΔGI.C.V.D.ΔG°.

4.Theinteractionsbetweenanenzymeandthesubstratetypicallyinvolvemultiple,weakandreversibleinteractions.WhichfactoristheLEASTabundantbetweenthesubstrateandenzyme?

A.VanderWaalsforcesB.HydrophobiceffectsC.CovalentbondingD.Hydrogenbonding

5.Iftheexamplereactiondescribedinthepassageisexothermicwitharate-limitingfirststep,whichofthefollowingcouldrepresenttheenergyprofileforthereaction?

A.

B.

Page 249: Edited By Deeangelee Pooran-Kublall, MD/MPH · 9.5 Preparing for the MCAT: Biochemistry in the Chemical and Physical Foundations of Biological Systems Section 9.6 Preparing for the

C.

D.

6.Supposeanenzymeisonlyactivewhenagivencarboxylicacidresidueisinthecarboxylateform.IftheKaoftheacidis4×10−5,whichofthefollowingcorrectlymatchesthepHofthesolutionwiththeratioofactivetoinactiveenzymes?

A.pH5.0and1:1B.pH4.4and10:1C.pH4.0and1:1D.pH3.4and1:10

GeneralChemistryPracticePassageExplanationsP1.Factorsmediatingreactions

E1.Rateoftheoverallreaction

E2.Relationshipbetweenfreeenergyandequilibrium

Page 250: Edited By Deeangelee Pooran-Kublall, MD/MPH · 9.5 Preparing for the MCAT: Biochemistry in the Chemical and Physical Foundations of Biological Systems Section 9.6 Preparing for the

P2.Enzymesloweractivationenergy

1.(D)

Thereactionvelocity,orrate,isgivenasV=v[I],whichis thejumping-offpointforthisquestion.ThepassagegivesKIastheequilibriumconstantforthetransitionbetween[R]and[I]andlookingatthesecondequationinthepassageitcanbeseenthatKIandΔGI,theactivationenergy,arerelated.TogetKIandsubsequentlyΔGIintotheequation,solvefor[I]intermsofKI.SinceKIistheequilibrium

constant,itsexpressionisgovernedby ,whichcanberearrangedto[I]=[R]KI.SolvingforKI

intermsofΔGIrequirestherearrangementofΔGI=−RTlnKItolnKI=−ΔGI/RT.SolvingtogetridofthenaturalloggivesKI=e−ΔGI/RT.Combiningthatequationwiththeearlierexpressionfortherategivesv[R]e−(ΔGI/RT).But this doesn’tmatch any answers.Onemore step is then required to solve this.WhensolvingforlnKI=−ΔGI/RTonecanmultiplybyafactorof2.3tochangefromnaturallogtologbase10 giving 2.3 logKI = −ΔGI/RT, which yields choice (D) or v[R]10−(Δ GI/2.3RT). Alternatively, using theprocessofelimination:(A)doesnotrelate“rateofthereactiontotheactivationenergy;”(B)impliese−(ΔGI/RT)=1(inordertosatisfyv[I]e−(ΔGI/RT)=v[I]),whichmeanstheactivationenergyisalwayszero,soitcannot be correct; finally, the passage states that the “lower the activation energy, the quicker thereaction”so(C)isout,becauseitshowsadirectrelationshipbetweenthetwo.

2.(C)

Determiningreactionratesrequiresseeinghowreactionsreactwhenchangingtheconcentrationsofonesubstratewhilekeepingtheothersconstant.ThisquestionfirstquarterstheconcentrationofAor

multiplies it by . This produces a drop in the reaction rate that is equivalent to the drop inconcentration, so the reaction rate is first orderwith respect toA. This eliminates choice (B) andchoice(D),astheybothhaveAasasecond-orderreactant.DeterminingtheorderofBasareactantisa little more complicated. The scientist increases the concentration of B by a factor of 4 and thereactionrate increases to4timestheoriginal.Thismeansthat thereactionrate is16timesgreater,

sincewehavealreadyloweredthereactionrateto oftheoriginal.ThismeansthatBisasecond-orderreactantandmatcheschoice(C).

3.(C)

Eliminatethosethatarenotincreased.Choice(A)canbeeliminatedbecauseenzymesdonotaltertheequilibrium constant; they just allow the reaction to reach equilibrium more quickly. From thepassage,itcanbeseenthatΔGIislowered,soeliminatechoice(B).Also,enzymesdonotchangethedifferenceinfreeenergybetweenthereactantsandproducts,ΔG°,justthefreeenergyofactivation—sochoice(D)isout.Enzymesincreasethereactionrate,soVincreases,meaningit’stheexceptiontothosethatdonotincrease—match(C).

4.(C)

Stabilizing transition states in biological systems involves making sure unstable parts of themolecules, i.e., parts that have charges that they don’t want, have something to interact with tostabilize them. The enzyme itself doesn’t usually contribute or accept electrons, but rather lets

Page 251: Edited By Deeangelee Pooran-Kublall, MD/MPH · 9.5 Preparing for the MCAT: Biochemistry in the Chemical and Physical Foundations of Biological Systems Section 9.6 Preparing for the

oppositechargesinteracttolessentheenergyofasystem.Thismeansforcesthatarenottransferringor sharing electrons will be more abundant than those that actually bond. There are many moreinstancesofthenon-covalentforcesastheseareusuallypresentinnumerousplacesinanenzyme’sactivesitetohelpstabilize.Enzymesdonotusuallycovalentlybondtotheirsubstrate,asthiswouldraise the energy required to dissociate the enzyme from the substrate andwould slow the reactiondown.Covalentbondingorchoice(C)willbetheleastprevalent.

5.(B)

Thereactioninthepassagehasadiscreteintermediateandisthusatwo-stepreaction.Thismeansthatthe correct graph will have two energy activation hills and will have a valley in the middlerepresentingtheintermediate.Thiseliminateschoices(A)and (D), since theyhave toofewand toomanypeaksrespectively.Thenextthingtoconsideristhequestionstem.Itindicatesthatthefirststepofthereaction,theconversionofreactanttointermediate,istherate-limitingstep.Thismeansthatitwillbe the slowest stepand thushave the largestactivationenergy,whichshows thatchoice(B) iscorrect.

6.(D)

ThepKaisafunctionoftheKa,whichdescribestheratioofconcentrationofconjugatebasetoacidatequilibrium.GiventheKathepKacanbecalculated[precisely]as4.4(ormorequicklyas4.6).Atacertain pH, the actual ratio of conjugate base to acid is described by the Henderson–Hasselbalch

equation.Theequation canberearrangedtoyield .SoatapH=pKa(4.4),theratioofactivetoinactiveis1:1,whichmeanswecaneliminatechoices(A),(B),and(C).AccordingtotheHHequation,atapHonepointbelowthepKa(3.4) theratioofactive toinactiveis1:10,whichmatcheschoice(D).*ThisconceptisonlytestedontheBiologicalandBiochemicalFoundationsofLivingSystemssection.**ThisconceptcanalsobetestedontheBiologicalandBiochemicalFoundationsofLivingSystemssection.*ThisconceptisonlytestedontheBiologicalandBiochemicalFoundationsofLivingSystemssection.**ThisconceptcanalsobetestedontheBiologicalandBiochemicalFoundationsofLivingSystemssection.**ThisconceptcanalsobetestedontheBiologicalandBiochemicalFoundationsofLivingSystemssection.

Page 252: Edited By Deeangelee Pooran-Kublall, MD/MPH · 9.5 Preparing for the MCAT: Biochemistry in the Chemical and Physical Foundations of Biological Systems Section 9.6 Preparing for the

CHAPTERTWELVE

OrganicChemistryOrganic chemistry has long been regarded as a significant hurdle tomedical school, full of longhoursinthelabandchallengingsynthesispathways.Overtheyears,theAAMChasbeenplacinglesssignificance on organic chemistry on the MCAT, and more emphasis on molecular biology,biochemistry,andgenetics.On theMCATin2015,5percentof thequestions in theBiologicalandBiochemicalFoundationsofLivingSystemssectionand15percentofthequestionsintheChemicaland Physical Foundations of Biological Systems section will be related to organic chemistry.Regardlessof these changes, organic chemistry is still important to achieve ahigh score.Organicchemistryknowledgeisstill requiredtoevaluatechemicalstructuresandtopredict thebehaviorofmoleculeswithinabiologicalsystem.YoucanexpecttheMCATtotestorganicchemistrywithinthecontext ofwhat is needed for further understandingof biochemistry and reactions essential for anunderstandingofmedicaltestingandpharmacology.

12.1ReadingthePassagePassagesrelatedtoorganicchemistrymaycomeintheformofapassagethatisexclusivelyrelatedtoorganicchemistryanditsapplicationtoabiologicalsystem,orapassagethatcoversmultiplesubjectareas, especially biochemistry. There is a clear relationship between these two sciences in thatunderstandingoforganicchemistrystructuresandreactionsisessentialtounderstandingthebehaviorof biochemical systems. Therefore, success in organic chemistry on theMCAT depends on yourabilitytoapplyyourknowledgeoforganicchemistrytobiochemicalsystems.

PASSAGETYPESOrganicchemistrypassageswilleitherpresentinformationoranexperiment.

•Informationpassages•Oftenshort,andaccompaniedbydiagramsillustratingreactionsormechanisms.•Mayalsobeintegratedwithothersubjectareas,suchasbiochemistryorbiology.•Maypresentanewreactionoraseriesofrelatedreactions,ordescribeacompoundor an experimental technique. However, organic chemistry passages will always berelatedtoalivingsystem.

•Experimentpassages•Presentationofoneormoreexperiments.•Experimentaldatamaybeintheformofpercentageyieldfromasynthesis,awrittensummaryofthereaction,ordescriptionoftheappearanceorspectroscopicpropertiesofaproduct.•Theproductofasynthesismaythenbeusedinabiochemistryexperiment,meaningthatthesynthesisreactionandthebiochemistryexperimentgohandinhand.

PASSAGETYPESINORGANICCHEMISTRY

Page 253: Edited By Deeangelee Pooran-Kublall, MD/MPH · 9.5 Preparing for the MCAT: Biochemistry in the Chemical and Physical Foundations of Biological Systems Section 9.6 Preparing for the

InformationPassages

ExperimentPassages

Content

Readslikeatextbook;mayintegrateconceptsoforganicchemistryandbiochemistry.

Readslikealabreportorajournalarticlethatsummarizesanexperimentalprocedure.Theproductofanorganicchemistryreactionmaybethesubstrateforabiochemistryexperiment.

Questions

Manyquestionswillnotrequireinformationfromthepassage.Thosethatdoarelikelytobemoretheoretical.

Willoftenfocusonthehypothesis,procedure,andoutcome.Ifmultipleexperiments,questionsarelikelytofocusontherelationshipsbetweentheexperiments.

OUTLININGTHEPASSAGEOrganicchemistrypassageswilloftenfeaturesynthesispathwaysormolecularstructures.Inaddition,the passagemay be amixture of organic chemistry and biochemistry, rather than a pure organicchemistrypassages.Whileapproachingapassage,besuretothinkabouthowtheorganicchemistryscienceappliestoalivingsystem,andlookfortheconnectionsbetweenthesciencespresentedinthepassage.

ScanforStructure•Startbyevaluatingtheoverallstructureofthepassage.Isthepassageentirelytext-based?Doesthepassagecontainbothtextanddiagrams?Whattypesofdiagramsarepresent?Charts,graphs,tables,synthesispathways,ordiagramsofanexperimentalprocedure?•Is thispassageone todonowor later?Answer thisquestion,and thenmoveon toeither thenextpassageorstartreadingthepassage.

ReadStrategically•Asyouread,identifythepurposeofeachparagraphandhowtheparagraphfitsintotheoverallstructureandargumentofthepassage.•Identifyanyconnectionsbetweenfiguresandthetext.•Considerhowtheorganicchemistrydescribedfitsintoalivingsystem.•Anticipatewhatothertopicsmightbeinvolvedinthepassage.• If the passage describes an experiment, be sure to identify the hypothesis, procedure, andoutcome.

LabelEachComponent•Writedownthepurposeofeachparagraphandfigure.•Ifafigureisdirectlyrelatedtosomeportionofthetext,identifythoseconnections.•Writedownwhatisrepresentedineachfigure.

Page 254: Edited By Deeangelee Pooran-Kublall, MD/MPH · 9.5 Preparing for the MCAT: Biochemistry in the Chemical and Physical Foundations of Biological Systems Section 9.6 Preparing for the

•Forexperimentalpassages,notethelocationofthehypothesis,procedure,andoutcome.

ReflectonYourOutline•Determine theoverallpurposeof thepassage.What isbeingdescribed in thepassage?Whywastheexperimentconducted?

Page 255: Edited By Deeangelee Pooran-Kublall, MD/MPH · 9.5 Preparing for the MCAT: Biochemistry in the Chemical and Physical Foundations of Biological Systems Section 9.6 Preparing for the

12.2AnsweringtheQuestionsQuestionsthatareexclusivelyrelatedtoorganicchemistryareoftenverystraightforward.However,questions that require integration of organic chemistry with another topic may be more difficult.Onceagain,thesamequestiontypesseenintheothersciencesapply.

•Discretequestions•Questionsnotassociatedwithadescriptivepassage.•Oftenprecededwithawarningsuchas,“Questions12–15areNOTassociatedwithadescriptivepassage.”•Likelytotestbasicprinciplesoforganicchemistry,suchasstructuresandreactions.

•Questionsthatstandalonefromthepassage•Oneofthemostcommonquestiontypesoforganicchemistryquestions.•Oftenrequiresanalysisofstructures.• When evaluating structures, many of the wrong answer choices will containstructuresthataresimplynotpossiblegiventhebasicconceptsoforganicchemistry.

•Questionsthatrequiredatafromthepassage•Willoftenrequireanalysisofdataorexperimentaldesign.•Mayoften require evaluationof a synthesis processwith adifferent substrate, thusrequiringapplicationofinformationfromthepassagetoanewsituation.

•Questionsthatrequirethegoalofthepassage•Thenatureofthesequestionswilldependonwhatelseisinthepassage.•Ifthepassageisexclusivelyorganicchemistry,thenthegoalofthepassageislikelytobetheoutcomeofanexperimentorsyntheticpathway.• If thepassage containsorganic chemistry integratedwithbiologyor biochemistry,thenthegoalofthepassagewilldependonthecontentofthepassageasawholeandthecontextinwhichorganicchemistryisdiscussed.

DETERMININGTHEPURPOSEOFTHEQUESTIONMCAT organic chemistry questionsmay be related only to organic chemistry ormay be organicchemistryinthecontextofanothersubjectarea.Inaddition,languageisoftenusedtodisguisesimpleorganicchemistryquestionsasmoredifficultones.Payspecialattentiontothetaskofthequestiontoensurethatquestionslikethesearediscoveredandansweredcorrectly.

1.AssesstheQuestion•Identifythetopicanddegreeofdifficultyofthequestion.•Determinethetimerequirementforthequestion.•Decidewhetherthisquestionisonetobeanswerednoworlater.

2.PlanYourAttack•Readthequestioncriticallytoaidinidentifyingthetaskofthequestion.• Identify whether the question is exclusively organic chemistry or requires integration withanothersubjectarea.•Determinewhetherthepassageisrequiredtoanswerthequestion.•Identifywheretheinformationislocated.

Page 256: Edited By Deeangelee Pooran-Kublall, MD/MPH · 9.5 Preparing for the MCAT: Biochemistry in the Chemical and Physical Foundations of Biological Systems Section 9.6 Preparing for the

•Ifdataanalysisisrequired,locatethecorrectdataset.•Identifythetaskofthequestion.•Paraphrase thequestion, ifnecessary, topreventmissingsimplequestionsdisguisedasmorecomplicatedones.

3.ExecuteYourPlan•Locatetheinformationandanalyzethedatatodetermineapossibleanswerforthequestion.

4.AnswertheQuestionbyMatching,Eliminating,orGuessing•Matchyouranswerwiththeanswerchoices.•Ifyouranswerdoesnotmatch,proceedtoelimination.•Wronganswerchoicesinorganicchemistryoftencontainmolecularstructuresthataresimplynotpossibleordonotmakesense.Look for incorrectchargedistribution,wrongnumbersofbondsforatomswithinthemolecule,doublebondsinunusuallocations,etc.• Ifguessing is required, try toeliminate twoanswerchoices.Then,choose theanswerchoicethatismostlikelycorrect,basedonyouranalysis.

Page 257: Edited By Deeangelee Pooran-Kublall, MD/MPH · 9.5 Preparing for the MCAT: Biochemistry in the Chemical and Physical Foundations of Biological Systems Section 9.6 Preparing for the

12.3GettingtheEdgeinOrganicChemistryWith the addition of biochemistry to the MCAT, many organic chemistry questions will requireintegrationwith concepts of biochemistry and biology. In this context, expect questions that assessyourunderstandingof the implicationsoforganicchemistry inbiochemistryandbiology.Simplerorganicchemistryquestionsarelikelytobedisguisedasmorecomplicatedones.Usingthetaskofthequestion,paraphrasethesequestionstoasimplerform.Doingsowillhelpyoutoidentifythesequestions and determine the correct answer. Many test-takers will become overwhelmed by thesequestions,whileacriticalthinkerwhoparaphrasesquestionswillseethesimplerquestionandansweritcorrectly.

Organicchemistryquestionsoftencontainstructuresasanswerchoices.Manywronganswerchoicesamongthesequestionswillbestructuresthatarenotpossiblegiventhebasicfundamentalsoforganicchemistry.Applyingthebasicfundamentalsoforganicchemistrytotheseanswerchoicescanallowforquickeliminationofwronganswerchoices.

Page 258: Edited By Deeangelee Pooran-Kublall, MD/MPH · 9.5 Preparing for the MCAT: Biochemistry in the Chemical and Physical Foundations of Biological Systems Section 9.6 Preparing for the

12.4Step-By-StepGuidetoOrganicChemistryPassages

OUTLININGTHEPASSAGE•Scanforstructure

•Determinewhethertodothispassagenoworlater.•Identifythestructureofthepassage,includingcharts,graphs,chemicalequations,andsynthesispathways.

•Readstrategically•Identifythetypeofpassage.•Payspecialattentiontotherelationshipsbetweenconcepts.•Inanexperimentpassage,determinethehypothesis,procedure,andoutcome.• In an information passage, identify how the information in each paragraph fitstogethertopresentaunifiedpicture.•Identifywhatinformationispresentedineachfigureorimage.

•Labeleachcomponent•Writedownthepurposeofeachparagraphandfigure.•Identifyanyconnectionsbetweenconceptswithinthepassage.•Determinethegoalofanysynthesispathwayandhowtheproductsofthatsynthesisfitintotherestofthepassage,i.e.,theproductofthesynthesismaybethesubstrateforabiochemicalprocess.

•Reflectonyouroutline•Determinethegoalofthepassageandwriteitdown.•Identifytheconceptswithinthepassageinanefforttoanticipatequestions.

ANSWERINGTHEQUESTIONS

1.Assessthequestion•Determinewhetherthisquestionshouldbedonenoworlater.•Identifythetopicandthedegreeofdifficulty.•Goodquestionstodonowinorganicchemistryareonesthatstandalonefromthepassageordonotrequireextensivedataanalysisinordertodeterminethecorrectanswer.

2.Planyourattack•Identifythetaskofthequestion.Ifnecessary,paraphrasethequestiontomakeitsimpler.•Determinewhatyoualreadyknow,andwhatinformationyouneed.•Identifywheretofindtherequiredinformation:thepassage,thequestion,youroutline,oryourownknowledge• If youhave to goback to the passage, determinewhere to find the required informationbyusingyouroutline.• If data analysis is required, identify the correct data set, as there may be multiple datarepresentations.

3.Executeyourplan

Page 259: Edited By Deeangelee Pooran-Kublall, MD/MPH · 9.5 Preparing for the MCAT: Biochemistry in the Chemical and Physical Foundations of Biological Systems Section 9.6 Preparing for the

•Analyzethedata,gobacktothepassage,andcarryoutyourplan.•Ifyougetstuckanalyzingdata,rememberthatthetrendofthedataisoftenenoughtoyieldacorrectanswerchoice.

4.Answerthequestionbymatching,eliminating,orguessing•Matchyouranswertotheanswerchoices.•Ifthereisnotamatch,eliminateincorrectanswerchoices.Someoftheanswerchoicesmaynotmakesense;eliminatethosefirst.•Iftheanswerchoicesaremolecularstructures,eliminatebystartingwithstructuresthatsimplydonotmakesense.•Ifeliminationdoesnotprovideaclearanswer,guessbetweentwoprobableanswers.

Page 260: Edited By Deeangelee Pooran-Kublall, MD/MPH · 9.5 Preparing for the MCAT: Biochemistry in the Chemical and Physical Foundations of Biological Systems Section 9.6 Preparing for the

12.5PreparingfortheMCAT:OrganicChemistryTheseare theorganicchemistry topics thatyouare likely toseeonTestDay.Allof the followingtopics appear in the Chemical and Physical Foundations of Biological Systems section unlessotherwisenoted.

AMINOACIDS•Descriptionsofaminoacidsintermsofabsoluteconfirmationattheαpositionandasdipolarions**•Classificationofaminoacidsasacidicorbasic,hydrophobicorhydrophilic**• Reactions of amino acids, including sulfur linkages for cysteine, peptide linkages inpolypeptidesandproteins,andhydrolysis**•SynthesisofαaminoacidsincludingtheStreckersynthesisandGabrielsynthesis

CARBOHYDRATES• Descriptions of carbohydrates, including nomenclature, classification, common names,absoluteconfiguration,cyclicstructure,conformationsofhexoses,epimers,andanomers**•Reactionsofcarbohydrates,includinghydrolysisoftheglycosidelinkage**•Definitionsandcharacteristicsofmonosaccharides**•Theconceptofketo-enoltautomerismofmonosaccharides

MOLECULARSTRUCTUREANDABSORPTIONSPECTRA• Identification and characteristics of the infrared region, including intramolecular vibrationsandrotations; recognitionofcommoncharacteristicsofgroupabsorptionsandthefingerprintregion• Identification and characteristics of the visible region, including creation of color byabsorptionandtheeffectofstructuralchangesonabsorption•Identificationandcharacteristicsoftheultravioletregion,includingπ-electronandnonbondingelectrontransitionsandconjugatedsystems•FundamentalconceptsofNMRspectroscopy,includingprotonsinamagneticfield,equivalentprotons,andspin-spinsplitting

SEPARATIONSANDPURIFICATIONS•Conceptsofextractionasadistributionofsolutebetweentwoimmisciblesolvents•Conceptsandprocessofdistillation• Process and basic principles of chromatography, including column chromatography, gas-liquidchromatography,highpressureliquidchromatography,paperchromatography,andthin-layerchromatography•Definitionandcharacteristicsofracemicmixtures,includingseparationofenantiomers

LIPIDS

Page 261: Edited By Deeangelee Pooran-Kublall, MD/MPH · 9.5 Preparing for the MCAT: Biochemistry in the Chemical and Physical Foundations of Biological Systems Section 9.6 Preparing for the

•Typesof lipids, includingthoseforstorage(triacylglycerolsandfreefattyacids),structural(phospholipids, phosphatides, sphingolipids, and waxes), and signals/cofactors (fat-solublevitaminsandsteroids)**•Theprocessandoutcomeofsaponification•Descriptionsandstructuresofsteroids,terpenes,andterpenoids*

ALDEHYDESANDKETONES•Descriptionsandcharacteristicsofaldehydesandketones,includingnomenclatureandphysicalproperties•Characteristicsofnucleophilicadditionreactionsatthecarbonylbond(C=O),includingacetal,hemiacetal,imine,enamine,hydridereagents,andcyanohydrin•Oxidationofaldehydes• Characteristics of reactions at adjacent positions (enolate chemistry), including keto-enoltautomerism (α-racemization), aldol condensation, retro-aldol, and kinetic vs. thermodynamicenolate• General principles of aldehyde and ketone chemistry, including effect of substituents onreactivityofC=O,sterichindrance,andacidityoftheα-H;carbanions

ALCOHOLS•Description,nomenclature,andphysicalpropertiesofalcohols,includingacidityandhydrogenbonding• Important reactions of alcohols, including oxidation, protection of alcohols, preparation ofmesylatesandtosylates,andsubstitutionreactions(SN1andSN2)

CARBOXYLICACIDS•Description,nomenclature,andphysicalpropertiesofcarboxylicacids•Importantreactionsofcarboxylicgroups,includingamides(andlactam),esters(andlactone),anhydrideformation,reduction,decarboxylation,andreactionsattheαcarbon,substitution

ACIDDERIVATIVES(ANHYDRIDES,AMIDES,ESTERS)•Descriptions,nomenclature,andphysicalpropertiesofacidderivatives•Importantreactionsofacidderivatives,includingnucleophilicsubstitution,transesterification,andhydrolysisofamides•Generalprinciplesofacidderivatives, including relative reactivityofacidderivatives, stericeffects,electroniceffects,andstrain(β-1actams)

PHENOLS•Oxidationandreductionofhydroquinonesandubiquinones;biological2e−redoxcenters

Page 262: Edited By Deeangelee Pooran-Kublall, MD/MPH · 9.5 Preparing for the MCAT: Biochemistry in the Chemical and Physical Foundations of Biological Systems Section 9.6 Preparing for the

POLYCYCLICANDHETEROCYCLICAROMATICCOMPOUNDS•Biologicalaromaticheterocyclics

Page 263: Edited By Deeangelee Pooran-Kublall, MD/MPH · 9.5 Preparing for the MCAT: Biochemistry in the Chemical and Physical Foundations of Biological Systems Section 9.6 Preparing for the

12.6OrganicChemistryWorkedExamplesPASSAGEI:REACTIONMECHANISMSEnzymatic catalysis of reactions is multifaceted and highly specific. The mechanisms by whichenzymesincreasereactionratesareasvariedasthereactionsthemselves.Enzymescaninducebondstraininsubstratesbyfavoringtheconformationofthetransitionstateoverthatofthesubstrate.Inthisway, thesubstratesaredestabilized, therebydecreasing thepotentialenergydifferencebetweenthesubstrateandthetransitionstate.Itisimportanttorecognizethatenzymes,viathismechanism,donotdirectlystabilizethetransitionstate.

Enzymescanalsobesaid to reduce theentropyof the reactants,because inorder fora reaction tooccur,thereactantsmustalignappropriately,whichishighlyunlikelyinsolution.Enzymesthereforefacilitatereactionsbycorrectlyorientatingthesubstrates.Acidicandbasicresiduesofenzymescanactivatenucleophilesorelectrophilesthroughprotonationordeprotonation.Leavinggroupscanbestabilizedbytheseresiduesaswell.Enzymescanformeithertransientionicorcovalentbondswiththesubstrate.Partialorcompleteionicbondingisusuallycarriedoutbyacidicorbasicresiduesidechainstostabilizedevelopingcharges.Covalentbondsmayformbetweenresiduesandthesubstratetolowertransitionstateenergy.However,thesebondsmustbebrokentoregeneratetheenzyme.Thehydrolysisofproteinrequiresthepresenceofproteasestocatalyzetheprocess.Generally,therearetwopathwaysbywhichproteinsarehydrolyzed.

Pathway1:Anucleophilicresidueofanenzymeattacksthesubstrateproteintoformanacyl-enzymeintermediate. An activated water molecule attacks causing the release of half of the product andregenerationoftheenzyme,asshowninFigure1.

Pathway 2: An activated water molecule performs nucleophilic attack on the peptide bond tohydrolyzeit,asshowninFigure2.

Page 264: Edited By Deeangelee Pooran-Kublall, MD/MPH · 9.5 Preparing for the MCAT: Biochemistry in the Chemical and Physical Foundations of Biological Systems Section 9.6 Preparing for the

Figure1.Mechanismofproteinbreakdownbyaserineprotease

Figure2.Mechanismofproteinbreakdownbyanasparticprotease

P1.

P2.

F1.

F2.

1.HowdoesaspartatecontributetothefirststepofthemechanismillustratedinFigure1?A.Itdirectlyincreasesthenucleophilicityoftheserineresiduethroughdeprotonation.B.Itfunctionstomaintainanacidicenvironmentforoptimalenzymaticactivity.C.ItincreasesthepKaofhistidinethroughelectrostaticinteractions.

D.Itservestoincreasethebasicityofhistidinebyprotectingitsaromaticity.

Page 265: Edited By Deeangelee Pooran-Kublall, MD/MPH · 9.5 Preparing for the MCAT: Biochemistry in the Chemical and Physical Foundations of Biological Systems Section 9.6 Preparing for the

2.InFigure1,activatedwatercanperformnucleophilicattackonlyafterhalfofthesubstratehasbeencleavedoff.Whichofthefollowingbestexplainswhythisisso?

A.Histidinewillpreferentiallyextractaprotonfromserinebeforeitwillactivatewater.B.Sterichindrancepreventedwaterfromperformingnucleophilicattack.C.Thecarbonylcarbonoftheamidewasinitiallynotsufficientlyelectrophilic.D.Thedeprotonatedserineisalesseffectiveleavinggroupthanthesubstratefragment.

3.Whycanasparticproteaseshydrolyzepeptidebondsinfewerstepsthanserineproteases?A.Asparticproteaseshydrogenbondwiththecarbonyloxygen,whileserineproteasesdonot.B.Theasparticacidinserineproteasesislocatedtoofarfromthepeptidebond.C.Histidineisnotastrongenoughbasetodeprotonatewater.D.Asparticproteasesareabletostabilizetetrahedralintermediates.

4.BasedonthemechanismillustratedinFigure2,whatmustbetrueoftheactivesiteofasparticproteases?

A.Itbindstohydrophobicproteinsmorereadilythantohydrophilicproteins.B.Itbindstobulkyproteinsmorereadilythantosmallerproteins.C.Itiskeptrelativelyacidic.D.Itiskeptrelativelybasic.

5.Whyiswaternecessarytocarryoutproteolysisinsteadofsimplyusinganaminoacidresiduealreadyfoundontheenzyme?

A.Waterisubiquitousandthereforemaximallyincreasesreactionkinetics.B.Waterisasmallmoleculethatcaneasilyperformnucleophilicattackwithoutsignificantsterichindrance.C.Watercanbeactivatedbybasicaminoacidresidues,makingitthemostnucleophilicentityinthemicroenvironmentoftheactivesite.D.Waterallowsfortherecyclingoftheenzymeinproteolysis.

6.Theaminoacidresiduesofserineproteases,Gly193andSer195,formanoxyanionhole,whichhydrogenbondstothesubstrate.Whenwilltheoxyanionholebeparticularlyimportant?

A.AfterthesubstratehasenteredtheactivesitebutbeforehydrolysishasbegunB.WhenserineperformsnucleophilicattackonthesubstrateC.WhilethesubstrateexhibitstetrahedralgeometryD.Whilewaterperformsnucleophilicattackonthesubstrate

Page 266: Edited By Deeangelee Pooran-Kublall, MD/MPH · 9.5 Preparing for the MCAT: Biochemistry in the Chemical and Physical Foundations of Biological Systems Section 9.6 Preparing for the

OrganicChemistryPassageIExplanation:USINGTHEKAPLANMETHODP1.Enzymefunctions:bondstrain

P2.Enzymefunctions:reduceentropy,acid/base,transientbonding

F1.Serineprotease–Pathway1

F2.Asparticprotease–Pathway2

KeyConceptsCarboxylicAcidDerivativemechanismofaction

1.HowdoesaspartatecontributetothefirststepofthemechanismillustratedinFigure1?A.Itdirectlyincreasesthenucleophilicityoftheserineresiduethroughdeprotonation.B.Itfunctionstomaintainanacidicenvironmentforoptimalenzymaticactivity.C.ItincreasesthepKaofhistidinethroughelectrostaticinteractions.

D.Itservestoincreasethebasicityofhistidinebyprotectingitsaromaticity.

AssessthequestionThequestionstem limits the scopeof thequestion toFigure1.Since thepassagedoesnotdirectlyaddresshowserineproteaseshydrolyzeproteins,itisnotnecessarytorefertothetext.Therefore,itisonlythemechanismthatmustbeanalyzed.

PlanyourattackInordertodeterminetheroleofaspartate,itisessentialtolookatboththereactantsandtheproductsfollowingthefirststep.Asisthecasewithmostorganicchemistryproblems,itisprudenttobeginbyidentifying relevant nucleophiles and electrophiles. To avoid being seduced by wrong answers,makingapredictionisagoodideahere.

Executetheplan

Page 267: Edited By Deeangelee Pooran-Kublall, MD/MPH · 9.5 Preparing for the MCAT: Biochemistry in the Chemical and Physical Foundations of Biological Systems Section 9.6 Preparing for the

Ataglance, it is clear that the serine isdeprotonatedand thenperformsnucleophilic attackon thecarbonyl carbon of the substrate. At the same time, histidine appears to be protonated but alsodeprotonated,andtheasparticacidresidueisprotonated.

Enzymes are highly specific, so the amino acid residues were not arranged as indicated bycoincidence.Eachresiduewillinteractwiththeadjacentresidue.Withthisunderstanding,itbecomesevident that aspartic acid must be stealing a proton from histidine. Removing the proton allowshistidine to undergo a rearrangement. One can infer that this rearrangement will help histidineactivate the serine residue through deprotonation. Because histidine is extracting a proton fromserine, it isbehavingas abase.Thus,put together, aspartic acidmustbe increasing thebasicityofhistidine.Nowitistimetolookattheanswerchoicestofindananswer.

Answerbymatching,eliminating,orguessingChoice(C)statesthatasparticacidincreasesthepKaofhistidine throughelectrostatic interactions.This is equivalent to stating that aspartic acid increases the basicity of histidine. As for theelectrostatic interactions,beforecompletedeprotonationoccurs,asparticacidhydrogenbondswithhistidine.Hence,choice(C)iscorrect.

Aspartic aciddoesnot deprotonate serinedirectly as stated in (A); it deprotonates histidine,whichdeprotonates serine. Histidine contains both protonated and deprotonated nitrogen atoms. If theenvironmentwereacidic,asdescribedin(B),thesenitrogenatomswouldbothbeprotonated.Choice(D)canbeeliminatedbecausealthoughasparticaciddoesincreasethebasicityofhistidine,itdoesnotdosobyprotectinghistidine’saromaticity.Thearomaticityofhistidine isnot in jeopardyand if itwere,itwouldnotreactregardlessofasparticacid’spresence.

KeyConceptsNucleophilicReactioninCarboxylicAcidDerivatives

2.InFigure1,activatedwatercanperformnucleophilicattackonlyafterhalfofthesubstratehasbeencleavedoff.Whichofthefollowingbestexplainswhythisisso?

A.Histidinewillpreferentiallyextractaprotonfromserinebeforeitwillactivatewater.B.Sterichindrancepreventedwaterfromperformingnucleophilicattack.C.Thecarbonylcarbonoftheamidewasinitiallynotsufficientlyelectrophilic.D.Thedeprotonatedserineisalesseffectiveleavinggroupthanthesubstratefragment.

Page 268: Edited By Deeangelee Pooran-Kublall, MD/MPH · 9.5 Preparing for the MCAT: Biochemistry in the Chemical and Physical Foundations of Biological Systems Section 9.6 Preparing for the

AssessthequestionThequestionstemspecificallyasksaboutFigure1.Itmightbeagoodideatoparaphrasethisone.Forinstance,thequestioncanbeparaphrasedas,“Whycan’twaterattackthecarbonylcarboninitially?”Thissimplifiesthequestionstemsothatitismoremanageable.

PlanyourattackRecognizingthedifferencebetweensubstratestructureoriginallyandsubstratestructureafterhalfofitiscleavedoffisessential.Activationofthewaterisirrelevant,sincewatercouldbeactivatedatanypoint in themechanism.Toavoidbeingseducedbywronganswers,makingaprediction isagoodidea.

ExecutetheplanForwater toperformnucleophilicattack, thecarbonylcarbonmustbesufficientlyelectrophilic.Atthebeginningofthemechanism,thecarbonylgroupispartofanamide,whilelaterit ispartofanester. Because amide groups have greater double-bond character, they aremore stable than esters.Therefore,waterwillnotbeabletoattacktheamide,butisabletoattacktheester.

Answerbymatching,eliminating,orguessingChoice(C)correctlyidentifiesthisreasoning.

There are a couple of ways water could be activated (aspartic acid could do this). Therefore,histidine’spreferredtargetdoesnotexplainwhyactivatedwatercannotperformnucleophilicattackuntilthesubstrateiscleavedinchoice(A).

Choice(B)canbeeliminatedbecausewaterisasmallmolecule,sosterichindrancewouldnotbeafactor here. Actually, in (D), the deprotonated serine is a better leaving group than the substratefragment.

KeyConceptsHydrolysis

3.Whycanasparticproteaseshydrolyzepeptidebondsinfewerstepsthanserineproteases?A.Asparticproteaseshydrogenbondwiththecarbonyloxygen,whileserineproteases

Page 269: Edited By Deeangelee Pooran-Kublall, MD/MPH · 9.5 Preparing for the MCAT: Biochemistry in the Chemical and Physical Foundations of Biological Systems Section 9.6 Preparing for the

donot.B.Theasparticacidinserineproteasesislocatedtoofarfromthepeptidebond.C.Histidineisnotastrongenoughbasetodeprotonatewater.D.Asparticproteasesareabletostabilizetetrahedralintermediates.

AssessthequestionThequestionstemasksforacomparisonbetweenasparticproteasesandserineproteases.Therefore,Figure 1 and Figure 2 must be compared. The written portion of the passage is not necessary toanswerthisquestion.

PlanyourattackThemaindifferencebetweenthemechanisminFigure1andthemechanisminFigure2iswhenwaterperformsnucleophilicattack.Therefore, todeterminetheanswer to thisquestion,onemustexplainthisdifference.

ExecutetheplanWater can perform nucleophilic attack earlier in the aspartic protease mechanism because thecarbonylcarbonisespeciallyelectrophilic.Theadditionalasparticacidresiduecanhydrogenbondwiththecarbonyloxygen.Asaresult,thecarbonyloxygenbecomesrelativelymoreelectronegative.Thecarbonyloxygenwillsapnegativechargefromthecarbonylcarbonsothatitbecomesrelativelyelectropositive.Watercanthenattackearlierwithasparticproteasesversusserineproteases.

Answerbymatching,eliminating,orguessingChoice(A)mustbecorrect.

Choice(B) is incorrectbecausetheenzymesarefoldedsuchthatallof theaminoacidresiduesarelocatedwithincloseproximityofeachother.Histidinedeprotonateswaterlaterinthemechanism,so(C) is not true. Both serine proteases and aspartic proteases are able to sufficiently stabilizetetrahedralintermediates.Ifthiswerenotthecase,asdescribedin(D),thentheywouldnotbeabletoeffectivelycatalyzeproteolysis.

Page 270: Edited By Deeangelee Pooran-Kublall, MD/MPH · 9.5 Preparing for the MCAT: Biochemistry in the Chemical and Physical Foundations of Biological Systems Section 9.6 Preparing for the

KeyConceptsAminoAcidChemistry

4.BasedonthemechanismillustratedinFigure2,whatmustbetrueoftheactivesiteofasparticproteases?

A.Itbindstohydrophobicproteinsmorereadilythantohydrophilicproteins.B.Itbindstobulkyproteinsmorereadilythantosmallerproteins.C.Itiskeptrelativelyacidic.D.Itiskeptrelativelybasic.

AssessthequestionThequestionstemasksforageneralizationabouttheasparticprotease’sactivesite.

PlanyourattackItisimportanttorecognizethatFigure2isonlyasnapshotofaportionoftheactivesiteofasparticproteases.Therefore,alimitednumberofconclusionscanbedrawn.

ExecutetheplanUpon analyzing themechanism inFigure2, it is apparent that one of the aspartic acid residues isprotonated. Since carboxylic acids are acidic and will lose a proton at a neutral pH, themicroenvironmentofasparticproteasesmustbeacidic.

Answerbymatching,eliminating,orguessingChoice(C)mustbecorrect.

TherearemanyotheraminoacidsthatarenotillustratedinFigure2thatarerelevant.Hence,choices(A)and (B) cannotbedrawnwith the information that is available. If themicroenvironmentof theactivesitewerebasic,asstatedin(D),thentheasparticacidresidueswouldnotbeprotonated.

Page 271: Edited By Deeangelee Pooran-Kublall, MD/MPH · 9.5 Preparing for the MCAT: Biochemistry in the Chemical and Physical Foundations of Biological Systems Section 9.6 Preparing for the

KeyConceptsUniquePropertiesofWater

5.Whyiswaternecessarytocarryoutproteolysisinsteadofsimplyusinganaminoacidresiduealreadyfoundontheenzyme?

A.Waterisubiquitousandthereforemaximallyincreasesreactionkinetics.B.Waterisasmallmoleculethatcaneasilyperformnucleophilicattackwithoutsignificantsterichindrance.C.Watercanbeactivatedbybasicaminoacidresiduesmakingitthemostnucleophilicentityinthemicroenvironmentoftheactivesite.D.Waterallowsfortherecyclingoftheenzymeinproteolysis.

AssessthequestionThequestionisaskingforanexplanationfortheneedofathird-partymolecule(specificallywater)forproteolysistooccur.

PlanyourattackThescopeofthisquestionisnotlimitedtoFigures1and2,butisextendedtoincludeallproteases.Analyzingwater ’s role inproteolysiswithasparticandserineproteaseswill stillbehelpful. Itwillalsobeusefultorecallbasicknowledgeofenzymes.

ExecutetheplanIftheaminoacidresiduesofanenzymeweretoattackthesubstrate,thenalittlepieceoftheenzymewould be lost during each reaction. Since an enzyme must be regenerated over the course of areaction,thiscannotoccur.Therefore,waterisnecessaryforproteolysis.

Answerbymatching,eliminating,orguessingChoice(D)mustbecorrect.

Wateriseverywhereandthusisasolidoptionforenzymecatalysis.However,water ’sviabilityasanoption in (A)doesnotexplainwhya third-partymolecule isessential.Therearemanyaminoacid

Page 272: Edited By Deeangelee Pooran-Kublall, MD/MPH · 9.5 Preparing for the MCAT: Biochemistry in the Chemical and Physical Foundations of Biological Systems Section 9.6 Preparing for the

residueswithsidechainsthatarepotentiallymuchmorenucleophilicandwouldnotbedramaticallyimpactedbysterichindrance.Thus,(B)cannotbecorrect.Waterisdefinitelynotnecessarilythemostnucleophilic entity in the microenvironment of the active site, as stated by (C), even after beingactivated.

KeyConceptsCarboxylicAcidDerivatives

6.Theaminoacidresiduesofserineproteases,Gly193andSer195,formanoxyanionhole,whichhydrogenbondstothesubstrate.Whenwilltheoxyanionhole,beparticularlyimportant?

A.Afterthesubstratehasenteredtheactivesite,butbeforehydrolysishasbegunB.WhenserineperformsnucleophilicattackonthesubstrateC.WhilethesubstrateexhibitstetrahedralgeometryD.Whilewaterperformsnucleophilicattackonthesubstrate

AssessthequestionThequestionstemmentionsserineproteases,soFigure1iskeyhere.

PlanyourattackHydrogenbondingconfersstability.Therefore,theanswerwillbethestepinwhichstabilityismostneeded.

ExecutetheplanThemostunstablesteps illustrated inFigure1are those that include intermediates. InFigure1, theintermediates all exhibit tetrahedral geometry.Anotherway to approach this problem is to use theinformationinthepassage.Thepassageexplainsthatenzymescatalyzereactionsbyinducingstraininthe substrate by favoring the transition states. “Favoring the transition” essentially means that theenzymesconferstabilitytothem.

Answerbymatching,eliminating,orguessing

Page 273: Edited By Deeangelee Pooran-Kublall, MD/MPH · 9.5 Preparing for the MCAT: Biochemistry in the Chemical and Physical Foundations of Biological Systems Section 9.6 Preparing for the

Choice(C)mustbecorrect.

If the substratewere stabilized toomuch, then the reactionwouldnot proceed forward. In (A), thesubstrate isheld inplaceby the enzyme,butnotmuchmore than that.Theaminoacid residuesofserineproteasesworktogether(insomesensestabilizeeachother)tofacilitatenucleophilicattackonthesubstrate,asdescribedin(B).However,thereisnoneedtoprovideextrastabilitytothesubstrate.(D) can be eliminated because the substrate is relatively stable at this point. It is not carrying anychargesandwouldnotrequireadditionalstability.

ORGANICCHEMISTRYPASSAGEII:ALDEHYDESANDKETONESAmphetamines are a class of potent nervous system stimulants popularized as performance andcognitive enhancers, and in some cases, used recreationally as aphrodisiacs or euphoriants.Amphetamines diffuse across the blood–brain barrier, as well as the placental barrier, makingunwanted side-effects a serious concern. In order tomitigate adverse effects, medical dosage andavailabilityisstrictlycontrolled.Whiledebatepersistsoveramphetamine’sneurotoxicityinhumans,there appears to be evidence that amphetaminemetabolism increases the concentration of reactiveoxygenspecies(ROS).

Amphetamines are legally and illicitly synthesized in a variety of ways. Figure 1 outlines theproductionofracemicamphetaminesviatwodifferentintermediates.

Figure1.Synthesisofmethamphetamineviaarrangement

Becausethebiologicalactivityofamphetaminesisstereospecificandamphetamineisoftenproducedasaracemicmixture,itisgenerallynecessarytoisolatethebiologicallyactiveenantiomer.Figure2outlines the resolution of racemic amphetamine via hot, basic tartaric acid. In addition, Figure 2showsastereospecificsyntheticpathway.

Page 274: Edited By Deeangelee Pooran-Kublall, MD/MPH · 9.5 Preparing for the MCAT: Biochemistry in the Chemical and Physical Foundations of Biological Systems Section 9.6 Preparing for the

Figure2.StereoselectiveproductionofD-amphetamine

Amphetamines have biological analogs termed trace amines. Trace amines are structurally andmetabolicallyrelatedtotraditionalmonoamineneurotransmitterslikedopamineandnorepinephrineandaresonamedbecausetheyareonlyfoundintraceamounts.

P1.

Fig1.

P2.

Fig2.

P3.

1.Epinephrine,shownbelow,isnotpermeabletotheblood–brainbarrier.Whichofthefollowingbestexplainswhyamphetaminesarepermeableandepinephrineisnot?

A.Amphetaminesaremostlylipidsolubleduetothearomaticringandrelativelackofpolarproticgroups.B.Themethylgroupofamphetaminesprovidesstereospecificityfortraceaminereceptors.C.ROSspeciesproducedbyamphetaminesfacilitatemembranetransfer.D.Thepolarityofthehydroxylgroupsreducesaffinityforplasma-membranetransporters.

2.Whatisthemostlikelyreasonwhymethamphetamine(asecondaryamineandamphetamineanalog)canelicitsympatheticresponsessimilartoepinephrinedespitedifferencesintheirchemicalmakeup?

A.Metabolitesofmethamphetaminestructurallyresembleepinephrinereceptors.

Page 275: Edited By Deeangelee Pooran-Kublall, MD/MPH · 9.5 Preparing for the MCAT: Biochemistry in the Chemical and Physical Foundations of Biological Systems Section 9.6 Preparing for the

B.Methamphetamineinhibitsepinephrinereuptakemechanisms.C.Methamphetamineisaconfigurationalisomerofepinephrineandreactssimilarly.D.Epinephrinereceptorshaveahighaffinityforthebenzeneringofbothmolecules.

3.Whatistheprimarydifficultyinsyntheticallyproducingtraceaminecompoundsfromketoneintermediates?

A.Carbonylsarepronetoring-closingmechanismsandthuscreateintermediatesunfavorabletoamination.B.Carbonylcompounds,particularlyketones,arerelativelyunreactivetoamines.C.Thereareanumberofsideproductspossiblefromcarbonylcompounds.D.Additiontothecarbonylcarbonisreversibleandsynthesisissubjecttoequilibriumconstraints.

4.Astudentwishingtoresolveopticallypureamphetamineusesmesotartaricacid,adiastereomerofD-tartaricacid.Isthestudentlikelytobesuccessful?

A.Yes,becausemesotartaricacidwillcombinewitheachoftheenantiomerstoformapairofdiastereomers,whichcanbeseparatedbyphysicalmeans.B.Yes,becausetheenantiomerofD-tartaricacidwillreactwiththeracematestoformdistinctsaltsthatcanbeseparatedbyphysicalmeans.C.No,becauseD-tartaricacidisnecessarytoformapairofdiastereomersthatcanbeseparated.D.No,becausemesotartaricacidwillreactwiththemixturetoformdistinctsaltsthataremirrorimages.

5.WhichofthefollowingstatementsbestdescribestherearrangementsinFigure1?A.IntheHofmannrearrangement,primaryamidesareconvertedtoderivativesbytheactionofhalohydroxidesorhalogensinalkalinesolution.Excessbasegeneratesaconjugateacidoftheproduct.B.IntheCurtiusrearrangement,anacylazideispreparedbyreactionofanacylchloridewithdiazoniumfollowedbytreatmentwithcoldnitrousacid.Subsequentheatingresultsindecomposition.C.BoththeHofmannandCurtiusrearrangementsinvolveacylnitrenesthatquicklyrearrangetoisocyanateisomers,whichareisolatedorreactedinacidicsolvents.D.BoththeHofmannandCurtiusrearrangementsinvolvetheadditionofwatertoisocyanatesinordertoproduceanunstablecarbamicacidthatdecomposestoanamineandcarbondioxide.

Page 276: Edited By Deeangelee Pooran-Kublall, MD/MPH · 9.5 Preparing for the MCAT: Biochemistry in the Chemical and Physical Foundations of Biological Systems Section 9.6 Preparing for the

OrganicChemistryPassageIIExplanation:USINGTHEKAPLANMETHODSP1.Amphetamineuses,permeabilityandtoxicity

Fig1.Syntheticpathwaysviarearrangements(Hoff/Curtius)

P2.Biologicalactivityisstereospecific

Fig2.Twopathwaysforopticallypureamphetamine

P3.Traceaminesdescribed,examples

KeyConceptsWhenpresentedwithcomplexmoleculesorreactions,lookforrecognizablefunctionalgroupsorreactionconditions.

1.Epinephrine,shownbelow,isnotpermeabletotheblood–brainbarrier.Whichofthefollowingbestexplainswhyamphetaminesarepermeableandepinephrineisnot?

A.Amphetaminesaremostlylipidsolubleduetothearomaticringandrelativelackofpolarproticgroups.B.Themethylgroupofamphetaminesprovidesstereospecificityfortraceaminereceptors.C.ROSspeciesproducedbyamphetaminesfacilitatemembranetransfer.D.Thepolarityofthehydroxylgroupsreducesaffinityforplasma-membranetransporters.

AssessthequestionChemicalstructureanalysisisrequiredtodeterminetheanswer,alongwithabasicunderstandingofpermeability.

Page 277: Edited By Deeangelee Pooran-Kublall, MD/MPH · 9.5 Preparing for the MCAT: Biochemistry in the Chemical and Physical Foundations of Biological Systems Section 9.6 Preparing for the

PlanyourattackInordertoanswerthisquestion,itisnecessarytofocusonthedifferencesbetweenthetwomolecules.Thosedifferencescombinedwithnewinformationfromthepassageandrequiredoutsideknowledgewillbeenoughtofindthecorrectanswer.Epinephrineisstructurallysimilartoamphetamines,withtheadditionofseveralhydroxylgroups.Lookforthistoshowupinthecorrectanswer.

ExecutetheplanParagraph1indicatesthatamphetaminescandiffuseacrosstheblood–brainbarrier.Theblood–brainbarrier is primarily lipid or small non-polar molecule soluble. Thus, it is possible that thehydrocarbon structure, including the aromatic ring, may allow the molecule to diffuse, while thepolar,hydroxylgroupsinepinephrinepreventitsdiffusion.

Choice(A)addressesbothcomponentsofthepredictionandisastronganswer.

Choice(B)isperhapstrue,butitdoesnotaddresswhyonemoleculewouldbepermeableandanotherwouldnot—soitcannotbethecorrectanswer(tothisquestion).

Choice(C)recallsanotherfactfromthepassage,however;eventhoughreactiveoxygenspeciesmaydamage cells and thereby compromise the integrity of the blood–brain barrier, it is not the ROSspeciesthatfacilitatesmembranetransfer,itisdiffusion—whichisaconsequenceofitsstructure.

Choice (D) addresses a key difference between the two molecules (the hydroxyl groups), but itneglectsthefactthatamphetaminesdiffuseacrossmembranesandthusdonotrequiretransporters.

Answerbymatching,eliminating,orguessingLookingforananswerthataddressesthekeydifferencesbetweenthemoleculesledto(A)and (D).Aftercarefulconsiderationof thepassage information, inconjunctionwithoutsideknowledge, it’sclearthat(A)isthebestanswer.

TakeawaysWhenaskedtoexplainadifferenceinfunction,lookforadifferenceinstructure.

ThingstoWatchOutForAnswerchoicesthatbringupunrelatedfactsaboutonemoleculeoranother.

Page 278: Edited By Deeangelee Pooran-Kublall, MD/MPH · 9.5 Preparing for the MCAT: Biochemistry in the Chemical and Physical Foundations of Biological Systems Section 9.6 Preparing for the

KeyConceptsFormdictatesfunction.

2.Whatisthemostlikelyreasonwhymethamphetamine(asecondaryamineandamphetamineanalog)canelicitsympatheticresponsessimilartoepinephrinedespitedifferencesintheirchemicalmakeup?

A.Metabolitesofmethamphetaminestructurallyresembleepinephrinereceptors.B.Methamphetamineinhibitsepinephrinereuptakemechanisms.C.Methamphetamineisaconfigurationalisomerofepinephrineandreactssimilarly.D.Epinephrinereceptorshaveahighaffinityforthebenzeneringofbothmolecules.

AssessthequestionMuch like the previous question, in order to answer question 2 it will be necessary to explain anobservation about twomolecules. Unlike its predecessor, the correct answer to this questionmustaddressasimilarityinfunctiondespiteadifferenceinform.

PlanyourattackIt’sbesttomakeafewpredictionsandthenusetheprocessofelimination.Thereareafewnotabledifferencesbetweenthetwomoleculesandonenotablesimilarity.Thesimilarityisagoodplacetostart.Inordertoelicitaresponsefromthesympatheticnervoussystem,theremustbestimulationoftheappropriatereceptors.Whileit’suncertainhowmethamphetamineworks,it’slikelyoneofafewcommonmechanisms.Themoleculecouldmimicepinephrineandbind toepinephrinereceptors. Itcould also prevent the degradation or reuptake of epinephrine. There are other, more obscuremechanisms,buttheirvalidityshouldbeaddressedonanindividualbasisusingpassageinformation,outsideknowledge,andlogic.

ExecutetheplanChoice(A)maybetempting,becauseitprovidesapossibleexplanation(thatmethamphetaminedoesnotitselfactivateepinephrinereceptors),whichtakesintoaccountstructuraldifferences;however,thelastportionofthisanswerisnotplausible.Thecorrectanswerwouldindicatethatthemetabolitesarestructurallysimilartoepinephrinenottheepinephrinereceptor.

Choice(B)matchesalikelymechanism.

Page 279: Edited By Deeangelee Pooran-Kublall, MD/MPH · 9.5 Preparing for the MCAT: Biochemistry in the Chemical and Physical Foundations of Biological Systems Section 9.6 Preparing for the

Choice (C) states that methamphetamine and epinephrine are configurational isomers—this isincorrect. Furthermore, configurational isomers can react very differently, so this is not the bestanswer.

Choice(D) provides apossible explanation for the similarity; it focuseson a commonelementofboth molecules. However, given the stereospecificity of amphetamine receptors and the fact thatmethamphetamineisananalog,it’sunlikelythattheachiralbenzeneringissufficienttostimulateasympatheticresponse.

Answerbymatching,eliminating,orguessingExecutingtheplanledtotheeliminationof(A),(C),and(D),indicatingthat(B)isthecorrectanswer.

TakeawaysConsiderwhatneedstobefoundinthecorrectanswer,butremainopentounconsideredexplanationsthatareinlinewithpassageinformationandoutsideknowledge.

ThingstoWatchOutForCommonly,wronganswerswilladdressthequestionbutcontradictinformationfromthepassage.

KeyConceptsTheMCATwillblurthelinesbetweenorganicchemistry,generalchemistry,biochemistry,andbiology.

3.Whatistheprimarydifficultyinsyntheticallyproducingtraceaminecompoundsfromketoneintermediates?

A.Carbonylsarepronetoring-closingmechanismsandthuscreateintermediatesunfavorabletoamination.B.Carbonylcompounds,particularlyketones,arerelativelyunreactivetoamines.C.Thereareanumberofsideproductspossiblefromcarbonylcompounds.D.Additiontothecarbonylcarbonisreversibleandsynthesisissubjecttoequilibriumconstraints.

Page 280: Edited By Deeangelee Pooran-Kublall, MD/MPH · 9.5 Preparing for the MCAT: Biochemistry in the Chemical and Physical Foundations of Biological Systems Section 9.6 Preparing for the

AssessthequestionThisquestionrequeststheprimarydifficultyexperiencedduringasynthesis.

PlanyourattackIt’s possible tomake some predictions based on the compounds in question, but ultimately it willrequireeliminatingthoseanswersthatarelessproblematic.

ExecutetheplanCarbonylgroupsundergoawidevarietyofreactions—apreliminaryprediction.Further,manyoftheintermediatesarealsoreactive.Thereisalsoanexampleofamphetamineproductionfromaketone(thesecondreactioninFigure2)thatcanbeusedforreference.Inthereaction,theamineattacksthecarbonylcarbon,forminganalcoholintermediate.Thereisthendehydrationofthealcoholtoformtheimineintermediate.

Choice (A) may seem reasonable, but it does not even mention ketones and does not answer thequestion.

Choice (B) indicates that carbonyl groups are unreactive, which is untrue—carbonyl groups,includingketones,undergoawidevarietyofreactions.

Choice(C)addressesafundamentalproblemwhenworkingwithreactivespecies.

Choice (D) must be eliminated because (while true) equilibrium reactions can be coerced viaapplicationofLeChâtelier ’sprinciple.

Answerbymatching,eliminating,orguessingSuchaquestionrequireselimination.Thoroughreasoningwilldictatethatthebestofthesechoicesischoice(C). In fact, synthetic routes foralmostallpharmaceuticalssuffer fromsomeformofyieldissuesbasedonchemicalstructure.

TakeawaysWhenpresentedwithmultipleanswerchoiceswithfactualinformation,eliminatethosethatareleastrelevanttothequestion.

Page 281: Edited By Deeangelee Pooran-Kublall, MD/MPH · 9.5 Preparing for the MCAT: Biochemistry in the Chemical and Physical Foundations of Biological Systems Section 9.6 Preparing for the

ThingstoWatchOutForWhenaskedforaprimarydifficulty,allanswerchoicesshouldbeconsideredbeforemakingaselection.

KeyConceptsThecarbonylisafavoritereactionpointontheMCAT;befamiliarwithitscommonmechanisms.

4.Astudentwishingtoresolveopticallypureamphetamineusesmesotartaricacid,adiastereomerofD-tartaricacid.Isthestudentlikelytobesuccessful?

A.Yes,becausemesotartaricacidwillcombinewitheachoftheenantiomerstoformapairofdiastereomers,whichcanbeseparatedbyphysicalmeans.B.Yes,becausetheenantiomerofD-tartaricacidwillreactwiththeracematestoformdistinctsaltsthatcanbeseparatedbyphysicalmeans.C.No,becauseD-tartaricacidisnecessarytoformapairofdiastereomersthatcanbeseparated.D.No,becausemesotartaricacidwillreactwiththemixturetoformdistinctsaltsthataremirrorimages.

AssessthequestionThis question asks about the reasoning behind the experimental design. The answers are yes/nofollowedbyabriefexplanation.

PlanyourattackIt’s best to approach the answerswith a prediction inmind.Once a prediction ismade, attack theanswer. The correct answer will not only answer the question, but also have sound reasoning. Ifunsureaboutthecontent,focusonthereasoningwithintheanswer.

Inorder to resolvea racemicmixture intoopticallypureenantiomers, themixturemustbe reactedwithanotheropticallyactivereagent.Commonly,acid/basepropertiesareexploitedtoformsalts.Thesalts thus formed will be diastereomers of each other. Diastereomers have different physicalpropertiesandcanbeseparatedbyphysicalmeans(crystallizationiscommonwithamines).

Mesotartaric acid is a meso compound and as such is optically inactive. Therefore, it will beincapableofresolvingaracemicmixture.It’snowtimetofindtheanswerthatmatchestheprediction,albeitinotherwords.

Page 282: Edited By Deeangelee Pooran-Kublall, MD/MPH · 9.5 Preparing for the MCAT: Biochemistry in the Chemical and Physical Foundations of Biological Systems Section 9.6 Preparing for the

ExecutetheplanChoices(A)and(B)canbeeliminatedbecausetheresolutionwillnotwork.Further,(B)claimsthattheenantiomerofD-tartaricacidreacts,whenthequestionstemclearlyidentifiesmesotartaricacidasadiastereomer(thetwotermsaremutuallyexclusive).

Choice(C)hasthefirstpartcorrect,however,itdistortsthesecondhalf.D-tartaricacidisuseful,butitisnotnecessary.Theremanyotheropticallyactiveresolvingagentsthatcouldbeused,includingL-tartaricacid.

Choice(D)mustbecorrectandmatchestheprediction.Distinctmoleculesthataremirrorimagesareenantiomers, which is what would result from a reaction of a racemic mixture and an opticallyinactive reagent. Enantiomers cannot be separated by physical means, so the student’s procedurewouldnotwork.

Answerbymatching,eliminating,orguessingExecutingtheplanledtotheeliminationof(A),(B),and(C),indicatingthat(D)iscorrect.

TakeawaysInordertoresolvearacemicmixture,anopticallyactivereagentmustbeused.

ThingstoWatchOutForExpecttoseethesameconceptmentionedindifferentterms;thetestwriterscommonlyemploysynonyms.

KeyConceptsDon’tbeafraidtomakeaquicksketch;oftentimesasketchwillelucidatethedifferencebetweenareasonablemechanism/productandanunreasonableone.

5.WhichofthefollowingstatementsbestdescribestherearrangementsinFigure1?A.IntheHofmannrearrangement,primaryamidesareconvertedtoderivativesbytheactionofhalohydroxidesorhalogensinalkalinesolution.Excessbasegeneratesaconjugateacidoftheproduct.B.IntheCurtiusrearrangement,anacylazideispreparedbyreactionofanacylchloridewithdiazoniumfollowedbytreatmentwithcoldnitrousacid.Subsequentheatingresultsindecomposition.

Page 283: Edited By Deeangelee Pooran-Kublall, MD/MPH · 9.5 Preparing for the MCAT: Biochemistry in the Chemical and Physical Foundations of Biological Systems Section 9.6 Preparing for the

C.BoththeHofmannandCurtiusrearrangementsinvolveacylnitrenesthatquicklyrearrangetoisocyanateisomers,whichareisolatedorreactedinacidicsolvents.D.BoththeHofmannandCurtiusrearrangementsinvolvetheadditionofwatertoisocyanatesinordertoproduceanunstablecarbamicacidthatdecomposestoanamineandcarbondioxide.

AssessthequestionThequestioncontains functionalgroups that arenot requiredknowledge for theMCAT. If there iscontentthatappearstobeyondthescopeoftheMCAT,remindyourselfthattheMCATisareasoningtestandwilloftenrequirethehigh-scoringtest-takertomakedeductions.

PlanyourattackThis question will require careful analysis of the answer choices. Follow along and look forobviouslywrongstatementsandeliminate.

ExecutetheplanStartingwithchoice(A)andfollowingthenarrative,everythingisconsistentexceptthelaststatementthattheconjugateacidisproduced—eliminate.

Choice (B) requires careful analysis and deduction regarding the Curtius pathway. The answermentionsadiazoniumwheninfactthereactantisanazide.Thiscanbededucedeitherbythenamesofthereactants(di-azoniumcompoundscontainthe functionalgroup)orbylookingatthenameoftheproduct(anazide).

Choice(C)canbeeliminatedbecausetheHofmannrearrangementinvolvesabasicworkup,sowhiletheworkup for theCurtius rearrangement isunknown, the fact that thisanswer saysboth,makes itincorrect.

Atthispoint,theprocessofeliminationshowsthatthecorrectresponseis(D).Thereisnoevidencethat would directly refute this answer and itmust therefore be chosen. Furthermore, based on thereactioninFigure1,theprocessdescribedin(D)seemsplausible.

Answerbymatching,eliminating,orguessingExecuting the plan led to the elimination of (A), (B), and (C) in sequence; therefore the correct

Page 284: Edited By Deeangelee Pooran-Kublall, MD/MPH · 9.5 Preparing for the MCAT: Biochemistry in the Chemical and Physical Foundations of Biological Systems Section 9.6 Preparing for the

answermustbe(D).

TakeawaysHighdifficultyproblemsmayrequireonetodeduceunstatedstepsofareaction.

ThingstoWatchOutForRefertothepassagetodouble-checkforconsistency.

KeyConceptsMechanismsareviabletopicsforanalysisontheMCAT.Besuretounderstandthereactantsandproductsofeveryreaction,butalsoextendyouranalysistosidereactionsthattheintermediatesmaybeinvolvedin.

Page 285: Edited By Deeangelee Pooran-Kublall, MD/MPH · 9.5 Preparing for the MCAT: Biochemistry in the Chemical and Physical Foundations of Biological Systems Section 9.6 Preparing for the

12.7OrganicChemistryPracticeORGANICCHEMISTRYPASSAGEIII(QUESTIONS1–6)Humanscanonlysynthesize11of theproteogenicaminoacids.Nineothersareknownasessentialamino acids and must be supplied via diet—although some essential amino acids may beinterconverted(thesulfur-containingaminoacidsareinterchangeableinthebody,asarethearomaticaminoacids).

De novo synthesis of amino acids usually starts with the non-essential amino acid glutamate (theconjugatebaseofglutamicacid).Glutamateisformedfromthemoleculeα-ketoglutarate,aproductof the Krebs cycle. In amino acid synthesis, α-ketoglutarate is aminated by ammonium to formglutamate.Glutamate can then be used to transaminate a number of different precursors into theirrespective amino acids. The transamination converts glutamate to α-ketoglutarate. For example,pyruvate,showninFigure1,canbeaminatedbyglutamatetoformalanine.

Amino acid synthesis in the lab follows a variety of other pathways using molecules not usuallyfoundinthehumanbody.Streckersynthesisstartswithacarefullychosenaldehyde.Thealdehydeisreactedwithammoniumionsleadingtoaniminiumintermediate.Theiminiumintermediate is thenattackedbyacyanideionthatformsanaminonitrile.Subsequently,thisaminonitrileisconvertedtoacarboxylic acid by the addition of water and acid, proceeding through a 1,2-diamino diolintermediate.

Aminoacidshaveunique isoelectricpoints (pI),apHwhere theaminoacidwillhaveanetneutralcharge.ThepIisdeterminedbythepKasofthevariousfunctionalgroups.Theisoelectricpointcanbefoundforindividualaminoacidsorforapolypeptidechain.Inthepolypeptidechain,mostofthecarboxylic acid and amino groups are bound and thus have no charge. Therefore the charge, andsubsequently,thepI,isinfluencedmostsignificantlybythesidechainsinthepolypeptide.

P1.

P2.

F1.

P3.

P4.

1.WhichofthefollowingaminoacidswillbenegativelychargedatphysiologicalpH?A.Glutamicacid

Page 286: Edited By Deeangelee Pooran-Kublall, MD/MPH · 9.5 Preparing for the MCAT: Biochemistry in the Chemical and Physical Foundations of Biological Systems Section 9.6 Preparing for the

B.ArginineC.ValineD.Phenylalanine

2.ThefigurebelowshowsanionizedformoftyrosineanditspKavalues.Basedonthisinformation,whatistyrosine’spI?

A.5.64B.6.17C.9.04D.9.57

3.WhichofthefollowingisasignificantdisadvantageofusingStreckeraminoacidsynthesistocreateaminoacidsforthebody?

A.Theammoniumioncausesthereactiontoproceedtooquicklytocontrol.B.Thenucleophileusedcanalsoattacksidechainswithcarbonyls.C.Thenucleophilicattackonthecarbonylcausesracemization.D.Allaminoacidsformedfromthissynthesisareuselessbiologically.

4.Giventhestructureofl-alanineandα-ketoglutaricacid,whatisthestructureofL-glutamicacid?

A.

B.

Page 287: Edited By Deeangelee Pooran-Kublall, MD/MPH · 9.5 Preparing for the MCAT: Biochemistry in the Chemical and Physical Foundations of Biological Systems Section 9.6 Preparing for the

C.

D.

5.Supposeaportionofapeptidechaincontainsalargeamountofphenylalanine,alanine,andvalineresidues.Ifthepeptideispartofanenzymethatisdissolvedinthecytoplasm,whereontheenzymeisthisregionlikelytobelocated?

A.IntheactivesiteoftheenzymeB.IntheallostericsiteoftheenzymeC.IntheinterioroftheenzymeD.Ontheexterioroftheenzyme

6.Whichofthefollowingsetupswouldbemostappropriateforisoelectricfocusingofproteinmolecules?

A.apHgradient(0–14)fromlefttoright,withapositivechargeontheleftandanegativechargeontherightB.apHgradient(0–14)fromrighttoleft,withapositivechargeontheleftandanegativechargeontherightC.apHgradient(0–14)frombottomtotop,withapositivechargeontherightandanegativechargeontheleftD.apHgradient(0–14)fromtoptobottom,withapositivechargeontherightandanegativechargeontheleft

OrganicChemistryPracticePassageExplanationsP1.Essentialaminoacids

P2.Processofdenovosynthesis

F1.Pyruvatestructure

P3.Streckersynthesis

P4.RoleofpI

1.(A)

AtphysiologicpH,thecarboxylicacidandtheaminogroupofanaminoacidhaveanegativechargeand positive charge, respectively. This implies that if a molecule is to be negatively charged at

Page 288: Edited By Deeangelee Pooran-Kublall, MD/MPH · 9.5 Preparing for the MCAT: Biochemistry in the Chemical and Physical Foundations of Biological Systems Section 9.6 Preparing for the

physiologicalpH,thenthesidechainmustcarryanegativecharge.Basedonthisprediction,lookforanaminoacidwithanacidicsidechain.Amatchisfoundwithan“acid”inchoice(A).

2.(A)

Whencalculating thepI for anaminoacid, the sidechainmustbeconsidered. In this case the sidechainisarelativelyunreactivephenolgroup—whichwillremainunchargeduntilitdonatesaproton(nearapHequaltoitspKa).AtalowpHtheaminegroupwillbeprotonatedandthecarboxylicacidgroupwillbeneutral(asshown).AtapHequaltothepKaofthecarboxylicacid,approximatelyhalfof thecarboxylicacidswillbedeprotonatedandcarryanegativecharge.Thismakes themoleculeneutralasawhole.AsthepHnearsthepKaoftheprotonatedaminogroup,themoleculewillbecomenegatively charged. The pH between these two pKas is the pI and is calculated as the average.

orchoice(A).

3.(C)

When the ammonium attacks the aldehyde, the carbonyl carbon is sp2 hybridized. This means theelectrophileisplanarandthenucleophilecanattackfromeitherthetoporthebottom.Thisimpliesthat therewill be a racemicmixtureof amino acids (for all amino acids except glycine).D-aminoacidsarenotusefulbiologicallyaspracticallyall aminoacids in thebodyareof theL form.Thismeans that approximately half of the amino acids producedwill not be useful, thus potentiating adisadvantagetoStreckersynthesis,orchoice(C).

4.(B)

L-amino acids are of the S configuration (according to Cahn–Ingold–Prelog rules) except forcysteine and glycine. The correct form of glutamic acidwill be similar in chirality to the alanineshown in thequestion.Thismeans that the configurationat theα-carbon shouldbeS.Thepassageprovides a few clues that can elucidate themolecular formula for glutamic acid; it’s stated in thesecondparagraphthatpyruvatecanbeaminatedbyglutamatetoformalanineandthatintheprocessglutamate is deaminated to α-ketoglutarate. (pyruvate + glutamate ↔ alanine + α-ketoglutarate).Therefore,theremustbefivecarbonsinglutamateanditsconjugateacid,glutamicacid.Thisrulesoutchoice(C).Choice(D)canbeeliminatedbecause it representsglutaricacid,notglutamicacid.Betweentheremaininganswers,(B)iscorrect.

5.(C)

Thelocationofacertainsectionofapolypeptidechaindependsonthetypesofaminoacidscontainedin that chain. The chain in this question contains nonpolar amino acids, which are also known ashydrophobicaminoacids.Theydonot like tobearoundwaterandwillgroup together toavoid it.Thismeansthattheyareprobablynotlocatedonanypartoftheenzymethatisexposedtowaterandchoice(C)isthecorrectanswer.

6.(A)

Thisquestionisaskinghowisoelectricfocusingworksinagar.TheideabehindisoelectricfocusingisthatatacertainpHthemoleculesareneutralandatotherpHsthemoleculeshavecharges.Charged

Page 289: Edited By Deeangelee Pooran-Kublall, MD/MPH · 9.5 Preparing for the MCAT: Biochemistry in the Chemical and Physical Foundations of Biological Systems Section 9.6 Preparing for the

particleswillabidebyCoulomb’sLaw,experienceaforcefromotherchargesandaccelerate.Whenthere is a positive charge on one end and a negative charge on the other, positivemoleculeswillmigratetowardsthenegativechargeandnegativemoleculeswillmigratetowardsthepositivecharge.Molecules becomemore positive as the conditions becomemore acidic. This means the negativechargeshouldbeon theoppositesideof theacidicsideso thatwhen theaminoacid isacidic (andpositive), itwill travel towards thebasic side (thenegative side).As itmoves towards thenegativeside,thepHincreasesandthemoleculebeginstoloseitspositivecharge.Onceithaslostitscharge,itwillnolongerexperienceaforcefromotherchargesanditwillstop.Thismatcheschoice(A).*ThisconceptisonlytestedontheBiologicalandBiochemicalFoundationsofLivingSystemssection.**ThisconceptcanalsobetestedontheBiologicalandBiochemicalFoundationsofLivingSystemssection.*ThisconceptisonlytestedontheBiologicalandBiochemicalFoundationsofLivingSystemssection.**ThisconceptcanalsobetestedontheBiologicalandBiochemicalFoundationsofLivingSystemssection.

Page 290: Edited By Deeangelee Pooran-Kublall, MD/MPH · 9.5 Preparing for the MCAT: Biochemistry in the Chemical and Physical Foundations of Biological Systems Section 9.6 Preparing for the

CHAPTERTHIRTEEN

PhysicsMany students approach MCAT physics as a series of equations to memorize. However, MCATphysics is not just about the equations, because the MCAT doesn’t award points for the simplerecollectionofequations.Inactuality,successontheMCATwithrespecttophysicsrequiresapplyingtheequationstonovelsituations.AnotherimportantpointtonoteisthattheAAMChaseliminatedanyphysicsquestionontheexamthatisnotapplicabletolifesciences.Whatdoesthismeanforyou?Itmeans thatwhile youwill be spared having to answer questions about awatermelon shot out of acannon; youwill have to answer questions about laminar blood flowwithin the vasculature usingyourknowledgeofbasicprinciplessuchasfluiddynamics.

ItisnomysterythatphysicsisoneofthemostdreadedcontentareastestedontheMCAT.However,understanding the fundamental concepts and the ability to apply those concepts can separate theaverage test-taker from an elite test-taker.MCATphysics is not like an undergraduate course.TheMCATfocusesonconceptualunderstandingaswellastheabilitytochoosethecorrectmathematicalprocess.All of the questionswill involve living systems, andwill require outside knowledge.Thebetterpreparedyouareforwhatyouaregoingtosee,themoreconfidentyouwillfeelonTestDay.

In this chapter, wewill explore howMCAT tests physics andwhat you need to know in order tomaximizeyourscore.

13.1ReadingthePassageOneof theworst thingsyoucandoasa test-taker is toapproachaphysicspassagewithanattitudesuchas,“I’mgoingtoreadthisentirepassage,memorizingallofthedetailsanddatapointsasIgoalong,sothatIwon’tneedtowastetimereferringtothepassagewhileIanswerthequestions.”Thistypeofapproach results ina tremendousamountof time lost to reading thepassage.Therearenopointsforreadingandmemorizingthepassage.Inaddition,thequestionswilldictatewhatyouneedfromthepassage.Therewillalwaysbeinformationinthepassagethatappearstestable,butissimplynottested.RememberthattheMCATisaskingyoutoapplywhatyouknowtothetopicathand.Forsomequestions,thetopicofthepassagewon’tevenbeimportant;you’llsimplyneedtoapplywhatyouknow.

PASSAGETYPESTheMCATfeaturestwotypesofpassagesinthephysicssection.Identifyingthetypeofpassageyouarereadinghelpsyoutopredictwhatisgoingtobeimportantforthequestions.

Information passages describe natural or manmade phenomenon, much like a textbook. Thesepassages will often provide definitions of new terms. Test questions are likely to focus on theinformationpresented,andyouwillbeasked to interpret the information in thepassage in lightofwhatyoualreadyknowaboutthetopic.

Page 291: Edited By Deeangelee Pooran-Kublall, MD/MPH · 9.5 Preparing for the MCAT: Biochemistry in the Chemical and Physical Foundations of Biological Systems Section 9.6 Preparing for the

Thesecondtypeofphysicspassagedescribesanexperiment.Inexperimentpassages,theexperimentconductedusuallyhasacleargoal.Somethingisvaried,somethingelseismeasured,andconclusionscanbeformed.Atableorgraphwithdatafromtheexperimentmaybepresented,anditislikelythatyouwillbeaskedtointerpretthedata.Whenmultipleexperimentsareperformed,itisthesimilaritiesanddifferencesbetweentheexperimentsthatarelikelytobetested: ifmakingasmallchangetoanexperimentcreatesaradicallydifferentresult,youcanbetthattherewillbeaquestionthatrequiresyoutounderstandwhyasmallchangeresultedinadifferentoutcome.

As you prepare for the MCAT, remember that your skill at identifying and absorbing what isimportantwithinapassageandskimmingoverwhatisnotimportantwilldirectlytranslateintotimesavedandmorepointsonTestDay.Withthatinmind,elitetest-takersdotwothingswhenreadingaphysicspassage:outlinethepassageandidentifythetopic.

MCATPhysics–PassageTypes

Information

Experiment

Goal

Topresentinformation

Tosummarizeanexperimentperformed

Contents

Informationaboutsomephenomena,presentedinapredictableway,anewequation,considerabledetailthatmayormaynotbeimportant

Ahypothesis,aprocedure,data(oftenintheformofchartsand/ortables),anewequation.Someexperimentpassageswillconsistoftwoexperiments.

HowtoReadIt

Quickly,identifywherethedetailsarelocated,butnoneedtomemorize.Getthegistofeachparagraph,andmoveontothenext.

Payattentiontothehypothesisbehindtheexperiment,theprocedure,andtheoutcome.Iftwoexperimentsareconductedwithdrasticallydifferentresults,payattentiontothedifferencesbetweentheexperiments.

Similarto:

Textbook,journalarticle

Labreport

OUTLININGTHEPASSAGEAs we have already discussed, memorizing the passage is not going to help you maximize yourpoints onTestDay.However, there is anotherway tomake sure that you are able to quickly findrequiredinformationwithinthepassage.Thisisknownasoutliningthepassage.Byemployingyourcriticalreadingskills,youshouldunderstandthegistofeachparagraph.Thebestwayofdoingthisisby answering this question after each paragraph, “What is this paragraph doing?Why is it here?”

Page 292: Edited By Deeangelee Pooran-Kublall, MD/MPH · 9.5 Preparing for the MCAT: Biochemistry in the Chemical and Physical Foundations of Biological Systems Section 9.6 Preparing for the

Remember,theMCATisnotahaphazardsetofpassagesandquestionssloppilythrowntogether.Thisisa test thathasbeencraftedby theAAMCasanaptitude test formedicalschool.Eachparagraph,eachword,andeachanswerchoicehasbeenplacedthereforareason.

ScanforStructureInthisstep,takeabrieflookatthevisualaspectsofthepassage.Isthepassagewordy?Doesitcontainanequationoradiagram?Aretherechartsandgraphspresent?Afteridentifyingthevisualelements,determine a degree of difficulty.The degree of difficultywill vary depending on the test-taker, aseachindividualhashisownstrengthsandweaknesses.Now,itistimeto“triage”thepassage.Triageisaway todeterminewhichpassages receivepriority.TheMCATdoesnotawardmorepoints forcorrectanswersonmoredifficultpassages.Sinceeachquestion isworth the same, thebestway tomaximizeyourpointsistodotheeasypassagesfirst.Thisallowsyoutotakecontrolofthetestandmakethebestofyourownstrengths.Ifyoudeterminethatapassageisfairlyeasyforyoutodo,thendoit.Ifit’smoredifficultorappearstobeverytime-consuming,doitlater.

ReadStrategicallyAsyoustarttoreadthepassage,itwillbecomeveryclearwhattypeofpassageyouarereading.Anexperiment passage will describe an experiment, while an information passage will presentinformation. Using this knowledge, you can adapt your critical reading skills such that you areactivelypullingouttheinformationthatislikelytobeimportantforthatpassagetype.Questionsonexperimentpassageswillfocusonthehypothesistested,thedetailsoftheexperimentprocedure,theoutcome of the experiment, and data analysis.As you read an experiment passage, paying specialattentiontotheseelementsofthepassagehelpsyoutopickoutwhatisimportant,andskimthroughwhatislesslikelytobetested.

Onaninformationpassage,themostimportantthingtopulloutofeachparagraphisthepurposeoftheparagraph.Readeachparagraphasthoughitwereastand-aloneparagraph,focusingonmeaningandthepurposeofthatparagraphwithinthepassageasawhole.Inphysics,thepassageislikelytofocusonatheoryordescriptionofnaturalphenomenathatisrelatedtowhatwaspresentedinyourphysics courses. Itwill likelybeunderstandable, but therewill be a tremendousnumberofdetails.While it is important foryou tounderstand thosedetails, donotwaste time trying to analyzeeachdetailandhowtheinformationfitsintothebigpicture.Simplynoticethatthedetailisthereandmoveon.Youonlyhavetounderstandenoughofthepassagetoanswerthequestions.

LabelEachComponentInthisstep,writedownabriefdescriptionofeachparagraph,equation,image,orchart.Thisshouldonly be a couple of words, or as much as a sentence. If there is a paragraph that describes onlyvariables,makeanoteof this,as it is likely thatyouwillneed this information later. If there isanequation,writedownthepurposeofthatequation.Ifaparagraphcontainsonlytheory,writedownthemainpointofthattheory.Besuretoconsidertheoverallfunctionofthatparagraph,suchas(butnotlimitedto)experimentprocedure,variables,experimentoutcomes,orexpandingonatheorypresentinpreviousparagraphs.Eachparagraph,equation,graph,table,andimagehasbeenplacedthereforareason.Identifying“why”canhelpyoutomaximizeyourpointswhenyougettothequestions.

ReflectonYourOutlineEverypassagehasareasonwhyitwaswritten.Inthisstep,identifythegoalofthepassage.Thegoalofthepassageis themainpointor topicof thepassage.Didthepassagedescribeanexperimentor

Page 293: Edited By Deeangelee Pooran-Kublall, MD/MPH · 9.5 Preparing for the MCAT: Biochemistry in the Chemical and Physical Foundations of Biological Systems Section 9.6 Preparing for the

discussatheory?Whatwouldbethereasonwhytheauthorsatdowntowritethispassage?Inphysics,thiswill tellyou theoverall topicof thepassage.However, forphysics, there isanadditional step.Manyofthetheoriesinphysicsareinterrelatedandthisposesanopportunityforthetestmakertoaskquestions regarding any concept thatmaybe related to the topicof thepassage.For example, it islikelythatadiscussionofcircuitsmaybefollowedbyquestionsregardingelectricfields,capacitors,andpotentialenergy. Identifying theconceptshelpsyou toanticipatewhere thequestionsmightgo.Thisisanothermethodoftakingcontrolofthetest,andanticipatingwhatiscomingnext.

THEPROCESSOFOUTLINING•Scanthepassagetodeterminewhetheritshouldbedonenoworlater.•Readthepassage.• Label each paragraph. Summarize each paragraph, including themeanings of new terms orequations.Eachparagraphlabelshouldbeveryshort,atleastawordortwo,butnolongerthanasentence.•Reflectonthepassage.Attheendofthepassage,identifythegoalofthepassage.•Then,onlyinphysics,identifytheconceptsthatcouldbetestedwiththepassageinquestion.

Page 294: Edited By Deeangelee Pooran-Kublall, MD/MPH · 9.5 Preparing for the MCAT: Biochemistry in the Chemical and Physical Foundations of Biological Systems Section 9.6 Preparing for the

13.2AnsweringtheQuestionsPhysics questions on the MCAT all demand the same thing: that the test-taker must have a solidfoundationinphysics.Thereareafew“plugandchug”orgraph-interpretationquestionsinMCATphysics,butmostofthequestionswillrequireahigherlevelofconceptualthinking.Therefore,well-developedcriticalthinkingskillsareanessentialrequirementforattainingahighscoreonTestDay,especiallywithregardtophysics.Historically,MCATquestionshavetakentwoforms:questionsthatrequirethetest-takertorecallaspecificpieceofinformationandquestionsthatrequirethetest-takertoapplypreviousknowledgetoanewsituationinordertodeterminetheanswer.However,withtherecentchangestotheMCATin2015,twonewquestionformshavebeenadded.Thesequestionswillrequirethetest-takertoanalyzerawdataaswellascritiquestudydesigns.Youwillneedtounderstandthe fundamentals of study design as well as the requirements of a study in order to producestatistically significant outcomes. In addition, these questions will require the ability to drawconclusionsfromrawdata,suchasthatpresentedinachartorgraph.

Asdiscussedpreviously, thereare four typesofquestions thatappear in thesciencesectionsof theMCAT.Let’sseehowthesefourtypesofquestionsconnecttoMCATphysics.

DiscreteQuestions•Donotaccompanyapassage• Always preceded by a warning such as, “Questions 12–15 are NOT based on a descriptivepassage.”•Willinvariablyrequireathoroughunderstandingofthesciencebehindthequestion.•Withasolidfoundationinphysics,thesequestionscanbeeasypointsonTestDay.•Alloftheinformationrequiredwillbeinthequestionstem,theanswerchoices,oryourownoutsideknowledge.

QuestionsthatStandAlonefromthePassage•Willfollowapassage,butthepassagewillnotberequiredtodeterminethecorrectanswer.•Thesearereallydiscretequestionshiddenwithinthepassage-basedquestions.• May be thematically related to the passage, but require no further information from thepassage.•Inphysics,thesequestionsareverycommon.

QuestionsthatRequireDatafromthePassage•Will require data from the passage, but an understanding of the passage as a whole is notrequired.•Inordertoanswerthequestion,youwillhavetofindtheinformationinthepassageandapplyittodeterminetheanswer.•Youwillhavetoknowhowtoapplythisinformationfromthepassagetoarriveatthecorrectanswer.•Theinformationinthepassageisusuallyintheformofavariableorknownquantitythatmustbeusedinordertofindtheanswer.

QuestionsthatRequiretheGoalofthePassage•Thisquestiontypeismostlikelytoappearfollowinganexperimentpassage.

Page 295: Edited By Deeangelee Pooran-Kublall, MD/MPH · 9.5 Preparing for the MCAT: Biochemistry in the Chemical and Physical Foundations of Biological Systems Section 9.6 Preparing for the

•Cannotbeansweredsolelybyoutsideknowledge;anunderstandingofatleastaportionofthepassagewillbenecessary.•Amethodicalapproachandcriticalreadingskills,suchastheKaplanway,willbeessentialforthesequestions.

DETERMININGTHEPURPOSEOFTHEQUESTIONManytest-takersregularlymisreadquestionsormissanimportantdetailthatdrasticallychangesananswer. Furthermore, many test-takers misinterpret the answer choices or miss a correct answerchoice.Thebestwaytoavoidthesemistakesistoadoptasystematicmethodforansweringquestionsandusethismethodoneverysinglequestion.Asareminder,theKaplanMethodconsistsoffoursteps(seeFigure13.1):

Figure13.1.TheKaplanMethodforquestions

1.AssesstheQuestionRead the question, but avoid reading the answers.Determine the type of question and the level ofdifficulty. Is thisquestiongoing tobemoredifficult foryou? Is thisquestionagooduseof time?Doesthequestionrequirealongcalculation?Nowitistimetomakeadecision.Isthisaquestionthatyouwanttodonoworlater?Goodquestionstodonowinphysicsarequestionsthatrequireasimplecalculationorinvolveatopicwithwhichyoufeelverycomfortable.Questionsthatarequickandeasytoanswerareworthjustasmuchasthemoredifficultquestions.

2.PlanYourAttackThisportionoftheKaplanmethodforphysicsinvolvesdeterminingwhatyouhavetodotoreachtheanswer.This isknownas thetask.Doyouknowwhat informationyoualreadyhave?Whatare theknownvariables?Givenwhatyoualreadyknowaboutthequestion,whichequationislikelytoyieldthe correct answer? Or is the question more theoretical? Is the information in the passage, youroutline,thequestionstem,orinyourfundofknowledge?Thetaskofthequestioncanbesimple—notall MCAT physics questions are going to be particularly difficult. Do not waste time, energy, orpointsonthoughtsof“Itcan’tbethiseasy.”Sometimes,itreallywillbeeasy.

3.ExecuteYourPlanNow that you have determined a plan of attack, it is time to carry out that plan. This is known asexecutingyourplan.Gobackandfindtheinformationortheequation.Dothecalculation.Applywhatyouknowtothequestionandpassagetodeterminetheanswer.

Page 296: Edited By Deeangelee Pooran-Kublall, MD/MPH · 9.5 Preparing for the MCAT: Biochemistry in the Chemical and Physical Foundations of Biological Systems Section 9.6 Preparing for the

4.AnswertheQuestionbyMatching,Eliminating,orGuessingWhen you have reached an answer from the execution of the plan, now you can read the answerchoices.Theideahereistofindtheanswerchoicethatisclosesttoyouranswer.Ifyoudonotseenanswerthatisclosetoyouranswer,thenyouhavetoreviseyouranswer.Theprocessofeliminationcanalsobeusedtodeterminethecorrectanswer.Thisisespeciallyeffectiveafteryou’vecarriedoutthestepsintheKaplanMethodbecause,bythispoint,youhavealsoconsideredpossibilitiesthatwillNOTanswerthequestioncorrectly.Finally,iftheprocessofeliminationdoesnotnarrowdowntheanswer, then take an educated guess. If you are down to two choices, use your reasoning skills todetermine which one is most likely to be correct. For physics, if you have to use the process ofeliminationorguessing,thesemethodscanbeenhancedbyidentifyinganswerchoice(s)containthecorrectunitsorsigns.Evenifyoucan’trememberthecorrectequation,chancesarethatyoudoknowif the answer should be positive or negative, or what units the correct answerwill have. Use thisinformationtoaidinyourselectionofananswer.Finally,theremaybesomeanswerchoicesthataresimplyNOTpossible.Eliminatetheseanswerchoices,andyouwillincreaseyouroddsofselectingthecorrectanswer.

Page 297: Edited By Deeangelee Pooran-Kublall, MD/MPH · 9.5 Preparing for the MCAT: Biochemistry in the Chemical and Physical Foundations of Biological Systems Section 9.6 Preparing for the

13.3GettingtheEdgeinPhysicsObtaininganelitescoreonTestDayrequiresyoutonotonlyunderstandthefundamentalconceptsofphysics but also have the ability to apply these concepts and perform the required mathematicaloperations. This ability starts with your foundation in physics. It is not enough to memorize theequationsandhopethatitwillallbe“plugandchug.”Yes,youneedtoknowtheequations,butyoualsoneedtohaveaconceptualunderstandingofthephysicsbehindtheequations.

OnTestDay,therewillbetwotypesofphysicspassages:informationandexperimentpassages.Onaninformationpassage,thequestionswillmostlyfocusontheinformationinthepassage.Youarelikelytoseemanyquestionsthatwillnotrequireinformationfromthepassage.Onexperimentpassages,acomplete understanding of the experiment is required; this includes the hypothesis tested by theexperiment,thedetailsofhowtheexperimentwasdone,andhowtointerpretthedataobtainedfromthe experiment.The questions accompanying an experiment passagewill likely require you to useinformationfoundinthepassage,aswellasdrawconclusionsandmakepredictionsbasedonthedataprovided.

Amethodical, systematicmethod for answering questions is required tomaximize your points onTestDay.Inphysics,theKaplanMethodhelpsyoutoavoidmakingunnecessarymistakes,whilealsomaximizingyourpotential.Remember, thatcorrectanswerchoiceswillmatch theappropriate signandunitsforthatparticularvectororscalarquantity.

Page 298: Edited By Deeangelee Pooran-Kublall, MD/MPH · 9.5 Preparing for the MCAT: Biochemistry in the Chemical and Physical Foundations of Biological Systems Section 9.6 Preparing for the

13.4Step-By-StepGuidetothePhysicsPassageandQuestions

READINGTHEPASSAGE•Identifythetypeofpassage

•Thisshouldbeevidentinthefirstcoupleofsentencesorbyglancingatthepassage.• If the paragraph is mostly words, with no charts or graphs, it is likely to be aninformation passage. If there are graphs and tables with data, it is likely to be anexperiment passage. If you find that there is a lot of theorywithmultiple points ofview, then it is a persuasive information passage (but it will probably look like aninformationpassageatfirst).

•Readcriticallythroughthepassage,outliningeachparagraph.• The outline of the paragraph should answer the question, “What is this paragraphdoinginthepassage?• Identify if a paragraph is mostly explanations of an equation, or if it is mainlydefiningvariables.•Iftherearechartsandgraphs,notewhatissummarizedbyeachchartandgraph.Noneedtoanalyze,justidentifywhatisrepresentedthere.•Ifthereisanequation,notewhattheequationisforwithoutanalyzingit.

•Identifythegoalofthepassage•Determine the goal of the passage by answering the question, “What is the authortryingtodowiththispassage?”• Identify the physics conceptswithin the passage by answering the question, “Whatconceptsarerepresentedhere?”

ANSWERINGTHEQUESTIONS

1.Assessthequestion•Identifythequestiontype.•Determinethepurposeofthequestionaswellasthedifficultylevelofthequestion.•Triage:isthisquestionworththetime?Doesitinvolvealengthycalculation?AmIlikelytogetthisquestioncorrectinatimelymanner?•Makeadecision:isthisquestionagoodonetodonowortocomebackto?•Ifitisagoodonetodonow,thenproceed.

2.Planyourattack•Determinewhatmustbedoneinordertoanswerthequestioncarefully.•Identifywheretofindtheinformationtoanswerthequestion.Isitinthepassage,youroutline,thequestionstem,oryourownknowledge?• If the question requires a calculation, determine the known variables and the unknownvariables.Then,selectthecorrectequation.• If the question requires you to apply your knowledge to a new situation, identifywhat youalreadyknowaboutthesituation.

Page 299: Edited By Deeangelee Pooran-Kublall, MD/MPH · 9.5 Preparing for the MCAT: Biochemistry in the Chemical and Physical Foundations of Biological Systems Section 9.6 Preparing for the

3.Executeyourplan•Findtherequiredinformation.Performthecalculation.Answerthequestionbyapplyingyourknowledge.

4.Answerbymatching,eliminating,orguessing•Readthequestionandtheanswerchoices.•Findamatchforyouranswer.•Ifacalculationwasrequired,besurethatyouranswerchoicehasthecorrectsignandunits.•Ifthereisnomatch,thenproceedtotheprocessofelimination.•Ifyoufindthattheprocessofeliminationisnotprovidingyouwithasolidanswer,thenmakeaneducatedguessandmoveontothenextquestion.

Page 300: Edited By Deeangelee Pooran-Kublall, MD/MPH · 9.5 Preparing for the MCAT: Biochemistry in the Chemical and Physical Foundations of Biological Systems Section 9.6 Preparing for the

13.5PreparingfortheMCAT:PhysicsAs you prepare for the Chemical and Physical Foundations of Biological Systems section of theMCAT,thesearethetopicsthatyouarelikelytoseeonTestDay.

TRANSLATIONALMOTION•TheSIunitsusedtomeasuredimensions•Thecomponentsofavector(magnitudeanddirection)•Howvectorscanbeaddedtodeterminedisplacement•Thedifferencebetweenspeedandvelocity•Theuseandapplicationofquantitiessuchasaverageandinstantaneousvelocity• The units of acceleration and how the value of acceleration can be used to identify a finalvelocityordistancetraveled

EQUILIBRIUM•TheconceptofforceandtheSIunitsusedtodescribeaforce•Theconceptoftorqueandhowmotiononaleverarmresultsintorque

FORCE•TheconceptofforceandtheSIunitsusedtodescribeaforce

WORK•Unitsusedtodescribework•TheworkdonebyaconstantforceisgivenbyW=Fdcosθ•WorkKineticEnergyTheoremanditsimplications• The pressure–volume diagram and how work can be calculated from this diagram bydeterminingtheareaunderthecurve•Theconceptofconservativeforcessuchasgravity•Mechanicaladvantageandthetrade-offofforceforvoltage

ENERGY•Thedefinitionofkineticenergyandtheunitsofkineticenergy•Equationusedtodescribekineticenergy:KE=½mv2•Potentialenergyanditsrelationshiptogravity,position,andsprings• The equations used to describe potential energy in different situations, includingPE =mgh(gravityandposition),PE=½kx2(spring)•Theconservationofenergy•Theunitsofpowerandtherelationshipsbetweenpower,work,andtime

THERMODYNAMICS

Page 301: Edited By Deeangelee Pooran-Kublall, MD/MPH · 9.5 Preparing for the MCAT: Biochemistry in the Chemical and Physical Foundations of Biological Systems Section 9.6 Preparing for the

• The pressure-volume diagram and how work can be calculated from this diagram bydeterminingtheareaunderthecurve•Themodesofheattransfer—conduction,convection,andradiation•Theconceptofexpansionofsolidsastheyheat,andthecoefficientofexpansion

FLUIDS•Thedefinitionofdensityanditsrelationshiptospecificgravity•ThedefinitionofbuoyancyanditsrelationshiptoArchimedes’Principle•Pascal’slawandhydrostaticpressure,includinghowhydrostaticpressurevarieswithdepth;P=ρgh•Viscosityandhowitaffectstheflowofafluid;Poiseuille’sLaw•Therelationshipbetweencross-sectionalareaandvelocityofafluid;continuityequation;Av=constant•Understandtherelationshipbetweenturbulenceandvelocityofafluid•HowBernoulli’sequationappliestononviscousfluids,anditscommonapplications•ThedefinitionoftheVenturieffectandtheuseofapitottubetomeasurefluidvelocity•Thedefinitionofsurfacetension,andhowsurfacetensionaffectsthebehaviorofafluid

GASPHASE•ThedefinitionoftheKineticMolecularTheoryofGases•ThedefinitionandapplicationofBoltzmann’sconstant• Definition of heat capacity, and the behavior of gases at a constant volume and constantpressure

ELECTROSTATICS•Thedefinitionofacharge,howchargesareconductedandconserved•Thedefinitionofaninsulatorandhowinsulatorsareused•Thedefinitionofanelectricfield(E)andhowanelectricfieldcanbedefinedbyfieldlinesandchargedistribution• Identificationandunderstandingofpotentialdifferences, including theabsolutepotentialatapointinspace

CIRCUITELEMENTS•Thedefinitionofcurrent,includingtheequationforcurrent(I=ΔQ/Δt)•Theconceptofaconductor•Understandingandapplicationofsignconventionsandunitstocircuits•Thedefinitionoftheelectromotiveforceanditsrelationshiptovoltage•Thedefinitionofresistanceanditsapplicationtocircuits•Calculationsandconceptualunderstandingofresistorsinparallelandinseries•Thedefinitionandequationforresistivity(ρ=R*A/L)• The definition of capacitance and its application to parallel plate capacitors and energy ofchargedcapacitors

Page 302: Edited By Deeangelee Pooran-Kublall, MD/MPH · 9.5 Preparing for the MCAT: Biochemistry in the Chemical and Physical Foundations of Biological Systems Section 9.6 Preparing for the

•Calculationsandconceptualunderstandingofcapacitorsinparallelandinseries•Thedefinitionandapplicationofdielectrics•Conductivityofmetallicandelectrolyticsubstances•Howmetersareusedtomeasurepotentialdifferencesandcurrents

MAGNETISM•DefinitionofamagneticfieldB•Motionofchargedparticlesinmagneticfields;LorentzForce

PERIODICMOTION•Thedefinitionsofamplitude,frequency,andphase,andhowtheyarecalculatedgivenawave•Thedifferencesbetweentransverseandlongitudinalwaves•Measurementsofwavelengthandpropagationspeed

SOUND•Howsoundisproduced•Thedifferencesbetweenthespeedsofsoundwithinvariousmediumsincludingsolids,liquids,andgases•Applicationofthedecibelasalogscaletodescribetheintensityofsound•Theuseofdampingorattenuation•Changesintheperceivedfrequencyfromtheemittedfrequencyasaresultofamovingsourceorobserver;theDopplereffect•Thedefinitionofpitch•Understandingtheresonanceofsoundwithindifferentstructures,includingpipesandstrings•Thedefinitionandapplicationofultrasound•Thedefinitionofshockwaves

LIGHTANDELECTROMAGNETICRADIATION• Conceptual understanding of interference and the implications of Young’s double-slitexperiment• The definition of diffraction as related to thin films, diffraction grating, and single-slitdiffraction•Identificationandunderstandingofotherdiffractionphenomena,includingX-raydiffraction•Thedefinitionsofpolarizationoflightandcircularpolarization• Understanding and application of the properties of electromagnetic radiation, includingconstantvelocitycinvacuo• Understanding of electromagnetic radiation as consisting of perpendicularly oscillatingelectricandmagneticfields,andthatthemovementofelectromagneticradiationisperpendiculartoboth•Howtheelectromagneticspectrumisclassified,includingthevisualspectrumandcolor•ThedefinitionofaphotonandthattheenergyofthephotonisE=hf

Page 303: Edited By Deeangelee Pooran-Kublall, MD/MPH · 9.5 Preparing for the MCAT: Biochemistry in the Chemical and Physical Foundations of Biological Systems Section 9.6 Preparing for the

GEOMETRICALOPTICS•Thenatureofreflectionfromaplanesurface,includingtheconceptthattheangleofincidence=angleofreflection•Thedefinitionofrefractionandrefractiveindexn•ApplicationofSnell’slaw:n1sinθ1=n2sinθ2• The definition of dispersion and how it relates to a change of index of refraction withwavelength•Thedefinitionoftotalinternalreflectionandtheconditionsrequired• Identificationof thecenterofcurvature, focal length,andrealorvirtual images insphericalmirrors•Understandingconverginganddiverginglenses•Applicationofthelensmaker ’sformulatolenssystems;1/p+1/q=1/fwithsignconventions• Performing calculations and demonstration of conceptual understanding of combinations oflenses•Thedefinitionoflensaberration•Generalfunctionofopticalinstruments,includingthehumaneye

Page 304: Edited By Deeangelee Pooran-Kublall, MD/MPH · 9.5 Preparing for the MCAT: Biochemistry in the Chemical and Physical Foundations of Biological Systems Section 9.6 Preparing for the

13.6PhysicsWorkedExamplesPHYSICSPASSAGEI:FLUIDDYNAMICSCardiovascular disease is the leading cause of death in the world. It is primarily caused byatherosclerosis, which is characterized by thickening and hardening of arterial walls due to thedeposition of cholesterol, triglycerides, and other substances. The accumulation of cholesterolplaqueswithinarteriescausesstenosis,ornarrowing,whichinseverecasescandrasticallyreducethebloodsupplytodownstreamtissuesandresultinischemia.

Atlowlevelsofstenosis,increasingstenosiscausesthedilationofthearteriestopreserveflowuptothepointwherethevascularbedismaximallydilated.Furtherstenosisincreasesimpedancetobloodflowto thepointwhere thebloodflowstarts todropandbecomesdependentonvascularpressure.Thus, at high levels of stenosis, increasing stenosis causes an increase in vascular pressure, but adecreaseinflow.

Anexperimentwasperformedtodeterminetherelationshipbetweentheseverityofstenosis,vascularbloodpressure,andthevolumetricflowrateofblood.Controlled,artificialstenoseswereinducedinthefemoralarteryofadogusinganexternalballooncatheter.Vascularpressureandflowrateweremeasured simultaneously just proximal to the stenosis. The resulting pressurewaveswere plottedagainsttheflowwavesintheformofpressure-flowloopareas(PFLAs)asshownbelowinFigure1.Each loop in the figure is the sum of ten cardiac cycles and corresponds to a specific degree ofstenosis.PFLAissometimesexpressedasnormalizedPFLA, theratioofPFLAatagivendegreeofstenosis to themaximumPFLA observed at any degree of stenosis. The loop slope, which can bedefinedastheslopeofthelinepassingthroughthelowestandhighestpointsontheloop,correspondstotherelativeincreaseinpressurerequiredtoincreasetheflowrate.

Figure1.Pressure-flowloopscorrespondingtodifferentstenosislevels.

P1.

P2.

Page 305: Edited By Deeangelee Pooran-Kublall, MD/MPH · 9.5 Preparing for the MCAT: Biochemistry in the Chemical and Physical Foundations of Biological Systems Section 9.6 Preparing for the

P3.

Fig1.

1.Basedontheresultsoftheexperiment,whichofthefollowinggraphscorrectlydepictstherelationshipbetweennormalizedPFLAandthedegreeofstenosis?

A.

B.

C.

D.

Page 306: Edited By Deeangelee Pooran-Kublall, MD/MPH · 9.5 Preparing for the MCAT: Biochemistry in the Chemical and Physical Foundations of Biological Systems Section 9.6 Preparing for the

2.Whichofthefollowingstatementsaboutthevariablesdescribedintheexperimentiscorrect?A.Pressureandflowareindependentvariables,whiledegreeofstenosisisthedependentvariable.B.Degreeofstenosisistheindependentvariable,whilepressureandflowaredependentvariables.C.Degreeofstenosis,pressure,andflowareallindependentvariables.D.Degreeofstenosis,pressure,andflowarealldependentvariables.

3.Thevelocityofbloodinasupinepatientis40cm/simmediatelyproximaltoastenosis,and80cm/simmediatelydistaltoit.Assumingthatthedensityofbloodequalsthedensityofwater,howwouldthevascularpressurescompareatthosetwopoints?

A.Thepressureattheproximalpointis240Pagreaterthanthepressureatthedistalpoint.B.Thepressureatthedistalpointis240Pagreaterthanthepressureattheproximalpoint.C.Thepressureattheproximalpointis480Pagreaterthanthepressureatthedistalpoint.D.Additionalinformationisrequiredtoanswerthisquestion.

4.Assumingafixedrateofbloodflow,whichofthefollowingstatementsaboutthevolumetricrateofbloodflowinthebodyisFALSE?

A.Atafixedrateofbloodflow,thecross-sectionalareaofthebloodvesselisinverselyproportionaltothevelocityofbloodflow.B.Flowrateisdirectlyproportionaltothepressuredropalongabloodvessel.C.Flowrateisinverselyproportionaltotheviscosityofblood.D.Flowrateisdirectlyproportionaltothelengthofthebloodvessel.

5.Basedoninformationinthepassage,whenthedegreeofstenosisincreases:A.PFLAincreasesandtheloopslopeincreases.B.PFLAincreasesandtheloopslopedecreases.C.PFLAdecreasesandtheloopslopeincreases.D.PFLAdecreasesandtheloopslopedecreases.

Page 307: Edited By Deeangelee Pooran-Kublall, MD/MPH · 9.5 Preparing for the MCAT: Biochemistry in the Chemical and Physical Foundations of Biological Systems Section 9.6 Preparing for the

PhysicsPassageIExplanation:USINGTHEKAPLANMETHODSP1.Introductiontostenosis

P2.Effectofstenosesonvascularpressureandbloodflowrate

P3.Experimenttodeterminerelationshipbetweenstenoticseverity,pressureandflow.

Fig1.Pressure-flowloopsatvaryingdegreesofstenosis.

1.Basedontheresultsoftheexperiment,whichofthefollowinggraphscorrectlydepictstherelationshipbetweennormalizedPFLAandthedegreeofstenosis?

A.

B.

C.

D.

Page 308: Edited By Deeangelee Pooran-Kublall, MD/MPH · 9.5 Preparing for the MCAT: Biochemistry in the Chemical and Physical Foundations of Biological Systems Section 9.6 Preparing for the

AssessthequestionThisSkill4questionteststheinformationpresentedinFigure1.ThecorrectchoicewillbeagraphconsistentwiththedatainFigure1.

PlanyourattackWeneed thedefinitionofnormalizedPFLA,which isgiven inparagraph3.Thenwe’ll lookat thegraphinFigure1todeterminetherelationshipbetweenPFLAandthedegreeofstenosis.

ExecutetheplanParagraph3statesthatnormalizedPFLA(thevariableplottedonthey-axisintheanswers)istheratioofPFLAatthatdegreeofstenosistothemaximumpossiblePFLAatanydegreeofstenosis,soit’sanumberbetween0and1.Figure1showsthatasthedegreeofstenosisincreases,thePFLAincreases.Asaresult,thenormalizedPFLAincreasesaswell.Thismeanswecaneliminateanyanswerchoicethat showsanegativecorrelationbetween thevariableson thex-andy-axes in theanswerchoices,namelychoice(D).

Since the remaininganswerchoices showapositivecorrelationbetween thevariableson theaxes,analyze Figure 1 further. With zero stenosis, the loop area is nonzero, which means the correctanswershouldhaveapositivey-intercept.

AnswerthequestionLookingattheremainingthreeanswerchoices,(A)and(B)havetheiry-interceptsatzero,sowecaneliminate them. That leaves choice (C) as the correct answer: the graph in choice (C) shows apositive,almostlinearcorrelationbetweennormalizedPFLAanddegreeofstenosis,whichmatchesthedatainFigure1.

Page 309: Edited By Deeangelee Pooran-Kublall, MD/MPH · 9.5 Preparing for the MCAT: Biochemistry in the Chemical and Physical Foundations of Biological Systems Section 9.6 Preparing for the

2.Whichofthefollowingstatementsaboutthevariablesdescribedintheexperimentiscorrect?A.Pressureandflowareindependentvariables,whiledegreeofstenosisisthedependentvariable.B.Degreeofstenosisistheindependentvariable,whilepressureandflowaredependentvariables.C.Degreeofstenosis,pressure,andflowareallindependentvariables.D.Degreeofstenosis,pressure,andflowarealldependentvariables.

AssessthequestionThisisaSkill3questionthatrequiresanunderstandingoftheconceptofindependentanddependentvariables.Acloselookattheanswerchoicesshowsthatpressureandflowarealwayslistedtogether,soweonlyneedtoconsideroneofthetwo.

PlanyourattackAn independent variable is a causative factor in an experiment—a variable we manipulate. Thedependentvariableisanoutputoreffectof theexperiment—avariablewemeasure.Toanswerthisquestion,weneedtodeterminetheroleofpressure,flow,anddegreeofstenosis.

ExecutetheplanParagraph 3 states that controlled, artificial stenoses are produced by a catheter. This degree ofstenosisinfluencesthevascularpressureandrateofbloodflow,whicharemeasuredjustproximaltothestenosis.Thus,degreeofstenosisisacausativefactor,andhenceanindependentvariable,whilepressureandflowareeffectsofthecausativefactor,andhencedependentvariables.

AnswerthequestionOurpredictionmatcheschoice(B).

3.Thevelocityofbloodinasupinepatientis40cm/simmediatelyproximaltoastenosis,and80cm/simmediatelydistaltoit.Assumingthatthedensityofbloodequalsthedensityofwater,howwouldthevascularpressurescompareatthosetwopoints?

A.Thepressureattheproximalpointis240Pagreaterthanthepressureatthedistal

Page 310: Edited By Deeangelee Pooran-Kublall, MD/MPH · 9.5 Preparing for the MCAT: Biochemistry in the Chemical and Physical Foundations of Biological Systems Section 9.6 Preparing for the

point.B.Thepressureatthedistalpointis240Pagreaterthanthepressureattheproximalpoint.C.Thepressureattheproximalpointis480Pagreaterthanthepressureatthedistalpoint.D.Additionalinformationisrequiredtoanswerthisquestion.

AssessthequestionThisisaSkill2questionthatrequirestheapplicationofBernoulli’sprinciple.Noinformationfromthepassageisrequiredtoanswerthisquestion.

PlanyourattackWritetheexpressionforBernoulli’sprincipleatthetwopointsofinterest.Forthisproblem,thetwopointsarethelocationsimmediatelyproximalanddistaltothestenosis.Hence,Bernoulli’sequationforthisproblemcanbewrittenas:

Sincethequestionstemstatesthatthepatientissupine,wecanassumethathp≈hdThiscancelsoutthethirdtermonbothsidesoftheequationtogive:

ExecutetheplanThe question stem provides the proximal and distal velocities, and states that we should use the

densityofwater, ,forthedensityofblood.Pluggingthesevaluesintotheequationfromthe“Plan”step,wecandeterminethedifferencebetweenthetwopressures,Pp−Pd.

Page 311: Edited By Deeangelee Pooran-Kublall, MD/MPH · 9.5 Preparing for the MCAT: Biochemistry in the Chemical and Physical Foundations of Biological Systems Section 9.6 Preparing for the

AnswerthequestionSincePp−Pdispositive,theproximalvascularpressureexceedsthedistalvascularpressureby240Pa.Thismatcheschoice(A).

4.Assumingafixedrateofbloodflow,whichofthefollowingstatementsaboutthevolumetricrateofbloodflowinthebodyisFALSE?

A.Flowrateisinverselyproportionaltothecross-sectionalareaofthebloodvessel.B.Flowrateisdirectlyproportionaltothepressuredropalongabloodvessel.C.Flowrateisinverselyproportionaltotheviscosityofblood.D.Flowrateisdirectlyproportionaltothelengthofthebloodvessel.

AssessthequestionThis is a Skill 1 question testing the relationship between blood flow and other parameters;specifically,weneedananswerthatisfalse.

PlanyourattackToanswerthisquestion,weneedtothinkaboutwhatequationsconnectflowratetotheparametersintheanswerchoices (cross-sectionalarea,pressuredrop,viscosity, and length).Thosevariablesarefoundintwoequations:thecontinuityequationandPoiseuille’slaw.Wewon’tneedanyinformationfromthepassage.

ExecutetheplanThe continuity equation states that for a closed system, the productAv (cross-sectional area timesvelocity),isaconstant.Poiseuille’slawholdsthat

whereΔP is thepressuredropacross thevessel, r is the radiusof thevessel,L is the lengthof thevessel,andηistheviscosityofthefluid.

Nowthatwehavetheequations,wecanlookattheanswerchoices,andfindtheonethatcontradicts

Page 312: Edited By Deeangelee Pooran-Kublall, MD/MPH · 9.5 Preparing for the MCAT: Biochemistry in the Chemical and Physical Foundations of Biological Systems Section 9.6 Preparing for the

oneoftheseequations.

Answerthequestion(A)isarestatementofthecontinuityequation,soitistrueandcanbeeliminated.Similarly,both(B)and (C) are consistentwithPoiseuille’s law: flow rate isdirectlyproportional toΔP and inverselyproportionaltoη.Thereforewecaneliminatethemaswell,leavingchoice(D)astheanswerweseek;accordingtoPoiseuille’slaw,flowrateisinverselyproportionaltolength,notdirectlyproportional.

5.Basedoninformationinthepassage,whenthedegreeofstenosisincreases:A.PFLAincreasesandtheloopslopedecreases.B.PFLAincreasesandtheloopslopeincreases.C.PFLAdecreasesandtheloopslopeincreases.D.PFLAdecreasesandtheloopslopedecreases.

AssessthequestionThis is another Skill 4 question that tests the information presented in Figure 1. To answer thisquestion,weneedtoknowhowthedegreeofstenosisinfluencesPFLAandloopslope.

PlanyourattackLook at the graph in Figure 1 to determine the relationship between PFLA, loop slope (which isdefinedinparagraph3),andthedegreeofstenosis.Sincetheloopschangeincrementallyfromoneextremetoanother,wecanusetheloopsatthetwoextremestodeterminetheeffectsofthedegreeofstenosis.

ExecutetheplanWithzerostenosis,theloopareaisthesmallest,andthelinebetweenthelowestandhighestpointsisnearlyhorizontal(thatis,closetozero).Attheotherextreme,99percentstenosis,theloopareagetslargerandlarger,soPFLAincreases.Theloopslopeismuchhigheraswell,sincethelinebetweenthelowestandhighestpointsisnearlyvertical.SobothPFLAandloopslopeshouldincrease.

Page 313: Edited By Deeangelee Pooran-Kublall, MD/MPH · 9.5 Preparing for the MCAT: Biochemistry in the Chemical and Physical Foundations of Biological Systems Section 9.6 Preparing for the

AnswerthequestionOncewe know that PFLA increases, we can eliminate (C) and (D). Knowing that loop slope alsoincreasesmakeschoice(A)thecorrectanswer.

KeyConceptsThermodynamics,RespiratorySystem

PHYSICSPASSAGEII:THERMODYNAMICSNormal respiratory function is an autonomic nervous system process regulated by the pons andmedulla oblongata.However, injury or diseasemay impact this function such that a patient’s ownsystem cannot sustain itselfwith adequate respiration. Patients in these circumstancesmust receiveoutside assistance throughmechanical ventilation. This assistance,meant to supplement or replacenormalspontaneousbreathing,canbeaccomplishedthrougheitheranegativepressuresystemorapositivepressuresystem.

Several early ventilators, including the “iron lung,” used negative pressure ventilation. Thismechanism simulates the normal function of the respiratory system. In an iron lung, the patient’sentirebody,excepttheheadandneck,isenclosedwithinalargechamber.Tosimulateinhalation,theironlungremovesairfromthechamber,decreasingthepressurebelowthatwithinthelungs,creatingapressuregradient.Asaresult,thelungsexpand,whichcausesairfromtheenvironmenttobesuckedintothelungs.Atypicalironlungmightcreateapressuregradientof−3mmHgbetweenthelungsandtheoutsideairtogenerateaninspirationof0.5liters.

Mostmodernventilators,however,relyonpositivepressureventilation.Suchamachinepressurizesthe air slightly before delivering it to the patient, who is often intubated. In both mechanisms,expiration is facilitatedwhen theventilator ceases itspressuregeneration.This allows the thoraciccavitytoreturntoinitialpressureandvolume,asthenaturalelasticityofthechestwallpushesairout.Theexpirationhappensquicklyenoughthatitcanbeconsideredanadiabaticprocess.

P1.

P2.

P3.

1.Forapatientonpositivepressuremechanicalventilation,whichofthefollowingmustnecessarilybetrueregardingtheairexpiredduringonebreath?

A.Noworkisdone.B.Nooverallchangeininternalenergytakesplace.C.Themagnitudeofworkdoneisequaltotheoverallchangeininternalenergythattakesplace.D.Theheatenergytransferredisequaltotheoverallchangeininternalenergythattakesplace.

Page 314: Edited By Deeangelee Pooran-Kublall, MD/MPH · 9.5 Preparing for the MCAT: Biochemistry in the Chemical and Physical Foundations of Biological Systems Section 9.6 Preparing for the

2.Fortheironlungdescribedinthepassage,howmuchworkdoesairdoininflatingthepatient’slungs?

(Note:1mmHg=133Pa;1L=10−3m3; or .)A.50.46JB.0.49JC.−0.49JD.−50.46J

3.Apulmonologistwantstoconductanexperimenttodetermineifinspirationisanadiabaticprocess.Toanswerthisquestion,whichofthesesetsofquantitieswouldheneedtobeabletomeasure?

I.PressurechangeswithinthelungII.TemperaturechangeswithinthelungIII.VolumechangeswithinthelungA.IonlyB.IIonlyC.IIandIIIonlyD.I,II,andIIIonly

4.Supposeamechanicalventilatorexistedthatusedthestepwiseprocessillustratedbelow.

WhichofthefollowingmustbeFALSEforthisprocess?A.ThetemperatureincreasesduringstepC.B.TheinternalenergyincreasesduringstepB.C.NoworkisdoneduringstepB.D.ThetemperatureremainsconstantduringstepA.

5.Forapatientonnegativepressureventilation,theprocessofinhalation,withrespecttothegaswithinthelungs,canbestbedescribedas:

A.spontaneous,withentropyincreasing.B.spontaneous,withentropydecreasing.C.non-spontaneous,withentropyincreasing.D.non-spontaneous,withentropydecreasing.

Page 315: Edited By Deeangelee Pooran-Kublall, MD/MPH · 9.5 Preparing for the MCAT: Biochemistry in the Chemical and Physical Foundations of Biological Systems Section 9.6 Preparing for the

PhysicsPassageIIExplanation:USINGTHEKAPLANMETHODSP1.Mechanicalventilation

P2.Negativepressureventilation

P3.Positivepressureventilation

KeyConceptsFirstlawofthermodynamics,Gaslaws

1.Forapatientonpositivepressuremechanicalventilation,whichofthefollowingmustnecessarilybetrueregardingtheairexpiredduringonebreath?

A.Noworkisdone.B.Nooverallchangeininternalenergytakesplace.C.Themagnitudeofworkdoneisequaltotheoverallchangeininternalenergythattakesplace.D.Theheatenergytransferredisequaltotheoverallchangeininternalenergythattakesplace.

AssessthequestionHere,we’reaskedtofindatruestatementaboutpositivepressureventilation.Lookingattheanswerchoices,ouranswerwillinvolveworkandinternalenergy.

PlanyourattackPositivepressureventilationisdescribedinparagraph3.We’llneedtousethedetailsprovidedthere,combined with our knowledge of work and internal energy in thermodynamic processes. Inparticular,we’relikelytoneedthefirstlawofthermodynamics,ΔU=Qin−Wout.

Executetheplan

Page 316: Edited By Deeangelee Pooran-Kublall, MD/MPH · 9.5 Preparing for the MCAT: Biochemistry in the Chemical and Physical Foundations of Biological Systems Section 9.6 Preparing for the

Thelastsentenceofparagraph3iscriticalhere.Ittellsusthatwecanassumethatexpirationcanbeconsidered anadiabatic process.Bydefinition, theheat exchanged in an adiabatic process is zero.Sincenoheatistransferred,Q=0and,accordingtothefirstlawofthermodynamics,ΔU=−W.

Answerbymatching,eliminating,orguessingLookingattheanswerchoicesoneatatime,wecaneliminate(A)becauseworkisdone.Similarly,sinceworkisdone,thereisachangeininternalenergy,so(B)isfalse.Choice(C),though,matchesour conclusion exactly, so it must be the correct answer. (D) is incorrect because no heat istransferredinanadiabaticprocess.

KeyConceptsWork,Gaslaws

2.Fortheironlungdescribedinthepassage,howmuchworkdoesairdoininflatingthe

patient’slungs?(Note:1mmHg=133Pa;1L=10−3m3; or .)A.50.46JB.0.49JC.−0.49JD.−50.46J

AssessthequestionThequestionstemtellsusthatwe’regoingtocalculatetheworkdoneininflatingthelungs.Aquicklookattheanswerchoicestellsusthatthemagnitudeandsignoftheansweraremoreimportantthantheexactvalue.

PlanyourattackToanswerthisquestion,we’llneedthedatafromparagraph2ontheironlung,aswellastheformulafor work. Given the information in the question stem, we’ll probably also need to do some unitconversions.

Page 317: Edited By Deeangelee Pooran-Kublall, MD/MPH · 9.5 Preparing for the MCAT: Biochemistry in the Chemical and Physical Foundations of Biological Systems Section 9.6 Preparing for the

ExecutetheplanSinceparagraph2givesusinformationonpressureandvolume,theworkformulawe’llneedisW=PΔV.ThekeyquestionhereiswhatvaluetouseforP.The−3mmHginparagraph2isthegradientformed, not the actual pressure involved.Since1 atm≈760mmHg, 3mmHg is a small enoughnumberthatwecanignoreit,andassumetheactualpressureis1atm.

Nowwecanuseourworkformula:W=PΔV=(1atm)(0.5L)=0.5L·atm

This gives us aworkvalue inL·atm, but the answer choices are all in joules.We canuse the twovalues of R provided, though, to make this conversion. Since the denominators are the same, itfollowsthat8.314J=0.0821L·atm.TheRconstantcanhelpusmakethisunitconversion.Noticeforthe twovaluesofR, thedenominatorsof theunitsare thesame.Thus,wecanthenuse that ratio toconverttheworktojoules:

Nowwehaveouranswerandallwehavetodoismatchittoananswerchoice.Bothchoice(A)andchoice(D)haveamagnitudeofworkclosetowhatwecalculated,butwhichiscorrect?ThequestionaskedforworkdoneBYtheairthatcausesexpansion,sosincetheairisdoingworkonthelungsweknowthatitshouldbeapositivevalue.Choice(A)isouranswer.

Answerbymatching,eliminating,orguessingBased on themagnitude of our estimate,we can eliminate (B) and (C).But is the sign positive ornegative?Here, the lungs havework doneon them, their internal energy should increase, and theworkdoneshouldbepositive.Thusthecorrectansweris(A).

ThingstoWatchOutForIfyourecognizethataproblemisgoingtoinvolveamultistepcalculation,considertriagingitforlater.

3.Apulmonologistwantstoconductanexperimenttodetermineifinspirationisanadiabaticprocess.Toanswerthisquestion,whichofthesequantitieswouldneedtobemonitoredduringtheexperiment?

I.PressurewithinthelungII.TemperaturewithinthelungIII.VolumewithinthelungA.Ionly

Page 318: Edited By Deeangelee Pooran-Kublall, MD/MPH · 9.5 Preparing for the MCAT: Biochemistry in the Chemical and Physical Foundations of Biological Systems Section 9.6 Preparing for the

B.IIonlyC.IIandIIIonlyD.I,II,andIIIonly

AssessthequestionThisquestion is aRomannumeralquestionaskingwhichquantities—outofpressure, temperature,and/orvolume—wouldneedtobemeasuredtodetermineifaprocessisadiabatic.

PlanyourattackThis question doesn’t require any information out of the passage. It will require, however, ourknowledgeofadiabaticprocesses.

ExecutetheplanBydefinition,inanadiabaticprocess,Q=0,asnoheatflowsinoroutofthesystem.Therefore,thechangeininternalenergyequalstheworkdone:ΔU=−Wout.So,todeterminewhethertheprocessisadiabaticornot,weneedtobeabletocalculatetheworkdoneandtheinternalenergy,anddetermineifthetwoquantitiesareequal.Sothe“real”questionbeingaskedhereisWhatquantitiesdoweneedtoknowtomeasureΔUand−Wout?

Answerbymatching,eliminating,orguessingLookingatouranswerchoices,itemII,temperature,comesupinthreechoices;ifit’swrong,we’re

done.Butinternalenergyis ,soweneedtemperaturetomeasureit.SoitemIIiscorrect,andwecaneliminate(A).

Wecaneliminatethetworemainingwronganswersbyconsideringthedefinitionofthermodynamicwork,W=PΔV,someasuringtheworkwouldrequireknowingbothtemperatureandvolume.Thus,thecorrectanswerischoice(D);weneedallthreequantities.

ThingstoWatchOutForSometimestrapanswersareanswersthatwouldbecorrectforaslightlydifferentquestion.Forexample,inthisquestion,(B)wouldbecorrectifthequestionwereaskingaboutanisothermalprocess.

Page 319: Edited By Deeangelee Pooran-Kublall, MD/MPH · 9.5 Preparing for the MCAT: Biochemistry in the Chemical and Physical Foundations of Biological Systems Section 9.6 Preparing for the

KeyConceptsFirstlawofthermodynamics,PVdiagram,Work

4.Supposeamechanicalventilatorexistedthatusedthestepwiseprocessillustratedbelow.

WhichofthefollowingmustbeFALSEforthisprocess?A.ThetemperatureincreasesduringstepC.B.TheinternalenergyincreasesduringstepB.C.NoworkisdoneduringstepB.D.ThetemperatureremainsconstantduringstepA.

AssessthequestionThisquestionisaskingforafalsestatementabouttheprocessshowninthediagram.

PlanyourattackSincethisquestiongivesusanewdiagramtostudy, it’sunlikelywe’llneedanyinformationin thepassage.Wewill,however,needourknowledgeofthermodynamics,especiallythetypesofprocessesdepictedinthediagram.TwoofthechoicesdealwithstepB,sothatmaybeagoodplacetostart.

ExecutetheplanStepBisisovolumetric(orisochoric):thepressureincreaseswhilevolumeremainsconstant.Sincepressure–volumeworkisgivenbyPΔV,noworkisdone.Fromtheidealgaslaw,PV=nRT,wecanalsoconclude that ifV isconstant, temperature isproportional topressure, so the temperaturewillincreaseaswell. (B)and (C)bothagreewith thesestatements, soneitherof them is theanswerweseek.

Page 320: Edited By Deeangelee Pooran-Kublall, MD/MPH · 9.5 Preparing for the MCAT: Biochemistry in the Chemical and Physical Foundations of Biological Systems Section 9.6 Preparing for the

InstepC,pressureisconstantwhilevolumechanges.UsingPV=nRToncemore,wecanconcludethatifpressureisconstantandvolumedecreases,Twilldecreaseaswell.

Answerbymatching,eliminating,orguessingChoice(A)saysthattemperatureincreasesduringstepC,whenitshoulddecrease;sinceitisafalsestatement,it’stheanswerwe’relookingfor.

AsforstepA,sincewedon’thaveenoughinformationtodeterminewhetherPVremainsconstantorchanges,wecan’tdefinitivelyconcludewhether(D)istrueorfalse.Thequestionstemkeepsusfromworrying about that, however: it asks for a statement thatmust be false; could be false isn’t goodenoughhere.

MCATExpertiseWhenaquestionasksforwhatmustbeorisnecessarilytrue/false,couldbeisnevergoodenough.

KeyConceptsSecondlawofthermodynamics,Entropy

5.Forapatientonnegativepressureventilation,theprocessofinhalation,withrespecttothegaswithinthelungs,canbestbedescribedas:

A.spontaneous,withentropyincreasing.B.spontaneous,withentropydecreasing.C.non-spontaneous,withentropyincreasing.D.non-spontaneous,withentropydecreasing.

AssessthequestionThisquestionasksustoconsiderinhalationinnegativepressureventilation,anddeterminewhetheritisaspontaneousprocessornot,andwhetherwewouldexpectentropytoincreaseordecrease.

Planyourattack

Page 321: Edited By Deeangelee Pooran-Kublall, MD/MPH · 9.5 Preparing for the MCAT: Biochemistry in the Chemical and Physical Foundations of Biological Systems Section 9.6 Preparing for the

To answer this question, we’ll need the description from paragraph 2 of how negative pressureventilationworks.We’llalsoneedourknowledgeofspontaneousprocessesandentropy.

ExecutetheplanParagraph2tellsusthatinnegativepressureventilation,theironlungremovesairfromthechamber,decreasingthepressurebelowthatwithinthelungs,creatingapressuregradient.Themovementofgasfromareasofhigherpressure toareasof lowerpressureoccursnaturally,andshouldproceedwithoutanyoutsideforces.Sotheexpansionshouldbeconsideredspontaneous.

Butwhathappens to theentropy?Entropy isameasureof randomnessordisorder, and, inpart, isproportionaltothenumberofgasparticlesinasystem.Sinceairentersthelungsduringinhalation,thenumberofmolesofgasinthelungstendstoincrease,andtherefore,theentropyshouldincreaseaswell.

Answerbymatching,eliminating,orguessingOnceweknowtheprocessisspontaneous,wecaneliminate(C)and(D).Determiningthattheentropyinthelungsmustincreaseallowsustoeliminate(B)andselectchoice(A)asthecorrectanswer.

Page 322: Edited By Deeangelee Pooran-Kublall, MD/MPH · 9.5 Preparing for the MCAT: Biochemistry in the Chemical and Physical Foundations of Biological Systems Section 9.6 Preparing for the

13.7PhysicsPracticePHYSICSPASSAGE(QUESTIONS1–5)Therhythmiccontractionoftheheartisinitiatedbythefiringofmyogenicelectricalimpulsesatthesinoatrialnodelocatedinthewalloftherightatrium.Anydeviationfromthenormalrangeof60–100 rhythmic heartbeats/min in adults is classified as cardiac dysrhythmia, a potentially fatalcondition.Defibrillatorscan“reset”theheartandreestablishthenormalfunctioningofthesinoatrialnode by delivering therapeutic doses of electrical energy to the heart. A circuit diagram of adefibrillatorisshowninFigure1.

Figure1.Circuitdiagramofadefibrillatorwithpaddleelectrodes

Atypicaldefibrillatorconsistsofacapacitor(C),inductor(L),andapowersupply.Whentheswitchisinposition1,thedefibrillatorisinchargingmode,andthepowersupplyEisusedtostorechargesacrosstheplatesofthecapacitor.TheworkdonetochargethecapacitorisstoredaspotentialenergyUinthecapacitor,andcanbecalculatedusingtheequationbelow:

whereA is theareaof thecapacitorplates,κ is thedielectricconstant,ε0 is thepermittivityof freespace,anddisthedistancebetweenthecapacitorplates.

Whentheswitchisinposition2,thedefibrillatorisindischargingmodeandthecircuitiscompletedbythepatient,whoisrepresentedasaresistorRinthecircuitdiagram.Metalpaddleswithinsulatedhandlesareheldonthepatient’sskinwithabout25lbsofforcetodeliverthestoredelectricalenergyof the capacitor to thepatient.Toprevent the capacitor fromdischarging anddelivering its storedenergytooquickly,theinductorisusedtoprolongthedurationofcurrentflow.

P1.

F1.

P2.

E1.

Page 323: Edited By Deeangelee Pooran-Kublall, MD/MPH · 9.5 Preparing for the MCAT: Biochemistry in the Chemical and Physical Foundations of Biological Systems Section 9.6 Preparing for the

P3.

1.Adefibrillatorconsistingofa5-mFcapacitorwithadistanceof10mmbetweenitsplatesispoweredbyastep-uptransformerthatsuppliespeakvoltagesof10,000V.Whatisthemaximumelectricalenergythatcanbedeliveredbythiscapacitortoapatient?

A.25×10−3JB.0.5JC.25JD.50J

2.Manystandarddefibrillatorsexhibitatypeofdefibrillationwaveformknownasabiphasicwaveform,asdepictedbelow.

Whatisthemagnitudeofthetotalchargedeliveredinasinglebiphasiccurrentpulseofsuchadefibrillator?A.85μCB.170μCC.85mCD.170mC

3.ThefigurebelowshowstherawECGdataofapersonwithahealthyheart.Thegraphshowssuperimposednoisefromthe60Hzpowersupplyandmotionartifactsfrombreathing.GiventhestandardEKGwavetotheleft,whataretheheartrateandrespiratoryrate,respectively,forthisindividual?

Page 324: Edited By Deeangelee Pooran-Kublall, MD/MPH · 9.5 Preparing for the MCAT: Biochemistry in the Chemical and Physical Foundations of Biological Systems Section 9.6 Preparing for the

A.1.5Hz,0.25HzB.25Hz,5HzC.60Hz,12HzD.80Hz,20Hz

4.Adefibrillatorisusedtodeliver50Jofenergytoapatientoveraperiodof20ms.Ifthecurrentflowingthroughtheinductorcomponentofthedefibrillatoris2A,whatistheeffectiveresistanceoftheelectricalpathwaythroughthepatient?

A.250ΩB.625ΩC.850ΩD.1,250Ω

5.Gelisappliedtotheskinwheretheelectrodesofadefibrillatorwillbeplaced.Thisismostlikelydoneinorderto:

A.minimizeelectricalconductanceofthebodyandreducethepossibilityofseriousburnstotheskin.B.maximizeelectricalconductanceofthebodyandreducethepossibilityofseriousburnstotheskin.C.minimizeelectricalconductanceofthebodyandensurethatasufficientlyhighamountofelectricalenergyisdeliveredtothemyocardialtissue.D.maximizeelectricalconductanceofthebodyandensurethatasufficientlyhighamountofelectricalenergyisdeliveredtothemyocardialtissue.

Page 325: Edited By Deeangelee Pooran-Kublall, MD/MPH · 9.5 Preparing for the MCAT: Biochemistry in the Chemical and Physical Foundations of Biological Systems Section 9.6 Preparing for the

PhysicsPracticePassageExplanationsUSINGTHEKAPLANMETHODSP1.Introductiontodefibrillators

Fig1.Circuitdiagramofdefibrillator

P2.Chargingphaseofdefibrillator

E1.Equationforpotentialenergystoredbycapacitor

P3.Dischargingphaseofdefibrillator

1.(C)

The first step is to set up an equation for energy stored by a capacitor. The maximum electricalenergy that can be delivered by a capacitor is equal to the energy stored by the capacitor, which,accordingtothepassage,isgivenby:

However,sincethequestionstemonlyprovidesuswiththevaluesforE,C,andd,weneedtorewrite

theequationintermsofthosevariables.Weknowthat ,whichrearrangestogiveAκε0=Cd.Hence,theequationforpotentialenergycanbewrittenas:

ThesecondstepistopluginthevaluesforE,C,andd:

Hence,thecorrectanswerischoice(C),25J.

2.(D)

Thefirststepistocalculatetheareaunderthepulsefrom0to4ms.Thatportionofthegraphcanbedividedintoatriangleandrectangle.Thetriangularportionhasaheightofabout5A,andhencethe

area of the triangle is . The rectangular portion has aheightofabout25A,andhencetheareaoftherectangleis25A×4ms=100millicoulombs.Hencethetotalchargedeliveredbythisportionofthebiphasicwaveformis110millicoulombs.Thenextstepis tocalculate theareaunder thepulsefrom4to8ms.Thetriangularportionin thisareaofthegraphhasaheightof5A,andhencetheareaofthetriangleis10millicoulombs,likeinstep1.However,therectangularportionherehasaheightofabout15A(remembertoonlyconsidermagnitude),andhence theareaof therectangle is15A×4ms=60millicoulombs.Thus, the total

Page 326: Edited By Deeangelee Pooran-Kublall, MD/MPH · 9.5 Preparing for the MCAT: Biochemistry in the Chemical and Physical Foundations of Biological Systems Section 9.6 Preparing for the

chargedeliveredbythisportionofthebiphasicwaveformis70millicoulombs.The final step is to calculate the total charge delivered by the biphasic pulse. The total chargedeliveredbyasinglebiphasiccurrentpulseissimplythesumofthetwocalculations:110mC+70mC=180millicoulombs.Thisisclosesttochoice(D),whichisthecorrectanswer.

3.(A)

Thefirststepistocalculatetheheartrate.AsingleheartbeattranslatesintoanECGsignalthatlookslikethewaveformbelow—knownasthePQRSTsignal.

TheECGdatainthequestionstemshowsthatthetimeperiodofonePQRSTsignalcycleisabout0.7s.Thefrequencyofaheartbeatishence1/0.7=1.43Hz.Next,weneedtocalculatetherespiratoryrate.ThePQRSTsignallookslikeitissuperimposedonaslowerwaveintheECGdatashowninthequestionstem.Theslowerwavemustbethemotionartifactfrombreathing,sincethepowersupplylinehasarelativelyhighfrequencyof60Hz.Thetimeperiodofthisslowersignalisabout4s,whichmeansitsfrequencyis¼=0.25Hz.Thisisclosesttochoice(A),whichisthecorrectanswer.(B)isoffbyaboutamagnitudeoftenforeachrate.(C)isthelowerendoftherangefortheheartrateandrespiratoryratewhenmeasuringthebeatsandbreathsperminute.(D)istheupperendoftherangefortheheartrateandrespiratoryratewhenmeasuringthebeatsandbreathsperminute.

4.(B)

Thequestiondescribesthedefibrillatorinthedischargingmode—thatis,theswitchisinposition2.First,calculatethepowerdeliveredbythedefibrillator:

Next,calculatetheresistanceofferedbythepatient.Sincetheinductorandtheeffectiveresistanceofthepatientareinseries,thecurrentflowingthroughtheinductoristhesameasthatflowingthroughtheresistance.ThepowerofthiseffectiveresistanceisgivenbyP=I2R,whichcanberearrangedtosolveforR.PlugginginthevaluesforIandP,weget:

Hence,theresistanceoftheelectricalpathwaythroughthepatientis625Ω,choice(B).

5.(B)

Page 327: Edited By Deeangelee Pooran-Kublall, MD/MPH · 9.5 Preparing for the MCAT: Biochemistry in the Chemical and Physical Foundations of Biological Systems Section 9.6 Preparing for the

Gelactsasaconductorandensuresabetterconnectionbetweenthepaddlesandskin.Thisreducestheelectrical resistance, and thus increases the conductance offered by the patient to the dischargingdefibrillator.Iftheresistanceofferedbytheskinweretoohigh,thepowerdeliveredtothepatientbythedefibrillatorwouldbetoohigh,sinceP=I2R.Thiswould transferdangerouslyhighamountsofenergytotheskin,whichcouldresultinskinburns.Choice(B)isthecorrectanswer.(A) is the opposite of the correct answer choice. Per choice (C), application of gel maximizeselectricalconductanceofthebody.Whileapplicationofgeldoesmaximizeconductanceofthebodyasstatedby(D),itwoulddecreasethepowerandthetotalenergydeliveredtothemyocardialtissue.

Page 328: Edited By Deeangelee Pooran-Kublall, MD/MPH · 9.5 Preparing for the MCAT: Biochemistry in the Chemical and Physical Foundations of Biological Systems Section 9.6 Preparing for the

IV

CriticalAnalysisandReasoningSkills

Page 329: Edited By Deeangelee Pooran-Kublall, MD/MPH · 9.5 Preparing for the MCAT: Biochemistry in the Chemical and Physical Foundations of Biological Systems Section 9.6 Preparing for the

CHAPTERFOURTEEN

ArgumentsandFormalLogicTheMCATisultimatelyatestofcriticalthinking,andassuch,theabilitytocharacterize,understandand assess arguments plays an important role, and the use of formal logic is an important tool inworking with arguments as they appear on the MCAT. Mastery of arguments and understandingformal logic allow for deeper understanding of the stated information, and will be essential toobtainingahighscoreonTestDay.OfallCARSquestions,30percentinvolveReasoningWithintheTextand40percentrequireReasoningBeyondtheText.Youcanexpectmore than three-quartersofyourpointsonCARStobenefitfromclear logical thinking.Inorder toprovideasolidfoundationforyourmasteryofargumentsandformallogic,we’llstartbyexaminingarguments,thenmoveontousingformallogicinarguments.

14.1WhatIsanArgument?Initsmostbasicform,anargumentissimplyastatementcomposedoftwostatedparts,theevidenceand conclusion, and one or more unstated, but implied parts, the assumption(s) or inference(s).Arguments as they appear on the MCAT have nothing to do with heated debates but are simplyconclusions the author makes (regardless of whether or not they are actually true or false), theevidence the author uses to back up his conclusion, and the unstated assumptions or inferencesimpliedintheargumenttoprovidefurtherevidencethatwillmaketheconclusionmorelikelytobetrue, at least from the author ’s point of view.There are three levels of arguments thatwe call thedomainsofdiscourse:thebasicdifferencebetweenthethreeisverysimplythingsvs.wordsvs.ideas.Thoughthedomainscanneverbeentirelyseparatedfromoneanother,eachhasdistinctivepartsandrelationshipsthatmustnotbeconfused.

DOMAINSOFDISCOURSE•Thenaturaldomain corresponds to object, events, and experiences—everything that can befoundintheworldaroundus.•Thetextualdomaincorrespondstowords,sentences,andparagraphs—everythingthatdirectlyfacesyouinanMCATpassage.• The conceptual domain corresponds to concepts, claims, and arguments—everything thatunderlieslogic.

KeyConceptsLogicistheformalstudyofarguments.Itfallsintotheconceptualdomainofdiscourse.

KeyConcepts

Page 330: Edited By Deeangelee Pooran-Kublall, MD/MPH · 9.5 Preparing for the MCAT: Biochemistry in the Chemical and Physical Foundations of Biological Systems Section 9.6 Preparing for the

Aconcepthasameaningordefinition,butisnotbyitselftrueorfalse.Ideasshouldbedistinguishedbothfromthetermsusedtorepresentthem,ontheonehand,andfromthenaturalobjectsoreventsthatmightbegivenasexamplesofthem.•Conceptsareideasthathavemeanings(definitionsorconnotations).• Ideascanbe related invariousways,often indicatedwithRelationkeywords inapassage.Relationkeywords are tools forrecognizingtheorganizationwithintexts.

•SimilarityandDifferenceareverycommonrelationkeywords.•Opposition,Sequence,andComparisonkeywordsarelesscommon,butsignificantinapassage.•TherelationbetweenthewholeanditspartscanbeimportantinsomeCARSpassages.

Page 331: Edited By Deeangelee Pooran-Kublall, MD/MPH · 9.5 Preparing for the MCAT: Biochemistry in the Chemical and Physical Foundations of Biological Systems Section 9.6 Preparing for the

14.2WhatAretheElementsofArguments?CONCEPTSItmightseemobviousbutthefundamentalelementofalogicalargumentisanidea,calledaconcept.Conceptshavemeanings,butarenotnecessarilytrueorfalse.Note,though,thatquestionsinCARScanemploysynonymsandparaphrasesof ideas indifferentwordingthanisusedin thepassage.Inshort, itwill be essential on TestDay to look for concept-for-concept correspondences, not exactword-for-wordmatches.

CLAIMSWhatdistinguishesaclaimfromamereconceptistruthvalue,thecapacitytobeeithertrueorfalse.Whileclaimsmaybequitecomplex(potentiallyconsistingofnumerousconceptsrelatedtogetherindiverseways),havingatruthvaluerequiresonlytwopartsatminimum:asubjectandapredicate,suchastheyetiistenfeettall.

•Claimscanalsobecalledassertions,statements,propositions,beliefs,orcontentions.•Claimsaremadeupofcombinationsofconceptsandrelationsofideas.•Theypossesstruthvalue,andcanthusbetrueorfalse.•Claimscanalsoberelatedthroughvariousrelationships.

•Iftwoclaimsareconsistent(compatibleorinagreement)withoneanother,thenbothcanbetruesimultaneously.•Iftwoclaimsareinconsistent(contradictoryorconflicting)withoneanother,thenitisimpossibleforbothtobetruesimultaneously.•Ifoneclaimsupportsanother,thenthisclaimbeingtruewouldmaketheotherclaimmorelikelytobetrueaswell.• If one claim challenges (refutes or objects to) another, then this claim being truewouldmaketheotherclaimmorelikelytobefalse.

KeyConceptsClaimsarethemiddlemeninthelogicalhierarchy,composedthemselvesofconceptsandtheirrelationships,and,inturn,composingarguments.Claimsconsistofatleastasubjectandapredicate,andhavebothmeaningandtruthvalue(thecapacitytobetrueorfalse).

INFERENCES,ASSUMPTIONANDIMPLICATIONSWhileconceptsandclaimsfallunderthestatedpartsofarguments,theunstatedpartsrequireyoutobridgethegapbetweenthestatedevidenceandconclusionortoidentifytheconclusionbasedonthestatedevidence.Youwilllikelyseeanabundanceofquestionsthatrequireyoutoidentifyinference,assumption,orconclusion.

• Inferences are unstated parts of arguments. One way to recognize an inference is by thenegativeeffectitwouldhaveontheargumentifitweredenied•Assumptionsareunstatedpiecesofevidence.•Implicationsareunstatedconclusions.

Page 332: Edited By Deeangelee Pooran-Kublall, MD/MPH · 9.5 Preparing for the MCAT: Biochemistry in the Chemical and Physical Foundations of Biological Systems Section 9.6 Preparing for the

•Inferencesareclaimsthatmustbetrueor—attheveryleast—mustbehighlyprobable.

COUNTERARGUMENTSThecounterargumentisanargumentmadeagainstaconclusion.Whilesomewriterswillonlyoffercounterarguments, their purpose being to argue against some claim, many authors raisecounterargumentsmerely for the sake of refuting them,which is an indirect way to support theirconclusion.

KeyConceptUnstatedclaimsinargumentsareknownasinferences.Inferencesareeitherassumptions(unstatedevidence)orimplications(unstatedconclusions).

KeyConceptCounterarguments,alsocalledrefutations,objections,orchallenges,aretheoppositeofevidencebecausetheygoagainsttheconclusion.

Page 333: Edited By Deeangelee Pooran-Kublall, MD/MPH · 9.5 Preparing for the MCAT: Biochemistry in the Chemical and Physical Foundations of Biological Systems Section 9.6 Preparing for the

14.3HowWillArgumentsBeTested?INFERENCEANDASSUMPTIONQUESTIONSYour skill in parsing arguments is mostly commonly tested on the MCAT by inference andassumptionquestions,oftenaskingwhatwouldweakenorstrengthenthem.

•Therearethreemainwaysofstrengtheninganargument:•Onecouldprovideanewpieceofevidencethatsupportstheconclusion.•Onecouldsupportevidencethatalreadyexiststosupporttheconclusion.•Onecouldchallengerefutationsagainsttheconclusion.

•Therearethreemainwaysofweakeninganargument:•Onecouldprovideanewrefutationthatgoesagainsttheconclusion.•Onecouldsupportrefutationsthatalreadyexist.•Onecouldchallengeevidencefortheconclusion.

ANALOGICALREASONINGQUESTIONSTheMCATwillalsoposequestionsbasedonanalogies,usingthesimilaritiesbetweentwothingstoargueforanadditionalcommonalitybetweenthem.Theknownentityistheonewithcharacteristicsthathavealreadybeenestablished.Theunknownentityistheonethatisonlypartiallyunderstood.Insomecases, thepassageprovides theknown term,with itsvariouscharacteristics, and thequestiongives the new context that establishes the unknown. In such questions, you are being asked toextrapolate, extend, or apply the ideas from the passage to a new situation, so you can take theinformationfromthepassageasagiven.

•Ananalogycanbestrengthenedbygreatersimilaritybetweentheknownandunknown.•Themorepointsofsimilaritybetweenthetwo,thestrongertheanalogy.•Themorerelevant(structuralasopposedtosuperficial)thesimilaritiesbetweenthetwo,thestrongertheanalogy.•Thefewerrelevantdifferencesbetweenthetwo,thestrongertheanalogy.

Page 334: Edited By Deeangelee Pooran-Kublall, MD/MPH · 9.5 Preparing for the MCAT: Biochemistry in the Chemical and Physical Foundations of Biological Systems Section 9.6 Preparing for the

14.4WhatIsFormalLogic?Themost abstract application of logic, formal logic, examines patterns of reasoning to determinewhichonesnecessarilyresultinvalidconclusions.Formallogicconsistsofaconditionalstatement,suchasifIaminPennsylvaniaandanecessaryresult,thenIamintheUnitedStates.Asyousee,theconditional statement is sufficient to necessarily bring about the result. On theMCAT, argumentsmade using conditional claims,with conditional relationships, are featured in some form in everypassageandplaysomeroleinmostCARSquestions.

MCATEXPERTISETheMCATwillnotdirectlytestyourunderstandingofformallogic.Forexample,noquestionwilldirectly ask you to form a contrapositive. Many questions, however, will implicitly test yourunderstanding.

Page 335: Edited By Deeangelee Pooran-Kublall, MD/MPH · 9.5 Preparing for the MCAT: Biochemistry in the Chemical and Physical Foundations of Biological Systems Section 9.6 Preparing for the

14.5WhatAretheElementsofFormalLogic?CONDITIONALSAconditionalisaunidirectionalrelationshipthatexistsbetweentwoterms.

•Conditionalscanberepresentedwithlanguage(ifX,thenY),orsymbols:X→Y.• The antecedent (X) can also be called a sufficient condition, evidence (in cases ofjustification),orcause(incasesofcausation).• The consequent (Y) can also be called a necessary condition, conclusion (in cases ofjustification),oreffect(incasesofcausation).•Aconditionalclaimistrueifitisimpossibletohaveatrueantecedentandafalseconsequentsimultaneously.•Operationsofformallogiccanberepresentedinatruthtable(Table14.1).

KeyConceptInTable14.1,allfourpossiblecombinationsofthetruthofXandYarepresented,aswellastheresultanttruthoftheconditionalclaiminthefinalcolumn:

X

Y

X→Y

true

true

true

true

false

false

false

true

true

false

false

true

Table14.1.Truthtableforconditionalclaims

APPLICATIONSFORCONDITIONALSConditionals can function in several different ways in CARS passages and questions. Of the listbelow,justificationandcausationarethemostcommonwaysthatconditionalswillbeused.

• Justification is the relationship of logical support between a piece of evidence and itsconclusion.•Causation is theone-wayrelationshipof theantecedent leadingto theconsequent(causeandeffect).•Correlationistherelationshipoftwoeventsaccompanyingoneanother.

Page 336: Edited By Deeangelee Pooran-Kublall, MD/MPH · 9.5 Preparing for the MCAT: Biochemistry in the Chemical and Physical Foundations of Biological Systems Section 9.6 Preparing for the

•Whole–partsrelationship.•Oneconceptcanbeapartofanotherconcept(thewhole)intheconceptualdomain.•Onecomponentorcharacteristiccanbepartofanobjectinthenaturaldomain.

SUFFICIENTVS.NECESSARYSufficient and necessary refer to the one-way relationship between the antecedent (X) and theconsequent(Y)andtheimpossibilityofhavinganantecedentwithoutitsconsequent.Iftheconsequentisnot true, thentheantecedent isalsonot true.Revisitingthepreviousexample, tobein theUnitedStatesitissufficienttobeinPennsylvania.However,itisnotnecessarytobeinPennsylvaniainordertobeintheUnitedStates;anystatewilldo.

CONTRAPOSITIVEThe contrapositive is logically equivalent to the original conditional, but carries a differentconnotation. Whenever an author makes any kind of conditional claim, it is always possible totranslatetheconditionalclaimintootherrelationships,mostnotably,thecontrapositive.Bydefinition,thecontrapositiveofifX,thenYisifnotY,thennotX.Notethatinthecontrapositive,theXandYtermsswitchpositions,withtheYtermnowfirst.Thecontrapositivecanberepresentedusingatilde(∼)tostandfornegation(∼XthusmeansnotXorthenegationofX):

Conditional:X→Y

Contrapositive:∼Y→∼X

Oneof themostuseful reasons for forming thecontrapositive is that it’saguaranteed inference,alogicalequivalentforanyconditionalclaimmadeinapassage.

Page 337: Edited By Deeangelee Pooran-Kublall, MD/MPH · 9.5 Preparing for the MCAT: Biochemistry in the Chemical and Physical Foundations of Biological Systems Section 9.6 Preparing for the

14.6HowWillFormalLogicBeTested?The MCAT will not provide you with a formal logic statement and require you to form thecontrapositive, but formal logic will allow you to dig deeper into claims. If, for example, theconditionisthatalllionsaremammals,andyouformthecontrapositivethatifit’snotamammal,it’snot a lion, you have the information you need to answer a question about mammals and lions.Appreciating the general characteristics of formal logicwill clarify your thinking andgarner youmanyadditionalpoints.

Page 338: Edited By Deeangelee Pooran-Kublall, MD/MPH · 9.5 Preparing for the MCAT: Biochemistry in the Chemical and Physical Foundations of Biological Systems Section 9.6 Preparing for the

14.7GettingtheEdgeUsingArgumentsandFormalLogicAs we said in the beginning, the MCAT is ultimately a test of critical thinking, and that skill ismanifestinyourabilitytorecognizeanargument,takeitapart(sometimesusingformallogic),andanswerquestionsbasedonit.Whileacademictextscanbevagueandsubjecttointerpretation,everyquestionyoufaceontheCARSsectionwillhaveonlyonedefensibleanswer.Thevalueoflogicisprecisely in its ability toclarifyyour thinking, allowingyou tomirror thecognitiveprocesses thetestmakersuse,andhoneinoncorrectresponses.

Buttheabstractknowledgeofargumentsisnotenough.TheonlywaytobesuccessfulwithCARSisto practice questions and passages. Every question that you get wrong on a practice passage orquestion is an opportunity to review how you approached the question. Systematically reviewingquestions to determine weaknesses in your critical thinking skills will help you to identify andaddressthoseweaknesses.Ifyoufindthatyouarestuckatascoreplateauonpracticetests,thenitisessentialtotakeanhonestlookatyourcriticalthinkingskillstobreakthroughthatplateau.

Page 339: Edited By Deeangelee Pooran-Kublall, MD/MPH · 9.5 Preparing for the MCAT: Biochemistry in the Chemical and Physical Foundations of Biological Systems Section 9.6 Preparing for the

CHAPTERFIFTEEN

CARSQuestionTypesTheMCAT is fundamentallya testofcritical thinking, and thisbecomesveryevident in theCARSsection. Here you are not reading for its own sake but thinking about what you read in order toanswer a range of question types. It’s important to be able to identify each type since differentquestion types are more efficiently answered with different approaches. As you go through thequestiontypesyou’llseeinthischapter,rememberthattherearenoMCATpointstobehadinreadinga passage; all your points are in the questions. Themore efficiently you can identify and attack aquestion,thegreateryouropportunitytogetitright.

15.1WhatKindsofQuestionsWillYouBeAsked?Questions accompanying CARS passages will range from ones that can be answered simply byreferringtothepassagetext,tothosethatrequirethinkingaboutwhatisnotstated,butimpliedinthetext.Thus,questiontypesrequiredifferentapproaches,involvedifferentlevelsofdifficulty,andaskyoutothinkindifferentways.Youwillhavequestionsthatrequireyoutosimplyusetheinformationin the text,calledFoundationofComprehensionquestions;questions that requiredeeper reasoningabouttheinformationinthetext;andquestionsthatrequirereasoningbeyondthetext.Regardlessofthe complexity of question type, all correct answers result in the same number of points. Thinkclearlyandcarefullyabouteachone.

Page 340: Edited By Deeangelee Pooran-Kublall, MD/MPH · 9.5 Preparing for the MCAT: Biochemistry in the Chemical and Physical Foundations of Biological Systems Section 9.6 Preparing for the

15.2FoundationofComprehensionQuestionsThoughthereareavarietyFoundationofComprehensionquestiontypes, theyhaveincommonthefactthattheanswerisinthetextanddoesnotrequireyoutomanipulatethetextinanyway.

MAINIDEAMainIdeaquestionsaskfortheauthor ’sprimarygoalandoftencontainwordslikecentral thesis,primarypurpose,ormainidea.Lesscommonly,thesequestionsmayaskforadifferentaspectoftherhetoricalsituationsuchastheaudienceorthemedium.

•Plan:LookatwhatyouwroteinyouroutlinefortheGoal.• Execute:Reread theGoal in your outline, taking note of the charge and degree of the verb(positiveversusnegative,extremeversusmoderate).•Answer:Match your expectations for the right answer. If there is no clearmatch, or if youcannot perform any of the earlier steps of the KaplanMethod for CARS Questions, use theprocessofelimination.

•Wronganswerchoicesmaybetoonarrow(FaultyUseofDetail)ortoobroad(OutofScope).•Wronganswerchoicesmayhavethewrongtone(positive,negative,ambivalent,orimpartial)ordegree(tooextremeortoomoderate).

MCATExpertiseAccordingtoourresearchofreleasedAAMCmaterial,MainIdeaquestionsmakeupabout5percentoftheCARSsection(aboutthreequestions).

DETAILDetail questions ask about what is stated explicitly in the passage and tend to use words like theauthorstatesoraccordingtothepassage,withdeclarativelanguagelikeisandare.Detailquestionsare themost likely touse theScattered format,whichusesRomannumeraloptionsorwords likeEXCEPT,NOT,orLEAST.

•Plan:Lookforcontentbuzzwordsinthequestionstemandyouroutlinetodeterminewheretherelevantinformationwillbefound.•Execute:Reread the relevant sentence, aswell as the sentencesbefore and after.Createyourpredictionbyputtingtheanswerinyourownwords.

•Makethepredictionbriefsoyoucanrepeatittoyourselfbetweenanswerchoices.•ForScatteredDetailquestions,locateallthreeofthewronganswersinthepassagetobeabletoeliminatethemfromtheoptions.

•Answer:Match your expectations for the right answer. If there is no clearmatch, or if youcannot perform any of the earlier steps of the KaplanMethod for CARS Questions, use theprocessofelimination.

Page 341: Edited By Deeangelee Pooran-Kublall, MD/MPH · 9.5 Preparing for the MCAT: Biochemistry in the Chemical and Physical Foundations of Biological Systems Section 9.6 Preparing for the

MCATExpertiseAccordingtoourresearchofreleasedAAMCmaterial,Detailquestionsmakeupabout16percentoftheCARSsection(aboutnineortenquestions).

FUNCTIONFunctionquestionsaskaboutwhattheauthoristryingtododuringthepassage.ThesequestionsaresimilartoMainIdeaquestions,althoughtheyfocusonthepurposeofonlyoneportionofthepassage(usuallyonesentenceoroneparagraph).Functionquestionstendtousewordslikepurpose,motive,orintention,orcompletionslikeinordertoorbecause.

•Plan:Useyouroutlinetolocatetherelevantparagraph.•Execute:Look at yourLabel for the relevant paragraph and theGoal at the bottomof youroutline. If buzzwords in the question stem direct to specific sentences, reread those portions,thinkingabouthowtheyfitintothepurposeoftheparagraphandtheoverallpassage.•Answer:Match your expectations for the right answer. If there is no clearmatch, or if youcannot perform any of the earlier steps of the KaplanMethod for CARS Questions, use theprocessofelimination,removinganyanswerthatconflictswiththeauthor ’smainargumentortheparagraph’spurpose.

KeyConceptBothMainIdeaandFunctionquestionscanoftenbeansweredsolelybylookingatyourpassageoutline.TheanswerstoMainIdeaquestionsshouldreflecttheauthor’sGoal,whereastheanswerstoFunctionquestionsshouldusuallyreflecttheLabelyou’veassignedforagivenparagraph.

DEFINITION-IN-CONTEXTDefinition-in-Context questions ask you to define a word or phrase as it is used in the passage.Thesequestionsoftencallattentiontothetermtobedefinedusingquotationmarksoritalics,butnotalways.

Definition-in-Contextquestionsalwaysreferenceaword,phrase,oranentireclaimfromthepassage.•Plan:Useyouroutlinetolocatetherelevantparagraph.•Execute:Reread the sentencewith theword or phrase, and perhaps the surrounding context.Rephrasetheauthor ’sdefinitionoftheterminyourownwords.•Answer:Match your expectations for the right answer. If there is no clearmatch, or if youcannot perform any of the earlier steps of the KaplanMethod for CARS Questions, use theprocessofelimination.

Page 342: Edited By Deeangelee Pooran-Kublall, MD/MPH · 9.5 Preparing for the MCAT: Biochemistry in the Chemical and Physical Foundations of Biological Systems Section 9.6 Preparing for the

15.3ReasoningWithintheTextQuestionsUnlikeFoundationofComprehensionquestions,whichdirectyouprimarily to theexplicit ideas inthe text,ReasoningWithin theText questionsrequire you to think aboutwhat is implicit, but notstated, in the text. The two most common types of these questions are Inference and Strengthen–Weaken.ReasoningWithintheTextquestionsaccountfor30percentofwhatyou’llencounteronTestDay,accordingtotheAAMC’sofficialstatements.

INFERENCEInferencequestionslookfortheunstatedpartsofarguments.Theanswersmustbetruegivenwhatisclaimedinthepassage.

•Unstatedpartsofarguments•Assumptionsareunstatedevidence.•Implicationsareunstatedconclusions.• These questions often contain words like assume,because, conclude, imply, infer,justify,reasonable,orsuggest.

• Plan: Determine whether you are looking for an assumption (evidence) or implication(conclusion).Thendeterminewhichclaimtheanswerissupposedtosupport(assumptions)orbesupportedby(implications).•Execute:Rereadtherelevantsentence,notingtheexplicitevidenceorconclusionsgiven.

•Forassumptionquestions,theansweriseithersimilartotheevidencegivenorlinkstheevidencetotheconclusions.•Forimplicationquestions,theansweriseithersimilartotheconclusionsgivenorisanotherlogicalconclusiononecoulddrawfromtheevidence.

•Answer:Match your expectations for the right answer. If there is no clearmatch, or if youcannotperformanyoftheearlierstepsoftheKaplanMethodforCARSQuestions,useaspecialformoftheprocessofeliminationcalledtheDenialTest:

•Negateeachanswerchoice.•Whichever answer choice—when negated—has the most detrimental effect on theargumentmadeinthepassageisthecorrectanswerchoice.

MCATExpertiseWhiletheDenialTestwillalwaysrevealthecorrectanswerinanInferencequestion,it’sverytime-consuming.IfyoucannotsetgoodexpectationsfortherightanswerduringtheExecutestep,triagethequestionandreturntoitlaterwiththeDenialTest.

STRENGTHEN–WEAKENStrengthen–Weakenquestions concern the logical relationshipbetweenconclusions andevidencethatstrengthens themor refutations thatweaken them.Thereare twosubtypesof these:strengthen–weaken within the passage, and strengthen–weaken outside the passage. Those using informationoutsidethepassagewillbediscussedlaterinthischapter.

•Thesequestionsoftencontainwordslikerelate,support,challenge,relevance,significance,orimpact.

Page 343: Edited By Deeangelee Pooran-Kublall, MD/MPH · 9.5 Preparing for the MCAT: Biochemistry in the Chemical and Physical Foundations of Biological Systems Section 9.6 Preparing for the

• These questions bring in a new piece of information rather than using information directlyfromthepassage.•Plan:Determinethetwoclaimsandtheconnectionbetweenthem;youwillusuallybegivenatleastoneoftheseelementsandwillhavetofindtheother(s).

•Identifywhereeachpieceoftheargumentcanbefound:inthequestionstem,inthepassage,orintheanswerchoices.• If no claims are given in the question stem, plan to triage it and answer it by theprocessofeliminationlater.•Ifoneclaimisgiveninthequestionstem,determinewhetheritisaconclusion,pieceofevidence,orarefutation.•Iftwoclaimsaregiveninthequestionstem,identifytherelationshipbetweenthem.

•Execute:Researchtherelevanttexttodeterminethemissingclaimortheconnectionbetweenthem.UseLogickeywordstohelpassembletheargument.•Answer:Match your expectations for the right answer. If there is no clearmatch, or if youcannot perform any of the earlier steps of the KaplanMethod for CARS Questions, use theprocessofelimination.

OTHERREASONINGWITHINTHETEXTQUESTIONSWhile youwillmostly see inference and assumption questions onCARSpassages, there are threemoretypesofquestionsthatyoushouldbeabletoidentifyinordertoapproachthemmostefficiently.

•Clarification questions ask for statements that are roughly synonymous, but the clarifyingstatementtendstobesupportingevidencefortheconclusionbecauseitismorespecificorexact.

•Thesequestionsoftencontainwordslikeclarify,explain,orreflect.•ApproachthesequestionslikeStrengthen–Weaken(Within)questions,exceptthatthemeaningsofthetwoclaimsshouldberoughlysynonymous.

•Weaknessquestionsaskforimplicitrefutationstoargumentsdiscussedinthepassage.• These questions often contain words like implicit weaknesses or reasonableobjections.•ApproachthesequestionsliketheDenialTestforInferencequestions,exceptthatthecorrect answer will be the most detrimental to the argument made in the passagewithoutbeingnegated.

•Paradoxquestionsaskfortheresolutionofanapparentlogicalcontradiction.•Thesequestionsoftencontainwordslikeparadox,dilemma,ordiscrepancy.•Approach thesequestionswith theprocessofelimination, crossingoutanyanswerchoice that is inconsistentwithoneorbothof theclaimsof theparadox,orwith thepassageasawhole.

KeyConceptAnsweringaWeaknessquestionisjustlikeusingtheDenialTestforInferencequestions.Thedifferenceisthatthecorrectanswerchoicewillbedetrimentaltotheargumentsinthepassagewithoutbeingnegated.

Page 344: Edited By Deeangelee Pooran-Kublall, MD/MPH · 9.5 Preparing for the MCAT: Biochemistry in the Chemical and Physical Foundations of Biological Systems Section 9.6 Preparing for the

KeyConceptAparadoxisasetoftwoclaimsthatappeartobeinconsistentonthesurface.ThecorrectanswerinaParadoxquestionwillbeconsistentwithbothoftheclaims,andwillusuallyattempttoexplainthesurfaceinconsistenciesbetweenthetwoclaims.

Page 345: Edited By Deeangelee Pooran-Kublall, MD/MPH · 9.5 Preparing for the MCAT: Biochemistry in the Chemical and Physical Foundations of Biological Systems Section 9.6 Preparing for the

15.4ReasoningBeyondtheTextQuestionsQuestions that fall into the broad category ofReasoning Beyond the Text are easy to identifybecausetheyalwaysinvolvenovelinformation(inthequestionstem,setofanswerchoices,orboth)thatisnotstatedorevensuggestedbythepassage,andwhichmaynotevenseemtoberelatedatfirst.There are two main categories of these questions: Apply and Strengthen–Weaken (Beyond). Thefundamentaldifferenceisoneofdirection:applyquestionsgofrompassagetonewsituation,while(Beyond)questionsgofromnewsituationtopassage.

MCATExpertiseIntensiveKaplanstudyofreleasedAAMCmaterialsseemstoindicatethatApplyquestionsappeartobeabitmorecommonthanStrengthen–Weaken(Beyond)questions.So,outoftheapproximately24ReasoningBeyondquestionsyou’llseeinaCARSsection,typicallyabout13willbeApplyquestions,andonlyaboutnineortenwillbeStrengthen–Weaken(Beyond)questions,withperhapsonethatdoesn’tneatlyfallintoeithertype.

APPLYApplyquestionsrequireyoutotaketheinformationgiveninthepassageandextrapolateittoanewcontext.Applyquestionsmayaskforoneofthreetasks.

•Theymayaskfortheauthor ’sresponsetoasituation,usingwordslikeresponse,reply,mostlikelytoagreewith,orleastconsistentwith.•Theymayaskforthemostprobableoutcomeinasituation,usingwordslikeoutcome,result,expectation,orconsequence.•Theymayaskforanexampleofanideadiscussedinthepassage,usingwordslikeexampleorinstance.•ThesequestionsoftenbeginwithwordslikeSuppose,Consider,orImagine.• Plan: If the question stem is long, jump to the end to determinewhat it’s asking. Read anyinformationgiveninthequestionstemclosely,lookingforhintsthatconnectittothepassage.•Execute:Rereadtherelevanttext,keepinginmindthespecifictypeofApplyquestioninvolved.

• For Response questions, determine the author ’s key beliefs, which are generallyreflectedinthepassageusingAuthorkeywords.• For Outcome questions, pay attention to cause–effect relationships in the passage,whicharegenerallyreflectedinthepassageusingLogickeywords.•ForExamplequestions, look for text thatprovidesdefinitions, explanations,or theauthor ’sownexample,notinganynecessaryorsufficientconditions.

•Answer:Match your expectations for the right answer. If there is no clearmatch, or if youcannot perform any of the earlier steps of the KaplanMethod for CARS Questions, use theprocessofelimination.

•Eliminateanyanswerchoicesthatareinconsistentwiththeauthor ’sviews,especiallyforResponsequestions.•Eliminateanyanswerchoicethatdoesnotcontainnecessaryconditions(whichmustoccurinallinstancesofaconcept),especiallyforExamplequestions.

STRENGTHEN–WEAKEN(BEYONDTHEPASSAGE)

Page 346: Edited By Deeangelee Pooran-Kublall, MD/MPH · 9.5 Preparing for the MCAT: Biochemistry in the Chemical and Physical Foundations of Biological Systems Section 9.6 Preparing for the

Like Strengthen–Weaken Within the Text questions, Strengthen–Weaken (Beyond the Passage)questionsconcernthelogicalrelationshipbetweenconclusionsandevidencethatstrengthensthemorrefutationsthatweakenthem.However,unliketheothertypeofStrengthen–Weakenquestion,atleastoneof theclaimsinvolvedwillnotbefromthepassage,butwillbeuniquetothequestionstemoranswerchoices.Strengthen–Weaken(Beyond)questionsarealsodistinctinthattheytreatthepassageasflexible,subjecttomodificationbyoutsideforces.

•Thesequestionsoftencontainwordslikerelate,support,challenge,relevance,significance,orimpact. In contrast to Strengthen–Weaken (Within the Passage) questions, they often containwordslikecouldorwould.•Readthequestionstemclosely,lookingforhintsofanalogytopartsofthepassage.•Plan:Determinethetwoclaimsandtheconnectionbetweenthem;youwillusuallybegivenatleastoneoftheseelementsandwillhavetofindtheother(s).

•Identifywhereeachpieceoftheargumentcanbefound:inthequestionstem,inthepassage,orintheanswerchoices.•Ifnoclaimsaregiveninthequestionstem,plantotriageitandansweritbyusingtheprocessofeliminationlater.•Ifoneclaimisgiveninthequestionstem,determinewhetheritisaconclusion,pieceofevidence,orrefutation.•Iftwoclaimsaregiveninthequestionstem,identifytherelationshipbetweenthem.

•Execute:Researchtherelevanttexttodeterminethemissingclaimortheconnectionbetweenthem.UseLogickeywordstohelpassembletheargument.•Answer:Match your expectations for the right answer. If there is no clearmatch, or if youcannot perform any of the earlier steps of the KaplanMethod for CARS Questions, use theprocessofelimination.

OTHERREASONINGBEYONDTHETEXTQUESTIONSThoughnot themostcommon typeofReasoningBeyond theTextquestions,ProbableHypothesis,Alternative Explanation, and Passage Alteration questions do occasionally appear. They can beidentifiedbythewordsinthequestionsthemselves.

•Probable Hypothesis questions ask for causes of new situations presented in the questionstem.

•Thesequestionsoftencontainwords likeprobablehypothesis, likelycause,ormostreasonableexplanation.• Approach these questions like Apply questions, except that you are looking foranalogouscause–effectrelationshipsinthepassage.

•Alternative Explanation questions ask for causes that differ from the ones given in thepassage,butwhichstillprovideanexplanationforaphenomenon.

• These questions often contain words like alternative explanation, other cause, ordifferentreason.•Approachthesequestionsbyeliminatinganyanswerchoicethatwouldnotleadtotheeffect in the question stem. If stuck between multiple answers, eliminate those thatconflictmostsignificantlywiththepassage.

•PassageAlterationquestionsaskforchangestheauthorcouldmaketothepassagetomakeitconsistentwithnewinformation.

•Thesequestionsoftencontainwordslikealter,change,orupdate.

Page 347: Edited By Deeangelee Pooran-Kublall, MD/MPH · 9.5 Preparing for the MCAT: Biochemistry in the Chemical and Physical Foundations of Biological Systems Section 9.6 Preparing for the

•Approachthesequestionsbylookingfortheanswerthatproducesthedesiredeffectwiththeleastamountofmodificationtotheideasinthepassage.

Page 348: Edited By Deeangelee Pooran-Kublall, MD/MPH · 9.5 Preparing for the MCAT: Biochemistry in the Chemical and Physical Foundations of Biological Systems Section 9.6 Preparing for the

15.5HowWillCARSQuestionTypesAppearontheExam?Asyou see in the reviewof all question types, questions canbeworded indifferentways, ask fordifferenttypesofanswers,andrequiredifferenttypesofthinkingandreasoning.However,allCARSquestions have one thing in common: they do not ask for any information outside the stated orimplicitinformationinthepassage,question,oranswers.Everythingyouneedtoansweraquestioncorrectlycanbeextrapolatedfromoneofthesethreeparts,evenwhenthequestionasksyoutoreasonanimplicitconclusionorapplyinformationtoanewsituation.Bringinginoutsideinformationwillsetyouonthewrongtrack,justasusinganirrelevantformulainsciencewilldeflectyoufromthecorrectanswer.Youcanbesure,then,thatregardlessofthequestiontypeorcomplexity,theMCATtestmakerhasgivenyoueverythingyouneedtoanswerthequestioncorrectly.

Page 349: Edited By Deeangelee Pooran-Kublall, MD/MPH · 9.5 Preparing for the MCAT: Biochemistry in the Chemical and Physical Foundations of Biological Systems Section 9.6 Preparing for the

15.6GettingtheEdgeUsingCARSQuestionTypesRememberwhat you read in the beginning of this chapter: there are no points in the passage; allpointsareinthequestions.Souseyourtimewisely.Don’tfocusandspendmostofyourtimereadingand understanding everything in the passage, and don’t even try tomemorize anything. Justmakeyour outline and refer to it to determinewhat part of the passage you need to review in order toanswerthequestion.Letthequestionsbeyourguidetowhatisimportantinthepassage.Ifthereisnoquestionaboutapartofthepassageyoureadslowlyandcarefully,youhavewastedyourtimeinthereading. Furthermore, when you can identify the question type, you can also identify the mostefficient and effective way to attack the question. As with all parts of theMCAT, the best way toimproveyourperformancewithCARSquestionsistopracticeandtheneffectivelyreview.Systematicreviewofhowyouanswerquestionswillhelpyoudetermineweaknessesandformulateastudyplantoamelioratethem.Ifyoufindthatyouarenotimprovingsteadily,takeanhonestlookatyourcriticalthinkingskillstodeterminewhatyouneedtodotoimproveyourscore.

Page 350: Edited By Deeangelee Pooran-Kublall, MD/MPH · 9.5 Preparing for the MCAT: Biochemistry in the Chemical and Physical Foundations of Biological Systems Section 9.6 Preparing for the

CHAPTERSIXTEEN

ReasoningWithinPassagesInthischapteroftheCriticalAnalysisandReasoningSkillsUnit,studentswill:

•Witness the utilization of the KaplanMethod for passages, questions, and arguments in thecontextofacoupleofexceptionallychallengingpassages• Practice the application of the KaplanMethod for passages, questions, and arguments withsomepracticepassages.

Page 351: Edited By Deeangelee Pooran-Kublall, MD/MPH · 9.5 Preparing for the MCAT: Biochemistry in the Chemical and Physical Foundations of Biological Systems Section 9.6 Preparing for the

16.1CARSWorkedExampleIAPHILOSOPHYPASSAGEEverystudentofthesciencesistaughttobewaryofmistakingcorrelationforcausation,butfewfullyappreciate the difference. Among the first to give an account of this distinction was David Hume(1711–76) in his early masterpiece A Treatise of Human Nature, the composition of whichcommencedattheprodigiousageof15,whenHumewashimselfbutastudent.Thoughoftenthoughtto be surpassed by his treatment of cause and effect in the more mature An Enquiry ConcerningHumanUnderstanding, thediscussion in theTreatise is remarkable for situatingcausation squarelywithinthecontextofhumanpsychology.

Hume’s analysis begins from a simple principle, “that all our ideas are copy’d from ourimpressions,”bywhichhemeans thatallknowledgeultimatelyderives fromsenseexperience—anaxiom he shares with fellow empiricists Locke and Berkeley. Causation is no different, and thusHumesetsout todetermine theoriginal impressions (todaymorecommonlycalledperceptions orsensations)whencethisideaderives.Accordingtohisanalysis,causationisnothingthatisintrinsictoanyparticularobjectbut ratheronlyemerges in the relationsbetween twoobjects,namely,acauseandaneffect.Henotesthreespecificrelationsthatare“essential”totheideaofcausation:contiguity(spatialproximity),temporalpriorityofcausebeforeeffect,andanadditional“necessaryconnexion”betweenthetwo.Thislast iswhatdistinguishescausationfrommerecoincidence,soHumedevotesseveralsectionstouncoveringwhatitis.

Hisconclusionmayshockthoseunaccustomedtoskepticalthinking.Humearguesthatthisnecessaryconnectionthatmakesoneentitythecauseofanotherispurelyacreationofthemind:“necessityisnothingbutthatdeterminationofthethoughttopassfromcausestoeffectsandfromeffectstocauses,according to their experienc’d union.” Such behavior is the product of custom, a mental habitestablishedafterwerepeatedlyperceivesimilarsequencesofcauseandeffect,suchaswhenamovingbilliard ball transfers momentum to a resting one after they collide—one of parlor gamesmanHume’sfavoriteexamples.Logicdoesnotdictatethatmomentumshouldbetransferredinacollision,for we could easily imagine one ball colliding into another and producing any number of otherresults;onlyexperienceshowsusitisso.

Hume’sinquiriesleadhimtoformulatethefollowingdefinition:“Acauseisanobjectprecedentandcontiguoustoanother,andsounitedwithit,thattheideaoftheonedeterminesthemindtoformtheidea of the other, and the impression of the one to form amore lively idea of the other.”He canremain confident that this determinationof themind is a customary connection, not a logical one,withhisastuteobservationthatallcausalreasoningpresupposes“thatthefutureresemblesthepast,”aclaimwhichneednotbetrue.Infact,suchaclaimcouldonlyeverbetakenonfaith—forhowcoulditbe proved? If onewere to argue that, in the past, whatwould become the future at that point hasalwaysturnedouttoresemblethepriorpast,sowecanexpectthesameinthefuture,onewouldbebeggingthequestion.Howdoweknowthelawsofnaturewon’tchangetomorrow?

IfHume is right, there is naught but the quirks of the psyche that properly distinguishes causationfromwhathedesignates“constantconjunction”(correlation).Laterthinkerswouldcometocallthisthe “problem of induction,” for it demonstrates how all inductive reasoning—which moves fromparticularpiecesofevidencetoauniversalconclusion—isultimatelyuncertain.

Page 352: Edited By Deeangelee Pooran-Kublall, MD/MPH · 9.5 Preparing for the MCAT: Biochemistry in the Chemical and Physical Foundations of Biological Systems Section 9.6 Preparing for the

P1.

P2.

P3.

P4.

P5.

1.Whichofthefollowingstatementsisassumedwithoutsupportinparagraph3?A.Humesaysthatthenecessaryconnectionbetweencauseandeffectissimplyamentalcustom.B.Momentumistransferredifanobjectinmotioncollideswithanobjectatrest.C.Themindinfersanecessaryconnectionafterexperiencingoneinstanceofacauseanditseffect.D.Thetruthofaclaimisnotlogicallydeterminedifitcanbeimaginedotherwise.

2.Accordingtothediscussioninthefinalparagraph,oneexampleof“inductivereasoning”wouldbeconcludingthatanautomobiletirewillnevergoflatonthebasisof:

A.repeateddailyobservationsofthetirestayingintact.B.thelogicalnecessityofalltiresbeingincapableofgoingflat.C.acustomaryhabitofjumpingtofaultyconclusions.D.knowledgethatthetireismadeofanindestructiblematerial.

3.Basedonthepassage,whatbestexplainshow“allcausalreasoningpresupposes‘thatthefutureresemblesthepast’”(paragraph4)?

A.Reasoningaboutcausalityisultimatelyfoundedonanassumptionestablishedbycustomratherthanbylogic.B.Thelawsofnaturemustbeunchangingfrompasttofuture.C.Itisnotnecessarilythecasethatthepastandfutureresembleoneanother.D.Pastconjunctionsofcauseandeffectwouldyieldnocausalknowledgeifthefutureoperatedbynewlawsofnature.

4.Theclaim“causationisnothingthatisintrinsictoanyparticularobject”mostnearlymeansthatobjects:

A.canneverbeadequatelycomprehendedbythehumanmind.B.arebynatureeffectsratherthancauses.C.canbeunderstoodascausesonlyrelativetootherentities.D.cannotbethecauseofotherobjects.

5.Whichofthefollowing,iftrue,wouldmostUNDERMINEHume’sconclusionsaboutcauseandeffect?

Page 353: Edited By Deeangelee Pooran-Kublall, MD/MPH · 9.5 Preparing for the MCAT: Biochemistry in the Chemical and Physical Foundations of Biological Systems Section 9.6 Preparing for the

A.Thereisnoreasontobelievethatthelawsofnaturewillchangetomorrow.B.Someknowledgeisattainablecompletelyindependentofexperience.C.Moststudentsofsciencefullyappreciatethedifferencebetweencorrelationandcausation.D.Claimsaboutcausalrelationscanalwaysbedoubted.

KeyConceptsQuestionTypesII:ReasoningWithintheTextInferenceQuestions–AssumptionSubtype

1.Whichofthefollowingstatementsisassumedwithoutsupportinparagraph3?A.Humesaysthatthenecessaryconnectionbetweencauseandeffectissimplyamentalcustom.B.Momentumistransferredifanobjectinmotioncollideswithanobjectatrest.C.Themindinfersanecessaryconnectionafterexperiencingoneinstanceofacauseanditseffect.D.Thetruthofaclaimisnotlogicallydeterminedifitcanbeimaginedotherwise.

Assessthequestion

Inadditiontothereferencetothethirdparagraph,themostimportantwordsinthequestionstemare“assumedwithoutsupport.”ThephrasingsuggestsanInferencequestion,mostlikelyanAssumption.Thecorrectchoicewillbeastatementthatmustbetrue,givenwhatissaidinthatparagraph,butthatis not backed upwith its own reasoning.Given that the scope is limited to only one paragraph, aquestionlikethisisprobablyworthdoingassoonasencountered.

PlanyourattackThereferencedoesnotspecifywhatpartofparagraph3tolookat,soitmaybeworthdoingaquickreread of the paragraph, or at least the label for P3 in the passage outline constructedwhile firstreading.Afterthispreparation,theprocessofeliminationisthewaytogo.Readeachchoice,andthenfindtherelevanttextinparagraph3.Ifachoiceisnotassumedbytheauthor,thenitcanbeeliminated.Butthat’snotall:thewordinginthestemsuggestsanotherwayananswercanberuledout;namely,ifachoiceisaclaimthattheauthorwouldendorsebutthatissupportedelsewhereinparagraph3.

Page 354: Edited By Deeangelee Pooran-Kublall, MD/MPH · 9.5 Preparing for the MCAT: Biochemistry in the Chemical and Physical Foundations of Biological Systems Section 9.6 Preparing for the

ExecutetheplanStartwithchoice(A),“Humesaysthatthenecessaryconnectionbetweencauseandeffectissimplyamentalcustom.”Therelevanttextisthefollowing:

“Humeargues that thisnecessaryconnection thatmakesoneentity thecauseofanother ispurelyacreationofthemind:‘necessityisnothingbutthatdeterminationofthethoughttopassfromcausestoeffectsandfromeffectstocauses,accordingtotheirexperienc’dunion.’Suchbehavioristheproductof custom, amental habit established afterwe repeatedly perceive similar sequences of cause andeffect...”

Clearlythelanguageinthischoiceiscobbledtogetherfromthesentencethatsurroundsthequotation,so itmust be something the authorwould endorse.However, this is ultimately a claim aboutwhatHumesays,andthebestwaytostrengthenclaimsthatofferinterpretationsofawriteristoquotethewriterdirectly.BecausetheauthordoesquoteHumedirectly,thisisnot“assumedwithoutsupport,”andsocanbeeliminated.

Forchoice(B),“Momentumistransferredifanobjectinmotioncollideswithanobjectatrest,”looktothispartoftheparagraph:

“Logic does not dictate that momentum should be transferred in a collision, for we could easilyimagineoneballcollidingintoanotherandproducinganynumberofotherresults;onlyexperienceshowsusitisso.”

Though the firstpartof this sentencestates that theclaimcannotbe supported logically, theclausefollowing the semicolon notes that this claim is demonstrated by experience. That reference tosupportrulesoutthischoiceaswell.

Withchoice(C),“Themindinfersanecessaryconnectionafterexperiencingoneinstanceofacauseanditseffect,”thereisactuallyaninconsistencywiththepassage.Thisisfoundwhentheauthornotesthatcustomis“amentalhabitestablishedafterwerepeatedlyperceivesimilarsequencesofcauseandeffect” (emphasis added). There’s no reason to believe that the mind does this after only oneexperience,socrossoffthisoptiontoo.

At thispoint,processofeliminationshows that thecorrect response ischoice(D), “The truthof aclaimisnotlogicallydeterminedifitcanbeimaginedotherwise.”

Answerbymatching,eliminating,orguessingExecuting theplan led to theeliminationof (A), (B),and (C), indicating that choice (D) is correct.ThisisconfirmedwiththefirstpartofthelastsentenceinP3:“Logicdoesnotdictatethatmomentumshouldbetransferredinacollision,forwecouldeasilyimagineoneballcollidingintoanotherandproducing any number of other results.” The Evidence keyword for suggests the author takes theability to imagine something to be otherwise as evidence that it is not dictated, or determined, bylogic.Butwhyshouldtheimaginationbeanyguideaboutwhatislogicallydetermined?Theauthor

Page 355: Edited By Deeangelee Pooran-Kublall, MD/MPH · 9.5 Preparing for the MCAT: Biochemistry in the Chemical and Physical Foundations of Biological Systems Section 9.6 Preparing for the

offersnoreasons,sothisassumptionisunsupported.

SIMILARQUESTIONS1.Whatwouldtheauthormostlikelyconsidertofallunderthecategoryof“anynumberofotherresults,”asdescribedattheendofparagraph3?

2.Inthethirdparagraph,theauthorusestheexampleofcollidingbilliardballsinorderto:3.Recentpsychologicalevidenceshowsthatpeoplesometimesassumeacausalconnectionafterseeingonlyoneexampleofacauseanditseffect.WhatrelevancedoesthishavetoHume’sargument?

TakeawaysWhenaquestionasksforastatement“assumedwithoutsupport,”thecorrectanswermaybedirectlystatedoronlyimpliedinthepassage.Regardless,theauthorwillwritenothingelseaboutwhytobelievesuchastatement.

ThingstoWatchOutForWatchoutforwhenaquestionstempresentsmultiplewaysofeliminatingincorrectresponses.

SOLUTIONSTOSIMILARQUESTIONS

1.Thecontextofthephraseis,“Logicdoesnotdictatethatmomentumshouldbetransferredinacollision,forwecouldeasilyimagineoneballcollidingintoanotherandproducinganynumberofotherresults.”Inshort,anyanswerthatindicatedsomeconsequenceotherthanthetransferofmomentum—suchasthefirstballpassingthroughthesecond,theballsbeingannihilated,orthefirstballliftingoffthegroundandhoveringinmid-airwhilethesecondballboresaholeintotheground—wouldwork.

2.ThepurposeofthebilliardballsexampleistoprovideamoreconcretebasisfortheabstractclaimsmadeinP3.ThecorrectanswertoaquestionlikethiswouldeithersaysomethingalongthoselinesorsimplystresshowitfunctionstosupportHume’saccountofcausation.

3.TherelevancetoHume’sargumentisrelativelyminimal.Thoughthepassagenotesthatcausalconnectionsaremadeafterrepeatedinstancesofseeingacauseanditseffect,itneversuggeststhatthisalwayshastohappen.Sincethequestionstemonlysays“sometimes,”thereisnotasignificantchallenge.Thecorrectchoicewillsaysomethingtotheeffectthatthisevidenceneitherstrengthensnorweakenstheargumentconsiderably.

KeyConcepts

Page 356: Edited By Deeangelee Pooran-Kublall, MD/MPH · 9.5 Preparing for the MCAT: Biochemistry in the Chemical and Physical Foundations of Biological Systems Section 9.6 Preparing for the

QuestionTypesIII:ReasoningBeyondtheTextApplyQuestions–ExampleSubtype

2.Accordingtothediscussioninthefinalparagraph,oneexampleof“inductivereasoning”wouldbeconcludingthatanautomobiletirewillnevergoflatonthebasisof:

A.repeateddailyobservationsofthetirestayingintact.B.thelogicalnecessityofalltiresbeingincapableofgoingflat.C.acustomaryhabitofjumpingtofaultyconclusions.D.knowledgethatthetireismadeofanindestructiblematerial.

AssessthequestionTheparagraphreference,quotation,anduseof the term“example”help to identify thisquestionasone of the subtypes of Apply questions. The stem provides a lot of clues, so this one is worthattemptingnow.

PlanyourattackInordertofurnishanexampleofaconceptfromapassage,itisessentialtobeclearonthemeaningof that concept. Go back to the passage and learn as much as possible about the cited term: “allinductivereasoning—whichmovesfromparticularpiecesofevidencetoauniversalconclusion—isultimatelyuncertain.”Thedashessetapartadefinitionfortheterm,recognizingtwokeycomponentsthatmakeupinduction.Thequestionstemprovideshalfofthis,the“conclu[sion]thatanautomobiletirewillnevergoflat.”Thisisauniversalclaim(onethatappliesineverycase),assuggestedbytheword “never,” which admits no exceptions. The words “on the basis of” confirm that the correctanswerwillbetheevidencethatsupportsthisconclusion.

ExecutetheplanThecorrectchoicemustexemplify“particularpiecesofevidence,”butwhatdoesthisreallymean?ThefinalparagraphnoteshowtheproblemthatHumeidentifiedwithcausationwaslatercalledthe“problemofinduction,”suggestingthatcausalreasoningandinductivereasoningarecloselyrelated(althoughtheauthordoesnotspecifypreciselywhatthisrelationis).Becauseofthisconnection,itispossibletousetheexamplesthatthepassageprovidesofcausalreasoningasabasisforpredictingwhatwillcountasinductivereasoning.

The third paragraph offers a crucial hint,with its reference to the formation of customary casual

Page 357: Edited By Deeangelee Pooran-Kublall, MD/MPH · 9.5 Preparing for the MCAT: Biochemistry in the Chemical and Physical Foundations of Biological Systems Section 9.6 Preparing for the

associations “after we repeatedly perceive similar sequences of cause and effect.” These repeatedperceptionsare the“particularpiecesofevidence”mentionedinP5.Putting itall togethernow, thecorrectanswershouldmakereferencetomultiplecasesinwhichthetiredoesnotgoflat.

Answerbymatching,eliminating,orguessingChoice(A) presents an immediatematch for these expectations. The general conclusion, “this tirewillnevergoflat”couldbeinductivelysupportedby“repeateddailyobservationsofthetirestayingintact.”

SIMILARQUESTIONS1.Whichofthefollowingcomesclosestinmeaningto“quirksofthepsyche”(paragraph5)?2.Whatisoneimplicationthatthe“problemofinduction”posesforknowledgeinthesciences?3.Judgingonthebasisofthepassageasawhole,whatconstitutesthedifferencebetweencorrelationandcausationthattheauthorasserts“fewfullyappreciate”(paragraph1)?

TakeawaysAnythingthatappearsinaquestionstemshouldbetakenforgranted,regardlessofhowpeculiaritmayseem.Theclaimthatatirewillnevergoflatisclearlyfalse—thephysicalworldprovidesampleevidencethateverythingeventuallysuccumbstoentropy.However,whatmattersforthisquestionisthereasoningbehindtheconclusion,notitstruthvalue.

ThingstoWatchOutForWhileeverychoiceprovidessomebasisformakingthatconclusion,themosttemptingis(C),whichechoesalotofthelanguagefromthepassage.Theproblemwithit,though,isthatthereisnoindicationinthepassagethatinductiveconclusionsmustbe“faulty”orwrong,butonlythattheyare“uncertain,”whichisfarlessnegative.

SOLUTIONSTOSIMILARQUESTIONS

1.Asnotedintheearlierdiscussioninthetwoprecedingparagraphs,it’sthecustomaryassociationmadebythemindbetweenacauseanditseffectthatconstitutestherelationshipofcausation,accordingtoHume.Theanswercouldsimplybetheword“custom”oranelaborationinmorewordsuponthatconcept.

2.Though“knowledgeinthesciences”isneverexplicitlymentionedinthepassage,thediscussioninP1directlyconnected“student[s]ofthesciences”tothedistinctionbetweencorrelationandcausation.Thus,thecorrectchoicemightpointoutthatitunderminesthatdistinctionbymakingitaproductofthemind,notofthephysicalobjectsthemselves.Alternatively,therightanswermightsimplyreflecttheideasattheendofP5andnotehowtheproblemmakesscientificconclusions,derivedfrominductivereasoningthatusesexperiments(scientificallycontrolledexperiences)as

Page 358: Edited By Deeangelee Pooran-Kublall, MD/MPH · 9.5 Preparing for the MCAT: Biochemistry in the Chemical and Physical Foundations of Biological Systems Section 9.6 Preparing for the

evidence,uncertain.

3.Theanswertothisquestionwillmostlikelybefoundinthelastparagraph,whentheauthorreturnstotheissueasitwasframedintheintroduction.Expectatrapanswertodiscussthedifferencebetweencausationandcoincidence,discussedinP2.Acorrelationor“constantconjunction”hastoinvolverepeatedinstances,makingitmorethanamerecoincidence.Whatdifferentiatesconstantconjunction(repeatedexamplesofBfollowingA)fromcausation(AbeingthecauseofB)issimplycustom,accordingtoHume,sothecorrectresponsewillsayasmuch.

3.Basedonthepassage,whatbestexplainshow“allcausalreasoningpresupposes‘thatthefutureresemblesthepast’”(paragraph4)?

A.Reasoningaboutcausalityisultimatelyfoundedonanassumptionestablishedbycustomratherthanbylogic.B.Thelawsofnaturemustbeunchangingfrompasttofuture.C.Itisnotnecessarilythecasethatthepastandfutureresembleoneanother.D.Pastconjunctionsofcauseandeffectwouldyieldnocausalknowledgeifthefutureoperatedbynewlawsofnature.

AssessthequestionQuestions that ask forexplanationswilloften fall intooneof the two“Other” types.Sincenonewsituationissuggested,thisisOtherReasoningWithintheText,ratherthanOtherReasoningBeyondtheText.SinceOtherquestionsareoftendifficult,thismaybeworthsavingforthelastquestionofthepassage,thoughthedirectquotationwillmakeresearchabiteasier.

PlanyourattackPutthephraseintocontext:

“[Hume]can remainconfident that thisdeterminationof themind isacustomaryconnection,notalogical one, with his astute observation that all causal reasoning presupposes ‘that the futureresembles thepast,’aclaimthatneednotbe true. Infact,suchaclaimcouldonlyeverbe takenonfaith—for how could it be proved? If onewere to argue that, in thepast,whatwould become thefutureatthatpointhasalwaysturnedouttoresemblethepriorpast,sowecanexpectthesameinthefuture, one would be begging the question. How do we know the laws of nature won’t changetomorrow?”

Thisisthemostintricateideaintheentirepassage,thoughit’sgivenarelativelybriefformulation.Theauthor(echoingHume)issayingthattheclaim“thefutureresemblesthepast”isanassumption

Page 359: Edited By Deeangelee Pooran-Kublall, MD/MPH · 9.5 Preparing for the MCAT: Biochemistry in the Chemical and Physical Foundations of Biological Systems Section 9.6 Preparing for the

in any argumentmadewith a conclusion about cause and effect. So, to rely upon an example thepassage itself furnishes, throughout human history there have been repeated observations that acollision causesmomentum to be transferred fromamoving object to a static (but not immobile)one, such as seenwith a rolling ball hitting a standing ball in games like billiards or croquet. Inphysics,achangeinmomentumisknownasanimpulse,soabbreviatethisideaas,“acollisioncausesanimpulse”orC→I.

Past observations have always confirmed C → I, but to be able to claim that this statement isuniversallyorgenerallytrue,onemustalsosayC→Iisequallytrueforthefuture.Sincethefutureissomethingthat,bydefinition,hasnotyetbeenobserved,therecanbenoexperiencetodrawontosupportC→I.ToarguethatC→Iistrue,onemustassumethatwhathasnotbeenexperienced(thefuture)willproducethesameresults(moreexamplesofC→I)aswhathasbeenexperienced(thepast).

Thisclaim,“thefutureresemblesthepast,”couldotherwisebethoughtofastheideathatthelawsofnature(suchasC→I)areunchanging.Ofcourse,itcanreceivethesametreatmentasC→Iinthepreviousparagraph.Backinthe18thcentury,whenHumewaswriting,thereweremanyobservationsthatsuggestedthelawsofnaturewereuniform,thatthefutureresembledthepast.Sincethattime,inwhat would be the future relative to the 18th century, there have been yet more observationssupporting this uniformity. Thus, it is safe to conclude that, in the past, the laws of nature haveremaineduniform(thefuturehasresembledthepast).Butnow,onceagain,therearenoobservationsof the future (relative to the present) to rely upon to confirm that this uniformity would holdtomorrowandbeyond.So,toprovethatthefuturealwaysresemblesthepast,onehastoassumethatinthisrespectthefuturewillresemblethepast,whichisknownas“beggingthequestion”or“circularreasoning.” This type of reasoning is fallacious because it takes for granted what it purports todemonstratewithevidence.

ExecutetheplanHaving unpacked the claim, now it’s time to find the answer choice that actually provides theexplanation requested. Begin with (A), “Reasoning about causality is ultimately founded on anassumption established by custom rather than by logic.” The last part is simply a reference to theclaim that the future resembles the past, which means that altogether this choice is simply arestatementofthequotedlinefromthequestionstem.Thisprovidesatmostaminimalexplanation,soitisunlikelytobethecorrectanswer,particularlysincethequestionasksfor“whatbestexplains.”

Thestatementinchoice(B)(“Thelawsofnaturemustbeunchangingfrompasttofuture”)is,aswasnoted in thePlan step, simply anotherwayof saying the future resembles thepast.This is anotherrestatement,thistimeofonlypartofthequotationinthequestionstem,soit’sevenlesshelpfulthanthepreviouschoice.Itcansafelyberuledout.

While (C), “It is not necessarily the case that the past and future resemble one another,” does notrehashthequotedclaim,itiseffectivelyareiterationofsomethingsaidinP4,thattheclaimthatthefutureresemblesthepast“neednotbetrue.”Thisisanotherwayofsayingtheclaimisnotlogicallyornecessarilytrue,butitdoesnotexplainhowtheclaimispresupposedincausalreasoning.Thus,thischoicecanalsobeeliminated.

Page 360: Edited By Deeangelee Pooran-Kublall, MD/MPH · 9.5 Preparing for the MCAT: Biochemistry in the Chemical and Physical Foundations of Biological Systems Section 9.6 Preparing for the

Theprocessofeliminationwouldsuggesttheanswermustbechoice(D),“Pastconjunctionsofcauseandeffectwouldyieldnocausalknowledgeifthefutureoperatedbynewlawsofnature.”Besuretoevaluateitfirsttoensurethatitissuperiortotheminimalexplanationgivenby(A).

Answerbymatching,eliminating,orguessingUpon examination, (D) provides by far the best explanation, going beyond themere repetition ofclaims to some of the underlying ideas in the text. “Causal knowledge” must be referring to theknowledgegainedfromcausalreasoning,thatis,theconclusions,whicharebackedbyobservation.Now, theseobservations (“past conjunctionsof cause and effect”)wouldbeworthless if the futurestopped resembling the past, because that past knowledge would no longer apply. The author ’srhetoricalquestionattheendofP4(“Howdoweknowthelawsofnaturewon’tchangetomorrow?”)ishintingatthissameidea,butthischoicespellsitoutexplicitly.

SIMILARQUESTIONS1.Whatprovidessupportfortheauthor ’scontentionthattheclaimconsideredinP4“couldonlyeverbetakenonfaith”?

2.Howwouldtheauthormostlikelyexplainthesuggestionmadeinthefourthparagraphthatit“neednotbetrue”thatthepastandfutureresembleeachother?

3.WhichofthefollowingwouldHumemostlikelycharacterizeasaninstanceof“theimpressionoftheone[determiningthemind]toformamorelivelyideaoftheother”?

TakeawaysWhenaskedtoprovideanexplanationforaclaim,thecorrectanswerwilldomorethanjustmakethesameassertioninslightlydifferentlanguage,butwillofferanaccountofwhyorhowitistrue.

ThingstoWatchOutForQuestionstemsthatusesuperlativelanguage(most,best,andother-estwords)willoccasionallyfeaturechoicesthatseemtoprovideminimalanswerstothequestion,butwhichwillpaleincomparisontothecorrectresponse.

SOLUTIONSTOSIMILARQUESTIONS

1.Theauthornotesthattheproofofferedforthisclaimbegsthequestion,thatis,thatthesupposeddemonstrationoftheclaimrequiresassumingtheclaimistruetobeginwith.Astatementcannotsupportitselfanymorethanahousecouldserveasitsownfoundation.Thisdemonstrationthatthereasoningiscircularinturnservesassupportforthecontentionfromthequestionstem.Aclaimtakenonfaithisonebelievedwithoutsufficientevidence,andcircularreasoningisdefinitelynotsufficientevidence.Theexactformofthecorrectchoicewillbetoughtopredict,butitshould

Page 361: Edited By Deeangelee Pooran-Kublall, MD/MPH · 9.5 Preparing for the MCAT: Biochemistry in the Chemical and Physical Foundations of Biological Systems Section 9.6 Preparing for the

generallyreflectthislineofthinking.

2.ThediscussioninP3wouldactuallybehelpfulinreachingananswerforthisquestion.Astatementthatneedstobetrue(isnecessarilytrue)isonethetruthofwhichisdictatedbylogicalone.RecallthattheexampleinP3suggestedaclaimwasnotlogicallytruebecauseotheroutcomescouldbeimagined:“Logicdoesnotdictatethatmomentumshouldbetransferredinacollision,forwecouldeasilyimagineoneballcollidingintoanotherandproducinganynumberofotherresults.”Thus,theauthorwouldlikelysaysomethingsimilarfortheclaimthatthefutureresemblesthepast,assertingperhapsthatonecouldimaginethelawsofnaturechanging.Ifitcanbeimaginedthatthefuturedoesnotresemblethepast,usingthesamereasoningfromP3,thenitneednotbetruethatthefutureresemblesthepast.Thecorrectanswershouldbeconsistentwiththisanalysis.

3.ThisquestionrequiresunpackingthearchaiclanguagethatHumeusesinhisdefinitionofcause.InP2,theauthorequatesimpressionswith“perceptionsorsensations,”whileit’sclearfromtheexamplesusedinP3,thatthetermideascouldbeusedtorefertoimages,suchascollidingbilliardballs.Consequently,thecorrectresponsewillprobablysuggestaperceptionofoneentity(eitheracauseoraneffect)leadingthemindtoimaginetheappropriatecounterpart.

KeyConceptsQuestionTypesI:FoundationsofComprehensionDefinition-in-ContextQuestions

4.Theclaim“causationisnothingthatisintrinsictoanyparticularobject”mostnearlymeansthatobjects:

A.canneverbeadequatelycomprehendedbythehumanmind.B.arebynatureeffectsratherthancauses.C.canbeunderstoodascausesonlyrelativetootherentities.D.cannotbethecauseofotherobjects.

AssessthequestionPhrases like “most nearlymeans” almost always indicates aDefinition-in-Context question,whichcarriesthetaskofidentifyingthemeaningofaterm,statement,orothersegmentofthetext.Thesetend tobemorestraightforward, soeven though theparagraph reference isnotgiven, thiskindofquestionisworthattemptingwhenfirstencountered.

Page 362: Edited By Deeangelee Pooran-Kublall, MD/MPH · 9.5 Preparing for the MCAT: Biochemistry in the Chemical and Physical Foundations of Biological Systems Section 9.6 Preparing for the

PlanyourattackAsthenameofthequestiontypesuggests,goingbacktothepassageforcontextwillbeessential.Therelevantpartof thepassageis locatedinP2:“Accordingto[Hume’s]analysis,causationisnothingthatisintrinsictoanyparticularobjectbutratheronlyemergesintherelationsbetweentwoobjects,namely, a cause and an effect.” The part left out of the question stem provides a useful contrast,suggestingadistinctionbetweenapropertythatcanbefoundinasingleobject(say,massorvolume)andonethatcanonlybefoundwhenmultipleobjectsarerelatedtogether.Thecontextsuggeststhatcausationisofthelattersort:itiswrongtolabelsomethingasacauseunlessitproducessomeeffect;norshouldsomethingbelabeledeffectwithouthavingacorrespondingcause.

ExecutetheplanAfterlocatingandanalyzingtherelevanttext,it’shelpfultomakeapredictionabouthowtocompletethesentenceafterthecolon.Forinstance,theequivalentclaimcouldbethat“objectscannotbecausesbythemselves”(whichstressesthefirsthalfofthesentencefromP2)orsomethinglike“objectsarenotcausesunlesstheyhaveeffects”(whichstressesthesecondhalf).

Answerbymatching,eliminating,orguessingChoice(C) presents amatch for the latter prediction, saying “objects can be understood as causesonlyrelativetootherentities.”

SIMILARQUESTIONS1.Whichofthefollowing,accordingtotheauthor ’saccountofHume,isNOTanessentialaspectofcausalrelationships?

2.AccordingtotheNewtonianaccountofgravitywidelyacceptedwhenHumewrotetheTreatise,bodiesexertgravitationalforcesononeanother,evenwhenseparatedatagreatdistance.WhatimpactdoesthishaveonHume’saccountofcausation?

3.Inthesecondparagraph,theauthorstatesthat“Humesetsouttodeterminetheoriginalimpressions…whencethisidea[ofcausation]derives.”Whatdoestheauthormostlikelymeanbythis?

TakeawaysHavingabroaderrangeofexpectations,suchasrecognizingtwoequivalentwaysofmakingthesameprediction,canleadtohominginonthecorrectchoicemorequickly.

Page 363: Edited By Deeangelee Pooran-Kublall, MD/MPH · 9.5 Preparing for the MCAT: Biochemistry in the Chemical and Physical Foundations of Biological Systems Section 9.6 Preparing for the

ThingstoWatchOutForAnswerslike(A)canbetempting,becausetheymightseemtohaveasimilartoneaslanguageinthepassage,suchas“ultimatelyuncertain”inP5.However,thischoiceexcludestheessentialelementofcausality,soitcannotcapturethemeaningofthequotedphrase.

SOLUTIONSTOSIMILARQUESTIONS

1.Thereferencetoessential(thoughleftoutofquotesinthestem)suggeststhelistofferedinP2:“contiguity(spatialproximity),temporalpriorityofcausebeforeeffect,andanadditional“necessaryconnexion”betweenthetwo.”Thewronganswersarelikelytobevariantsofeachofthosethreeitems,whilethecorrectresponsewillbeanotheridea,quitepossiblyliftedfromanotherportionofthepassagetoseemmoreplausible,suchas“havingauniversalconclusion”(whichappliestoinductivereasoning,nottheconceptofacause-and-effectrelationship).

2.OneoftherequirementsofHume’sunderstandingofcausationnotedinP2is“contiguity(spatialproximity),”thatis,nearnessinspace.Ifoneobjectcausesanotherobjectatagreatdistanceawaytobeattracted,thenthiswouldsuggestthatHume’srequirementisnotbeingsatisfied.Consequently,thecorrectresponsewouldhavetosaythatHume’saccountwasinsomewayweakened.

3.ThecorrectresponsetothisDefinition-in-Contextquestionwillreflectthefollowingconcerns.Thepassagenotesinaparenthesishowthetermimpressionsisequivalentto“perceptionsorsensations,”thatis,thecomponentsofsenseexperience.Thus,ifHumeisrightinsayingthatallknowledgeisderivedfromexperience,theideaofcausationshouldbederivedfromexperienceaswell.TheauthorissimplydescribinghowHumeusesthat“simpleprinciple”todirecthisanalysis.

KeyConceptsQuestionTypesIII:ReasoningBeyondtheTextStrengthen–Weaken(BeyondPassage)Questions–WeakenSubtype

5.Whichofthefollowing,iftrue,wouldmostUNDERMINEHume’sconclusionsaboutcauseandeffect?

A.Thereisnoreasontobelievethatthelawsofnaturewillchangetomorrow.B.Someknowledgeisattainablecompletelyindependentofexperience.C.Moststudentsofsciencefullyappreciatethedifferencebetweencorrelationandcausation.D.Claimsaboutcausalrelationscanalwaysbedoubted.

Page 364: Edited By Deeangelee Pooran-Kublall, MD/MPH · 9.5 Preparing for the MCAT: Biochemistry in the Chemical and Physical Foundations of Biological Systems Section 9.6 Preparing for the

AssessthequestionThe phrase “if true” typically indicates a Strengthen–Weaken (Beyond Passage) question, andundermineisjustasynonymofweaken.Thevaguelyworded“Hume’sconclusionsaboutcauseandeffect” could refer to just about any part of the passage. This question will likely require someprocessof elimination (especiallygiven thewordmost) andmayalso require research inmultipleparagraphs, so it’sonebest reserved for later, after severalotherquestionshavebeen successfullytackledandthepassageismorefamiliar.

PlanyourattackEvaluatetheimpactthateachchoice’struthwouldhaveonargumentsmadeinthepassage.Itdoesnotmatterhowstrongthelanguageisorhowimplausiblethesituationdescribedmaybe—thequestionstem asks only for themost powerful negative effect onHume’s ideas in particular. Plan to checkeverychoiceandbewaryifthechallengeisbarelysignificant(suchachoicewouldonlybetrueifallthealternativeswerecompletelyoffthemark,whichisrelativelyrare).

Theanswerchoicesmaycontainlanguagesimilartothepassage,theymaybequitenovel,ortheremayevenbeamix.Regardlessofhowclosethechoicesmirrortheoriginaltext,locatetherelevantportions of the passage for each one, and look out for outright contradictions and other types ofchallenges.BecauseHumeisthetarget,limitthissearchtoquotations,paraphrases,andexplanationsofHume’sideas—nottheauthor ’sownopinions.

ExecutetheplanFor(A),“Thereisnoreasontobelievethat thelawsofnaturewillchangetomorrow,”therelevanttextisinP4,attheendofananalysisofadirectquotefromHume,therhetoricalquestion,“Howdoweknowthelawsofnaturewon’tchangetomorrow?”Asarhetoricalquestion,ithasoneanswertheauthor immediately expects readers to jump to: “Wedon’t know.” In short, this rhetorical questionamountstotheclaimthatit’snotpossibletoknowwhetherthelawsofnaturewillremainthesameinthefuture.

Evenifitweretruethatthereisnoreasonatalltoexpectthoselawstochangetomorrow,theabsenceof evidence is not equivalent to the evidence of absence. In other words, just because there is noreasontobelievethatsomeeventwillhappendoesnotbyitselfgiveanaffirmativereasontobelievethatitwon’thappen.Whatthismeansisthatthetruthof(A)doesnotsignificantlychallengethetruthofthestatementthatthelawsofnaturewillchangetomorrow—yettheactualassertionattributedbytheauthortoHumeisnotthatthiswilldefinitelyhappen,butonlythatit’snotpossibletoknow,oneway or another, whether it will.Choice (A) certainly does not undermine that idea, since it alsoamountstoaclaimofignorance.

Choice(B)directlypertainstothediscussionatthestartofP2:

Page 365: Edited By Deeangelee Pooran-Kublall, MD/MPH · 9.5 Preparing for the MCAT: Biochemistry in the Chemical and Physical Foundations of Biological Systems Section 9.6 Preparing for the

“Hume’s analysis begins from a simple principle, ‘that all our ideas are copy’d from ourimpressions,’ bywhichhemeans that all knowledgeultimatelyderives fromsense experience—anaxiomheshareswithfellowempiricistsLockeandBerkeley.”

Thelanguageofbegins,simpleprinciple,andaxiom(anothernameforafoundationalassumption)allindicatethatthisisaclaimthatHumetakesforgranted,whilethecolorfulquotationshowsthatit’sanacknowledgedassumption,onethatHumedeliberatelycallsattentionto.Asanaxiom,itservesasanecessaryconditionfortheconclusionsthatHumereaches.Thus,ifit’snottruethat“allknowledgeultimatelyderivesfromsenseexperience,”thenHume’sargumentisinconsiderabletrouble.

Given that (B) states that “Someknowledge is attainable completely independent of experience,” itposesadirectcontradiction toHume’s foundationalprinciple. Itwouldbedifficult toofferamoresubstantial challenge than attacking agrounding assumption, so this is almost certainly the correctchoice.IfrunninglowontimeonTestDay,selectthisandmoveon.Iftimingisnotanissue,itwouldbefinetocontinuecheckingtheotherchoicestoensurethatthisonehasthemostnegativeimpact.

Although (C), “Most students of science fully appreciate the difference between correlation andcausation,”directlyconflictswiththeopeningsentence,“Everystudentofthesciencesistaughttobewary of mistaking correlation for causation, but few fully appreciate the difference,” this claimconstitutestheauthor ’slead-intothediscussionofHume.Itunderminestheauthor’sframingoftheissue,butitisnotrelevanttoanyideatiedtoHumehimself.

As expected, choice (D) also fails to have a negative impact. In fact, the statement “Claims aboutcausal relations can always be doubted,” is completely consistent with the author ’s concludingthoughtinP5,that“allinductivereasoning. . . isultimatelyuncertain.”Sinceitsupportstheauthor,andtheauthorisnevercriticalofHume,thereisnowaythischoicecouldundermineHume.

Answerbymatching,eliminating,orguessingAftercheckingtheotheranswers,it’sclearthatonlychoice(B)hadasignificantlynegativeimpactonanythingthatcanbeconnectedimmediatelytoHume.Infact,itsimpactwassodetrimental(analogousto demolishing a house’s foundation) that such an answer can safely be taken as correct withoutconsultingtheremainingoptions.

SIMILARQUESTIONS1.Whichofthefollowingobservations,ifvalid,wouldmostbolstertheHumeanaccountofcausalityconsideredinthepassage?

2.Whatroledoesthequotedclaim“thatallourideasarecopy’dfromourimpressions”playinthepassage?

3.Whatistheauthor ’sprimaryargumentaboutcausation?

Takeaways

Page 366: Edited By Deeangelee Pooran-Kublall, MD/MPH · 9.5 Preparing for the MCAT: Biochemistry in the Chemical and Physical Foundations of Biological Systems Section 9.6 Preparing for the

It’salwayspossibletoreviseaPlanasitisbeingExecuted,iftheanswerchoicesleadinsuchadirection.Justbecauseyoubegintheprocessofeliminationdoesnotmeanthatyouhavetofinishitwhenyoucomeacrossachoicethatperfectlyanswersthequestion.Savethetimereadingtheremainingwrongoptionsforworkingonanotherquestion.

ThingstoWatchOutForInpassageswithmultipleviewpoints,suchastheauthorandanotherwriterwhotheauthordiscusses,watchoutforchoicesinquestionsofalltypesthatreflectthewrongviews.

SOLUTIONSTOSIMILARQUESTIONS

1.ThisisessentiallytheStrengthencounterparttothequestionoriginallyposed.Liketheoriginal,italsoposessomedifficultyinsettingexpectations.Thecorrectchoicemaysimplymimicapieceofsupportingevidencefromthepassage,orinsteadoffersomethingthat’sonlyanalogous.EvenwithReasoningBeyondtheText,therewillalwaysbesomeconnectiontotheactualevidencepresentedinthepassage.

2.Asnotedabove,thisisaprinciplethatservesasaprerequisite(necessarycondition)forHume’streatmentofcauseandeffect.Thecorrectchoicemightcallitanassumptionorsomeequivalentphrase.

3.Thecorrectresponsetothisquestionwilleithersimplypresenttheconclusiontotheargument,namelythatonlycustomseparatescausationfromcorrelation,oritwillpresenttheconclusionalongwithsomeevidence.Inthelattercase,besuretheevidenceisconsistentwithwhatispresentedinthepassage,becausetwoanswerchoicescouldhavethesameconclusion(thelanguagemayvaryonlyslightlyorevenbeidentical)anddifferonlywithrespecttotheevidencecited.

KeyConceptsQuestionTypesII:ReasoningWithintheTextOtherReasoningWithintheTextQuestions

Page 367: Edited By Deeangelee Pooran-Kublall, MD/MPH · 9.5 Preparing for the MCAT: Biochemistry in the Chemical and Physical Foundations of Biological Systems Section 9.6 Preparing for the

16.2CARSWorkedExampleIIASOCIALSCIENCEPASSAGEAccordingtoa2014reportbyOxfamInternational,theworld’srichest85peoplepossessasmuchasthepoorest3.5billion,andnearlyhalfofallwealthisownedbyjust1percentoftheglobalpopulace.Many measures of inequality, when plotted annually, adopt a potentially disconcerting U-shape:decliningafterthereformsimplementedbymanyindustrializedpopulationsinthewakeoftheGreatDepression, inequality is now returning to levels not seen since the 1920s—and on the verge ofsurpassingthem.Withstatisticslikethese,elitefigures,fromthePresidentoftheUnitedStatestothePopeoftheCatholicChurch,havehadtoadmitthatinequalityhasbecomeaprominentissue.

Thetrulyinterestingquestionconcernsneithertheexistenceofeconomicinequalitynorthefactofitscontinuinggrowth,butitsorigin.Whathascaused—andevenmorecrucially,whatisperpetuating—this polarization inwealth?Afterweighing the evidence,my contention is that this development isultimately apolitical outcome, not aneconomic one.By this Imean it is the product of deliberatedecisionsbypoliticalleaders(elected,appointed,orotherwise)andotherinfluentialsocioeconomicelites,ratherthanthenaturalresultofmarketforces,asmanyotherscholarshavesuggested.

At issue are the differing political fortunes of two factions, the centrality of which have beenrecognizedbyeconomists fromKarlMarx toThomasPiketty, laborersandcapitalists, theclassofworkersandtheownerswhoemploythem.Whenlaborwaspoliticallyascendant(forinstance,intheaftermathofFDR’sNewDeal),inequalitydecreased.However,withtheriseofthepoliticalideologyofneoliberalism(embracedbyleadersonbothendsoftheacceptedpoliticalspectrum,liketheUnitedKingdom’s Margaret Thatcher and Tony Blair and the United States’s Ronald Reagan and BillClinton),inequalitybegantorebound.

Neoliberalismpurports topromote“freemarkets,”butperhapsabettercharacterizationof it is thepromotionofthefreemovementofcapital.Capitaltendstohaveitsownlawofgravity,exceptthatitseemstofallupwards,accumulatinginfloatingparadisesknownastaxhavensthatcontainthecoffersoftheplanet’swealthiest.Fewpeoplewouldvoteexplicitlyforthisprogram,yetmostgovernmentsindemocraticnationsthroughouttheworldarefilledwithofficialswhoactinwaysthatfurtherthepolarizationofwealth,whetherknowinglyorunwittingly.

Muchoftheascendanceofneoliberalideologystemsfromadiscrepancyinorganization.TheUnitedStates presents a clear-cut example of one side of this phenomenon. From 1940 to 1980, betweenaboutone-fifthandone-quarterofallemployedworkersintheUnitedStatesweremembersoflaborunions. According to Piketty et al., 1940–80 is also the bottom of the inequality U-curve.Subsequently, there was a precipitous decline in union participation in the 1980s, followed bycontinuingerosion,sothatnowonlyaboutone-ninthofallUSworkersareunionmembers—allthewhile,inequalityhassteadilyclimbedupwards.

Ontheotherside,thecapitalistshaveonlybecomebetterorganized.Infact,theirmobilization—notcoincidentally—shortlyprecedes theextensivedisempowermentofunions inandaround the1980s.Forexample,theinfamousyetinfluentialPowellMemorandum(writtenin1971byamanwhowouldbecome a US Supreme Court Associate Justice) explicitly advocated coordinated action amongcapitalists: “Strength lies in organization, in careful long-range planning and implementation, in

Page 368: Edited By Deeangelee Pooran-Kublall, MD/MPH · 9.5 Preparing for the MCAT: Biochemistry in the Chemical and Physical Foundations of Biological Systems Section 9.6 Preparing for the

consistency of action over an indefinite period of years, in the scale of financing available onlythrough joint effort, and in the political power available only through united action and nationalorganizations.”That“carefullong-rangeplanning”haspaidseriousdividends.

P1.

P2.

P3.

P4.

P5.

P6.

1.Whichofthefollowingwouldspecificallybolstertheauthor ’sprimarylineofargumentation?I. Evidencethatcoordinationamongcapitalistsinthe1970sdirectlycontributedtothedeclineoflaborunionsinthe1980sII.Findingsdemonstratingthatcurrentlevelsofinequalityhaveeclipsedthehistoricalrecordssetinthe1920sIII.Astudyshowingthattherateofpolarizationofwealthhasincreasedsincetheglobalfinancialcrisisof2007–8

A.IonlyB.IandIIonlyC.IIandIIIonlyD.I,II,andIII

2.Basedonthepassage,whatistheauthor ’smostlikelyreasonforregardingaU-shapedcurveas“potentiallydisconcerting”(paragraph1)?

A.InequalityhasreturnedtoalevelnotseensincebeforetheGreatDepression.B.Deliberatechoicesbypoliticalleadershaveledtoanincreaseininequality.C.Theextremepolarizationofwealthhasdetrimentalconsequencesforsociety.D.Thegrowthininequalityshowshowcapitalistsaremotivatedsolelybygreed.

3.Whichofthefollowing,whentakeninconjunctionwiththeinformationpresentedbythepassage,wouldbestexplaintheauthor ’suseof“notcoincidentally”(paragraph6)?

A.Theascendanceofneoliberalismindemocraticpoliticsislargelyresponsiblefortheriseininequality.B.Thecapitalistsbegancoordinatingtheireffortsimmediatelyafterrecognizingunionswerelosingtheirpower.C.Amajorityofcitizensdecidedtorelinquishmembershipintheworkingclassandbecomecapitalistsinstead.

Page 369: Edited By Deeangelee Pooran-Kublall, MD/MPH · 9.5 Preparing for the MCAT: Biochemistry in the Chemical and Physical Foundations of Biological Systems Section 9.6 Preparing for the

D.Thedemocraticallyelectedpoliticiansfinancedbyneoliberalorganizationsenactedanti-laborpolicies.

4.TheauthorsuggestsacorrelationbetweeneachofthefollowingpairsofphenomenaEXCEPT:A.morecoordinationamongcapitalistsandlesscoordinationamonglaborers.B.higherparticipationinlaborunionsandlowerlevelsofequality.C.strengthoforganizationandpoliticalsuccessindemocraticnations.D.theriseofneoliberalismandawideningofthegapbetweenrichandpoor.

5.InCitizensUnitedv.FederalElectionCommission(2010),theUSSupremeCourtruledthatrestrictionsonpoliticaladvocacyspendingbycapitalistcorporations,laborunions,andotherassociationswereprohibitedbytheFirstAmendment.Assumingthatsuchspendingiseffective,themostreasonableexpectationbasedonthepassageisthatinequalityintheUnitedStateswill:

A.stopincreasingbecausetherulingeliminatesregulationsthatwerepromotingthepolarizationofwealth.B.continueincreasingbecausecorporationswillbeabletooutspendunions,resultinginmorepro-capitalistpolicies.C.begindecreasingbecauseunionswillbeabletooutspendcorporations,resultinginmorepro-laborpolicies.D.remainatitspresentlevelbecausecorporationsandlaborunionsaretreatedequallyundertheruling.

1.Whichofthefollowingwouldspecificallybolstertheauthor ’sprimarylineofargumentation?

I.Evidencethatcoordinationamongcapitalistsinthe1970sdirectlycontributedtothedeclineoflaborunionsinthe1980sII.Findingsdemonstratingthatcurrentlevelsofinequalityhaveeclipsedthehistoricalrecordssetinthe1920sIII. Astudyshowingthattherateofpolarizationofwealthhasincreasedsincetheglobalfinancialcrisisof2007–8

A.IonlyB.IandIIonlyC.IIandIIIonlyD.I,II,andIII

Assessthequestion

Page 370: Edited By Deeangelee Pooran-Kublall, MD/MPH · 9.5 Preparing for the MCAT: Biochemistry in the Chemical and Physical Foundations of Biological Systems Section 9.6 Preparing for the

ThewordbolsterindicatesaStrengthenquestionofsomekind,withthesubjunctivewouldsuggestingReasoningBeyondtheText.Thefactthatthestemislookingspecificallyforaneffectontheauthor’sprimary lineofargumentation suggests that theRoman numeral optionsmay include evidence thatbolsters related arguments that are only incidental to the author ’s real concerns.Given the lackofreferences to particular parts of the text and the fact that there aremultiple pieces of evidence toconsider,thisquestionmightbebettersavedforlater.

PlanyourattackWithaRomannumeralquestion,adivideandconquerstrategyisthebestapproach.Beforedoingaclose reading of theRoman numerals, look at the answer choices to see if any of them appear inexactly two of the four options. This is the one you’llwant to test first: if it’s true, then you caneliminatethetwochoicesinwhichitdoesnotappear;ifit’sfalse,youcaneliminatethetwochoicesinwhichitappears.Then,withthetworemainingoptions,you’llonlyhavetoevaluateonenumeralthat differs between them. (Ifmore than one numeral appears twice, then startwithwhichever oneseemseasierforyou.Ifnonumeralappearstwice,youmayhavetotestallofthem,sojustbeginwiththeeasiest.)

With this question, numeral III appears in twochoices, (C) and (D). If III is true, then (A) and (B)couldbeeliminated,andonlynumeralIwouldhavetobetested,becauseitappearsin(D)butnot(C).IfIIIisfalse,then(C)and(D)couldbeeliminated,andonlynumeralIIwouldhavetobetestedin(B)butnot(A).You’lldefinitelywanttoevaluateIIIfirst.

One more piece of preparation is essential before beginning the attack: clarifying the author ’sprimaryargument.Thekeylinesarefoundinparagraph2:

“Thetrulyinterestingquestionconcernsneithertheexistenceofeconomicinequalitynorthefactofitscontinuinggrowth,butitsorigin.Whathascaused—andevenmorecrucially,whatisperpetuating—thispolarizationinwealth?Afterweighingtheevidence,mycontentionisthatthisdevelopmentisultimately apolitical outcome, not aneconomic one.By this Imean it is the product of deliberatedecisionsbypoliticalleaders(elected,appointed,orotherwise)andotherinfluentialsocioeconomicelites...”

Thismakesitclearthattheauthor ’sargumentconcernsacause-and-effectrelationship,sosupportingevidence would have to indicate causality. Moreover, the suggestion that neither the existence ofeconomicinequalitynorthefactofitscontinuinggrowthareinterestingisworthnoting,becausethiscanhelpsetexpectationsforthewrongkindofsupport.

ExecutetheplanFollowing the Plan outlined previously, start with numeral III, A study showing that the rate ofpolarizationofwealthhasincreasedsincetheglobalfinancialcrisisof2007–8.Whilesuchevidencewould support the claim that inequality is continuing to grow, this is precisely what the authorsuggestedwasuninteresting,definitelynottheauthor’sprimarylineofargument.Tosupportthemain

Page 371: Edited By Deeangelee Pooran-Kublall, MD/MPH · 9.5 Preparing for the MCAT: Biochemistry in the Chemical and Physical Foundations of Biological Systems Section 9.6 Preparing for the

argument, this item would need to offer some indication that decisions by socioeconomic elitescausedthisacceleratinggrowth,butthemerementionofthefinancialcrisisisinsufficienttoestablishthat.

Given thatnumeral III is false,both (C)and (D) are ruledout, and it canbe inferred that I is true.Looking at II,Findings demonstrating that current levels of inequality have eclipsed the historicalrecordssetinthe1920s,itisreadilyapparentthatthisismoresupportforinequality’sexistenceandgrowth,notforthecauseproposedbytheauthor.NumeralIIisthusfalseforroughlythesamereasonthatIIIis,andchoice(B)canalsobeeliminated.

Answerbymatching,eliminating,orguessingThedivideandconquerapproachrevealedthatchoice(A)iscorrectafterevaluatingonlytwoofthenumerals.Thoughyouwouldwanttoselect(A)onTestDaywithoutfurtherdelay,wecanconfirmthat only I is true by considering the impact of the sentence Evidence that coordination amongcapitalistsinthe1970sdirectlycontributedtothedeclineoflaborunionsinthe1980s.Thispointstoanother paragraph, P6, where the author states that capitalist mobilization—not coincidentally—shortlyprecedestheextensivedisempowermentofunionsinandaroundthe1980sandcitesthePowellMemorandum.NumeralIwoulddefinitelyhelptosupporttheauthor ’saccountofhowinequalitywascausedtoincreaseinthe1980s,andwouldthusbolstertheprimaryargument.

SIMILARQUESTIONS1.Theauthorofthepassageisespeciallyconcernedabout:2.Whowouldtheauthormostlikelyconsidertobeanexampleof“otherinfluentialsocioeconomicelites”(paragraph2)?

3.Theauthor ’sapparentintentionwithparagraph2isto:

TakeawaysWithRomannumeralquestions,useadivideandconquerstrategy:evaluateanumeralthatappearsintheanswerchoicesexactlytwicetocutthepossibilitiesinhalfafteronetest.

ThingstoWatchOutForWhenaquestionstemstressesstrengtheningorweakeningaparticularargument,bewaryofoptionsthataffectseparateclaims,evenonesthatmayseemrelated.

SOLUTIONSTOSIMILARQUESTIONS

1.MainIdeaquestionsoftentakeanabbreviatedform,suchaswiththisexample.Thecorrect

Page 372: Edited By Deeangelee Pooran-Kublall, MD/MPH · 9.5 Preparing for the MCAT: Biochemistry in the Chemical and Physical Foundations of Biological Systems Section 9.6 Preparing for the

responsewouldhavetonotespecificallytheauthor ’sargumentthatdeliberateactionsbyeconomicandpoliticalelitescausedinequalitytoincrease.

2.AlthoughthereferenceinthisApplyquestionistoP2,understandingtheauthor ’sintentrequirespayingattentiontothedetailsthatemergesubsequently.InP3,theauthorintroducesthedistinctionbetweenlaborersandcapitalists,thenmakesitclearthatthelattergrouphasbecomeascendant(anideafurtherelaboratedinP6).Theanswermightusethewordscapitalist,owner,employer,wealthy,orrichtoindicatemembersofthatclass.Incorrectchoicescouldincludeelectedorappointedpoliticalofficials,whowerementionedseparatelyfromthisothercategory.

3.ThisisjustaFunctionquestionaboutP2.UsetheLabelinyouroutlinetomakeaprediction.Thecorrectanswermightbegeneral,suchasarticulatethethesisofthepassage,oritmightbequitespecific,likecontendthatinequalityiscausedbypoliticalchoicesratherthannaturaleconomicforces.

KeyConceptsQuestionTypesII:ReasoningWithintheTextInferenceQuestions–AssumptionSubtype

2.Basedonthepassage,whatistheauthor ’smostlikelyreasonforregardingaU-shapedcurveas“potentiallydisconcerting”(paragraph1)?

A.InequalityhasreturnedtoalevelnotseensincebeforetheGreatDepression.B.Deliberatechoicesbypoliticalleadershaveledtoanincreaseininequality.C.Theextremepolarizationofwealthhasdetrimentalconsequencesforsociety.D.Thegrowthininequalityshowshowcapitalistsaremotivatedsolelybygreed.

AssessthequestionThe question stem asks for themost likely reason behind a judgment the author makes, which isanotherway of asking for an assumption. There’s a direct reference to paragraph 1,whichmightmake this question a bit more workable and worth trying now, when you first encounter it, eventhoughitconcernsimplicitpartsofanargument.

PlanyourattackStart by returning to the context of the quoted words:Many measures of inequality, when plotted

Page 373: Edited By Deeangelee Pooran-Kublall, MD/MPH · 9.5 Preparing for the MCAT: Biochemistry in the Chemical and Physical Foundations of Biological Systems Section 9.6 Preparing for the

annually,adoptapotentiallydisconcertingU-shape:decliningafterthereformsimplementedbymanyindustrializedpopulationsinthewakeoftheGreatDepression,inequalityisnowreturningtolevelsnot seen since the 1920s—and on the verge of surpassing them. The clauses following the colonexplaintheU-shape,butdonotreallygetatthereasonforitbeingpotentiallydisconcerting,thatis,whyitmightbealarmingortroubling.Thecorrectanswerwillhavetodomore,explainingwhythereturn to high levels of inequality is a negative outcome. Evaluate each answer choice to assesswhetheritaccomplishesthis.

ExecutetheplanChoice(A),InequalityhasreturnedtoalevelnotseensincebeforetheGreatDepression,mayseemtemptingbecauseitmentionstheGreatDepression,butitsimplyrestatestheauthor ’sdescriptionoftheU-curvewithoutexplainingit,atypicalFaultyUseofDetailtrap.

The issue with choice (B), Deliberate choices by political leaders have led to an increase ininequality, isdifferent: it offers anexplanation,but thewrongone. Itmaybe true that theauthor ’sprimaryargumentconcernsthecausesofthisincreaseininequality,butthisquestionasksspecificallyabouttheevaluationtheauthormakesofthisincrease.Askingforthereasonthatanoutcomeisgoodorbadisconceptuallydistinctfromaskingforthecauseofthatoutcome.

Although(C),Theextremepolarizationofwealthhasdetrimentalconsequencesforsociety,doesnotemploythewordinequalityliketheotheroptions,itdoesuseasynonymousphrase.Moreover,unliketheprevious choices, this actuallygives a reason forwhy this trend couldbedisturbing: it carriesnegative social consequences. The author does suggest that thewealth inequality of the 1920swasresponsible for an event so negative that it came to be known as theGreatDepression, so this ispreciselywhatwewerelookingfor.

Answerbymatching,eliminating,orguessingChoice(C)matchedthepredictionmadeinthePlanstep.Itcanbeconfirmedasthecorrectoptionbyabriefconsiderationofthefinalcontender,(D),Thegrowthin inequalityshowshowcapitalistsaremotivatedsolelybygreed.Whilegreedmaynotbeanemotionthatmostpeopleapplaud,choice(D)issimilarto(B) insofaras itaddresses thecauseof theoutcomerather thananappraisalof itsvalue.Thus,wecanbeconfidentthat(C)isright.

SIMILARQUESTIONS1.Whatistheprimarypurposeofparagraph1?2.WhatroledoestheU-curveplayintheauthor ’sargument?3.Manyprominentpoliticiansandcorporateexecutivesdenythatinequalityisaseriousproblem.Whatimpactdoesthisfacthaveontheclaimsmadeinthefirstparagraph?

Page 374: Edited By Deeangelee Pooran-Kublall, MD/MPH · 9.5 Preparing for the MCAT: Biochemistry in the Chemical and Physical Foundations of Biological Systems Section 9.6 Preparing for the

TakeawaysSometimesInferencequestionswillaskforveryspecificassumptionsorimplications,suchaspiecesofinformationthatexplainanormative(value)judgment.

ThingstoWatchOutForAvalidinferencefromthepassagecouldstillbeanincorrectanswerifitfailstomeettherequirementsofthequestionstem.Usethecluesthestemprovidestomakeamorethoroughprediction,andsuchtrapswillbemuchlesstempting.

SOLUTIONSTOSIMILARQUESTIONS

1.ThisstraightforwardFunctionquestionisbestapproachedbymakingapredictionwithyourLabelforP1.Itservesthepurposeofestablishingsomebackgroundontheproblem,offeringevidencethatinequalityisasignificantissue,onethatisonlybecomingworse.Wrongoptionsmightsuggestthatit’spartofthemainargument,whenreallyit’sonlyincidentaltotheauthor ’sconcerns,asnotedbythebeginningofP2.

2.ThecurveismentionedinbothP1andP5,sothecorrectresponsetothisStrengthen–Weaken(WithinPassage)questioncouldindicatetheroleitplaysineitherorboth.Inthefirstparagraph,itisusedtoestablishtheexistenceandgrowthofinequality,whileinP5itisusedtoshowaconnectionbetweentheorganizationoflaborandlevelsofinequality.

3.ThisisaStrengthen–Weaken(BeyondPassage)questionwithascopelimitedtoP1.Thefactthatmanypeopledenyaclaimforwhichthereisindependentevidencehaslittleimpactonthetruthofthatclaim,soitwouldnotsignificantlychallengetheauthor ’ssuggestionthatinequalityisserious.TheonlyfeasibleimpactitmighthaveiswithrespecttothementionofthePresident,thePope,andotherelitefigures:itwouldexplainwhytheauthorusesthephrasehavehadtoadmit,whichsuggestsakindofreluctantacceptance.

KeyConceptsQuestionTypesIII:ReasoningBeyondtheTextOtherReasoningBeyondtheTextQuestions

3.Whichofthefollowing,whentakeninconjunctionwiththeinformationpresentedbythepassage,wouldbestexplaintheauthor ’suseof“notcoincidentally”(paragraph6)?

A.Theascendanceofneoliberalismindemocraticpoliticsislargelyresponsiblefortheriseininequality.B.Thecapitalistsbegancoordinatingtheireffortsimmediatelyafterrecognizingunionswerelosingtheirpower.

Page 375: Edited By Deeangelee Pooran-Kublall, MD/MPH · 9.5 Preparing for the MCAT: Biochemistry in the Chemical and Physical Foundations of Biological Systems Section 9.6 Preparing for the

C.Amajorityofcitizensdecidedtorelinquishmembershipintheworkingclassandbecomecapitalistsinstead.D.Thedemocraticallyelectedpoliticiansfinancedbyneoliberalorganizationsenactedanti-laborpolicies.

AssessthequestionThe line in conjunctionwith the informationpresentedby thepassage suggests the answer choiceswill present new ideas, meaning that this is Reasoning Beyond the Text. Since it asks for anexplanation of the author ’s use of a term, this would not exactly fall into either the Apply orStrengthen–Weaken (Beyond Passage) types, but falls under the Other heading. There’s a specificreferencetothetext,whichmaymakethisquestionmoremanageable,butOtherquestionscanoftenbechallenging,soyoumaywanttotriagethisone.

PlanyourattackA question that asks for the best explanation, especially one that seems likely to bring in newinformation,isonethatwilllikelyrequireexaminingeveryanswerchoice.Agoodexplanationwillmake the author ’s intention with the phrase clear, so begin your preparation by returning to therelevant part of the text.After establishing in P5 that labor has become less organized, the authorwrites:

Ontheotherside,thecapitalistshaveonlybecomebetterorganized.Infact,theirmobilization—notcoincidentally—shortly precedes the extensive disempowerment of unions in and around the 1980s.Forexample,theinfamousyetinfluentialPowellMemorandum(writtenin1971byamanwhowouldbecome a US Supreme Court Associate Justice) explicitly advocated coordinated action amongcapitalists...

The suggestion is that the improved organization of the capitalists is directly connected (is not acoincidence) to thediminutionoforganized labor.Given that theauthor stresses that theoneeventshortly precedes the other, there is the hint that the two are causally connected (because a causetypically comes immediately before its effect).Thus, the correct response should account for howbetterorganizationamongcapitalistscouldleadtoworseorganizationamonglaborers.

ExecutetheplanWhile(A)(Theascendanceofneoliberalismindemocraticpoliticsislargelyresponsiblefortheriseininequality)mimicsthelanguageofthepassage,evenechoingtheauthor ’sthesis,itfailstoaccountfor the use of this particular phrase. It is too general to account for the specific phenomenonreferencedinP6.Eliminateit.

Page 376: Edited By Deeangelee Pooran-Kublall, MD/MPH · 9.5 Preparing for the MCAT: Biochemistry in the Chemical and Physical Foundations of Biological Systems Section 9.6 Preparing for the

Choice(B)seemsabitmorepromising,becauseitconnectstheactivitiesofthetwoclasses.However,Thecapitalistsbegancoordinatingtheireffortsimmediatelyafterrecognizingunionswerelosingtheirpowergetsthetimingbackwards.Theauthorsaysthatthecapitalistcoordinationcamefirst,andthenthedeclineoftheunions,so(B)isalsowrong.

Itmayseemlike(C)(Amajorityofcitizensdecidedtorelinquishmembershipintheworkingclassandbecomecapitalistsinstead)providesapossibleexplanation,butitfailsforanumberofreasons.Thevery first line of the passage notes the huge discrepancy in numbers between thewealthy class ofowners and the poor class of laborers. Though the author mentions that deliberate decisions areimportantinP2,thesearethedecisionsofpoliticalleadersandotherelites,notthedecisionsofthemajority of people—it’s not clear from the passage that laborers have the power simply to switchrolesfromemployeetoemployer.Evenifsomeemployeescouldbecomeemployers(forinstance,entrepreneursandtheself-employed),itseemshighlyunlikelythatamajorityofcitizenscoulddoso.Finally,thischoicedoesnottakecaretodistinguishbetweenmembershipinorganizedlaborunionsandmembershipintheclassoflaborers(employees).Thepreviousparagraphdiscussedadeclineinunionmembership,notadeclineinmembershipintheworkingclass.Thus,choice(C)clasheswiththepassagetoomuchtobeaplausibleanswer.

ThePlanwastocheckallfouranswers,so(D)shouldalsobeevaluated,thoughwenowexpectit’scorrect. The statement The democratically elected politicians financed by neoliberal organizationsenacted anti-labor policies would indeed provide the necessary explanation, one that is consistentwiththediscussioninthepassage.ThequotefromthePowellMemorandumevenmentionsfinancingasameansofattainingpoliticalpower,anumberofneoliberalpoliticiansarenoted inP3,andP4citestheactionsofdemocraticofficialswhopromotethepolarizationofwealth(inotherwords,anti-laborpolicies),sotheagreementbetweenthischoiceandthepassageisresounding.

Answerbymatching,eliminating,orguessingExecutingthePlanledtotheeliminationofthewrongoptions,revealingchoice(D)ascorrect.

SIMILARQUESTIONS1.Whatdoestheauthormostlikelyintendtosuggestbythephrase“seriousdividends”inthelastsentenceofthepassage?

2.Whichofthefollowingcanmostreasonablybeinferrediftheauthoriscorrectthatneoliberalismis“embracedbyleadersonbothendsoftheacceptedpoliticalspectrum”(paragraph3)?

3.TheauthorcitesthePowellMemorandumtosupporttheclaimthat:

TakeawaysQuestionsthataskforpossibleexplanationscanoftenbetricky.Beclearonexactlywhatitisthatmustbeexplained.

Page 377: Edited By Deeangelee Pooran-Kublall, MD/MPH · 9.5 Preparing for the MCAT: Biochemistry in the Chemical and Physical Foundations of Biological Systems Section 9.6 Preparing for the

ThingstoWatchOutForAnswerchoicesinReasoningBeyondtheTextquestionscanpresententirelynewsituationscompletelyunaddressedbythepassagewithoutsuchchoicesautomaticallybeingwrong.However,watchoutforsituationsthatdirectlyconflictwiththepassage—thesewillmostlikelybeincorrect.

SOLUTIONSTOSIMILARQUESTIONS

1.Thephraseisusedtosuggestthatthecapitalistswereextremelysuccessfulintheireffortsatcoordinatedaction,sotheanswertothisDefinition-in-Contextquestionwouldsuggestsomethinglikepoliticalsuccess,oreventheeconomicsuccessthatcameasaresultofpoliciesthatpromotedtheupwardflowofcapital.

2.ThisInferencequestionhasanumberofpossibleresponses,sincetherearemultipleimplicationstobedrawnfromthatlineandthesurroundingcontext.Forinstance,oneconclusionthatcanbedrawnisthattheacceptedpoliticalspectrumonlyallowspro-capitalistviews,whileanotheristhattheoptionsgiventovotersincontemporarydemocraciesareactuallyquitelimited.

3.ThisisaStrengthen–Weaken(WithinPassage)question,simplyaskingforhowthequotationinthelastparagraphisusedasevidence.Specifically,thisisintendedtosupporttheideathatthecapitalistswereworkingtogethertoadvancetheirpoliticalandeconomicfortunes.

KeyConceptsQuestionTypesII:ReasoningWithintheTextInferenceQuestions–ImplicationSubtypeScatteredQuestions

4.TheauthorsuggestsacorrelationbetweeneachofthefollowingpairsofphenomenaEXCEPT:

A.morecoordinationamongcapitalistsandlesscoordinationamonglaborers.B.higherparticipationinlaborunionsandlowerlevelsofequality.C.strengthoforganizationandpoliticalsuccessindemocraticnations.D.theriseofneoliberalismandawideningofthegapbetweenrichandpoor.

AssessthequestionTheprominentEXCEPTcallsattentiontothefactthisisaScatteredvariantofeitheraDetailquestionoranInferencequestion—thewordsuggests is regularlyused inboth types.However,since itasksfor something as complex as a correlation, this question will most likely require deducing some

Page 378: Edited By Deeangelee Pooran-Kublall, MD/MPH · 9.5 Preparing for the MCAT: Biochemistry in the Chemical and Physical Foundations of Biological Systems Section 9.6 Preparing for the

implications from the text. Given the amount of time that is potentially involved in answering aScattered Implication question (a lot of searching through the passage for ideas thatmight not beexplicitlystated,onlyimplied),thiswouldbestbesavedforlaterinaquestionset.

PlanyourattackScatteredquestions(featuringwordslikeEXCEPT,NOT,andLEAST)typicallyrequiretheprocessofelimination to answer.However, it’s stillworthwhile to set some expectations. The correct answerwilleitherbesomethingthatthetextfailstomentionor—morelikely—itwillbeatwistedversionofinformationcontainedinthepassage.Anychoicethatcontradictedthepassageorotherwisedistortedits contentwould have to be right, and could potentially be selectedwithout testing the remainingoptions.

Ontheotherhand,wronganswerchoiceswillbepairsofphenomenaforwhichtheauthorstatesorimplies a correlation. Keep inmind that cause-and-effect relationships always entail a correlationbetween the cause and the effect, so any indication by the author of causationwould thereby be asuggestionofcorrelationaswell.

ExecutetheplanFor(A)(morecoordinationamongcapitalistsandlesscoordinationamonglaborers),therelevanttextisatthestartofP6:Ontheotherside,thecapitalistshaveonlybecomebetterorganized.Infact,theirmobilization—not coincidentally—shortly precedes the extensive disempowerment of unions in andaround the 1980s. The fact that the author stresses this is not a coincidence reinforces that therelationship is intended to bemore significant, potentially cause and effect.That there is at least acorrelationisclear,sochoice(A)shouldberuledout.

Withchoice(B)(higherparticipationinlaborunionsandlowerlevelsofequality),however,thereisanissuewhenreturningtotherelevanttext,thistimefromP5:From1940to1980,betweenaboutone-fifth and one-quarter of all employedworkers in theUnited States weremembers of labor unions.AccordingtoPikettyetal.,1940–80isalsothebottomoftheinequalityU-curve.There’sdefinitelyacorrelationbeinghighlighted,but it’sbetweenhighunionparticipationand low inequality,not lowequality. If inequality is low, thenequalitymustberelativelyhigh,so thischoice is theoppositeofwhatthepassagesuggests.Thus,withoutfurtherado,it’ssafetoconcludethat(B)iscorrect.

Answerbymatching,eliminating,orguessingAfterfindingacontradictionin(B),itshouldbechosenasthecorrectanswer.Thiscouldbedouble-checked by finding the other choices in the text. The correlation in choice (C) (strength oforganizationandpoliticalsuccessindemocraticnations)isimpliedinthediscussionbeginningwithMuch of the ascendance of neoliberal ideology stems from a discrepancy in organization, andcontinuingthroughoutthefinaltwoparagraphs.For(D)(theriseofneoliberalismandawideningof

Page 379: Edited By Deeangelee Pooran-Kublall, MD/MPH · 9.5 Preparing for the MCAT: Biochemistry in the Chemical and Physical Foundations of Biological Systems Section 9.6 Preparing for the

thegapbetweenrichandpoor),therelevanttextisfromP3:withtheriseofthepoliticalideologyofneoliberalism...inequalitybegantorebound.Anincreaseininequalityentailsawideningofthegapbetweenrichandpoor.

SIMILARQUESTIONS1.Whichofthefollowingwouldtheauthorbemostlikelytoregardasanexampleofneoliberalideology?

2.By“freemarkets”(paragraph4),theauthorismostlikelyreferringto:3.Whatisthegreatestflawintheauthor ’sattempttoexplainwhyinequalitybegantoincreaseagaininthe1980s?

TakeawaysOnoccasion,Scatteredquestionscanbeansweredwithoutactuallyhuntingdowninformationforeverychoice.Ifyoufindadirectcontradiction(oranotherobviousdistortion),you’velikelyfoundyouranswer.

ThingstoWatchOutForWhenyoufindananswerchoicethatperfectlyanswersthequestion,don’twasteanymoretimereadingwronganswers.Getthepointandusethatextratimewhereitcouldgainyouevenmore!

SOLUTIONSTOSIMILARQUESTIONS

1.ForthisApplyquestion,theanswerwillhavetobesomeideathatpromotestheinterestsofwealthycapitalistsoverlaborers.Forexample,theideathatpeoplearepooronlybecausetheydeserveitwoulddefinitelyqualify,sincethissuggeststhatrichpeoplearemorallyjustifiedinhoardingalltheirwealthwhileothersstruggletosurvive.

2.ForthisDefinition-in-Context,thecorrectresponsewilllikelyreflectthecontrastthattheauthordraws:Neoliberalismpurportstopromote“freemarkets,”butperhapsabettercharacterizationofitisthepromotionofthefreemovementofcapital.Inotherwords,theterm“freemarkets”isabitofpublicrelations,theattempttoputamorepositivespinonanideathatwouldnotbesopopularifdescribedmoreaccurately.

3.Whilerare,theMCATwillincludequestionsthatrequireyoutoevaluatethereasoninginthepassage,atypeofOtherReasoningWithintheText.Thereareanumberofpossibilities,buttheanswerlikelyhassomethingtodowithhowweaklyestablishedanumberofthecausallinksare.Theauthoroffersmanycorrelations,butsuggestscausalconnectionswithoutsufficientevidence.There’snoconcreteevidenceprovided,forexample,thatthecapitalistscolludingtogetherledtothedownfalloftheunions.

Page 380: Edited By Deeangelee Pooran-Kublall, MD/MPH · 9.5 Preparing for the MCAT: Biochemistry in the Chemical and Physical Foundations of Biological Systems Section 9.6 Preparing for the

KeyConceptsQuestionTypesIII:ReasoningBeyondtheTextApplyQuestions–OutcomeSubtype

5.InCitizensUnitedv.FederalElectionCommission(2010),theUSSupremeCourtruledthatrestrictionsonpoliticaladvocacyspendingbycapitalistcorporations,laborunions,andotherassociationswereprohibitedbytheFirstAmendment.Assumingthatsuchspendingiseffective,themostreasonableexpectationbasedonthepassageisthatinequalityintheUnitedStateswill:

A.stopincreasingbecausetherulingeliminatesregulationsthatwerepromotingthepolarizationofwealth.B.continueincreasingbecausecorporationswillbeabletooutspendunions,resultinginmorepro-capitalistpolicies.C.begindecreasingbecauseunionswillbeabletooutspendcorporations,resultinginmorepro-laborpolicies.D.remainatitspresentlevelbecausecorporationsandlaborunionsaretreatedequallyundertheruling.

AssessthequestionThe lengthyquestion stem,chock fullofnew information, immediately revealsReasoningBeyondtheText,while the languageofmost reasonable expectation tells us that this is anApply question,specificallyonethatasksforanOutcome.Thesekindsofquestionscantakeawhile(especiallywithlonganswerchoices),sosavingthemforlater,afterworkingonsomeeasierones,isnotabadidea.

PlanyourattackWithanewscenario,thekeywillbefindingthepointsofconnection(oranalogy)withthepassage.Thestemsaysthatrestrictionsonpoliticaladvocacyspendingwerejudgedtobeprohibited,andthatthisaffectedcorporations,unions,andothergroups.Inshort,thatmeanstheseorganizationsgainedadditionalfreedomtospendmoneyonpoliticalissuesattheirdiscretion.ThequestionstemgoesontosayexplicitlyAssumingthatsuchspendingiseffective,which tellsyou to take it forgranted thatthis spending can have an impact on who gets elected and what policies get enacted. The actualquestion isacompletion(withacolon),but itamounts to thefollowing:given thenewfreedomofthese groups to spendmoney and assuming it works, what will likely happen to inequality in theUnitedStates?Answeringthisisthetaskpresented.

Page 381: Edited By Deeangelee Pooran-Kublall, MD/MPH · 9.5 Preparing for the MCAT: Biochemistry in the Chemical and Physical Foundations of Biological Systems Section 9.6 Preparing for the

ExecutetheplanBeforetacklingthosewordyanswerchoices,it’sworthwhiletomakeapredictionforthecorrectone.StartbyclarifyingthenatureofthetrendininequalityleadinguptotheSupremeCourtruling.Thefirstparagraphmentionsthemorerecentdateof2014,withinonlyafewyearsofthedatefromthequestionstem,soitoffersagoodreferencepoint.There,we’retoldthatinequalityisnowreturningtolevels not seen since the 1920s—and on the verge of surpassing them. This is hardly an isolatedstatement,sincetheideathatinequalityisincreasingisnotedthroughoutthepassage,whichlargelyseekstoanswerthequestionofwhythisoccurs.

Nowthequestionbecomes:howwillthisupwardtrendbeaffected?Thethirdparagraphofferssomeguidanceonthisquestion:Whenlaborwaspoliticallyascendant . . . inequalitydecreased.However,with theriseof thepolitical ideologyofneoliberalism . . . inequalitybegan torebound.Thus, if therulingbenefitslabormore,weshouldexpecttoseeinequalitydecrease,oratleastseetherateofitsincrease slow or stop. However, if the ruling benefits the capitalists more, then inequality shouldcontinueitsriseorevenaccelerate.

So, who benefits more? Even though the ruling applies to both capitalist corporations and laborunions, the discussion in the last two paragraphs suggests that capitalists are in a much strongerpositiontotakeadvantageoftheruling.InP5,theauthornotesthatabouthalfasmanyUSworkersbelongtounionsasoncedid(one-ninthversusone-fourthorone-fifth),whileP6isexplicitaboutthesuccessofcapitalistsinworkingtogetherevenhintingthattheircoordinatedeffortsmayhavecausedthedecline inorganizationamonglaborers.Sincethecapitalistshavetheadvantage, therulingwilllikelyonlyexacerbatethediscrepancybetweenrichandpoor,leadingtoevengreaterinequality.

Answerbymatching,eliminating,orguessingMorethoroughpreparationmeansaquickermatchwhenyoufinallylookattheanswerchoices.Onlychoice(B) (continue increasing because corporationswill be able to outspend unions, resulting inmorepro-capitalistpolicies)isconsistentwiththepredictionmade.

Choices (A) (stop increasing because the ruling eliminates regulations that were promoting thepolarization of wealth) and (C) (begin decreasing because unions will be able to outspendcorporations, resulting inmore pro-labor policies) can both immediately be discounted, since theyindicateadownwardtrend.Moreover,there’snoreasontosupposethattherestrictionsprohibitedbytherulingwerepromotingthepolarizationofwealth,nortosuspectthatunionswouldhavetheabilitytooutspendcorporations.

Choice(D) (remain at its present level because corporations and labor unions are treated equallyundertheruling)comesclosertothetruth,butfailstotakeintoaccounttherealitiesdescribedinthepassage. If corporations and unions began on equal footing, then it would be true that this rulingwould help neither, because it grants the same freedom to both. However, the passage makes itabundantlyclearthatthetwogroupsarenotatallonequalfooting,thatthecapitalistswereascendantwhenthisrulingwashandeddown.Tosuggestthatinequalityremainsatitspresentlevelswouldalsobetoignorethefact that itcontinuestoincrease: if therulingtrulyhadnoeffectonthestatusquo,

Page 382: Edited By Deeangelee Pooran-Kublall, MD/MPH · 9.5 Preparing for the MCAT: Biochemistry in the Chemical and Physical Foundations of Biological Systems Section 9.6 Preparing for the

thenpresumablyinequalitywouldjustkeepfollowingtheupwardtrenddescribedinthepassage.

SIMILARQUESTIONS1.Inthefourthparagraph,theauthorstatesthat“mostgovernmentsindemocraticnationsthroughouttheworldarefilledwithofficialswhoactinwaysthatfurtherthepolarizationofwealth,”butalsothatfewcitizensofthesenations“wouldvoteexplicitlyforthisprogram.”Howwouldtheauthormostlikelyresolvethisapparentparadox?

2.A2014studybyNASApredictedcatastrophicconsequencesforcivilizationduetoirreconcilabledifferencesbetween“Elites,”whocontrolaccesstoincreasinglyscarcenaturalresources,and“Commoners,”themassofpeoplewhoareresource-poor.Ifaccurate,whatimpactdoesthishaveontheargumentputforthinthepassage?

3.Howwouldtheauthormostlikelyrespondiftoldthatinequalityisactuallydecreasinginsomepartsofthedevelopingworld?

TakeawaysWhilealengthyquestionstemtakeslongertoread,italsogivesyoumorematerialtoworkwith.Besuretousethattoyouradvantageandformulateamorethoroughpredictionforthecorrectanswer.

ThingstoWatchoutForSomewronganswerchoicesmaybecompletelyconsistentlogicallybutareincorrectbecausetheyneglectvitalinformationfromthepassage.

SOLUTIONSTOSIMILARQUESTIONS

1.ThislengthyquestionisactuallyatypeofOtherReasoningWithintheText,thekindinwhichtwoseeminglyconflictingstatementsarereconciledbyfindingathirdstatementthatexplainshowtheycanbothbetrue.Forthisparadoxresolution,ahelpfulcluecomesfromP3’ssuggestionthatneoliberalismisembracedbyleadersonbothendsoftheacceptedpoliticalspectrum.Iftheonlyoptionsvotershavetochoosefromareneoliberals,thatis,politiciansbackedbycapitalistswhofavorpoliciesthatincreaseinequality,thenitexplainshowsuchpeopleareinpowerdespitewhatvoterswouldwant.

2.ThekeyforthisStrengthen–Weaken(BeyondPassage)questionistounderstandthatthisNASAstudyconcernstheeffectsofinequality.Becausetheauthorisconcernedwiththecausesofinequality,astudyofitseffectswouldhavenosignificantimpact.

3.ForthisApplyquestion,thereareafewpossibilities.Theauthorcouldclaimthatthisnewdataisoutsidethescopeofthediscussion,whichprimarilyconcernsdevelopeddemocracies.Alternatively,theauthormightrespondbyaskingtoseehoworganizedlaborisinthesepartsoftheworld,toseeifitcarriessupportforthethesisthatinequalitywilldecreasewhenlaborispolitically

Page 383: Edited By Deeangelee Pooran-Kublall, MD/MPH · 9.5 Preparing for the MCAT: Biochemistry in the Chemical and Physical Foundations of Biological Systems Section 9.6 Preparing for the

ascendant.

KeyConceptsQuestionTypesIII:ReasoningBeyondtheTextStrengthen–Weaken(BeyondPassage)Questions–StrengthenSubtypeRomanNumeralQuestions

Page 384: Edited By Deeangelee Pooran-Kublall, MD/MPH · 9.5 Preparing for the MCAT: Biochemistry in the Chemical and Physical Foundations of Biological Systems Section 9.6 Preparing for the

16.3CARSPracticePassageI(QUESTIONS1–6)Hope and fear are the quintessential political emotions, for both take as their object an unknownfuture, rife with possibilities for cultural flourishing or social dissolution. The one alwaysaccompanies the other, for we inevitably dread that our aspirations might remain unrealized, andcannotbutyearnforouranxietiestoproveunwarranted.Aneramaycometobedominatedbyoneorthe other pole, but its opposite can only be repressed temporarily—witness the opportunisticascendanceofthethemeof“Hope”inUSpoliticssubsequenttotheunabashedfear-mongeringofthe“WaronTerror.”

Given the centrality of hope and fear in politics, there can thus be no question that the mostauthenticallypoliticalofallworksoffictionarenovelsofutopiaanddystopia.Theformerterm,ahybrid of “good place” (the ancientGreek eutopos) and “no place” (outopos), is a coinage of SirThomasMore,whose1516Utopiaisregardedastheurtextofbothgenres,notwithstandingthattheliterary construction of ideal societies is found as early as Plato’sRepublic nearly two millenniaprior. Utopian fiction celebrates human potential, particularly the power of reason, which issupposedly capable of engineering amore perfect world than the one that the arbitrary forces ofnatureandtraditionhaveyielded.

Thefirstgreatdystopiannovelwasnotpublisheduntil1921,morethanfourcenturiesafterMore’soriginalUtopia.YevgenyZamyatin’sWedepictsthedarksideofhumanreason,whathascometobeknownas“instrumental rationality,”a robotic logic inwhichefficiency isvalued for itsownsake,andcitizensaremeremeansfortheadvancementofpoliticalends,whichmustremainunexamined.Zamyatin’s literary personae are granted numbers rather than names and treated accordingly.Instrumental rationality is readily apparent inother iconsofdystopia:AldousHuxley’sBraveNewWorld,GeorgeOrwell’sNineteenEighty-Four,MargaretAtwood’sTheHandmaid’sTale,and(morerecently)SuzanneCollins’sTheHungerGames.Thougheachnoveluniquelypaintsadiretomorrow,allportrayhumansascogs inavastsocialcontraption, theoverarching illogicofwhichbelies thetidysensibilityofitseverydayoperations.

Aswiththeiremotionalantecedents,utopiaanddystopiaareinextricablyintertwined,evidentintheirdeepstructuralcommonalities.Eachinitsdistinctivewayemphasizesthefateofthetransgressor,theindividualwhowould privilege personal desires over the ironclad imperatives of state. Such freespirits cannot be toleratedwithin the body politic anymore than a cancerous cellwithin the bodyphysical.Theanalogyis imperfect,ofcourse,sinceatumordoesnotfeel.Fromthetransgressor ’sperspective, a vantage taken up far more commonly in novels of dystopia, the well-oiled socialmachinebecomesatortureapparatus.Indeed,atremendousamountoftoilandviolencearerequiredfor the proper maintenance of a device that runs so contrary to nature; the most effective sociallubricantsareblood,sweat,andtears.

Ironically, utopias and dystopias are intended to be immutable and self-perpetuating,whichwouldprecludetheverypossibilityofpolitics;acertainfuturewillbringeitherdespairorconfidence,butnot the restless blend of hope and fear that accompanies uncertainty. In other words, both genresrepresent humancoexistence as aproblemand the state as its solution.Theutopiannovel seeks toanswerthepoliticalquestion,whileitscounterpartcallsintoquestionanemerginganswer.Themost

Page 385: Edited By Deeangelee Pooran-Kublall, MD/MPH · 9.5 Preparing for the MCAT: Biochemistry in the Chemical and Physical Foundations of Biological Systems Section 9.6 Preparing for the

profound works of speculative political fiction reject this dynamic entirely—which, in its crudestform, merely recapitulates instrumental rationality—suggesting perhaps the real problem isenvisioninghumanlifeasaproblemtobesolved.

P1.

P2.

P3.

P4.

P5.

1.Whichofthefollowingstatements,iftrue,wouldmoststrengthentheauthor ’sclaimthat“utopiaanddystopiaareinextricablyintertwined”(paragraph4)?

A.Adystopiansocietyalwaysseemslikeanidealpoliticalordertothemembersofitsrulingclass.B.Mostutopiansocietiesallowforthequestioningofoverarchingpoliticalobjectives.C.Dystopiasdonotactuallyrequiresignificantamountsofviolentforcetobemaintained.D.Manyworksofspeculativefictioncanbeclassifiedasneitherutopiannordystopian.

2.Whichofthefollowingdoestheauthorconsidertobeapointofdifferencebetweenutopiananddystopianliterature?

I.ThedepictionofviolenceasnecessaryformaintainingsocialorderII.AnemphasisontherolethatreasonplaysinshapingsocietyIII.Thelikelihoodofconsideringthepointofviewofasocialdeviant

A.IonlyB.IIIonlyC.IIandIIIonlyD.I,II,andIII

3.Bystatingthat“theanalogyisimperfect”inthefourthparagraph,theauthormostlikelyintendstosuggestthat:

A.peopleinasocietyshouldberegardedasmorethanjustpartsmakingupawhole.B.humansocietiesarefarmorecomplexthanthecellsthatconstituteasinglehumanbody.C.comparisonsbetweenanytwoideascanonlyeverbeimprecise.D.transgressorsarenottreatedidenticallyunderutopiananddystopiansocialorders.

4.TheauthorreferstoTheHandmaid’sTaleinparagraph3inorderto:A.arguethatwomenarejustastalentedasmenatwritingspeculativefiction.B.challengetheideathatsocietyshouldbeorganizedrationally.

Page 386: Edited By Deeangelee Pooran-Kublall, MD/MPH · 9.5 Preparing for the MCAT: Biochemistry in the Chemical and Physical Foundations of Biological Systems Section 9.6 Preparing for the

C.giveanexampleofutopianliteraturethatexplorestheconceptofinstrumentalrationality.D.offeraninstanceofanovelinwhichhumansaretreatedasmeansratherthanends.

5.Basedonthediscussioninparagraph4,whichofthefollowingwouldbeLEASTlikelytoberegardedasa“transgressor”?

A.AcitizenofadystopiawhotriestoleadarebellionagainstthepowersthatbeB.AcitizenofautopiawhoneglectspoliticaldutiestospendmoretimewithlovedonesC.AnofficialinadystopiansocietywhousestorturetoreprogramdisobedientcitizensD.Acriminalinautopiansocietywhoispunishedforquestioningthestate’slegitimacy

6.Theauthor ’sprimaryconcerninthepassageisto:A.advocateforthesuperiorityofdystopianoverutopianfiction.B.discussthecharacteristicsofutopiananddystopianliterature.C.challengethenotionthathumanlifeisaproblemtobesolved.D.arguethatthemostpoliticallyrelevantemotionsarehopeandfear.

Page 387: Edited By Deeangelee Pooran-Kublall, MD/MPH · 9.5 Preparing for the MCAT: Biochemistry in the Chemical and Physical Foundations of Biological Systems Section 9.6 Preparing for the

16.4PracticePassageII(QUESTIONS7–11)Inthelate18thcentury,citizensthroughoutruralMassachusettsshutdowncourthousesattemptingtoconduct debt collection hearings, farmers in western Pennsylvania and other parts of the westernfrontierrefusedtopayanexciseonwhiskey,andmembersofthePennsylvaniaDutchcommunityinthe east of the stateharassedofficials attempting to assess adirect taxonhouses. In eachcase, thegovernment’sinitialresponsetoprotestsof“taxationwithoutrepresentation”ledtoanexacerbationoftensions:radicalizedcitizensbandedtogether,creatingarmedmilitiasinopenrebellionagainsttherulingregime.

These popular uprisings against taxation and economic hardship were not—as many Americanswouldnowassumeuponhearingsuchdescriptions—revoltsagainsttheBritishmonarchyinpreludeto theAmericanRevolution (1775–83).Rather,Shays’Rebellion (1786–87), theWhiskeyRebellion(1791–94), and Fries’s Rebellion (1798–1800) occurred after the British had been vanquished.Thougheachepisodehasdistinctivehistorical significance, it isparticularly instructive toexaminetheevolvingreactiontopopularprotestbytheincipientUnitedStatesgovernment.

In the case of the uprisings throughout western and central Massachusetts that would comecollectivelytobeknownas“Shays’Rebellion,”thefederalgovernmentexistedinamuchattenuatedform,enfeebleddue to theconsiderableamountof sovereigntyceded to the thirteenoriginal statesundertheArticlesofConfederation.Aftersubsistencefarmers,veteransoftheContinentalArmy,andotherruralcitizensfoundthemselveshard-pressedin1786bydebtsincurredduringhardtimesandtaxesnewlyleviedbytheMassachusettsgovernment,theybegantorevolt,atfirstjustclosingdowncourtsbutsoonorganizingarmedmilitias,culminatinginanattemptledbyveteranDanielShaystoseizeafederalarmoryinSpringfield.Thefederalgovernmentlackedthefundstoassembleitsownmilitiaandcountertheuprising,soitwaslefttothegovernorofMassachusetts,JamesBowdoin,tohandle—andhehad to turn to assistance frommore thanahundredwealthymerchants tobankrollmercenaries,whoquashedtherebels.

The moneyed and propertied interests—creditors to whom many debts were owed—had beenunnervedbytheeventsinMassachusetts,andwereinstrumentalinthecreationandratificationofthenew Constitution, which greatly concentrated power in a more robust central government. Whenmanywestern farmers refused to pay a 1791 excise tax onwhiskey, the newly empowered federalgovernmentwasabletomusteraformidableresponseafterresistancegrewmoreorganized.In1794,PresidentWashingtonhimself led amassive federalizedmilitia of nearly13,000 troops thatwouldeffortlesslyscattertheresistanceforces.ThereactionbyPresidentAdamstothesmallerrebellionledbyJohnFriesyearslaterwouldbesimilarlyheavy-handed.

Thistendencytowardincreasedcentralizationofpowerhasonlyworsenedsincethe18thcentury.Asthefederalgovernmenthasaccumulatedstrength,stateandmunicipalgovernments—and,ultimately,thepeople—havelosttheirsovereignty.Andwhilethemoneyedhadtofootthebilldirectlytoprotecttheirproperty(andcontinuecollectingtheirrents)inquellingShays’Rebellion,sincetheadoptionofthenewConstitutionin1789,thefederalgovernmenthasbeenabletomakethepeoplepaydirectlyfor their own repression—a fact recently highlighted in the assault, covertly orchestrated acrossseveralcitiesby theFederalBureauof InvestigationandDepartmentofHomelandSecurity,on the

Page 388: Edited By Deeangelee Pooran-Kublall, MD/MPH · 9.5 Preparing for the MCAT: Biochemistry in the Chemical and Physical Foundations of Biological Systems Section 9.6 Preparing for the

2011Occupymovement. In theend, thepeoplehaveonly tradedonemaster foranother: thefeudalrelicofBritishmonarchyhasbeenusurpedbyamodernbureaucraticbehemoth,ultimatelyinthralltothenouveauaristocracyofcorporate“persons”andtherapaciousclassofexecutivesthatconstitutethehomunculiwithin.

P1.

P2.

P3.

P4.

P5.

7.Theauthorwritesinparagraph5that“thefederalgovernmenthasbeenabletomakethepeoplepaydirectlyfortheirownrepression.”Judgingbasedontherestofthepassage,thisismostlikelyintendedtosignifythat:

A.popularuprisingsnolongeroccurintheUnitedStatesduetomoresuccessfulcontrolofcitizens.B.imprisonedprotestorsaresentabillfortheexpensesaccruedwhiletheyarebehindbars.C.protestingultimatelyincursworseconsequencesforindividualstodaythanitdidinthe18thcentury.D.thegovernmentrequirescitizenstopaytaxes,whicharepartlyusedtofundpoliceandmilitaryresponsestoprotests.

8.Theauthor ’sattitudetoward“moneyedandpropertiedinterests”(paragraph4)canbestbecharacterizedas:

A.indifferent.B.positive.C.negative.D.ambivalent.

9.Theauthormostlikelyomitsspecificdetailsoftheeventsinthefirstparagraphinorderto:A.setanexpectationthatisreversedinthefollowingparagraph.B.expresstheprimarythesisofthepassagemoreconcisely.C.downplaythesignificanceoftheeventsbeingaddressed.D.concealagenerallackofknowledgeonthesubjectmatter.

10.Whichofthefollowingisanassumptionmadebytheauthorinthesecondparagraph?A.TheresponsetoFries’sRebellionwasmoreheavy-handedthantheresponsetoShays’Rebellion.B.TheBritishmonarchyisentirelyunliketheUSfederalgovernmentthateventuallyreplacedit.

Page 389: Edited By Deeangelee Pooran-Kublall, MD/MPH · 9.5 Preparing for the MCAT: Biochemistry in the Chemical and Physical Foundations of Biological Systems Section 9.6 Preparing for the

C.AsignificantnumberofAmericanstodayareunfamiliarwiththerebellionsthatoccurredaftertheRevolution.D.TheBritishplayedacovertroleintherebellionsthattookplaceaftertheirdefeatintheAmericanRevolution.

11.Somescholarshavearguedthatinresponsetothe2007–2008financialcrisis,theUSfederalgovernmentdidlesstoprotectcitizenswhosehomesweretakenawayinfraudulentforeclosuresthantodefendthebanksthatengagedinthiscriminalbehavior.Iftrue,whatimpactdoesthishaveonthepassage?A.Itchallengestheauthor ’scentralargument.B.Itsupportstheauthor ’scentralargument.C.Itweakenstheassertionthatthepeoplehaveexchangedonemasterforanother.D.ItstrengthenstheclaimthatthewealthyshapedthecreationoftheUSConstitution.

Page 390: Edited By Deeangelee Pooran-Kublall, MD/MPH · 9.5 Preparing for the MCAT: Biochemistry in the Chemical and Physical Foundations of Biological Systems Section 9.6 Preparing for the

PracticePassageExplanations

PRACTICEPASSAGEI:AHUMANITIESPASSAGE

SampleOutlineP1.Hopeandfear=politicalemotions,inseparable

P2.Utopian and dystopian novels aremost political; utopian fiction celebrates reason (ex.More’sUtopia)

P3. Zamyatin’sWe = 1st dystopia; dystopian fiction explores dark side of reason, “instrumentalrationality”(listofexamples)

P4. Commonalities between utopia and dystopia: focus on fate of transgressors, need for toil andviolence

P5.Moreincommon:nopolitics,viewhumanlifeasproblem(Author:profoundfictionquestionsthis)

Goal:todiscussthecharacteristicsofutopiananddystopianfiction,emphasizingtheirsimilarities

1.(A)

Look at the line in context to get a better sense of what to expect: “As with their emotionalantecedents, utopia and dystopia are inextricably intertwined, evident in their deep structuralcommonalities.”The “emotional antecedents” line refers to the discussion of hope and fear in P1,whichnotedhow“onealwaysaccompaniestheother,”eventhoughonemightcometodominateforatime.Choice(A),whichsuggests thatadystopiansocietyalways looks likeautopianonefromtheperspectiveoftherulers,woulddefinitelysupportthispoint,showinghowutopiaisinseparablefromdystopia.(B)Opposite. InP3, theauthornotesthat“instrumentalrationality,”featuredregularlyindystopianfiction, creates a society inwhich “citizens aremeremeans for the advancement of political ends,whichmust remain unexamined.” If utopias allowed the examination of political ends, this wouldconstituteapointofdifference,ratherthansimilarity.(C)Opposite.ViolenceisaddressedpredominatelyinP4,wheretheauthordiscussescommonalitiesbetween the two. Because utopias are suggested to be as violent, this choicewould create anotherpointofdifferenceiftrue.(D)FaultyUseofDetail.While thischoicedoesnotweaken theclaim,as theotherwronganswerchoicesdo, it is largelyirrelevant.Justbecausetheyarebothabsentsometimesdoesnotmeantheymustalwaysoccurtogether,as“inextricablyintertwined”suggests.

2.(B)

Romannumeral (I) appears twice, so beginwith it.Violence is discussed in P4 as a commonalitybetweenutopiananddystopianliterature,sothisisnotapointofdifference.Since(I)isfalse,choices(A) and (D) can be eliminated. Numeral (III) must be true, since it appears in the two remaining

Page 391: Edited By Deeangelee Pooran-Kublall, MD/MPH · 9.5 Preparing for the MCAT: Biochemistry in the Chemical and Physical Foundations of Biological Systems Section 9.6 Preparing for the

options(confirmedbyalinefromP4:“Fromthetransgressor ’sperspective,avantagetakenupfarmorecommonlyinnovelsofdystopia...”),soonly(II)needstobeevaluated.AttheendofP2,theauthorasserts,“Utopianfictioncelebrateshumanpotential,particularlythepowerofreason...,”soutopian literature does emphasize reason. In P3, the author discusses the crucial role that“instrumental rationality” (“the dark side of human reason”) plays inWe and a number of otherdystopiannovels.Asanotherpointofsimilarity,(II)isthusfalseand(C)iswrong.Becauseonly(III)istrue,choice(B)iscorrect.

3.(A)

Returntothelineincontext:“Suchfreespiritscannotbetoleratedwithinthebodypoliticanymorethana cancerouscellwithin thebodyphysical.Theanalogy is imperfect,of course, sincea tumordoes not feel.” The suggestion is that humans are different than cells, that they deserve to beconsidered asmore than just expendable parts of a largerwhole, because they (unlike tumors andothercells)canfeel.Choice(A)matchesmostcloselywiththisreasoning.(B) In theanalogy,cells(partsofthelargerwholethatisthebody)arebeinglikenedtopeople(partsofthelargerwholethatis society). This choice mischaracterizes the analogy because it compares the whole on one side(society)tothepartsontheother(cells).(C)Distortion.Theauthorisnotsuggestingthateveryanalogyisimperfect,onlythattheparticularanalogybeingdiscussedis.(D)While thismight pose a different kind of problem for the analogy, it does notmake sense incontext.Theauthorcites“atumordoesnotfeel”asareasonforthebreakdown,whichhasnothingtodowiththedifferencesbetweenutopiaanddystopia.

4.(D)

InP3,theauthorlaysoutthevariouscharacteristicsofdystopianfiction,emphasizingespeciallytheideaof“instrumentalrationality,”whichisdescribedas“aroboticlogicinwhichefficiencyisvaluedfor its own sake, and citizens aremeremeans for the advancement of political ends, whichmustremainunexamined.”Theauthormentions“MargaretAtwood’sTheHandmaid’sTale”aspartofalistof “icons of dystopia,” and so is giving an example of a novel that explores the concept ofinstrumentalrationality.Thus,choice(D)iscorrect.(A)OutofScope.Theauthorneverdiscussesgenderexplicitly,norcomparesauthorsofdifferentgenders.(B)While the author seems to be critical of instrumental reason in this paragraph, the idea is notsignificantlychallengeduntilP5.(C)Opposite.TheHandmaid’sTale issaid tobeoneof the“iconsofdystopia,”so itwouldnotbeutopianfiction,assuggestedinthischoice.

5.(C)

Todeterminetheleastlikelyexample,firstclarifywhattheauthormeansbya“transgressor”inP4:“the individual who would privilege personal desires over the ironclad imperatives of state.”Someonewouldnotbea transgressor if theywereactingonbehalfof thestate,suchasanofficialwhowastryingtoreprogramactualtransgressorsinadystopiansociety,asinchoice(C).(A)Opposite.Thiswouldbeatextbookcaseoftransgression,sincethecitizenisfightingagainstthestate(“thepowersthatbe”).

Page 392: Edited By Deeangelee Pooran-Kublall, MD/MPH · 9.5 Preparing for the MCAT: Biochemistry in the Chemical and Physical Foundations of Biological Systems Section 9.6 Preparing for the

(B)Opposite.Thoughthismightnotseemsobad,thiswouldcountasprivilegingsomeothervalueoverthestate,sotheindividualwouldbeatransgressor.(D)Opposite.Questioningitslegitimacywouldclearlybeactingagainsttheimperativesofstate,sothiscriminalwouldcountasatransgressor.

6.(B)

ThoughtheauthorbeginswithadiscussionofhopeandfearinP1,theprimaryfocusofeveryotherparagraphisonthequalitiesofutopiananddystopianfiction,asinchoice(B).(A)Out of Scope. The author primarily emphasizes the similarities between the two genres, andneverreallysuggeststhatoneisbetterthantheother.(C)FaultyUseofDetail.Thisisonlyraisedinthefinalparagraph,soitdoesnotaddressthepassageasawhole.(D)FaultyUseofDetail.TheargumentconcerninghopeandfearisalmostentirelylimitedtoP1.Itservesasanintroductiontoutopia(whichrepresentshope)anddystopia(whichrepresentsfear),butisnottheprimaryconcernoftheentirepassage.

PRACTICEPASSAGEII:AHISTORYPASSAGE

SampleOutlineP1.Governmentresponsestotaxprotestsin18thC.Massachusetts/Pennsylvanialedtoarmedmilitias

P2.Shays’,Whiskey,andFries’sRebellions—againstUSauthorities,notBritish

P3.Shays’Rebellion:federalgovernmenthadlittlepower;merchantshadtopayformilitiatostoprebels

P4. Whiskey Rebellion: after new Constitution, federal government had power to muster militia(Fries’too)

P5. Author: centralized power has only gotten worse; government now serves corporations andexecutives

Goal: to discuss the US federal government response to 18th Century tax protests and argue thatpowerhasbecomemorecentralized

7.(D)

Readthequotefromthequestionstemincontexttogetasenseofwhattolookfor:“Andwhilethemoneyedhad to foot thebill directly…inquellingShays’Rebellion, since the adoptionof thenewConstitutionin1789,thefederalgovernmenthasbeenabletomakethepeoplepaydirectlyfortheirownrepression.”ThecontrastwithShays’Rebellionis instructive,sincetheauthornotesinP3that“[t]he federal government lacked the funds to assemble its ownmilitia and counter the uprising,”whilethisisnotaprobleminP4withthefederalresponsetotheWhiskeyRebellion.Theinferencetobedrawn is that thenewgovernmentcan levy taxes,which it can thenuse to respond toapopularuprising—evenifthatuprisingisitselfareactiontothetaxeslevied,aswasthecasewiththeWhiskeyRebellion.Theonlyanswerthatreflectsthislineofthinkingischoice(D).

Page 393: Edited By Deeangelee Pooran-Kublall, MD/MPH · 9.5 Preparing for the MCAT: Biochemistry in the Chemical and Physical Foundations of Biological Systems Section 9.6 Preparing for the

(A)Opposite.Theauthorcitesarecentexample(the2011Occupymovement)ofakindofpopularuprisingimmediatelyafterraisingthispoint,sothischoiceiscontradictedbythepassage.(B)OutofScope.Whilethisoffersapossibleexplanation,thepassageneverdiscussesanythingofthissort.(C)OutofScope.Nocomparison isevermadebetween thekindsofconsequencesdissenters facetodayversusthe18thcentury,sothischoicecouldnotreflectthepassage.

8.(C)

While the language used to describe the “moneyed and propertied interests” tends to be relativelyneutralinP4,theauthor ’snegativeattitudetowardsthewealthycomesthroughinP5,particularlyinthe closing sentence, with the mention of a “rapacious class of executives.” Thus, choice (C) iscorrect.(A)WhiletheauthorisrelativelyneutralinP4,thelanguageusedinP5suggeststhattheauthorisfarfromindifferent.(B)Opposite.Theauthorneversaysanythingpositiveaboutthemoneyed.(D)Sincetheauthorsaysnothingtopraisethewealthybutonlyusesnegativelanguageindescribingthem indicates that the author ’s attitude is not one of ambivalence (amix of positive and negativefeelings).

9.(A)

Though this is ostensibly a question about the first paragraph, properly answering it requiresunderstandinghowP1connectstotherestofthepassage.AkeyhintcomesinthetransitionintoP2:“These popular uprisings against taxation and economic hardship were not—as many Americanswouldnowassumeuponhearingsuchdescriptions—revoltsagainst theBritishmonarchy . . .”TheauthorhasmadethedescriptionsinP1deliberatelyambiguousinordertocreateanexpectation(thesetaxprotestsareagainstunfairBritish taxes) that isalmost immediatelyoverturned(theprotestsareactually against taxes imposedbyAmerican authorities),which serves tohighlight the fact that theAmericanofficialswereactingjustasunfairlyastheBritish.Thiscorrespondsmostcloselytochoice(A).(B)Thoughconciseexpressionwouldbeareasontoomitdetails,theprimarythesisdoesnotreallyemergeuntillaterinthepassage,particularlyinthefinalparagraph.(C)Opposite.ThischoiceiscontradictedbythediscussioninP2,wheretheauthornotesthat“eachepisodehasdistinctivehistoricalsignificance,”goingontostatehow“particularlyinstructive”theircontrastis.(D)Opposite. Plenty of details are provided in P3 and P4, so it’s clear that the author does notgenerallylackknowledgeaboutthesubject.

10.(C)

BesuretosticktothediscussioninP2,sincethequestionstemspecificallyreferencesit.There,theauthorsuggeststhat“manyAmericanswouldnowassume”thattherebellionsdescribedinP1wereagainstBritishauthorities,whentheywereactuallyagainstAmericanones.InsuggestingthatmanyAmericanswillmakethatassumption,theauthorisactuallyassumingtheseindividualsareunfamiliarwith the events described, and would not be able to recognize them from the descriptions. Thismatcheswithchoice(C).

Page 394: Edited By Deeangelee Pooran-Kublall, MD/MPH · 9.5 Preparing for the MCAT: Biochemistry in the Chemical and Physical Foundations of Biological Systems Section 9.6 Preparing for the

(A)ThoughthereisthesuggestionattheendofP4thatthereactionstotheWhiskeyRebellionandFries’Rebellionwereboth“heavy-handed,”thisisnotanassumptionmadeinP2.(B) Opposite. The author deliberately compares the American Revolution against the Britishmonarchy to these rebellions against American authorities in order to highlight similarities, notdifferences.(D)Out of Scope. There is no suggestion in the passage that theBritish played any role in theseRebellions.TheBritisharemerelyraisedasapointofcomparison.

11.(B)

Thesituationdescribedinthequestionstemseemstoechotheideafromthelastsentence,thattheUSfederalgovernmentis“ultimatelyinthralltothenouveauaristocracyofcorporate‘persons’andtherapaciousclassofexecutivesthatconstitutethehomunculiwithin.”Thisisanaspectoftheauthor ’scentralargument, that thepeoplehavebeendisempoweredasmorepowerhasbeenaccumulated inthe federal government, and that this government represents the interests of the wealthy first andforemost.Thus,choice(B)isright.(A) Opposite. As explained above, the author ’s argument is actually bolstered by the newinformation.(C)Opposite.Ifanything,thisclaimwouldbestrengthened,sincetheevidenceinthequestionstemmakesitclearthatthepeoplearenotincharge.(D)Whilethenewsituationmakesitclearthatthewealthyhaveinfluenceinthe21stcentury,thisinitselfprovesnothingaboutwhathappenedinthe18thcentury,whenthenewConstitutionwascreatedandratified(explainedinP4).

Page 395: Edited By Deeangelee Pooran-Kublall, MD/MPH · 9.5 Preparing for the MCAT: Biochemistry in the Chemical and Physical Foundations of Biological Systems Section 9.6 Preparing for the

©2014byKaplan,Inc.

PublishedbyKaplanPublishing,adivisionofKaplan,Inc.395HudsonStreetNewYork,NY10014

AllrightsreservedunderInternationalandPan-AmericanCopyrightConventions.Bypaymentoftherequiredfees,youhavebeengrantedthenon-exclusive,non-transferablerighttoaccessandreadthetextofthiseBookonscreen.Nopartofthistextmaybereproduced,transmitted,downloaded,decompiled,reverseengineered,orstoredinorintroducedintoanyinformationstorageandretrievalsystem,inanyformorbyanymeans,whetherelectronicormechanical,nowknownorhereinafterinvented,withouttheexpresswrittenpermissionofthepublisher.

ISBN:978-1-61865-634-6